Π‘ΠΎΠ΄Π΅Ρ€ΠΆΠ°Π½ΠΈΠ΅

ΠœΠ΅Ρ‚ΠΎΠ΄Ρ‹ расчСта слоТных элСктричСских Ρ†Π΅ΠΏΠ΅ΠΉ постоянного Ρ‚ΠΎΠΊΠ° — Начало. ΠžΡΠ½ΠΎΠ²Ρ‹. — Π‘ΠΏΡ€Π°Π²ΠΎΡ‡Π½ΠΈΠΊ

ΠœΠ΅Ρ‚ΠΎΠ΄Ρ‹ расчСта слоТных элСктричСских Ρ†Π΅ΠΏΠ΅ΠΉ постоянного Ρ‚ΠΎΠΊΠ°

Β 

1. ΠœΠ΅Ρ‚ΠΎΠ΄ ΡƒΠ·Π»ΠΎΠ²Ρ‹Ρ… ΠΈ ΠΊΠΎΠ½Ρ‚ΡƒΡ€Π½Ρ‹Ρ… ΡƒΡ€Π°Π²Π½Π΅Π½ΠΈΠΉ

Π’ основС расчСта Π»Π΅ΠΆΠ°Ρ‚ ΠΏΠ΅Ρ€Π²Ρ‹ΠΉ ΠΈ Π²Ρ‚ΠΎΡ€ΠΎΠΉ Π·Π°ΠΊΠΎΠ½Ρ‹ ΠšΠΈΡ€Ρ…Π³ΠΎΡ„Π°.

Β Β Β Β Β Β Β  Β Β Β Β Β Β Β Β Β Β Β Β Β Β Β Β Β Β Β Β Β Β Β Β  Β  Β  Β  Β  Β Β  Β  βˆ‘I=0

Β Β Β Β Β Β Β Β Β Β Β Β Β Β Β Β Β Β Β Β Β Β Β Β Β  Β Β Β Β Β Β Β Β Β Β Β Β Β Β Β Β Β Β Β Β βˆ‘E=βˆ‘IR

ΠŸΠΎΡ€ΡΠ΄ΠΎΠΊ расчСта

  1. ΠŸΡ€ΠΎΠΈΠ·Π²ΠΎΠ»ΡŒΠ½ΠΎ Π²Ρ‹Π±ΠΈΡ€Π°Π΅ΠΌ Π½Π°ΠΏΡ€Π°Π²Π»Π΅Π½ΠΈΠ΅ Ρ‚ΠΎΠΊΠ° Π² вСтвях.
  2. ΠŸΡ€ΠΎΠΈΠ·Π²ΠΎΠ»ΡŒΠ½ΠΎ Π²Ρ‹Π±ΠΈΡ€Π°Π΅ΠΌ Π½Π°ΠΏΡ€Π°Π²Π»Π΅Π½ΠΈΠ΅ ΠΎΠ±Ρ…ΠΎΠ΄Π° ΠΊΠΎΠ½Ρ‚ΡƒΡ€ΠΎΠ².
  3. Зная ΠΏΠΎΠ»ΡΡ€Π½ΠΎΡΡ‚ΡŒ источников, проставляСм Π½Π°ΠΏΡ€Π°Π²Π»Π΅Π½ΠΈΠ΅ Π­Π”Π‘.
  4. БоставляСм уравнСния ΠΏΠΎ ΠΏΠ΅Ρ€Π²ΠΎΠΌΡƒ Π·Π°ΠΊΠΎΠ½Ρƒ ΠšΠΈΡ€Ρ…Π³ΠΎΡ„Π°. Π˜Ρ… Π΄ΠΎΠ»ΠΆΠ½ΠΎ Π±Ρ‹Ρ‚ΡŒ Π½ΠΎ ΠΎΠ΄Π½ΠΎ мСньшС, Ρ‡Π΅ΠΌ ΡƒΠ·Π»ΠΎΠ².
  5. БоставляСм уравнСния ΠΏΠΎ Π²Ρ‚ΠΎΡ€ΠΎΠΌΡƒ Π·Π°ΠΊΠΎΠ½Ρƒ ΠšΠΈΡ€Ρ…Π³ΠΎΡ„Π° ΠΈΠ· расчСта, Ρ‡Ρ‚ΠΎ ΠΎΠ±Ρ‰Π΅Π΅ число ΡƒΡ€Π°Π²Π½Π΅Π½ΠΈΠΉ Π΄ΠΎΠ»ΠΆΠ½ΠΎ Π±Ρ‹Ρ‚ΡŒ Ρ€Π°Π²Π½ΠΎ числу нСизвСстных Ρ‚ΠΎΠΊΠΎΠ².
  6. РСшаСм систСму ΡƒΡ€Π°Π²Π½Π΅Π½ΠΈΠΉ ΠΈ опрСдСляСм нСизвСстныС Ρ‚ΠΎΠΊΠΈ. Если Π² Ρ€Π΅Π·ΡƒΠ»ΡŒΡ‚Π°Ρ‚Π΅ Ρ€Π΅ΡˆΠ΅Π½ΠΈΡ ΠΊΠ°ΠΊΠΎΠΉ-Π»ΠΈΠ±ΠΎ Ρ‚ΠΎΠΊ окаТСтся со Π·Π½Π°ΠΊΠΎΠΌ Β«-Β», Ρ‚ΠΎ Π½Π°ΠΏΡ€Π°Π²Π»Π΅Π½ΠΈΠ΅ Π΅Π³ΠΎ ΠΏΡ€ΠΎΡ‚ΠΈΠ²ΠΎΠΏΠΎΠ»ΠΎΠΆΠ½ΠΎ Π²Ρ‹Π±Ρ€Π°Π½Π½ΠΎΠΌΡƒ.

ΠŸΡ€ΠΈΠ²Π΅Π΄Π΅ΠΌ ΠΏΡ€ΠΈΠΌΠ΅Ρ€.

Π”Π°Π½ΠΎ:

  1. 1=r2=0;
  2. 1=0,3 Ом;
  3. 2=1 Ом;
  4. 3=24 Ом;

Π•1=246 Π’;

Π•2=230Π’

Найти:

I1,I2,I3.


Β 

РСшСниС:

Π˜Ρ‚Π°ΠΊ, Π½Π° схСмС рисуСм направлСния Ρ‚ΠΎΠΊΠΎΠ² (1), согласно этим направлСниям рисуСм направлСния ΠΎΠ±Ρ…ΠΎΠ΄Π° ΠΊΠΎΠ½Ρ‚ΡƒΡ€ΠΎΠ² (2), согласно полярности источников питания ставим направлСния Π­Π”Π‘ (3).

Богласно ΠΏΠ΅Ρ€Π²ΠΎΠΌΡƒ Π·Π°ΠΊΠΎΠ½Ρƒ ΠšΠΈΡ€Ρ…Π³ΠΎΡ„Π°:

Β Β Β Β Β Β Β Β Β Β Β Β Β Β Β Β Β Β Β Β Β Β Β Β Β Β Β Β Β Β Β Β Β Β Β  I1-I2-I3=0 β†’ -I2=I3-I1

Π’Π΅ΠΏΠ΅Ρ€ΡŒ составляСм уравнСния ΠΏΠΎ Π²Ρ‚ΠΎΡ€ΠΎΠΌΡƒ Π·Π°ΠΊΠΎΠ½Ρƒ ΠšΠΈΡ€Ρ…Π³ΠΎΡ„Π°:

Β Β Β Β Β Β Β Β Β Β Β Β Β Β Β Β Β Β Β Β Β Β Β  Β Β Β Β Β Β Β Β Β Β Β Β Β Β Β Β Β Β Β Β Β Β Β E1=I1R1+I3R3

Β Β Β Β Β Β Β Β Β Β Β Β Β Β Β Β Β Β Β Β Β Β Β Β  Β Β Β Β Β Β Β Β Β Β Β Β Β Β Β Β Β Β Β Β Β Β Π•2=-I2R2+I3R3

ΠŸΠΎΠ»ΡƒΡ‡ΠΈΠ»ΠΈ систСму ΠΈΠ· Ρ‚Ρ€Π΅Ρ… ΡƒΡ€Π°Π²Π½Π΅Π½ΠΈΠΉ. РСшаСм.

Β Β Β Β Β Β Β Β Β Β Β Β Β Β Β Β Β Β Β Β Β Β Β Β  Β Β Β Β Β Β Β Β Β Β Β Β Β Β Β Β Β Β Β Β Β E2

=(I3-I1)R2+I3R3

Β Β Β Β Β Β Β Β Β Β Β Β Β Β Β Β Β Β Β Β Β Β Β Β Β  Β Β Β Β Β Β Β Β Β Β Β Β Β Β Β Β Β Β Β 230=I3(1+R3)-I1=25I3-I1 β†’ I1= 25I3-230

Β Β Β Β Β Β Β Β Β Β Β Β Β Β Β Β Β Β Β Β Β Β Β Β Β  Β Β Β Β Β Β Β Β Β Β Β Β Β Β Β Β Β Β Β E1=I1R1+I3R3=(25I3-230)R1+I3R3

Β Β Β Β Β Β Β Β Β Β Β Β Β Β Β Β Β Β Β Β Β Β Β Β  Β Β Β Β Β Β Β Β Β Β Β Β Β Β Β Β Β Β 246=0,3(25I3-230)+24I3

Β Β Β Β Β Β Β Β Β Β  Β Β Β Β Β Β Β Β Β Β Β Β Β Β Β Β Β Β Β Β Β Β Β Β Β Β Β Β Β Β Β Β 246=7,5I3-69+24I3

Β Β Β Β Β Β Β Β Β Β Β Β Β Β Β Β Β Β Β Β Β Β Β Β Β Β Β  Β Β Β Β Β Β Β Β Β Β Β Β Β Β Β 31,5I3=315

Β Β Β Β Β Β Β Β Β Β  Β Β Β Β Β Β Β Β Β Β Β Β Β Β Β Β Β Β Β Β Β Β Β Β Β Β Β Β Β Β Β Β I3=10A

Β Β Β Β Β Β Β Β Β Β Β Β Β Β Β Β Β Β Β Β Β Β Β Β Β  Β Β Β Β Β Β Β Β Β Β Β Β Β Β Β Β Β I1=25βˆ™10-230=20A

Β Β Β Β Β Β Β Β Β Β Β Β Β Β Β Β Β Β Β Β Β Β Β Β  Β Β Β Β Β Β Β Β Β Β Β Β Β Β Β Β Β Β I2=I1-I3=20-10=10A

Β 

Β 

2. ΠœΠ΅Ρ‚ΠΎΠ΄ ΠΊΠΎΠ½Ρ‚ΡƒΡ€Π½Ρ‹Ρ… Ρ‚ΠΎΠΊΠΎΠ²

Π­Ρ‚ΠΎΡ‚ ΠΌΠ΅Ρ‚ΠΎΠ΄ основан Π½Π° Π²Ρ‚ΠΎΡ€ΠΎΠΌ Π·Π°ΠΊΠΎΠ½Π΅ ΠšΠΈΡ€Ρ…Π³ΠΎΡ„Π°

Β Β Β Β Β Β Β Β Β Β Β Β Β Β Β Β Β Β Β Β Β Β Β Β Β Β Β Β Β Β Β Β Β Β Β Β Β Β Β Β Β Β Β Β Β Β Β Β Β Β Β Β Β Β Β Β Β Β Β Β 

  1. ΠŸΡ€ΠΎΠΈΠ·Π²ΠΎΠ»ΡŒΠ½ΠΎ Π²Ρ‹Π±ΠΈΡ€Π°Π΅ΠΌ направлСния ΠΊΠΎΠ½Ρ‚ΡƒΡ€Π½Ρ‹Ρ… Ρ‚ΠΎΠΊΠΎΠ² (рис.2)
  2. БоставляСм уравнСния ΠΏΠΎ Π²Ρ‚ΠΎΡ€ΠΎΠΌΡƒ Π·Π°ΠΊΠΎΠ½Ρƒ ΠšΠΈΡ€Ρ…Π³ΠΎΡ„Π°.

Β Β Β Β Β Β Β Β Β Β Β Β Β Β Β Β Β Β Β Β Β Β Β Β Β Β Β Β Β Β Β Β Β Β Β Β Β Β Β Β Β Β Β Β Β Β Β Β Β 

Β Β Β Β Β Β Β Β  Β Β Β Β Β Β Β Β Β Β Β Β Β Β Β Β Β Β Β Β Β Β Β Β Β Β Β Β Β Β Β Β E1-E2=I1(R1+R2)-I2R2

Β Β Β Β Β Β Β Β Β Β Β Β Β Β Β  Β Β Β Β Β Β Β Β Β Β Β Β Β Β Β Β Β Β Β Β Β Β Β Β Β E2=I2(R2+R3)-I1R2

Β 

Β Β Β Β Β Β Β Β Β Β Β Β Β  Β Β Β Β Β Β Β Β Β Β Β Β Β Β Β Β Β Β Β Β Β Β Β Β Β Β Β 246-230=I1(0,3+1)-I2 β†’ 16=1,3I1-I2 β†’ I2=1,3I1-16

Β Β Β Β Β Β Β Β Β Β Β Β Β Β Β Β Β Β  Β Β Β Β Β Β Β Β Β Β  Β  Β  Β  Β  Β Β  230=25(1,3I1-16)-I1

Β Β Β  Β Β Β Β Β Β Β Β Β Β Β Β Β Β Β Β Β Β Β Β Β Β Β Β Β  Β  Β  Β  Β  Β Β  31,5I1=630

Β Β Β Β Β Β Β Β Β  Β Β Β Β Β Β Β Β Β Β Β Β Β Β Β Β Β Β Β Β Β Β Β Β Β Β Β Β Β Β Β  I1=20A

Β Β Β Β Β Β  Β Β Β Β Β Β Β Β Β Β Β Β Β Β Β Β Β Β Β Β Β Β Β Β Β Β Β Β Β Β Β Β Β Β  I

2=1,3βˆ™20-16=10A

Β 

3. ΠžΠΏΡ€Π΅Π΄Π΅Π»ΡΠ΅ΠΌ истинныС Ρ‚ΠΎΠΊΠΈ.

Β Β Β Β Β Β Β Β Β Β Β Β Β Β  Β Β Β Β Β Β Β Β Β Β Β Β Β  Β  Β  Β  Β  Β  Β  I1=I1=20A

Β Β  Β Β Β Β Β Β Β Β Β Β Β Β Β Β Β Β Β Β Β Β Β Β  Β  Β  Β  Β  Β Β  Β Β Β Β  I2=I1-I2=10A

Β Β Β Β Β Β  Β Β Β Β Β Β Β Β Β Β Β Β Β Β Β Β Β Β Β Β  Β  Β  Β  Β  Β Β  Β Β  I3=I2=10A

Β 

3. ΠœΠ΅Ρ‚ΠΎΠ΄ Π΄Π²ΡƒΡ… ΡƒΠ·Π»ΠΎΠ²

Π­Ρ‚ΠΎΡ‚ ΠΌΠ΅Ρ‚ΠΎΠ΄ ΠΏΡ€ΠΈΠΌΠ΅Π½ΠΈΠΌ для схСм, ΠΈΠΌΠ΅ΡŽΡ‰ΠΈΡ… Π΄Π²Π° ΡƒΠ·Π»Π°

Β Β Β Β Β Β Β Β Β Β Β Β Β Β Β Β Β Β Β Β Β Β Β Β Β Β Β Β Β Β Β Β Β Β Β Β Β Β Β Β Β Β Β Β Β Β Β Β Β Β Β Β Β Β Β Β Β Β Β Β Β Β Β Β Β Β Β Β 

  1. Π’Ρ‹Π±ΠΈΡ€Π°Π΅ΠΌ ΠΏΡ€ΠΎΠΈΠ·Π²ΠΎΠ»ΡŒΠ½ΠΎ направлСния Ρ‚ΠΎΠΊΠΎΠ² Π² вСтвях Π² ΠΎΠ΄Π½Ρƒ ΠΈ Ρ‚Ρƒ-ΠΆΠ΅ сторону (см. рис.3 – стрСлки со ΡˆΡ‚Ρ€ΠΈΡ…Π°ΠΌΠΈ).
  2. ΠžΠΏΡ€Π΅Π΄Π΅Π»ΡΠ΅ΠΌ проводимости Π²Π΅Ρ‚Π²Π΅ΠΉ:

Β 

Β Β Β Β Β Β Β Β Β Β Β Β Β Β Β Β Β Β Β Β Β Β Β Β Β Β Β  Β  Β  Β  Β  Β  q1=1/R1=1/0,3=3,33 Π‘ΠΈΠΌ.

Β Β Β Β Β Β Β Β Β Β Β Β Β Β Β Β Β Β Β Β Β Β Β Β  Β Β Β  Β  Β  Β  Β Β  q2=1/R2=1 Π‘ΠΈΠΌ.

Β Β Β Β Β Β Β Β Β Β Β Β Β Β  Β Β Β Β Β Β Β Β Β Β Β Β Β Β  Β  Β  Β  Β  q3=1/R3=1/24=0,0416 Π‘ΠΈΠΌ.

Β 

  1. ΠžΠΏΡ€Π΅Π΄Π΅Π»ΡΠ΅ΠΌ напряТСниС ΠΌΠ΅ΠΆΠ΄Ρƒ двумя ΡƒΠ·Π»Π°ΠΌΠΈ ΠΏΠΎ Ρ„ΠΎΡ€ΠΌΡƒΠ»Π΅:

Β Β Β Β Β Β Β Β Β Β Β Β Β Β Β Β Β Β Β Β Β Β Β Β Β Β Β Β Β Β Β Β Β Β Β Β Β  U=βˆ‘Eq/βˆ‘arq=(E1+E2q2)/(q1+q2+q3)=(246βˆ™3,31+230)/4,3716=240 Π’

  1. ΠžΠΏΡ€Π΅Π΄Π΅Π»ΡΠ΅ΠΌ Ρ‚ΠΎΠΊΠΈ Π² вСтвях

Β Β Β Β Β Β Β Β Β Β Β  Β Β Β Β Β Β Β Β Β Β Β Β Β Β Β Β Β Β  Β  Β  Β  I=(E-U)q

Β Β Β Β Β Β Β Β Β Β Β Β  Β Β Β Β Β Β Β Β Β Β Β Β Β Β Β Β Β Β Β Β Β Β Β  I1=(E1-U)q1=(246-240)3,33=20A

Β Β Β Β Β Β Β Β Β Β Β Β  Β Β Β Β Β Β Β Β Β Β Β Β Β Β Β Β Β  Β  Β  Β  I2=(E2-U)q2=230-240=-10A

Β Β Β Β Β Β Β Β Β Β Β Β  Β Β Β Β Β Β Β Β Β Β Β Β Β Β Β Β Β  Β  Β  Β  I3=-Uq3=240βˆ™0,0416=-10А

Π’Π°ΠΊ ΠΊΠ°ΠΊ, значСния I2 ΠΈ I3 ΠΏΠΎΠ»ΡƒΡ‡ΠΈΠ»ΠΈΡΡŒ ΠΎΡ‚Ρ€ΠΈΡ†Π°Ρ‚Π΅Π»ΡŒΠ½Ρ‹ΠΌΠΈ, Ρ‚ΠΎ эти Ρ‚ΠΎΠΊΠΈ Π±ΡƒΠ΄ΡƒΡ‚ ΠΏΡ€ΠΎΡ‚ΠΈΠ²ΠΎΠΏΠΎΠ»ΠΎΠΆΠ½Ρ‹ΠΌΠΈ ΠΏΠΎ Π½Π°ΠΏΡ€Π°Π²Π»Π΅Π½ΠΈΡŽ (Π½Π° рисункС ΠΏΠΎΠΊΠ°Π·Π°Π½Ρ‹ ΠΆΠΈΡ€Π½Ρ‹Π΅ ΡΠΏΠ»ΠΎΡˆΠ½Ρ‹Π΅ стрСлки).

Β 

4. ΠœΠ΅Ρ‚ΠΎΠ΄ налоТСния ΠΈΠ»ΠΈ ΠΌΠ΅Ρ‚ΠΎΠ΄ супСрпозиции

ΠœΠ΅Ρ‚ΠΎΠ΄ основан Π½Π° Ρ‚ΠΎΠΌ, Ρ‡Ρ‚ΠΎ любой Ρ‚ΠΎΠΊ Π² Ρ†Π΅ΠΏΠΈ создаСтся совмСстным дСйствиСм всСх источников питания. ΠŸΠΎΡΡ‚ΠΎΠΌΡƒ ΠΌΠΎΠΆΠ½ΠΎ Ρ€Π°ΡΡΡ‡ΠΈΡ‚Π°Ρ‚ΡŒ частичныС Ρ‚ΠΎΠΊΠΈ ΠΎΡ‚ дСйствия ΠΊΠ°ΠΆΠ΄ΠΎΠ³ΠΎ источника питания ΠΎΡ‚Π΄Π΅Π»ΡŒΠ½ΠΎ, Π° Π·Π°Ρ‚Π΅ΠΌ, Π½Π°ΠΉΡ‚ΠΈ истинныС Ρ‚ΠΎΠΊΠΈ ΠΊΠ°ΠΊ Π°Ρ€ΠΈΡ„ΠΌΠ΅Ρ‚ΠΈΡ‡Π΅ΡΠΊΡƒΡŽ ΡΠΎΡΡ‚Π°Π²Π»ΡΡŽΡ‰ΡƒΡŽ частичных.

РСшСниС

1. Рис. 4. Π•2=0; r2β‰ 0

Β Β Β Β Β Β Β Β Β Β Β Β Β Β Β Β Β Β Β Β Β Β Β Β Β Β Β Β Β Β Β Β Β Β Β Β Β Β Β Β Β Β Β Β Β Β Β Β Β Β Β 

                                      Rэ=R2R3/(R2+R3)+R1=24/25+0,3=0,96+0,3=1,26 Ом

Β Β Β Β Β Β Β Β Β Β Β Β Β Β Β Β Β Β Β Β Β Β Β Β Β Β Β Β  Β  Β  Β  Β Β  I’1=E1/Rэ=246/1,26=195,23 Ом

Β Β Β Β Β Β Β Β Β Β Β Β Β Β Β Β Β Β Β Β Β Β Β Β Β Β Β  Β  Β  Β  Β  Β  Uab=I’1R23=195,23βˆ™0,96=187,42 Π’

Β Β Β Β Β Β Β Β Β Β Β Β Β Β Β Β Β Β Β Β Β Β Β Β Β Β Β Β  Β  Β  Β  Β Β  I’2=Uab/R2=187,42 A

Β Β Β Β Β Β Β Β Β Β Β Β Β Β Β Β Β Β Β Β Β Β Β Β Β Β  Β  Β  Β  Β Β  Β  I’3= Uab/R3=187,42/24=7,8 A

Β 

2. Рис. 5. E1=0; R1β‰ 0

Β Β Β Β Β Β Β Β Β Β Β Β Β Β Β Β Β Β Β Β Β Β Β Β Β Β Β Β Β Β Β Β Β Β Β Β Β Β Β 

Β Β Β Β Β Β Β Β Β Β Β Β Β Β Β Β Β Β Β Β Β Β Β Β Β Β Β  Β  Β  Β  Β  Β  Rэ=R1R3/(R1+R3)+R2=0,3βˆ™24/24,3+1=0,29+1=1,29 Ом

Β Β Β Β Β Β Β Β Β Β Β Β Β Β Β Β Β Β Β Β Β Β Β Β Β Β Β Β  Β  Β  Β  Β  Β  I”

2=E2/Rэ=230/1,29=178,29 A

Β Β Β Β Β Β Β Β Β Β Β Β Β Β Β Β Β Β Β Β Β Β Β Β Β Β Β Β Β Β Β Β Β Β Β Β Β Β  Uab=I”2R13=178,29βˆ™0,29=51,7 Π’

Β Β Β Β Β Β Β Β Β Β Β Β Β Β Β Β Β Β Β Β Β Β Β Β Β Β Β Β  Β  Β  Β  Β  Β Β  I”1=Uab/R1=51,7/0,3=172,4 A

Β Β Β Β Β Β Β Β Β Β Β Β Β Β Β Β Β Β Β Β Β Β Β Β Β Β Β  Β  Β  Β  Β  Β  Β  I”3=Uab/R3=51,7/24=2,15 A

3. ΠžΠΏΡ€Π΅Π΄Π΅Π»ΡΠ΅ΠΌ истинныС Ρ‚ΠΎΠΊΠΈ.

Β Β Β Β Β Β Β Β Β Β Β Β Β Β Β Β Β Β Β Β Β Β Β Β Β Β Β Β Β Β Β Β Β Β Β Β Β Β Β  I1=I’1-I”1=195,23-172,4=22,83 A

Β Β Β Β Β Β Β Β Β Β Β Β Β Β Β Β Β Β Β Β Β Β Β Β Β Β Β Β Β Β Β Β Β Β Β Β Β Β Β  I2=I’2-I”2=187,42-178,29=9,13 A

Β Β Β Β Β Β Β Β Β Β Β Β Β Β Β Β Β Β Β Β Β Β Β Β Β Β Β Β Β Β Β Β Β Β Β Β Β Β Β  I3=I’3-I”3=7,8-2,15=5,65 A

ΠœΠ΅Ρ‚ΠΎΠ΄Ρ‹ расчСта слоТных элСктричСских Ρ†Π΅ΠΏΠ΅ΠΉ

ΠœΠ΅Ρ‚ΠΎΠ΄Ρ‹ расчСта слоТных элСктричСских Ρ†Π΅ΠΏΠ΅ΠΉ

Π‘Π»ΠΎΠΆΠ½ΠΎΠΉ элСктричСской Ρ†Π΅ΠΏΡŒΡŽ Π½Π°Π·Ρ‹Π²Π°ΡŽΡ‚ Ρ€Π°Π·Π²Π΅Ρ‚Π²Π»Π΅Π½Π½ΡƒΡŽ Ρ†Π΅ΠΏΡŒ с нСсколькими источниками элСктричСской энСргии. ΠŸΡ€ΠΈΠΌΠ΅Π½Π΅Π½ΠΈΠ΅ ΠΌΠ΅Ρ‚ΠΎΠ΄ΠΎΠ² эквивалСнтных ΠΏΡ€Π΅ΠΎΠ±Ρ€Π°Π·ΠΎΠ²Π°Π½ΠΈΠΉ Π² Ρ‚Π°ΠΊΠΈΡ… цСпях, ΠΊΠ°ΠΊ ΠΏΡ€Π°Π²ΠΈΠ»ΠΎ, Π½Π΅ эффСктивно, Ρ‚Π°ΠΊ ΠΊΠ°ΠΊ Π½Π΅ ΠΏΠΎΠ·Π²ΠΎΠ»ΡΡŽΡ‚ ΡƒΠΏΡ€ΠΎΡΡ‚ΠΈΡ‚ΡŒ Π΅Π΅ Π΄ΠΎ ΠΎΠ΄Π½ΠΎΠΊΠΎΠ½Ρ‚ΡƒΡ€Π½ΠΎΠΉ Ρ†Π΅ΠΏΠΈ ΠΈΠ»ΠΈ Ρ†Π΅ΠΏΠΈ с двумя ΡƒΠ·Π»Π°ΠΌΠΈ. Для расчСта Ρ‚Π°ΠΊΠΈΡ… Ρ†Π΅ΠΏΠ΅ΠΉ ΠΈΡΠΏΠΎΠ»ΡŒΠ·ΡƒΡŽΡ‚ Π±ΠΎΠ»Π΅Π΅ ΠΎΠ±Ρ‰ΠΈΠ΅ ΠΌΠ΅Ρ‚ΠΎΠ΄Ρ‹.

ΠœΠ΅Ρ‚ΠΎΠ΄ нСпосрСдствСнного примСнСния Π·Π°ΠΊΠΎΠ½ΠΎΠ² ΠšΠΈΡ€Ρ…Π³ΠΎΡ„Π°

ΠœΠ΅Ρ‚ΠΎΠ΄ Π·Π°ΠΊΠ»ΡŽΡ‡Π°Π΅Ρ‚ΡΡ Π² составлСнии систСмы ΡƒΡ€Π°Π²Π½Π΅Π½ΠΈΠΉ с ΠΏΡ€ΠΈΠΌΠ΅Π½Π΅Π½ΠΈΠ΅ΠΌ ΠΏΠ΅Ρ€Π²ΠΎΠ³ΠΎ ΠΈ Π²Ρ‚ΠΎΡ€ΠΎΠ³ΠΎ Π·Π°ΠΊΠΎΠ½ΠΎΠ² ΠšΠΈΡ€Ρ…Π³ΠΎΡ„Π° для Π·Π°Π΄Π°Π½Π½ΠΎΠΉ элСктричСской Ρ†Π΅Π½ΠΈ, Ρ€Π΅ΡˆΠ΅Π½ΠΈΠ΅ ΠΊΠΎΡ‚ΠΎΡ€ΠΎΠΉ позволяСт ΠΎΠΏΡ€Π΅Π΄Π΅Π»ΠΈΡ‚ΡŒ Ρ‚ΠΎΠΊΠΈ всСх Π²Π΅Ρ‚Π²Π΅ΠΉ Ρ†Π΅ΠΏΠΈΡŽ.

РСализация ΡΡ‚ΠΎΡŽ ΠΌΠ΅Ρ‚ΠΎΠ΄Π°, ΠΊΠ°ΠΊ ΠΈ любого Π΄Ρ€ΡƒΠ³ΠΎΠ³ΠΎ ΠΌΠ΅Ρ‚ΠΎΠ΄Π° расчСта слоТной элСктричСской Ρ†Π΅Π½ΠΈ, начинаСтся с ΠΏΡ€Π΅Π΄Π²Π°Ρ€ΠΈΡ‚Π΅Π»ΡŒΠ½ΠΎΠ³ΠΎ Π°Π½Π°Π»ΠΈΠ·Π° Π΅Π΅ схСмы с Ρ†Π΅Π»ΡŒΡŽ опрСдСлСния числа ΡƒΠ·Π»ΠΎΠ² , числа Π²Π΅Ρ‚Π²Π΅ΠΉ , числа нСзависимых ΠΊΠΎΠ½Ρ‚ΡƒΡ€ΠΎΠ² , числа Π²Π΅Ρ‚Π²Π΅ΠΉ с источниками Ρ‚ΠΎΠΊΠΎΠ², выяснСния возмоТности упрощСния схСмы.

ΠŸΡ€Π΅ΠΆΠ΄Π΅ всСго ΠΎΠΏΡ€Π΅Π΄Π΅Π»ΡΡŽΡ‚ число нСизвСстных Ρ‚ΠΎΠΊΠΎΠ², ΠΊΠΎΡ‚ΠΎΡ€ΠΎΠ΅ Ρ€Π°Π²Π½ΠΎ β€” . Для ΠΊΠ°ΠΆΠ΄ΠΎΠΉ Π²Π΅Ρ‚Π²ΠΈ Π·Π°Π΄Π°ΡŽΡ‚ ΠΏΠΎΠ»ΠΎΠΆΠΈΡ‚Π΅Π»ΡŒΠ½ΠΎΠ΅ Π½Π°ΠΏΡ€Π°Π²Π»Π΅Π½ΠΈΠ΅ Ρ‚ΠΎΠΊΠ°.

Π”Π°Π»Π΅Π΅ ΠΏΠΎ ΠΏΠ΅Ρ€Π²ΠΎΠΌΡƒ Π·Π°ΠΊΠΎΠ½Ρƒ ΠšΠΈΡ€Ρ…Π³ΠΎΡ„Π° ΡΠΎΡΡ‚Π°Π²Π»ΡΡŽΡ‚ β€” 1 нСзависимых ΡƒΡ€Π°Π²Π½Π΅Π½ΠΈΠΉ.

Π—Π°Ρ‚Π΅ΠΌ ΠΏΠΎ Π²Ρ‚ΠΎΡ€ΠΎΠΌΡƒ Π·Π°ΠΊΠΎΠ½Ρƒ ΡΠΎΡΡ‚Π°Π²Π»ΡΡŽΡ‚ ΡƒΡ€Π°Π²Π½Π΅Π½ΠΈΠΉ. ΠŸΡ€ΠΈ этом Π²Ρ‹Π±ΠΈΡ€Π°ΡŽΡ‚ нСзависимыС ΠΊΠΎΠ½Ρ‚ΡƒΡ€Ρ‹, Π½Π΅ содСрТащиС источников Ρ‚ΠΎΠΊΠ°.

ΠžΠ±Ρ‰Π΅Π΅ число составлСнных ΠΏΠΎ ΠΏΠ΅Ρ€Π²ΠΎΠΌΡƒ ΠΈ Π²Ρ‚ΠΎΡ€ΠΎΠΌΡƒ Π·Π°ΠΊΠΎΠ½Π°ΠΌ ΠšΠΈΡ€Ρ…Π³ΠΎΡ„Π° Π΄ΠΎΠ»ΠΆΠ½ΠΎ Π±Ρ‹Ρ‚ΡŒ Ρ€Π°Π²Π½ΠΎ числу нСизвСстных Ρ‚ΠΎΠΊΠΎΠ².

Рассмотрим ΠΏΡ€ΠΈΠΌΠ΅Π½Π΅Π½ΠΈΠ΅ Π·Π°ΠΊΠΎΠ½ΠΎΠ² ΠšΠΈΡ€Ρ…Π³ΠΎΡ„Π° для опрСдСлСния Ρ‚ΠΎΠΊΠΎΠ² Π² вСтвях Ρ†Π΅ΠΏΠΈ, схСма ΠΊΠΎΡ‚ΠΎΡ€ΠΎΠΉ ΠΏΡ€ΠΈΠ²Π΅Π΄Π΅Π½Π° Π½Π° рис. 1.25. ΠŸΡƒΡΡ‚ΡŒ Π­Π”Π‘ ΠΈΠ΄Π΅Π°Π»ΡŒΠ½Ρ‹Ρ… источников напряТСния , сопротивлСния . ВрСбуСтся ΠΎΠΏΡ€Π΅Π΄Π΅Π»ΠΈΡ‚ΡŒ всС Ρ‚ΠΎΠΊΠΈ схСмы с ΠΏΠΎΠΌΠΎΡ‰ΡŒΡŽ ΠΌΠ΅Ρ‚ΠΎΠ΄Π° нСпосрСдствСнного примСнСния Π·Π°ΠΊΠΎΠ½ΠΎΠ² ΠšΠΈΡ€Ρ…Π³ΠΎΡ„Π°.

Π‘Ρ…Π΅ΠΌΠ° содСрТит 6 Π²Π΅Ρ‚Π²Π΅ΠΉ с нСизвСстными Ρ‚ΠΎΠΊΠ°ΠΌΠΈ ΠΈ Ρ‡Π΅Ρ‚Ρ‹Ρ€Π΅ ΡƒΠ·Π»Π°. Па схСмС ΡƒΠ·Π»Ρ‹ ΠΎΠ±ΠΎΠ·Π½Π°Ρ‡Π΅Π½Ρ‹ арабскими Ρ†ΠΈΡ„Ρ€Π°ΠΌΠΈ, ΠΏΠΎΠΊΠ°Π·Π°Π½Ρ‹ принятыС направлСния Ρ‚ΠΎΠΊΠΎΠ² ΠΈ направлСния ΠΎΠ±Ρ…ΠΎΠ΄Π° ΠΊΠΎΠ½Ρ‚ΡƒΡ€ΠΎΠ² А, Π‘ ΠΈ Π’.

Боставим систСму ΠΈΠ· 6 ΡƒΡ€Π°Π²Π½Π΅Π½ΠΈΠΉ. УравнСния ΠΏΠΎ ΠΏΠ΅Ρ€Π²ΠΎΠΌΡƒ Π·Π°ΠΊΠΎΠ½Ρƒ ΠšΠΈΡ€Ρ…Π³ΠΎΡ„Π° запишСм для ΡƒΠ·Π»ΠΎΠ² 1, 2, 3, уравнСния ΠΏΠΎ Π²Ρ‚ΠΎΡ€ΠΎΠΌΡƒ Π·Π°ΠΊΠΎΠ½Ρƒ ΠšΠΈΡ€Ρ…Π³ΠΎΡ„Π° запишСм для ΠΊΠΎΠ½Ρ‚ΡƒΡ€ΠΎΠ² А, Π‘, Π’:

РСшив эту систСму ΡƒΡ€Π°Π²Π½Π΅Π½ΠΈΠΉ, ΠΏΠΎΠ»ΡƒΡ‡ΠΈΠΌ . ΠžΡ‚Ρ€ΠΈΡ†Π°Ρ‚Π΅Π»ΡŒΠ½ΠΎΠ΅ Π·Π½Π°Ρ‡Π΅Π½ΠΈΠ΅ Ρ‚ΠΎΠΊΠ° , ΡƒΠΊΠ°Π·Ρ‹Π²Π°Π΅Ρ‚ Π½Π° Ρ‚ΠΎ, Ρ‡Ρ‚ΠΎ Π²Ρ‹Π±Ρ€Π°Π½Π½ΠΎΠ΅ ΠΏΡ€ΠΈ составлСнии ΡƒΡ€Π°Π²Π½Π΅Π½ΠΈΠΉ Π½Π°ΠΏΡ€Π°Π²Π»Π΅Π½ΠΈΠ΅ этого Ρ‚ΠΎΠΊΠ° Π½Π΅ соотвСтствуСт Π΄Π΅ΠΉΡΡ‚Π²ΠΈΡ‚Π΅Π»ΡŒΠ½ΠΎΡΡ‚ΠΈ. ΠŸΡ€Π°Π²ΠΈΠ»ΡŒΠ½ΠΎΠ΅ Π½Π°ΠΏΡ€Π°Π²Π»Π΅Π½ΠΈΠ΅ β€” ΠΎΡ‚ ΡƒΠ·Π»Π° 3 ΠΊ ΡƒΠ·Π»Ρƒ 4.

Для ΠΏΡ€ΠΎΠ²Π΅Ρ€ΠΊΠΈ вычислСний с ΠΏΠΎΠΌΠΎΡ‰ΡŒΡŽ ΠΏΡ€ΠΎΠ³Ρ€Π°ΠΌΠΌΡ‹ схСмотСхничСского модСлирования Micro Π‘Π°Ρ€ Π²Ρ‹ΠΏΠΎΠ»Π½Π΅Π½ Π°Π½Π°Π»ΠΈΠ· ΠΏΠΎ постоянному Ρ‚ΠΎΠΊΡƒ схСмы, ΠΈΠ·ΠΎΠ±Ρ€Π°ΠΆΠ΅Π½Π½ΠΎΠΉ Π½Π° рис. 1.25. Π˜Π·ΠΎΠ±Ρ€Π°ΠΆΠ΅Π½Π½Ρ‹Π΅ Π½Π° рис. 1.26,Π° значСния Ρ‚ΠΎΠΊΠΎΠ² Π²Π΅Ρ‚Π²Π΅ΠΉ (Π² мА) ΠΏΠΎΠ΄Ρ‚Π²Π΅Ρ€ΠΆΠ΄Π°ΡŽΡ‚ ΠΏΡ€Π°Π²ΠΈΠ»ΡŒΠ½ΠΎΡΡ‚ΡŒ Π²Ρ‹ΠΏΠΎΠ»Π½Π΅Π½Π½Ρ‹Ρ… расчСтов. Π˜Π·ΠΎΠ±Ρ€Π°ΠΆΠ΅Π½Π½Ρ‹Π΅ Π½Π° рис. 1.26,Π± ΡƒΠ·Π»ΠΎΠ²Ρ‹Π΅ ΠΏΠΎΡ‚Π΅Π½Ρ†ΠΈΠ°Π»Ρ‹ схСмы (Π² Π’) ΠΏΠΎΠ·Π²ΠΎΠ»ΡΡŽΡ‚ ΠΎΠΏΡ€Π΅Π΄Π΅Π»ΠΈΡ‚ΡŒ Π½Π°ΠΏΡ€Π°Π²Π»Π΅Π½ΠΈΠ΅ Ρ‚ΠΎΠΊΠΎΠ² Π²Π΅Ρ‚Π²Π΅ΠΉ.

ΠœΠ΅Ρ‚ΠΎΠ΄ ΠΊΠΎΠ½Ρ‚ΡƒΡ€Π½Ρ‹Ρ… Ρ‚ΠΎΠΊΠΎΠ²

ΠœΠ΅Ρ‚ΠΎΠ΄ ΠΊΠΎΠ½Ρ‚ΡƒΡ€Π½Ρ‹Ρ… Ρ‚ΠΎΠΊΠΎΠ² Π½Π°ΠΈΠ±ΠΎΠ»Π΅Π΅ часто примСняСтся Π½Π° ΠΏΡ€Π°ΠΊΡ‚ΠΈΠΊΠ΅ для расчСта слоТных Ρ†Π΅ΠΏΠ΅ΠΉ, Ρ‚Π°ΠΊ ΠΊΠ°ΠΊ ΠΎΠ½ позволяСт Π½Π°Ρ…ΠΎΠ΄ΠΈΡ‚ΡŒ всС нСизвСстныС Π²Π΅Π»ΠΈΡ‡ΠΈΠ½Ρ‹ ΠΏΡ€ΠΈ числС ΡƒΡ€Π°Π²Π½Π΅Π½ΠΈΠΉ, мСньшСм числа нСизвСстных Π²Π΅Π»ΠΈΡ‡ΠΈΠ½.

По этому ΠΌΠ΅Ρ‚ΠΎΠ΄Ρƒ Π² ΠΊΠ°ΠΆΠ΄ΠΎΠΌ нСзависимом ΠΊΠΎΠ½Ρ‚ΡƒΡ€Π΅ схСмы вмСсто Π΄Π΅ΠΉΡΡ‚Π²ΠΈΡ‚Π΅Π»ΡŒΠ½Ρ‹Ρ… Ρ‚ΠΎΠΊΠΎΠ² Π² вСтвях вводят условный ΠΊΠΎΠ½Ρ‚ΡƒΡ€Π½Ρ‹ΠΉ Ρ‚ΠΎΠΊ. Π”Π΅ΠΉΡΡ‚Π²ΠΈΡ‚Π΅Π»ΡŒΠ½Ρ‹ΠΉ Ρ‚ΠΎΠΊ Π² любой Π²Π΅Ρ‚Π²ΠΈ, ΠΏΡ€ΠΈΠ½Π°Π΄Π»Π΅ΠΆΠ°Ρ‰Π΅ΠΉ Ρ‚ΠΎΠ»ΡŒΠΊΠΎ ΠΎΠ΄Π½ΠΎΠΌΡƒ ΠΊΠΎΠ½Ρ‚ΡƒΡ€Ρƒ, числСнно Ρ€Π°Π²Π΅Π½ ΠΊΠΎΠ½Ρ‚ΡƒΡ€Π½ΠΎΠΌΡƒ Ρ‚ΠΎΠΊΡƒ. Π”Π΅ΠΉΡΡ‚Π²ΠΈΡ‚Π΅Π»ΡŒΠ½Ρ‹ΠΉ Ρ‚ΠΎΠΊ Π² любой Π²Π΅Ρ‚Π²ΠΈ, ΠΏΡ€ΠΈΠ½Π°Π΄Π»Π΅ΠΆΠ°Ρ‰Π΅ΠΉ нСскольким ΠΊΠΎΠ½Ρ‚ΡƒΡ€Π°ΠΌ Ρ€Π°Π²Π΅Π½ алгСбраичСской суммС ΠΊΠΎΠ½Ρ‚ΡƒΡ€Π½Ρ‹Ρ… Ρ‚ΠΎΠΊΠΎΠ², проходящих Ρ‡Π΅Ρ€Π΅Π· эту Π²Π΅Ρ‚Π²ΡŒ.

УравнСния для расчСта ΠΊΠΎΠ½Ρ‚ΡƒΡ€Π½Ρ‹Ρ… Ρ‚ΠΎΠΊΠΎΠ² ΡΠΎΡΡ‚Π°Π²Π»ΡΡŽΡ‚ΡΡ ΠΏΠΎ Π²Ρ‚ΠΎΡ€ΠΎΠΌΡƒ Π·Π°ΠΊΠΎΠ½Ρƒ ΠšΠΈΡ€Ρ…Π³ΠΎΡ„Π°. ΠŸΡ€ΠΈ этом ΡƒΡ‡ΠΈΡ‚Ρ‹Π²Π°ΡŽΡ‚ΡΡ напряТСния Π½Π° всСх пассивных элСмСнтах ΠΊΠΎΠ½Ρ‚ΡƒΡ€Π° ΠΎΡ‚ собствСнного ΠΊΠΎΠ½Ρ‚ΡƒΡ€Π½ΠΎΠ³ΠΎ Ρ‚ΠΎΠΊΠ° ΠΈ Π² смСТных элСмСнтах -ΠΎΡ‚ ΠΊΠΎΠ½Ρ‚ΡƒΡ€Π½Ρ‹Ρ… Ρ‚ΠΎΠΊΠΎΠ² сосСдних ΠΊΠΎΠ½Ρ‚ΡƒΡ€ΠΎΠ². НаправлСниС ΠΊΠΎΠ½Ρ‚ΡƒΡ€Π½ΠΎΠ³ΠΎ Ρ‚ΠΎΠΊΠ° Π² нСзависимом ΠΊΠΎΠ½Ρ‚ΡƒΡ€Π΅ Π²Ρ‹Π±ΠΈΡ€Π°ΡŽΡ‚ ΠΏΡ€ΠΎΠΈΠ·Π²ΠΎΠ»ΡŒΠ½ΠΎ. НаправлСниС ΠΎΠ±Ρ…ΠΎΠ΄Π° ΠΊΠΎΠ½Ρ‚ΡƒΡ€Π° ΠΎΠ±Ρ‹Ρ‡Π½ΠΎ Π²Ρ‹Π±ΠΈΡ€Π°ΡŽΡ‚ ΡΠΎΠ²ΠΏΠ°Π΄Π°ΡŽΡ‰ΠΈΠΌ с Π½Π°ΠΏΡ€Π°Π²Π»Π΅Π½ΠΈΠ΅ΠΌ собствСнного ΠΊΠΎΠ½Ρ‚ΡƒΡ€Π½ΠΎΠ³ΠΎ Ρ‚ΠΎΠΊΠ°.

ПадСниС напряТСния ΠΏΡ€ΠΈ ΠΏΡ€ΠΎΡ…ΠΎΠΆΠ΄Π΅Π½ΠΈΠΈ Ρ‚ΠΎΠΊΠ° смСТного ΠΊΠΎΠ½Ρ‚ΡƒΡ€Π° Π² элСмСнтС ΠΏΡ€ΠΈΠ½ΠΈΠΌΠ°ΡŽΡ‚ ΠΏΠΎΠ»ΠΎΠΆΠΈΡ‚Π΅Π»ΡŒΠ½Ρ‹ΠΌ, Ссли Π½Π°ΠΏΡ€Π°Π²Π»Π΅Π½ΠΈΠ΅ Ρ‚ΠΎΠΊΠ° Π² смСТном ΠΊΠΎΠ½Ρ‚ΡƒΡ€Π΅ совпадаСт с Π½Π°ΠΏΡ€Π°Π²Π»Π΅Π½ΠΈΠ΅ΠΌ ΠΎΠ±Ρ…ΠΎΠ΄Π°, Если Π½Π°ΠΏΡ€Π°Π²Π»Π΅Π½ΠΈΠ΅ Ρ‚ΠΎΠΊΠ° смСТного ΠΊΠΎΠ½Ρ‚ΡƒΡ€Π° Π½Π΅ совпадаСт с Π½Π°ΠΏΡ€Π°Π²Π»Π΅Π½ΠΈΠ΅ΠΌ ΠΎΠ±Ρ…ΠΎΠ΄Π°, ΠΏΠ°Π΄Π΅Π½ΠΈΠ΅ напряТСния ΡΡ‡ΠΈΡ‚Π°ΡŽΡ‚ ΠΎΡ‚Ρ€ΠΈΡ†Π°Ρ‚Π΅Π»ΡŒΠ½Ρ‹ΠΌ. Π—Π½Π°Ρ‡Π΅Π½ΠΈΠ΅ Π­Π”Π‘ бСрСтся со Π·Π½Π°ΠΊΠΎΠΌ плюс, Ссли Π½Π°ΠΏΡ€Π°Π²Π»Π΅Π½ΠΈΠ΅ ΠΎΠ±Ρ…ΠΎΠ΄Π° ΠΊΠΎΠ½Ρ‚ΡƒΡ€Π° совпадаСт с ΠΏΠΎΠ»ΠΎΠΆΠΈΡ‚Π΅Π»ΡŒΠ½Ρ‹ΠΌ Π½Π°ΠΏΡ€Π°Π²Π»Π΅Π½ΠΈΠ΅ΠΌ Π­Π”Π‘, ΠΈ со Π·Π½Π°ΠΊΠΎΠΌ минус β€” Ссли Π½Π΅ совпадаСт.

ΠœΠ΅Ρ‚ΠΎΠ΄ ΠΊΠΎΠ½Ρ‚ΡƒΡ€Π½Ρ‹Ρ… Ρ‚ΠΎΠΊΠΎΠ² рассмотрим Π½Π° ΠΏΡ€ΠΈΠΌΠ΅Ρ€Π΅ схСмы элСктричСской Ρ†Π΅ΠΏΠΈ, ΠΈΠ·ΠΎΠ±Ρ€Π°ΠΆΠ΅Π½Π½ΠΎΠΉ Π½Π° рис. 1.27. Π‘Ρ…Π΅ΠΌΠ° ΠΈΠΌΠ΅Π΅Ρ‚ Ρ‚Ρ€ΠΈ нСзависимых ΠΊΠΎΠ½Ρ‚ΡƒΡ€Π°: А, Π‘, Π’. Π§Π΅Ρ€Π΅Π· сопротивлСния ΠΊΠ°ΠΆΠ΄ΠΎΠ³ΠΎ ΠΊΠΎΠ½Ρ‚ΡƒΡ€Π° ΠΏΡ€ΠΎΡ…ΠΎΠ΄ΠΈΡ‚ свой ΠΊΠΎΠ½Ρ‚ΡƒΡ€Π½Ρ‹ΠΉ Ρ‚ΠΎΠΊ . НаправлСния ΠΎΠ±Ρ…ΠΎΠ΄Π° ΠΊΠ°ΠΆΠ΄ΠΎΠ³ΠΎ ΠΊΠΎΠ½Ρ‚ΡƒΡ€Π° совпадаСт с Π½Π°ΠΏΡ€Π°Π²Π»Π΅Π½ΠΈΠ΅ΠΌ ΠΊΠΎΠ½Ρ‚ΡƒΡ€Π½ΠΎΠ³ΠΎ Ρ‚ΠΎΠΊΠ° этого ΠΊΠΎΠ½Ρ‚ΡƒΡ€Π°. Π­Π”Π‘ ΠΈΠ΄Π΅Π°Π»ΡŒΠ½Ρ‹Ρ… источников напряТСния , сопротивлСния ΠΈ .

УравнСния, составлСнныС ΠΏΠΎ Π²Ρ‚ΠΎΡ€ΠΎΠΌΡƒ Π·Π°ΠΊΠΎΠ½Ρƒ ΠšΠΈΡ€Ρ…Π³ΠΎΡ„Π°, для ΠΊΠΎΠ½Ρ‚ΡƒΡ€ΠΎΠ² А, Π‘ ΠΈ Π’:

ΠŸΠΎΠ΄ΡΡ‚Π°Π²ΠΈΠ² Π² эту систСму ΡƒΡ€Π°Π²Π½Π΅Π½ΠΈΠΉ числСнныС значСния Π­Π”Π‘ источников ΠΈ сопротивлСний ΠΈ Ρ€Π΅ΡˆΠΈΠ² Π΅Π΅, ΠΏΠΎΠ»ΡƒΡ‡ΠΈΠΌ

Π”Π΅ΠΉΡΡ‚Π²ΠΈΡ‚Π΅Π»ΡŒΠ½Ρ‹Π΅ Ρ‚ΠΎΠΊΠΈ Π²Π΅Ρ‚Π²Π΅ΠΉ схСмы:

ΠŸΠΎΠ»ΡƒΡ‡Π΅Π½Π½Ρ‹Π΅ значСния ΠΏΠΎΠ»Π½ΠΎΡΡ‚ΡŒΡŽ ΡΠΎΠ²ΠΏΠ°Π΄Π°ΡŽΡ‚ с Ρ€Π΅Π·ΡƒΠ»ΡŒΡ‚Π°Ρ‚Π°ΠΌΠΈ Ρ€Π°Π½Π΅Π΅ ΠΏΡ€ΠΎΠ΄Π΅Π»Π°Π½Π½ΠΎΠ³ΠΎ расчСта этой ΠΆΠ΅ Ρ†Π΅Π½ΠΈ ΠΏΠΎ ΠΌΠ΅Ρ‚ΠΎΠ΄Ρƒ нСпосрСдствСнного примСнСния Π—Π°ΠΊΠΎΠ½ΠΎΠ² ΠšΠΈΡ€Ρ…Π³ΠΎΡ„Π°.

ΠœΠ΅Ρ‚ΠΎΠ΄ ΡƒΠ·Π»ΠΎΠ²Ρ‹Ρ… ΠΏΠΎΡ‚Π΅Π½Ρ†ΠΈΠ°Π»ΠΎΠ²

ΠŸΠΎΡ‚Π΅Π½Ρ†ΠΈΠ°Π» любой Ρ‚ΠΎΡ‡ΠΊΠΈ элСктричСской Ρ†Π΅ΠΏΠΈ опрСдСляСтся напряТСниСм ΠΌΠ΅ΠΆΠ΄Ρƒ Π΄Π°Π½Π½ΠΎΠΉ Ρ‚ΠΎΡ‡ΠΊΠΎΠΉ ΠΈ Ρ‚ΠΎΡ‡ΠΊΠΎΠΉ Ρ†Π΅ΠΏΠΈ с ΠΏΠΎΡ‚Π΅Π½Ρ†ΠΈΠ°Π»ΠΎΠΌ Ρ€Π°Π²Π½Ρ‹ΠΌ Π½ΡƒΠ»ΡŽ.

ΠœΠ΅Ρ‚ΠΎΠ΄ ΡƒΠ·Π»ΠΎΠ²Ρ‹Ρ… ΠΏΠΎΡ‚Π΅Π½Ρ†ΠΈΠ°Π»ΠΎΠ² Π·Π°ΠΊΠ»ΡŽΡ‡Π°Π΅Ρ‚ΡΡ Π² Ρ‚ΠΎΠΌ, Ρ‡Ρ‚ΠΎ Π²Π½Π°Ρ‡Π°Π»Π΅ ΠΏΠΎΠ»Π°Π³Π°ΡŽΡ‚ Ρ€Π°Π²Π½Ρ‹ΠΌ Π½ΡƒΠ»ΡŽ ΠΏΠΎΡ‚Π΅Π½Ρ†ΠΈΠ°Π» Π½Π΅ΠΊΠΎΡ‚ΠΎΡ€ΠΎΠ³ΠΎ базисного ΡƒΠ·Π»Π° ΠΈ для ΠΎΡΡ‚Π°Π²ΡˆΠΈΡ…ΡΡ ( -1) ΡƒΠ·Π»ΠΎΠ² ΡΠΎΡΡ‚Π°Π²Π»ΡΡŽΡ‚ уравнСния ΠΏΠΎ ΠΏΠ΅Ρ€Π²ΠΎΠΌΡƒ Π·Π°ΠΊΠΎΠ½Ρƒ ΠšΠΈΡ€Ρ…Π³ΠΎΡ„Π°: алгСбраичСская сумма Ρ‚ΠΎΠΊΠΎΠ² всСх Π²Π΅Ρ‚Π²Π΅ΠΉ, ΠΏΠΎΠ΄ΠΊΠ»ΡŽΡ‡Π΅Π½Π½Ρ‹Ρ… ΠΊ рассматриваСмому ΡƒΠ·Π»Ρƒ Ρ€Π°Π²Π½Π° Π½ΡƒΠ»ΡŽ. , ΠΏΠΎΠ»ΡƒΡ‡Π΅Π½Π½Ρ‹ΠΌΠΈ ΠΏΡ€ΠΈ Π²Ρ‹ΠΏΠΎΠ»Π½Π΅Π½ΠΈΠΈ с ΠΏΠΎΠΌΠΎΡ‰ΡŒΡŽ ΠΏΡ€ΠΎΠ³Ρ€Π°ΠΌΠΌΡ‹ Micro-Π‘Π°Ρ€ Π°Π½Π°Π»ΠΈΠ·Π° ΠΏΠΎ постоянному Ρ‚ΠΎΠΊΡƒ схСмы, ΠΈΠ·ΠΎΠ±Ρ€Π°ΠΆΠ΅Π½Π½ΠΎΠΉ Π½Π° рис. 1.28,Π°.

ΠŸΡ€ΠΈΠΌΠ΅Π½ΠΈΠ² ΠΎΠ±ΠΎΠ±Ρ‰Π΅Π½Π½Ρ‹ΠΉ Π·Π°ΠΊΠΎΠ½ Ома для ΠΊΠ°ΠΆΠ΄ΠΎΠΉ Π²Π΅Ρ‚Π²ΠΈ схСмы, ΠΏΠΎΠ»ΡƒΡ‡ΠΈΠΌ искомыС Ρ‚ΠΎΠΊΠΈ:

ΠŸΠΎΠ»ΡƒΡ‡Π΅Π½Π½Ρ‹Π΅ значСния Ρ‚ΠΎΠΊΠΎΠ² ΡΠΎΠ²ΠΏΠ°Π΄Π°ΡŽΡ‚ с Ρ€Π΅Π·ΡƒΠ»ΡŒΡ‚Π°Ρ‚Π°ΠΌΠΈ расчСта этой Ρ†Π΅ΠΏΠΈ ΠΌΠ΅Ρ‚ΠΎΠ΄ΠΎΠΌ нСпосрСдствСнного примСнСния Π·Π°ΠΊΠΎΠ½ΠΎΠ² ΠšΠΈΡ€Ρ…ΠΎΡ„Π° ΠΈ ΠΌΠ΅Ρ‚ΠΎΠ΄ΠΎΠΌ ΠΊΠΎΠ½Ρ‚ΡƒΡ€Π½Ρ‹Ρ… Ρ‚ΠΎΠΊΠΎΠ².

НаправлСния Π½Π°ΠΉΠ΄Π΅Π½Π½Ρ‹Ρ… Ρ‚ΠΎΠΊΠΎΠ² ΡƒΠΊΠ°Π·Π°Π½Ρ‹ Π½Π° Π³Ρ€Π°Ρ„Π΅ Ρ†Π΅ΠΏΠΈ Π½Π° рис. 1.28,6. Π“Ρ€Π°Ρ„ΠΎΠΌ Ρ†Π΅ΠΏΠΈ Π½Π°Π·Ρ‹Π²Π°ΡŽΡ‚ Ρ‚Π°ΠΊΠΎΠ΅ ΠΈΠ·ΠΎΠ±Ρ€Π°ΠΆΠ΅Π½ΠΈΠ΅ схСмы элСктричСской Ρ†Π΅ΠΏΠΈ, Π² ΠΊΠΎΡ‚ΠΎΡ€ΠΎΠΌ всС Π²Π΅Ρ‚Π²ΠΈ Π·Π°ΠΌΠ΅Π½Π΅Π½Ρ‹ линиями, источники напряТСния Π·Π°ΠΊΠΎΡ€ΠΎΡ‡Π΅Π½Ρ‹, Π° источники Ρ‚ΠΎΠΊΠ° Ρ€Π°Π·ΠΎΠΌΠΊΠ½ΡƒΡ‚Ρ‹. ВсС Π²Π΅Ρ‚Π²ΠΈ ΠΈ всС ΡƒΠ·Π»Ρ‹ ΡΠΎΡ…Ρ€Π°Π½ΡΡŽΡ‚ΡΡ.

ΠœΠ΅Ρ‚ΠΎΠ΄ ΡƒΠ·Π»ΠΎΠ²Ρ‹Ρ… ΠΏΠΎΡ‚Π΅Π½Ρ†ΠΈΠ°Π»ΠΎΠ² ΠΈΠΌΠ΅Π΅Ρ‚ прСимущСство ΠΏΠ΅Ρ€Π΅Π΄ ΠΌΠ΅Ρ‚ΠΎΠ΄ΠΎΠΌ ΠΊΠΎΠ½Ρ‚ΡƒΡ€Π½Ρ‹Ρ… Ρ‚ΠΎΠΊΠΎΠ² Π² Ρ‚ΠΎΠΌ случаС, ΠΊΠΎΠ³Π΄Π° число ΡƒΡ€Π°Π²Π½Π΅Π½ΠΈΠΉ, записанных ΠΏΠΎ ΠΏΠ΅Ρ€Π²ΠΎΠΌΡƒ Π·Π°ΠΊΠΎΠ½Ρƒ ΠšΠΈΡ€Ρ…Π³ΠΎΡ„Π°, мСньшС числа ΡƒΡ€Π°Π²Π½Π΅Π½ΠΈΠΉ, записанных ΠΏΠΎ Π²Ρ‚ΠΎΡ€ΠΎΠΌΡƒ Π·Π°ΠΊΠΎΠ½Ρƒ ΠšΠΈΡ€Ρ…Π³ΠΎΡ„Π°.

ΠœΠ΅Ρ‚ΠΎΠ΄ Π΄Π²ΡƒΡ… ΡƒΠ·Π»ΠΎΠ² являСтся частным Π²Π°Ρ€ΠΈΠ°Π½Ρ‚ΠΎΠΌ ΠΌΠ΅Ρ‚ΠΎΠ΄ ΡƒΠ·Π»ΠΎΠ²Ρ‹Ρ… ΠΏΠΎΡ‚Π΅Π½Ρ†ΠΈΠ°Π»ΠΎΠ². Он примСняСтся Π² Ρ‚Π΅Ρ… случаях, ΠΊΠΎΠ³Π΄Π° анализируСмая схСма содСрТит Ρ‚ΠΎΠ»ΡŒΠΊΠΎ Π΄Π²Π° ΡƒΠ·Π»Π° (для опрСдСлСнности ΡƒΠ·Π»Ρ‹ ΠΈ ) ΠΈ большоС число ΠΏΠ°Ρ€Π°Π»Π»Π΅Π»ΡŒΠ½Ρ‹Ρ… Π²Π΅Ρ‚Π²Π΅ΠΉ, содСрТащих ΠΈ Π½Π΅ содСрТащих источники Π­Π”Π‘. Богласно ΠΌΠ΅Ρ‚ΠΎΠ΄Ρƒ Π΄Π²ΡƒΡ… ΡƒΠ·Π»ΠΎΠ² ΠΌΠ΅ΠΆΡƒΠ·Π»ΠΎΠ²ΠΎΠ΅ напряТСниС

Π³Π΄Π΅ β€” алгСбраичСская сумма ΠΏΡ€ΠΎΠΈΠ·Π²Π΅Π΄Π΅Π½ΠΈΠΉ Π­Π”Π‘ Π²Π΅Ρ‚Π²Π΅ΠΉ (Π­Π”Π‘ ΡΡ‡ΠΈΡ‚Π°ΡŽΡ‚ΡΡ ΠΏΠΎΠ»ΠΎΠΆΠΈΡ‚Π΅Π»ΡŒΠ½Ρ‹ΠΌΠΈ, Ссли ΠΎΠ½ΠΈ Π½Π°ΠΏΡ€Π°Π²Π»Π΅Π½Ρ‹ ΠΊ ΡƒΠ·Π»Ρƒ , ΠΈ ΠΎΡ‚Ρ€ΠΈΡ†Π°Ρ‚Π΅Π»ΡŒΠ½Ρ‹ΠΌΠΈ, Ссли ΠΎΡ‚ ΡƒΠ·Π»Π° ΠΊ ΡƒΠ·Π»Ρƒ ) Π½Π° проводимости этих Π²Π΅Ρ‚Π²Π΅ΠΉ; β€” сумма проводимости всСх Π²Π΅Ρ‚Π²Π΅ΠΉ, ΡΠΎΠ΅Π΄ΠΈΠ½ΡΡŽΡ‰ΠΈΡ… ΡƒΠ·Π»Ρ‹ ΠΈ .

Π­Ρ‚Π° тСория взята со страницы ΠΏΠΎΠΌΠΎΡ‰ΠΈ с заданиями ΠΏΠΎ элСктротСхникС:

ΠŸΠΎΠΌΠΎΡ‰ΡŒ ΠΏΠΎ элСктротСхникС

Π’ΠΎΠ·ΠΌΠΎΠΆΠ½ΠΎ эти страницы Π²Π°ΠΌ Π±ΡƒΠ΄ΡƒΡ‚ ΠΏΠΎΠ»Π΅Π·Π½Ρ‹:

Π Π΅ΠΊΠΎΠΌΠ΅Π½Π΄Π°Ρ†ΠΈΠΈ ΠΏΠΎ Ρ€Π΅ΡˆΠ΅Π½ΠΈΡŽ Π½Π΅Ρ‚Ρ€Π°Π΄ΠΈΡ†ΠΈΠΎΠ½Π½Ρ‹Ρ… Π·Π°Π΄Π°Ρ‡ Π½Π° расчСт элСктричСских Ρ†Π΅ΠΏΠ΅ΠΉ постоянного Ρ‚ΠΎΠΊΠ°

Π’Π²Π΅Π΄Π΅Π½ΠΈΠ΅

РСшСниС Π·Π°Π΄Π°Ρ‡ — Π½Π΅ΠΎΡ‚ΡŠΠ΅ΠΌΠ»Π΅ΠΌΠ°Ρ Ρ‡Π°ΡΡ‚ΡŒ обучСния Ρ„ΠΈΠ·ΠΈΠΊΠ΅, ΠΏΠΎΡΠΊΠΎΠ»ΡŒΠΊΡƒ Π² процСссС Ρ€Π΅ΡˆΠ΅Π½ΠΈΡ Π·Π°Π΄Π°Ρ‡ происходит Ρ„ΠΎΡ€ΠΌΠΈΡ€ΠΎΠ²Π°Π½ΠΈΠ΅ ΠΈ ΠΎΠ±ΠΎΠ³Π°Ρ‰Π΅Π½ΠΈΠ΅ физичСских понятий, развиваСтся физичСскоС ΠΌΡ‹ΡˆΠ»Π΅Π½ΠΈΠ΅ учащихся ΠΈ ΡΠΎΠ²Π΅Ρ€ΡˆΠ΅Π½ΡΡ‚Π²ΡƒΠ΅Ρ‚ΡΡ ΠΈΡ… Π½Π°Π²Ρ‹ΠΊΠΈ примСнСния Π·Π½Π°Π½ΠΈΠΉ Π½Π° ΠΏΡ€Π°ΠΊΡ‚ΠΈΠΊΠ΅.

Π’ Ρ…ΠΎΠ΄Π΅ Ρ€Π΅ΡˆΠ΅Π½ΠΈΡ Π·Π°Π΄Π°Ρ‡ ΠΌΠΎΠ³ΡƒΡ‚ Π±Ρ‹Ρ‚ΡŒ поставлСны ΠΈ ΡƒΡΠΏΠ΅ΡˆΠ½ΠΎ Ρ€Π΅Π°Π»ΠΈΠ·ΠΎΠ²Π°Π½Ρ‹ ΡΠ»Π΅Π΄ΡƒΡŽΡ‰ΠΈΠ΅ дидактичСскиС Ρ†Π΅Π»ΠΈ:

  • Π’Ρ‹Π΄Π²ΠΈΠΆΠ΅Π½ΠΈΠ΅ ΠΏΡ€ΠΎΠ±Π»Π΅ΠΌΡ‹ ΠΈ созданиС ΠΏΡ€ΠΎΠ±Π»Π΅ΠΌΠ½ΠΎΠΉ ситуации;
  • ΠžΠ±ΠΎΠ±Ρ‰Π΅Π½ΠΈΠ΅ Π½ΠΎΠ²Ρ‹Ρ… свСдСний;
  • Π€ΠΎΡ€ΠΌΠΈΡ€ΠΎΠ²Π°Π½ΠΈΠ΅ практичСских ΡƒΠΌΠ΅Π½ΠΈΠΉ ΠΈ Π½Π°Π²Ρ‹ΠΊΠΎΠ²;
  • ΠŸΡ€ΠΎΠ²Π΅Ρ€ΠΊΠ° Π³Π»ΡƒΠ±ΠΈΠ½Ρ‹ ΠΈ прочности Π·Π½Π°Π½ΠΈΠΉ;
  • Π—Π°ΠΊΡ€Π΅ΠΏΠ»Π΅Π½ΠΈΠ΅, ΠΎΠ±ΠΎΠ±Ρ‰Π΅Π½ΠΈΠ΅ ΠΈ ΠΏΠΎΠ²Ρ‚ΠΎΡ€Π΅Π½ΠΈΠ΅ ΠΌΠ°Ρ‚Π΅Ρ€ΠΈΠ°Π»Π°;
  • РСализация ΠΏΡ€ΠΈΠ½Ρ†ΠΈΠΏΠ° ΠΏΠΎΠ»ΠΈΡ‚Π΅Ρ…Π½ΠΈΠ·ΠΌΠ°;
  • Π Π°Π·Π²ΠΈΡ‚ΠΈΠ΅ творчСских способностСй учащихся.

Наряду с этим ΠΏΡ€ΠΈ Ρ€Π΅ΡˆΠ΅Π½ΠΈΠΈ Π·Π°Π΄Π°Ρ‡ Ρƒ школьников Π²ΠΎΡΠΏΠΈΡ‚Ρ‹Π²Π°ΡŽΡ‚ΡΡ Ρ‚Ρ€ΡƒΠ΄ΠΎΠ»ΡŽΠ±ΠΈΠ΅, ΠΏΡ‹Ρ‚Π»ΠΈΠ²ΠΎΡΡ‚ΡŒ ΡƒΠΌΠ°, смСкалка, ΡΠ°ΠΌΠΎΡΡ‚ΠΎΡΡ‚Π΅Π»ΡŒΠ½ΠΎΡΡ‚ΡŒ Π² суТдСниях, интСрСс ΠΊ ΡƒΡ‡Π΅Π½ΠΈΡŽ, воля ΠΈ Ρ…Π°Ρ€Π°ΠΊΡ‚Π΅Ρ€, упорство Π² достиТСнии поставлСнной Ρ†Π΅Π»ΠΈ. Для Ρ€Π΅Π°Π»ΠΈΠ·Π°Ρ†ΠΈΠΈ пСрСчислСнных Ρ†Π΅Π»Π΅ΠΉ особСнно ΡƒΠ΄ΠΎΠ±Π½ΠΎ ΠΈΡΠΏΠΎΠ»ΡŒΠ·ΠΎΠ²Π°Ρ‚ΡŒ Π½Π΅Ρ‚Ρ€Π°Π΄ΠΈΡ†ΠΈΠΎΠ½Π½Ρ‹Π΅ Π·Π°Π΄Π°Ρ‡ΠΈ.

Β§1. Π—Π°Π΄Π°Ρ‡ΠΈ ΠΏΠΎ расчСту элСктричСских Ρ†Π΅ΠΏΠ΅ΠΉ постоянного Ρ‚ΠΎΠΊΠ°

По школьной ΠΏΡ€ΠΎΠ³Ρ€Π°ΠΌΠΌΠ΅ Π½Π° рассмотрСниС Π΄Π°Π½Π½ΠΎΠΉ Ρ‚Π΅ΠΌΡ‹ ΠΎΡ‡Π΅Π½ΡŒ ΠΌΠ°Π»ΠΎ отводится Π²Ρ€Π΅ΠΌΠ΅Π½ΠΈ, поэтому учащиСся Π±ΠΎΠ»Π΅Π΅ ΠΈΠ»ΠΈ ΠΌΠ΅Π½Π΅Π΅ ΡƒΡΠΏΠ΅ΡˆΠ½ΠΎ ΠΎΠ²Π»Π°Π΄Π΅Π²Π°ΡŽΡ‚ ΠΌΠ΅Ρ‚ΠΎΠ΄Π°ΠΌΠΈ Ρ€Π΅ΡˆΠ΅Π½ΠΈΡ Π·Π°Π΄Π°Ρ‡ Π΄Π°Π½Π½ΠΎΠ³ΠΎ Ρ‚ΠΈΠΏΠ°. Но часто Ρ‚Π°ΠΊΠΈΠ΅ Ρ‚ΠΈΠΏΡ‹ Π·Π°Π΄Π°Ρ‡ Π²ΡΡ‚Ρ€Π΅Ρ‡Π°ΡŽΡ‚ΡΡ ΠΎΠ»ΠΈΠΌΠΏΠΈΠ°Π΄Π½Ρ‹Ρ… заданиях, Π½ΠΎ Π±Π°Π·ΠΈΡ€ΡƒΡŽΡ‚ΡΡ ΠΎΠ½ΠΈ Π½Π° школьном курсС.

К Ρ‚Π°ΠΊΠΈΠΌ, нСстандартным Π·Π°Π΄Π°Ρ‡Π°ΠΌ ΠΏΠΎ расчСту элСктричСских Ρ†Π΅ΠΏΠ΅ΠΉ постоянного Ρ‚ΠΎΠΊΠ° ΠΌΠΎΠΆΠ½ΠΎ отнСсти Π·Π°Π΄Π°Ρ‡ΠΈ, схСмы ΠΊΠΎΡ‚ΠΎΡ€Ρ‹Ρ…:

1) содСрТат большоС число элСмСнтов – рСзисторов ΠΈΠ»ΠΈ кондСнсаторов;

2) симмСтричны;

3) состоят ΠΈΠ· слоТных ΡΠΌΠ΅ΡˆΠ°Π½Π½Ρ‹Ρ… соСдинСний элСмСнтов.

Π’ ΠΎΠ±Ρ‰Π΅ΠΌ случаС Π²ΡΡΠΊΡƒΡŽ Ρ†Π΅ΠΏΡŒ ΠΌΠΎΠΆΠ½ΠΎ Ρ€Π°ΡΡΡ‡ΠΈΡ‚Π°Ρ‚ΡŒ, ΠΈΡΠΏΠΎΠ»ΡŒΠ·ΡƒΡ Π·Π°ΠΊΠΎΠ½Ρ‹ ΠšΠΈΡ€Ρ…Π³ΠΎΡ„Π°. Однако эти Π·Π°ΠΊΠΎΠ½Ρ‹ Π½Π΅ входят Π² ΡˆΠΊΠΎΠ»ΡŒΠ½ΡƒΡŽ ΠΏΡ€ΠΎΠ³Ρ€Π°ΠΌΠΌΡƒ. К Ρ‚ΠΎΠΌΡƒ ΠΆΠ΅, ΠΏΡ€Π°Π²ΠΈΠ»ΡŒΠ½ΠΎ Ρ€Π΅ΡˆΠΈΡ‚ΡŒ систСму ΠΈΠ· большого числа ΡƒΡ€Π°Π²Π½Π΅Π½ΠΈΠΉ со ΠΌΠ½ΠΎΠ³ΠΈΠΌΠΈ нСизвСстными ΠΏΠΎΠ΄ силу Π½Π΅ ΠΌΠ½ΠΎΠ³ΠΈΠΌ учащимся ΠΈ этот ΠΏΡƒΡ‚ΡŒ Π½Π΅ являСтся Π»ΡƒΡ‡ΡˆΠΈΠΌ способом Ρ‚Ρ€Π°Ρ‚ΠΈΡ‚ΡŒ врСмя. ΠŸΠΎΡΡ‚ΠΎΠΌΡƒ Π½ΡƒΠΆΠ½ΠΎ ΡƒΠΌΠ΅Ρ‚ΡŒ ΠΏΠΎΠ»ΡŒΠ·ΠΎΠ²Π°Ρ‚ΡŒΡΡ ΠΌΠ΅Ρ‚ΠΎΠ΄Π°ΠΌΠΈ, ΠΏΠΎΠ·Π²ΠΎΠ»ΡΡŽΡ‰ΠΈΠΌΠΈ быстро Π½Π°ΠΉΡ‚ΠΈ сопротивлСния ΠΈ Смкости ΠΊΠΎΠ½Ρ‚ΡƒΡ€ΠΎΠ².

Β§2. ΠœΠ΅Ρ‚ΠΎΠ΄ эквивалСнтных схСм

ΠœΠ΅Ρ‚ΠΎΠ΄ эквивалСнтных схСм Π·Π°ΠΊΠ»ΡŽΡ‡Π°Π΅Ρ‚ΡΡ Π² Ρ‚ΠΎΠΌ, Ρ‡Ρ‚ΠΎ ΠΈΡΡ…ΠΎΠ΄Π½ΡƒΡŽ схСму Π½Π°Π΄ΠΎ ΠΏΡ€Π΅Π΄ΡΡ‚Π°Π²ΠΈΡ‚ΡŒ Π² Π²ΠΈΠ΄Π΅ ΠΏΠΎΡΠ»Π΅Π΄ΠΎΠ²Π°Ρ‚Π΅Π»ΡŒΠ½Ρ‹Ρ… участков, Π½Π° ΠΊΠ°ΠΆΠ΄ΠΎΠΌ ΠΈΠ· ΠΊΠΎΡ‚ΠΎΡ€Ρ‹Ρ… соСдинСниС элСмСнтов схСмы Π»ΠΈΠ±ΠΎ ΠΏΠΎΡΠ»Π΅Π΄ΠΎΠ²Π°Ρ‚Π΅Π»ΡŒΠ½ΠΎ, Π»ΠΈΠ±ΠΎ ΠΏΠ°Ρ€Π°Π»Π»Π΅Π»ΡŒΠ½ΠΎ. Для Ρ‚Π°ΠΊΠΎΠ³ΠΎ прСдставлСния схСму Π½Π΅ΠΎΠ±Ρ…ΠΎΠ΄ΠΈΠΌΠΎ ΡƒΠΏΡ€ΠΎΡΡ‚ΠΈΡ‚ΡŒ. Под ΡƒΠΏΡ€ΠΎΡ‰Π΅Π½ΠΈΠ΅ΠΌ схСмы Π±ΡƒΠ΄Π΅ΠΌ ΠΏΠΎΠ½ΠΈΠΌΠ°Ρ‚ΡŒ соСдинСниС ΠΈΠ»ΠΈ Ρ€Π°Π·ΡŠΠ΅Π΄ΠΈΠ½Π΅Π½ΠΈΠ΅ ΠΊΠ°ΠΊΠΈΡ…-Π»ΠΈΠ±ΠΎ ΡƒΠ·Π»ΠΎΠ² схСмы, ΡƒΠ΄Π°Π»Π΅Π½ΠΈΠ΅ ΠΈΠ»ΠΈ Π΄ΠΎΠ±Π°Π²Π»Π΅Π½ΠΈΠ΅ рСзисторов, кондСнсаторов, добиваясь Ρ‚ΠΎΠ³ΠΎ, Ρ‡Ρ‚ΠΎΠ±Ρ‹ новая схСма ΠΈΠ· ΠΏΠΎΡΠ»Π΅Π΄ΠΎΠ²Π°Ρ‚Π΅Π»ΡŒΠ½ΠΎ ΠΈ ΠΏΠ°Ρ€Π°Π»Π»Π΅Π»ΡŒΠ½ΠΎ соСдинСнных элСмСнтов Π±Ρ‹Π»Π° эквивалСнтна исходной.

ЭквивалСнтная схСма – это такая схСма, Ρ‡Ρ‚ΠΎ ΠΏΡ€ΠΈ ΠΏΠΎΠ΄Π°Ρ‡Π΅ ΠΎΠ΄ΠΈΠ½Π°ΠΊΠΎΠ²Ρ‹Ρ… напряТСний Π½Π° ΠΈΡΡ…ΠΎΠ΄Π½ΡƒΡŽ ΠΈ ΠΏΡ€Π΅ΠΎΠ±Ρ€Π°Π·ΠΎΠ²Π°Π½Π½ΡƒΡŽ схСмы, Ρ‚ΠΎΠΊ Π² ΠΎΠ±Π΅ΠΈΡ… цСпях Π±ΡƒΠ΄Π΅Ρ‚ ΠΎΠ΄ΠΈΠ½Π°ΠΊΠΎΠ² Π½Π° ΡΠΎΠΎΡ‚Π²Π΅Ρ‚ΡΡ‚Π²ΡƒΡŽΡ‰ΠΈΡ… участках. Π’ этом случаС всС расчСты производятся с ΠΏΡ€Π΅ΠΎΠ±Ρ€Π°Π·ΠΎΠ²Π°Π½Π½ΠΎΠΉ схСмой.

Π§Ρ‚ΠΎΠ±Ρ‹ Π½Π°Ρ‡Π΅Ρ€Ρ‚ΠΈΡ‚ΡŒ ΡΠΊΠ²ΠΈΠ²Π°Π»Π΅Π½Ρ‚Π½ΡƒΡŽ схСму для Ρ†Π΅ΠΏΠΈ со слоТным ΡΠΌΠ΅ΡˆΠ°Π½Π½Ρ‹ΠΌ соСдинСниСм рСзисторов ΠΌΠΎΠΆΠ½ΠΎ Π²ΠΎΡΠΏΠΎΠ»ΡŒΠ·ΠΎΠ²Π°Ρ‚ΡŒΡΡ нСсколькими ΠΏΡ€ΠΈΠ΅ΠΌΠ°ΠΌΠΈ. ΠœΡ‹ ограничимся рассмотрСниСм Π² подробностях лишь ΠΎΠ΄Π½ΠΎΠ³ΠΎ ΠΈΠ· Π½ΠΈΡ… – способа ΡΠΊΠ²ΠΈΠΏΠΎΡ‚Π΅Π½Ρ†ΠΈΠ°Π»ΡŒΠ½Ρ‹Ρ… ΡƒΠ·Π»ΠΎΠ².

Π­Ρ‚ΠΎΡ‚ способ Π·Π°ΠΊΠ»ΡŽΡ‡Π°Π΅Ρ‚ΡΡ Π² Ρ‚ΠΎΠΌ, Ρ‡Ρ‚ΠΎ Π² симмСтричных схСмах ΠΎΡ‚Ρ‹ΡΠΊΠΈΠ²Π°ΡŽΡ‚ΡΡ Ρ‚ΠΎΡ‡ΠΊΠΈ с Ρ€Π°Π²Π½Ρ‹ΠΌΠΈ ΠΏΠΎΡ‚Π΅Π½Ρ†ΠΈΠ°Π»Π°ΠΌΠΈ. Π­Ρ‚ΠΈ ΡƒΠ·Π»Ρ‹ ΡΠΎΠ΅Π΄ΠΈΠ½ΡΡŽΡ‚ΡΡ ΠΌΠ΅ΠΆΠ΄Ρƒ собой, ΠΏΡ€ΠΈΡ‡Π΅ΠΌ, Ссли ΠΌΠ΅ΠΆΠ΄Ρƒ этими Ρ‚ΠΎΡ‡ΠΊΠ°ΠΌΠΈ Π±Ρ‹Π» Π²ΠΊΠ»ΡŽΡ‡Π΅Π½ ΠΊΠ°ΠΊΠΎΠΉ-Ρ‚ΠΎ участок схСмы, Ρ‚ΠΎ Π΅Π³ΠΎ ΠΎΡ‚Π±Ρ€Π°ΡΡ‹Π²Π°ΡŽΡ‚, Ρ‚Π°ΠΊ ΠΊΠ°ΠΊ ΠΈΠ·-Π·Π° равСнства ΠΏΠΎΡ‚Π΅Π½Ρ†ΠΈΠ°Π»ΠΎΠ² Π½Π° ΠΊΠΎΠ½Ρ†Π°Ρ… Ρ‚ΠΎΠΊ ΠΏΠΎ Π½Π΅ΠΌΡƒ Π½Π΅ Ρ‚Π΅Ρ‡Π΅Ρ‚ ΠΈ этот участок Π½ΠΈΠΊΠ°ΠΊ Π½Π΅ влияСт Π½Π° ΠΎΠ±Ρ‰Π΅Π΅ сопротивлСниС схСмы.

Π’Π°ΠΊΠΈΠΌ ΠΎΠ±Ρ€Π°Π·ΠΎΠΌ, Π·Π°ΠΌΠ΅Π½Π° Π½Π΅ΡΠΊΠΎΠ»ΡŒΠΊΠΈΡ… ΡƒΠ·Π»ΠΎΠ² Ρ€Π°Π²Π½Ρ‹Ρ… ΠΏΠΎΡ‚Π΅Π½Ρ†ΠΈΠ°Π»ΠΎΠ² ΠΏΡ€ΠΈΠ²ΠΎΠ΄ΠΈΡ‚ ΠΊ Π±ΠΎΠ»Π΅Π΅ простой эквивалСнтной схСмС. Но ΠΈΠ½ΠΎΠ³Π΄Π° Π±Ρ‹Π²Π°Π΅Ρ‚ цСлСсообразнСС обратная Π·Π°ΠΌΠ΅Π½Π° ΠΎΠ΄Π½ΠΎΠ³ΠΎ ΡƒΠ·Π»Π°

нСсколькими ΡƒΠ·Π»Π°ΠΌΠΈ с Ρ€Π°Π²Π½Ρ‹ΠΌΠΈ ΠΏΠΎΡ‚Π΅Π½Ρ†ΠΈΠ°Π»Π°ΠΌΠΈ, Ρ‡Ρ‚ΠΎ Π½Π΅ Π½Π°Ρ€ΡƒΡˆΠ°Π΅Ρ‚ элСктричСских условий Π² ΠΎΡΡ‚Π°Π»ΡŒΠ½ΠΎΠΉ части.

Рассмотрим ΠΏΡ€ΠΈΠΌΠ΅Ρ€Ρ‹ Ρ€Π΅ΡˆΠ΅Π½ΠΈΡ Π·Π°Π΄Π°Ρ‡ эти ΠΌΠ΅Ρ‚ΠΎΠ΄ΠΎΠΌ.

Π— Π° Π΄ Π° Ρ‡ Π° β„–1

Π Π°ΡΡΡ‡ΠΈΡ‚Π°Ρ‚ΡŒ сопротивлСниС ΠΌΠ΅ΠΆΠ΄Ρƒ Ρ‚ΠΎΡ‡ΠΊΠ°ΠΌΠΈ А ΠΈ Π’ Π΄Π°Π½Π½ΠΎΠ³ΠΎ участка Ρ†Π΅ΠΏΠΈ. ВсС рСзисторы ΠΎΠ΄ΠΈΠ½Π°ΠΊΠΎΠ²Ρ‹ ΠΈ ΠΈΡ… сопротивлСния Ρ€Π°Π²Π½Ρ‹ r.

РСшСниС:

Π’ силу симмСтричности Π²Π΅Ρ‚Π²Π΅ΠΉ Ρ†Π΅ΠΏΠΈ Ρ‚ΠΎΡ‡ΠΊΠΈ Π‘ И Π” ΡΠ²Π»ΡΡŽΡ‚ΡΡ ΡΠΊΠ²ΠΈΠΏΠΎΡ‚Π΅Π½Ρ†ΠΈΠ°Π»ΡŒΠ½Ρ‹ΠΌΠΈ. ΠŸΠΎΡΡ‚ΠΎΠΌΡƒ рСзистор ΠΌΠ΅ΠΆΠ΄Ρƒ Π½ΠΈΠΌΠΈ ΠΌΡ‹ ΠΌΠΎΠΆΠ΅ΠΌ ΠΈΡΠΊΠ»ΡŽΡ‡ΠΈΡ‚ΡŒ. Π­ΠΊΠ²ΠΈΠΏΠΎΡ‚Π΅Π½Ρ†ΠΈΠ°Π»ΡŒΠ½Ρ‹Π΅ Ρ‚ΠΎΡ‡ΠΊΠΈ Π‘ ΠΈ Π” соСдиняСм Π² ΠΎΠ΄ΠΈΠ½ ΡƒΠ·Π΅Π». ΠŸΠΎΠ»ΡƒΡ‡Π°Π΅ΠΌ ΠΎΡ‡Π΅Π½ΡŒ ΠΏΡ€ΠΎΡΡ‚ΡƒΡŽ ΡΠΊΠ²ΠΈΠ²Π°Π»Π΅Π½Ρ‚Π½ΡƒΡŽ схСму:

Π‘ΠΎΠΏΡ€ΠΎΡ‚ΠΈΠ²Π»Π΅Π½ΠΈΠ΅ ΠΊΠΎΡ‚ΠΎΡ€ΠΎΠΉ Ρ€Π°Π²Π½ΠΎ:

RАВ=Rac+Rcd=r*r/r*r+r*r/r+r=r.

Π— Π° Π΄ Π° Ρ‡ Π° β„– 2

РСшСниС:

Π’ Ρ‚ΠΎΡ‡ΠΊΠ°Ρ… F ΠΈ F` ΠΏΠΎΡ‚Π΅Π½Ρ†ΠΈΠ°Π»Ρ‹ Ρ€Π°Π²Π½Ρ‹, Π·Π½Π°Ρ‡ΠΈΡ‚ сопротивлСниС ΠΌΠ΅ΠΆΠ΄Ρƒ Π½ΠΈΠΌΠΈ ΠΌΠΎΠΆΠ½ΠΎ ΠΎΡ‚Π±Ρ€ΠΎΡΠΈΡ‚ΡŒ. ЭквивалСнтная схСма выглядит Ρ‚Π°ΠΊ:

БопротивлСния участков DNB;F`C`D`; D`, N`, B`; FCD Ρ€Π°Π²Π½Ρ‹ ΠΌΠ΅ΠΆΠ΄Ρƒ собой ΠΈ Ρ€Π°Π²Π½Ρ‹ R1:

1/R1=1/2r+1/r=3/2r

R1=2/3*r

Π‘ ΡƒΡ‡Π΅Ρ‚ΠΎΠΌ этого получаСтся новая эквивалСнтная схСма:

Π•Π΅ сопротивлСниС ΠΈ сопротивлСниС исходной Ρ†Π΅ΠΏΠΈ RАВ Ρ€Π°Π²Π½ΠΎ:

1/RАВ=1/r+R1+R1+1/r+R1+R1=6/7r

RАВ=(7/6)*r.

Π— Π° Π΄ Π° Ρ‡ Π° β„– 3.

РСшСниС:

Π’ΠΎΡ‡ΠΊΠΈ Π‘ ΠΈ Π” ΠΈΠΌΠ΅ΡŽΡ‚ Ρ€Π°Π²Π½Ρ‹Π΅ ΠΏΠΎΡ‚Π΅Π½Ρ†ΠΈΠ°Π»Ρ‹. Π˜ΡΠΊΠ»ΡŽΡ‡Π΅Π½ΠΈΠ΅ΠΌ сопротивлСниС ΠΌΠ΅ΠΆΠ΄Ρƒ Π½ΠΈΠΌΠΈ. ΠŸΠΎΠ»ΡƒΡ‡Π°Π΅ΠΌ ΡΠΊΠ²ΠΈΠ²Π°Π»Π΅Π½Ρ‚Π½ΡƒΡŽ схСму:

ИскомоС сопротивлСниС RАВ Ρ€Π°Π²Π½ΠΎ:

1/RАВ=1/2r+1/2r+1/r=2/r

RАВ=r/2.

Π— Π° Π΄ Π° Ρ‡ Π° β„– 4.

РСшСниС:

Как Π²ΠΈΠ΄Π½ΠΎ ΠΈΠ· схСмы ΡƒΠ·Π»Ρ‹ 1,2,3 ΠΈΠΌΠ΅ΡŽΡ‚ Ρ€Π°Π²Π½Ρ‹Π΅ ΠΏΠΎΡ‚Π΅Π½Ρ†ΠΈΠ°Π»Ρ‹. Π‘ΠΎΠ΅Π΄ΠΈΠ½ΠΈΠΌ ΠΈΡ… Π² ΡƒΠ·Π΅Π» 1. Π£Π·Π»Ρ‹ 4,5,6 ΠΈΠΌΠ΅ΡŽΡ‚ Ρ‚ΠΎΠΆΠ΅ Ρ€Π°Π²Π½Ρ‹Π΅ ΠΏΠΎΡ‚Π΅Π½Ρ†ΠΈΠ°Π»Ρ‹- соСдиним ΠΈΡ… Π² ΡƒΠ·Π΅Π» 2. ΠŸΠΎΠ»ΡƒΡ‡ΠΈΠΌ Ρ‚Π°ΠΊΡƒΡŽ ΡΠΊΠ²ΠΈΠ²Π°Π»Π΅Π½Ρ‚Π½ΡƒΡŽ схСму:

Π‘ΠΎΠΏΡ€ΠΎΡ‚ΠΈΠ²Π»Π΅Π½ΠΈΠ΅ Π½Π° участкС А-1, R 1-Ρ€Π°Π²Π½ΠΎ ΡΠΎΠΏΡ€ΠΎΡ‚ΠΈΠ²Π»Π΅Π½ΠΈΡŽ Π½Π° участкС 2-Π’,R3 ΠΈ Ρ€Π°Π²Π½ΠΎ:

R1=R3=r/3

Π‘ΠΎΠΏΡ€ΠΎΡ‚ΠΈΠ²Π»Π΅Π½ΠΈΠ΅ Π½Π° участкС 1-2 Ρ€Π°Π²Π½ΠΎ: R2=r/6.

Π’Π΅ΠΏΠ΅Ρ€ΡŒ получаСтся эквивалСнтная схСма:

ΠžΠ±Ρ‰Π΅Π΅ сопротивлСниС RАВ Ρ€Π°Π²Π½ΠΎ:

RАВ= R1+ R2+ R3=(5/6)*r.

Π— Π° Π΄ Π° Ρ‡ Π° β„– 5.

РСшСниС:

Π’ΠΎΡ‡ΠΊΠΈ C ΠΈ F-эквивалСнтныС. Π‘ΠΎΠ΅Π΄ΠΈΠ½ΠΈΠΌ ΠΈΡ… Π² ΠΎΠ΄ΠΈΠ½ ΡƒΠ·Π΅Π». Π’ΠΎΠ³Π΄Π° эквивалСнтная схСма Π±ΡƒΠ΄Π΅Ρ‚ ΠΈΠΌΠ΅Ρ‚ΡŒ ΡΠ»Π΅Π΄ΡƒΡŽΡ‰ΠΈΠΉ Π²ΠΈΠ΄:

Π‘ΠΎΠΏΡ€ΠΎΡ‚ΠΈΠ²Π»Π΅Π½ΠΈΠ΅ Π½Π° участкС АБ:

Rас=r/2

Π‘ΠΎΠΏΡ€ΠΎΡ‚ΠΈΠ²Π»Π΅Π½ΠΈΠ΅ Π½Π° участкС FN:

RFN =

Π‘ΠΎΠΏΡ€ΠΎΡ‚ΠΈΠ²Π»Π΅Π½ΠΈΠ΅ Π½Π° участкС DB:

RDB =r/2

ΠŸΠΎΠ»ΡƒΡ‡Π°Π΅Ρ‚ΡΡ эквивалСнтная схСма:

ИскомоС ΠΎΠ±Ρ‰Π΅Π΅ сопротивлСниС Ρ€Π°Π²Π½ΠΎ:

RAB= r.

Π—Π°Π΄Π°Ρ‡Π° β„–6

РСшСниС:

Π—Π°ΠΌΠ΅Π½ΠΈΠΌ ΠΎΠ±Ρ‰ΠΈΠΉ ΡƒΠ·Π΅Π» О трСмя ΡƒΠ·Π»Π°ΠΌΠΈ с Ρ€Π°Π²Π½Ρ‹ΠΌΠΈ ΠΏΠΎΡ‚Π΅Π½Ρ†ΠΈΠ°Π»Π°ΠΌΠΈ О, О1 , О2. ΠŸΠΎΠ»ΡƒΡ‡ΠΈΠΌ ΡΠΊΠ²ΠΈΠ²Π°Π»Π΅Π½Ρ‚Π½ΡƒΡŽ систСму:

Π‘ΠΎΠΏΡ€ΠΎΡ‚ΠΈΠ²Π»Π΅Π½ΠΈΠ΅ Π½Π° участкС ABCD:

R1=(3/2)*r

Π‘ΠΎΠΏΡ€ΠΎΡ‚ΠΈΠ²Π»Π΅Π½ΠΈΠ΅ Π½Π° участкС A`B`C`D`:

R2= (8/3)*r

Π‘ΠΎΠΏΡ€ΠΎΡ‚ΠΈΠ²Π»Π΅Π½ΠΈΠ΅ Π½Π° участкС ACΠ’

R3 = 2r.

ΠŸΠΎΠ»ΡƒΡ‡Π°Π΅ΠΌ ΡΠΊΠ²ΠΈΠ²Π°Π»Π΅Π½Ρ‚Π½ΡƒΡŽ схСму:

ИскомоС ΠΎΠ±Ρ‰Π΅Π΅ сопротивлСниС Ρ†Π΅ΠΏΠΈ RAB Ρ€Π°Π²Π½ΠΎ:

RAB= (8/10)*r.

Π—Π°Π΄Π°Ρ‡Π° β„–7.

РСшСниС:

β€œΠ Π°Π·Π΄Π΅Π»ΠΈΠΌβ€ ΡƒΠ·Π΅Π» О Π½Π° Π΄Π²Π° ΡΠΊΠ²ΠΈΠΏΠΎΡ‚Π΅Π½Ρ†ΠΈΠ°Π»ΡŒΠ½Ρ‹Ρ… ΡƒΠ³Π»Π° О1 ΠΈ О2. Π’Π΅ΠΏΠ΅Ρ€ΡŒ схСму ΠΌΠΎΠΆΠ½ΠΎ ΠΏΡ€Π΅Π΄ΡΡ‚Π°Π²ΠΈΡ‚ΡŒ, ΠΊΠ°ΠΊ ΠΏΠ°Ρ€Π°Π»Π»Π΅Π»ΡŒΠ½Ρ‹Π΅ соСдинСниС Π΄Π²ΡƒΡ… ΠΎΠ΄ΠΈΠ½Π°ΠΊΠΎΠ²Ρ‹Ρ… Ρ†Π΅ΠΏΠ΅ΠΉ. ΠŸΠΎΡΡ‚ΠΎΠΌΡƒ достаточно ΠΏΠΎΠ΄Ρ€ΠΎΠ±Π½ΠΎ Ρ€Π°ΡΡΠΌΠΎΡ‚Ρ€Π΅Ρ‚ΡŒ ΠΎΠ΄Π½Ρƒ ΠΈΠ· Π½ΠΈΡ…:

Π‘ΠΎΠΏΡ€ΠΎΡ‚ΠΈΠ²Π»Π΅Π½ΠΈΠ΅ этой схСмы R1 Ρ€Π°Π²Π½ΠΎ:

R1 = 3r

Π’ΠΎΠ³Π΄Π° сопротивлСниС всСй Ρ†Π΅ΠΏΠΈ Π±ΡƒΠ΄Π΅Ρ‚ Ρ€Π°Π²Π½ΠΎ:

RAB = (3/2)*r

Π— Π° Π΄ Π° Ρ‡ Π° β„–8

РСшСниС:

Π£Π·Π»Ρ‹ 1 ΠΈ 2 – ΡΠΊΠ²ΠΈΠΏΠΎΡ‚Π΅Π½Ρ†ΠΈΠ°Π»ΡŒΠ½Ρ‹Π΅, поэтому соСдиним ΠΈΡ… Π² ΠΎΠ΄ΠΈΠ½ ΡƒΠ·Π΅Π» I. Π£Π·Π»Ρ‹ 3 ΠΈ 4 Ρ‚Π°ΠΊΠΆΠ΅ ΡΠΊΠ²ΠΈΠΏΠΎΡ‚Π΅Π½Ρ†ΠΈΠ°Π»ΡŒΠ½Ρ‹Π΅ – соСдинимих Π² Π΄Ρ€ΡƒΠ³ΠΎΠΉ ΡƒΠ·Π΅Π» II. ЭквивалСнтная схСма ΠΈΠΌΠ΅Π΅Ρ‚ Π²ΠΈΠ΄:

Π‘ΠΎΠΏΡ€ΠΎΡ‚ΠΈΠ²Π»Π΅Π½ΠΈΠ΅ Π½Π° участкС A- I Ρ€Π°Π²Π½ΠΎ ΡΠΎΠΏΡ€ΠΎΡ‚ΠΈΠ²Π»Π΅Π½ΠΈΡŽ Π½Π° участкС B- II ΠΈ Ρ€Π°Π²Π½ΠΎ:

RI =

Π‘ΠΎΠΏΡ€ΠΎΡ‚ΠΈΠ²Π»Π΅Π½ΠΈΠ΅ участка I-5-6- II Ρ€Π°Π²Π½ΠΎ:

RII = 2r

CΠΎΠΏΡ€ΠΎΡ‚ΠΈΠ²Π»Π΅Π½ΠΈΠ΅ участка I- II Ρ€Π°Π²Π½ΠΎ:

RIII =

ΠŸΠΎΠ»ΡƒΡ‡Π°Π΅ΠΌ ΠΎΠΊΠΎΠ½Ρ‡Π°Ρ‚Π΅Π»ΡŒΠ½ΡƒΡŽ ΡΠΊΠ²ΠΈΠ²Π°Π»Π΅Π½Ρ‚Π½ΡƒΡŽ схСму:

ИскомоС ΠΎΠ±Ρ‰Π΅Π΅ сопротивлСниС Ρ†Π΅ΠΏΠΈ RAB=(7/12)*r.

Π— Π° Π΄ Π° Ρ‡ Π° β„–9

Π’ Π²Π΅Ρ‚Π²ΠΈ ОБ Π·Π°ΠΌΠ΅Π½ΠΈΠΌ сопротивлСниС Π½Π° Π΄Π²Π° ΠΏΠ°Ρ€Π°Π»Π»Π΅Π»ΡŒΠ½ΠΎ соСдинСнных сопротивлСния ΠΏΠΎ 2r. Π’Π΅ΠΏΠ΅Ρ€ΡŒ ΡƒΠ·Π΅Π» Π‘ ΠΌΠΎΠΆΠ½ΠΎ Ρ€Π°Π·Π΄Π΅Π»ΠΈΡ‚ΡŒ Π½Π° 2 ΡΠΊΠ²ΠΈΠΏΠΎΡ‚Π΅Π½Ρ†ΠΈΠ°Π»ΡŒΠ½Ρ‹Ρ… ΡƒΠ·Π»Π° Π‘1 ΠΈ Π‘2. ЭквивалСнтная схСма Π² этом случаС выглядит Ρ‚Π°ΠΊ:

Π‘ΠΎΠΏΡ€ΠΎΡ‚ΠΈΠ²Π»Π΅Π½ΠΈΠ΅ Π½Π° участках ОБIB ΠΈ DCIIB ΠΎΠ΄ΠΈΠ½Π°ΠΊΠΎΠ²Ρ‹ ΠΈ Ρ€Π°Π²Π½Ρ‹, ΠΊΠ°ΠΊ Π»Π΅Π³ΠΊΠΎ ΠΏΠΎΠ΄ΡΡ‡ΠΈΡ‚Π°Ρ‚ΡŒ 2r. ΠžΠΏΡΡ‚ΡŒ Ρ‡Π΅Ρ€Ρ‚ΠΈΠΌ ΡΠΎΠΎΡ‚Π²Π΅Ρ‚ΡΡ‚Π²ΡƒΡŽΡ‰ΡƒΡŽ ΡΠΊΠ²ΠΈΠ²Π°Π»Π΅Π½Ρ‚Π½ΡƒΡŽ схСму:

Π‘ΠΎΠΏΡ€ΠΎΡ‚ΠΈΠ²Π»Π΅Π½ΠΈΠ΅ Π½Π° участкС AOB Ρ€Π°Π²Π½ΠΎ ΡΠΎΠΏΡ€ΠΎΡ‚ΠΈΠ²Π»Π΅Π½ΠΈΡŽ Π½Π° участкС ADB ΠΈ Ρ€Π°Π²Π½ΠΎ (7/4)*r. Π’Π°ΠΊΠΈΠΌ ΠΎΠ±Ρ€Π°Π·ΠΎΠΌ ΠΏΠΎΠ»ΡƒΡ‡Π°Π΅ΠΌ ΠΎΠΊΠΎΠ½Ρ‡Π°Ρ‚Π΅Π»ΡŒΠ½ΡƒΡŽ ΡΠΊΠ²ΠΈΠ²Π°Π»Π΅Π½Ρ‚Π½ΡƒΡŽ схСму ΠΈΠ· Ρ‚Ρ€Π΅Ρ… ΠΏΠ°Ρ€Π°Π»Π»Π΅Π»ΡŒΠ½ΠΎ соСдинСнных сопротивлСний:

Π•Π΅ ΠΎΠ±Ρ‰Π΅Π΅ сопротивлСниС Ρ€Π°Π²Π½ΠΎ RAB= (7/15)*r

Π— Π° Π΄ Π° Ρ‡ Π° β„– 10

Π’ΠΎΡ‡ΠΊΠΈ БОD ΠΈΠΌΠ΅ΡŽΡ‚ Ρ€Π°Π²Π½Ρ‹Π΅ ΠΏΠΎΡ‚Π΅Π½Ρ†ΠΈΠ°Π»Ρ‹ – соСдиним ΠΈΡ… Π² ΠΎΠ΄ΠΈΠ½ ΡƒΠ·Π΅Π» ОI .ЭквивалСнтная схСма ΠΈΠ·ΠΎΠ±Ρ€Π°ΠΆΠ΅Π½Π° Π½Π° рисункС :

Π‘ΠΎΠΏΡ€ΠΎΡ‚ΠΈΠ²Π»Π΅Π½ΠΈΠ΅ Π½Π° участкС А ОI Ρ€Π°Π²Π½ΠΎ . На участкС ОIΠ’ сопротивлСниС Ρ€Π°Π²Π½ΠΎ .ΠŸΠΎΠ»ΡƒΡ‡Π°Π΅ΠΌ совсСм ΠΏΡ€ΠΎΡΡ‚ΡƒΡŽ ΡΠΊΠ²ΠΈΠ²Π°Π»Π΅Π½Ρ‚Π½ΡƒΡŽ схСму:

Π•Π• сопротивлСниС Ρ€Π°Π²Π½ΠΎ искомому ΠΎΠ±Ρ‰Π΅ΠΌΡƒ ΡΠΎΠΏΡ€ΠΎΡ‚ΠΈΠ²Π»Π΅Π½ΠΈΡŽ

RAB=(5/6)*r

Π—Π°Π΄Π°Ρ‡ΠΈ β„– 11 ΠΈ β„– 12 Ρ€Π΅ΡˆΠ°ΡŽΡ‚ΡΡ нСсколько ΠΈΠ½Ρ‹ΠΌ способом, Ρ‡Π΅ΠΌ ΠΏΡ€Π΅Π΄Ρ‹Π΄ΡƒΡ‰ΠΈΠ΅. Π’ Π·Π°Π΄Π°Ρ‡Π΅ β„–11 для Π΅Π΅ Ρ€Π΅ΡˆΠ΅Π½ΠΈΡ ΠΈΡΠΏΠΎΠ»ΡŒΠ·ΡƒΠ΅Ρ‚ΡΡ особоС свойство бСсконСчных Ρ†Π΅ΠΏΠ΅ΠΉ, Π° Π² Π·Π°Π΄Π°Ρ‡Π΅ β„– 12 примСняСтся способ упрощСния Ρ†Π΅ΠΏΠΈ.

Π—Π°Π΄Π°Ρ‡Π° β„– 11

РСшСниС

Π’Ρ‹Π΄Π΅Π»ΠΈΠΌ Π² этой Ρ†Π΅ΠΏΠΈ бСсконСчно ΠΏΠΎΠ²Ρ‚ΠΎΡ€ΡΡŽΡ‰Π΅Π΅ΡΡ Π·Π²Π΅Π½ΠΎ, ΠΎΠ½ΠΎ состоит Π² Π΄Π°Π½Π½ΠΎΠΌ случаС ΠΈΠ· Ρ‚Ρ€Π΅Ρ… ΠΏΠ΅Ρ€Π²Ρ‹Ρ… сопротивлСний. Если ΠΌΡ‹ отбросим это Π·Π²Π΅Π½ΠΎ, Ρ‚ΠΎ ΠΏΠΎΠ»Π½ΠΎΠ΅ сопротивлСниС бСсконСчной Ρ†Π΅ΠΏΠΈ R Π½Π΅ ΠΈΠ·ΠΌΠ΅Π½ΠΈΡ‚ΡŒΡΡ ΠΎΡ‚ этого , Ρ‚Π°ΠΊ ΠΊΠ°ΠΊ получится Ρ‚ΠΎΡ‡Π½ΠΎ такая ΠΆΠ΅ бСсконСчная Ρ†Π΅ΠΏΡŒ. Π’Π°ΠΊ ΠΆΠ΅ Π½ΠΈΡ‡Π΅Π³ΠΎ Π½Π΅ ΠΈΠ·ΠΌΠ΅Π½ΠΈΡ‚ΡŒΡΡ, Ссли ΠΌΡ‹ Π²Ρ‹Π΄Π΅Π»Π΅Π½Π½ΠΎΠ΅ Π·Π²Π΅Π½ΠΎ ΠΏΠΎΠ΄ΠΊΠ»ΡŽΡ‡ΠΈΠΌ ΠΎΠ±Ρ€Π°Ρ‚Π½ΠΎ ΠΊ бСсконСчному ΡΠΎΠΏΡ€ΠΎΡ‚ΠΈΠ²Π»Π΅Π½ΠΈΡŽ R, Π½ΠΎ ΠΏΡ€ΠΈ этом слСдуСт ΠΎΠ±Ρ€Π°Ρ‚ΠΈΡ‚ΡŒ Π²Π½ΠΈΠΌΠ°Π½ΠΈΠ΅ , Ρ‡Ρ‚ΠΎ Ρ‡Π°ΡΡ‚ΡŒ Π·Π²Π΅Π½Π° ΠΈ бСсконСчная Ρ†Π΅ΠΏΡŒ сопротивлСниСм R соСдинСны ΠΏΠ°Ρ€Π°Π»Π»Π΅Π»ΡŒΠ½ΠΎ. Π’Π°ΠΊΠΈΠΌ ΠΎΠ±Ρ€Π°Π·ΠΎΠΌ ΠΏΠΎΠ»ΡƒΡ‡Π°Π΅ΠΌ ΡΠΊΠ²ΠΈΠ²Π°Π»Π΅Π½Ρ‚Π½ΡƒΡŽ схСму :

ΠŸΠΎΠ»ΡƒΡ‡Π°Π΅Ρ‚ΡΡ уравнСния

RAB=2Ρ‡ +

RAB = R

РСшая систСму этих ΡƒΡ€Π°Π²Π½Π΅Π½ΠΈΠΉ, ΠΏΠΎΠ»ΡƒΡ‡Π°Π΅ΠΌ:

R=Ρ‡ (1+ ).

Β§3. ΠžΠ±ΡƒΡ‡Π΅Π½ΠΈΠ΅ Ρ€Π΅ΡˆΠ΅Π½ΠΈΡŽ Π·Π°Π΄Π°Ρ‡ ΠΏΠΎ расчСту элСктричСских Ρ†Π΅ΠΏΠ΅ΠΉ способом ΡΠΊΠ²ΠΈΠΏΠΎΡ‚Π΅Π½Ρ†ΠΈΠ°Π»ΡŒΠ½Ρ‹Ρ… ΡƒΠ·Π»ΠΎΠ²

Π—Π°Π΄Π°Ρ‡Π° – это ΠΏΡ€ΠΎΠ±Π»Π΅ΠΌΠ°, для Ρ€Π°Π·Ρ€Π΅ΡˆΠ΅Π½ΠΈΡ ΠΊΠΎΡ‚ΠΎΡ€ΠΎΠΉ ΡƒΡ‡Π΅Π½ΠΈΠΊΡƒ ΠΏΠΎΡ‚Ρ€Π΅Π±ΡƒΡŽΡ‚ΡΡ логичСскиС рассуТдСния ΠΈ Π²Ρ‹Π²ΠΎΠ΄Ρ‹. БтроящиСся Π½Π° основС Π·Π°ΠΊΠΎΠ½ΠΎΠ² ΠΈ ΠΌΠ΅Ρ‚ΠΎΠ΄ΠΎΠ² Ρ„ΠΈΠ·ΠΈΠΊΠΈ. Π’Π°ΠΊΠΈΠΌ ΠΎΠ±Ρ€Π°Π·ΠΎΠΌ, с ΠΏΠΎΠΌΠΎΡ‰ΡŒΡŽ Π·Π°Π΄Π°Ρ‡ происходит активизация Ρ†Π΅Π»Π΅Π½Π°ΠΏΡ€Π°Π²Π»Π΅Π½Π½ΠΎΠ³ΠΎ ΠΌΡ‹ΡˆΠ»Π΅Π½ΠΈΡ учащихся.

Π’ Ρ‚ΠΎ ΠΆΠ΅ врСмя. ВСорСтичСскиС знания ΠΌΠΎΠΆΠ½ΠΎ ΡΡ‡ΠΈΡ‚Π°Ρ‚ΡŒ усвоСнными Ρ‚ΠΎΠ»ΡŒΠΊΠΎ Ρ‚ΠΎΠ³Π΄Π°, ΠΊΠΎΠ³Π΄Π° ΠΎΠ½ΠΈ ΡƒΠ΄Π°Ρ‡Π½ΠΎ ΠΏΡ€ΠΈΠΌΠ΅Π½ΡΡŽΡ‚ΡΡ Π½Π° ΠΏΡ€Π°ΠΊΡ‚ΠΈΠΊΠ΅. Π—Π°Π΄Π°Ρ‡ΠΈ ΠΏΠΎ Ρ„ΠΈΠ·ΠΈΠΊΠ΅ ΠΎΠΏΠΈΡΡ‹Π²Π°ΡŽΡ‚ часто Π²ΡΡ‚Ρ€Π΅Ρ‡Π°ΡŽΡ‰ΠΈΠ΅ΡΡ Π² ΠΆΠΈΠ·Π½ΠΈ ΠΈ Π½Π° производствС ΠΏΡ€ΠΎΠ±Π»Π΅ΠΌΡ‹, ΠΊΠΎΡ‚ΠΎΡ€Ρ‹Π΅ ΠΌΠΎΠ³ΡƒΡ‚ Π±Ρ‹Ρ‚ΡŒ Ρ€Π΅ΡˆΠ΅Π½Ρ‹ с ΠΏΠΎΠΌΠΎΡ‰ΡŒΡŽ Π·Π°ΠΊΠΎΠ½ΠΎΠ² Ρ„ΠΈΠ·ΠΈΠΊΠΈ ΠΈ, Ссли ΡƒΡ‡Π΅Π½ΠΈΠΊ ΡƒΡΠΏΠ΅ΡˆΠ½ΠΎ Ρ€Π΅ΡˆΠ°Π΅Ρ‚ Π·Π°Π΄Π°Ρ‡ΠΈ, Ρ‚ΠΎ ΠΌΠΎΠΆΠ½ΠΎ ΡΠΊΠ°Π·Π°Ρ‚ΡŒ, Ρ‡Ρ‚ΠΎ ΠΎΠ½ Ρ…ΠΎΡ€ΠΎΡˆΠΎ Π·Π½Π°Π΅Ρ‚ Ρ„ΠΈΠ·ΠΈΠΊΡƒ.

Для Ρ‚ΠΎΠ³ΠΎ, Ρ‡Ρ‚ΠΎΠ±Ρ‹ ΡƒΡ‡Π΅Π½ΠΈΠΊΠΈ ΡƒΡΠΏΠ΅ΡˆΠ½ΠΎ Ρ€Π΅ΡˆΠ°Π»ΠΈ Π·Π°Π΄Π°Ρ‡ΠΈ, нСдостаточно ΠΈΠΌΠ΅Ρ‚ΡŒ Π½Π°Π±ΠΎΡ€ ΠΌΠ΅Ρ‚ΠΎΠ΄ΠΎΠ² ΠΈ способов Ρ€Π΅ΡˆΠ΅Π½ΠΈΡ Π·Π°Π΄Π°Ρ‡, Π½Π΅ΠΎΠ±Ρ…ΠΎΠ΄ΠΈΠΌΠΎ Π΅Ρ‰Π΅ ΡΠΏΠ΅Ρ†ΠΈΠ°Π»ΡŒΠ½ΠΎ ΡƒΡ‡ΠΈΡ‚ΡŒ школьников ΠΏΡ€ΠΈΠΌΠ΅Π½Π΅Π½ΠΈΡŽ этих способов.

Рассмотрим ΠΏΠ»Π°Π½ Ρ€Π΅ΡˆΠ΅Π½ΠΈΡ Π·Π°Π΄Π°Ρ‡ ΠΏΠΎ расчСту элСктричСских Ρ†Π΅ΠΏΠ΅ΠΉ постоянного Ρ‚ΠΎΠΊΠ° ΠΌΠ΅Ρ‚ΠΎΠ΄ΠΎΠΌ ΡΠΊΠ²ΠΈΠΏΠΎΡ‚Π΅Π½Ρ†ΠΈΠ°Π»ΡŒΠ½Ρ‹Ρ… ΡƒΠ·Π»ΠΎΠ².

  1. Π§Ρ‚Π΅Π½ΠΈΠ΅ условия.
  2. ΠšΡ€Π°Ρ‚ΠΊΠ°Ρ запись условия.
  3. ΠŸΠ΅Ρ€Π΅Π²ΠΎΠ΄ Π² Π΅Π΄ΠΈΠ½ΠΈΡ†Ρ‹ БИ.
  4. Анализ схСмы:
    1. ΡƒΡΡ‚Π°Π½ΠΎΠ²ΠΈΡ‚ΡŒ, являСтся Π»ΠΈ схСма симмСтричной;
    2. ΡƒΡΡ‚Π°Π½ΠΎΠ²ΠΈΡ‚ΡŒ Ρ‚ΠΎΡ‡ΠΊΠΈ Ρ€Π°Π²Π½ΠΎΠ³ΠΎ ΠΏΠΎΡ‚Π΅Π½Ρ†ΠΈΠ°Π»Π°;
    3. Π²Ρ‹Π±Ρ€Π°Ρ‚ΡŒ, Ρ‡Ρ‚ΠΎ цСлСсообразнСС ΡΠ΄Π΅Π»Π°Ρ‚ΡŒ – ΡΠΎΠ΅Π΄ΠΈΠ½ΠΈΡ‚ΡŒ Ρ‚ΠΎΡ‡ΠΊΠΈ Ρ€Π°Π²Π½Ρ‹Ρ… ΠΏΠΎΡ‚Π΅Π½Ρ†ΠΈΠ°Π»ΠΎΠ² ΠΈΠ»ΠΈ ΠΆΠ΅, Π½Π°ΠΎΠ±ΠΎΡ€ΠΎΡ‚, Ρ€Π°Π·Π΄Π΅Π»ΠΈΡ‚ΡŒ ΠΎΠ΄Π½Ρƒ Ρ‚ΠΎΡ‡ΠΊΡƒ Π½Π° нСсколько Ρ‚ΠΎΡ‡Π΅ΠΊ Ρ€Π°Π²Π½Ρ‹Ρ… ΠΏΠΎΡ‚Π΅Π½Ρ†ΠΈΠ°Π»ΠΎΠ²;
    4. Π½Π°Ρ‡Π΅Ρ€Ρ‚ΠΈΡ‚ΡŒ ΡΠΊΠ²ΠΈΠ²Π°Π»Π΅Π½Ρ‚Π½ΡƒΡŽ схСму;
    5. Π½Π°ΠΉΡ‚ΠΈ участки Ρ‚ΠΎΠ»ΡŒΠΊΠΎ с ΠΏΠΎΡΠ»Π΅Π΄ΠΎΠ²Π°Ρ‚Π΅Π»ΡŒΠ½Ρ‹ΠΌ ΠΈΠ»ΠΈ Ρ‚ΠΎΠ»ΡŒΠΊΠΎ с ΠΏΠ°Ρ€Π°Π»Π»Π΅Π»ΡŒΠ½Ρ‹ΠΌ соСдинСниСм ΠΈ Ρ€Π°ΡΡΡ‡ΠΈΡ‚Π°Ρ‚ΡŒ ΠΎΠ±Ρ‰Π΅Π΅ сопротивлСниС Π½Π° ΠΊΠ°ΠΆΠ΄ΠΎΠΌ участкС ΠΏΠΎ Π·Π°ΠΊΠΎΠ½Π°ΠΌ ΠΏΠΎΡΠ»Π΅Π΄ΠΎΠ²Π°Ρ‚Π΅Π»ΡŒΠ½ΠΎΠ³ΠΎ ΠΈ ΠΏΠ°Ρ€Π°Π»Π»Π΅Π»ΡŒΠ½ΠΎΠ³ΠΎ соСдинСния;
    6. Π½Π°Ρ‡Π΅Ρ€Ρ‚ΠΈΡ‚ΡŒ ΡΠΊΠ²ΠΈΠ²Π°Π»Π΅Π½Ρ‚Π½ΡƒΡŽ схСму, замСняя участки ΡΠΎΠΎΡ‚Π²Π΅Ρ‚ΡΡ‚Π²ΡƒΡŽΡ‰ΠΈΠΌΠΈ ΠΈΠΌ расчСтными сопротивлСниями;
    7. ΠΏΡƒΠ½ΠΊΡ‚Ρ‹ 5 ΠΈ 6 ΠΏΠΎΠ²Ρ‚ΠΎΡ€ΡΡ‚ΡŒ Π΄ΠΎ Ρ‚Π΅Ρ… ΠΏΠΎΡ€, ΠΏΠΎΠΊΠ° Π½Π΅ останСтся ΠΎΠ΄Π½ΠΎ сопротивлСниС, Π²Π΅Π»ΠΈΡ‡ΠΈΠ½Π° ΠΊΠΎΡ‚ΠΎΡ€ΠΎΠ³ΠΎ ΠΈ Π±ΡƒΠ΄Π΅Ρ‚ Ρ€Π΅ΡˆΠ΅Π½ΠΈΠ΅ΠΌ Π·Π°Π΄Π°Ρ‡ΠΈ.
  5. Анализ Ρ€Π΅Π°Π»ΡŒΠ½ΠΎΡΡ‚ΠΈ ΠΎΡ‚Π²Π΅Ρ‚Π°.

ΠŸΠΎΠ΄Ρ€ΠΎΠ±Π½Π΅Π΅ ΠΎΠ± Π°Π½Π°Π»ΠΈΠ·Π΅ схСмы

Π°) ΡƒΡΡ‚Π°Π½ΠΎΠ²ΠΈΡ‚ΡŒ, являСтся Π»ΠΈ схСма симмСтричной.

ΠžΠΏΡ€Π΅Π΄Π΅Π»Π΅Π½ΠΈΠ΅. Π‘Ρ…Π΅ΠΌΠ° симмСтрична, Ссли ΠΎΠ΄Π½Π° Π΅Π΅ ΠΏΠΎΠ»ΠΎΠ²ΠΈΠ½Π° являСтся Π·Π΅Ρ€ΠΊΠ°Π»ΡŒΠ½Ρ‹ΠΌ ΠΎΡ‚Ρ€Π°ΠΆΠ΅Π½ΠΈΠ΅ΠΌ Π΄Ρ€ΡƒΠ³ΠΎΠΉ. ΠŸΡ€ΠΈΡ‡Π΅ΠΌ симмСтрия Π΄ΠΎΠ»ΠΆΠ½Π° Π±Ρ‹Ρ‚ΡŒ Π½Π΅ Ρ‚ΠΎΠ»ΡŒΠΊΠΎ гСомСтричСской, Π½ΠΎ Π΄ΠΎΠ»ΠΆΠ½Ρ‹ Π±Ρ‹Ρ‚ΡŒ симмСтричны ΠΈ числСнныС значСния сопротивлСний ΠΈΠ»ΠΈ кондСнсаторов.

ΠŸΡ€ΠΈΠΌΠ΅Ρ€Ρ‹:

1)

Π‘Ρ…Π΅ΠΌΠ° симмСтричная, Ρ‚Π°ΠΊ ΠΊΠ°ΠΊ Π²Π΅Ρ‚Π²ΠΈ АБВ ΠΈ АДВ симмСтричны гСомСтричСски ΠΈ ΠΎΡ‚Π½ΠΎΡˆΠ΅Π½ΠΈΠ΅ сопротивлСния Π½Π° ΠΎΠ΄Π½ΠΎΠΌ участкС АБ:АД=1:1 Ρ‚Π°ΠΊΠΎΠ΅ ΠΆΠ΅, ΠΊΠ°ΠΊ ΠΈ Π½Π° Π΄Ρ€ΡƒΠ³ΠΎΠΌ участкС Π‘Π”:Π”Π’=1:1.

2)

Π‘Ρ…Π΅ΠΌΠ° симмСтричная, Ρ‚Π°ΠΊ ΠΊΠ°ΠΊ ΠΎΡ‚Π½ΠΎΡˆΠ΅Π½ΠΈΠ΅ сопротивлСний Π½Π° участкС АБ:АД=1:1 Ρ‚Π°ΠΊΠΎΠ΅ ΠΆΠ΅, ΠΊΠ°ΠΊ ΠΈ Π½Π° Π΄Ρ€ΡƒΠ³ΠΎΠΌ участкС Π‘Π’:Π”Π’=3:3=1:1

3)

Π‘Ρ…Π΅ΠΌΠ° Π½Π΅ симмСтрична, Ρ‚Π°ΠΊ ΠΊΠ°ΠΊ ΠΎΡ‚Π½ΠΎΡˆΠ΅Π½ΠΈΡ сопротивлСний числСнно

Π½Π΅ симмСтричны -1:2 ΠΈ 1:1.

Π±) ΡƒΡΡ‚Π°Π½ΠΎΠ²ΠΈΡ‚ΡŒ Ρ‚ΠΎΡ‡ΠΊΠΈ Ρ€Π°Π²Π½Ρ‹Ρ… ΠΏΠΎΡ‚Π΅Π½Ρ†ΠΈΠ°Π»ΠΎΠ².

ΠŸΡ€ΠΈΠΌΠ΅Ρ€:

Из сообраТСний симмСтрии Π΄Π΅Π»Π°Π΅ΠΌ Π²Ρ‹Π²ΠΎΠ΄, Ρ‡Ρ‚ΠΎ Π² симмСтричных Ρ‚ΠΎΡ‡ΠΊΠ°Ρ… ΠΏΠΎΡ‚Π΅Π½Ρ†ΠΈΠ°Π»Ρ‹ Ρ€Π°Π²Π½Ρ‹. Π’ Π΄Π°Π½Π½ΠΎΠΌ случаС симмСтричными Ρ‚ΠΎΡ‡ΠΊΠ°ΠΌΠΈ ΡΠ²Π»ΡΡŽΡ‚ΡΡ Ρ‚ΠΎΡ‡ΠΊΠΈ Π‘ ΠΈ Π”. Π’Π°ΠΊΠΈΠΌ ΠΎΠ±Ρ€Π°Π·ΠΎΠΌ, Ρ‚ΠΎΡ‡ΠΊΠΈ Π‘ ΠΈ Π” – ΡΠΊΠ²ΠΈΠΏΠΎΡ‚Π΅Π½Ρ†ΠΈΠ°Π»ΡŒΠ½Ρ‹Π΅ Ρ‚ΠΎΡ‡ΠΊΠΈ.

Π²) Π²Ρ‹Π±Ρ€Π°Ρ‚ΡŒ, Ρ‡Ρ‚ΠΎ цСлСсообразно ΡΠ΄Π΅Π»Π°Ρ‚ΡŒ – ΡΠΎΠ΅Π΄ΠΈΠ½ΠΈΡ‚ΡŒ Ρ‚ΠΎΡ‡ΠΊΠΈ Ρ€Π°Π²Π½Ρ‹Ρ… ΠΏΠΎΡ‚Π΅Π½Ρ†ΠΈΠ°Π»ΠΎΠ² ΠΈΠ»ΠΈ ΠΆΠ΅, Π½Π°ΠΎΠ±ΠΎΡ€ΠΎΡ‚, Ρ€Π°Π·Π΄Π΅Π»ΠΈΡ‚ΡŒ ΠΎΠ΄Π½Ρƒ Ρ‚ΠΎΡ‡ΠΊΡƒ Π½Π° нСсколько Ρ‚ΠΎΡ‡Π΅ΠΊ Ρ€Π°Π²Π½Ρ‹Ρ… ΠΏΠΎΡ‚Π΅Π½Ρ†ΠΈΠ°Π»ΠΎΠ².

ΠœΡ‹ Π²ΠΈΠ΄ΠΈΠΌ Π² этом ΠΏΡ€ΠΈΠΌΠ΅Ρ€Π΅, Ρ‡Ρ‚ΠΎ ΠΌΠ΅ΠΆΠ΄Ρƒ Ρ‚ΠΎΡ‡ΠΊΠ°ΠΌΠΈ Ρ€Π°Π²Π½Ρ‹Ρ… ΠΏΠΎΡ‚Π΅Π½Ρ†ΠΈΠ°Π»ΠΎΠ² Π‘ ΠΈ Π” Π²ΠΊΠ»ΡŽΡ‡Π΅Π½ΠΎ сопротивлСниС, ΠΏΠΎ ΠΊΠΎΡ‚ΠΎΡ€ΠΎΠΌΡƒ Ρ‚ΠΎΠΊ Π½Π΅ Π±ΡƒΠ΄Π΅Ρ‚ Ρ‚Π΅Ρ‡ΡŒ. Π‘Π»Π΅Π΄ΠΎΠ²Π°Ρ‚Π΅Π»ΡŒΠ½ΠΎ, ΠΌΡ‹ ΠΌΠΎΠΆΠ΅ΠΌ ΠΎΡ‚Π±Ρ€ΠΎΡΠΈΡ‚ΡŒ это сопротивлСниС, Π° Ρ‚ΠΎΡ‡ΠΊΠΈ Π‘ ΠΈ Π” ΡΠΎΠ΅Π΄ΠΈΠ½ΠΈΡ‚ΡŒ Π² ΠΎΠ΄ΠΈΠ½ ΡƒΠ·Π΅Π».

Π³) Π½Π°Ρ‡Π΅Ρ€Ρ‚ΠΈΡ‚ΡŒ ΡΠΊΠ²ΠΈΠ²Π°Π»Π΅Π½Ρ‚Π½ΡƒΡŽ схСму.

Π§Π΅Ρ€Ρ‚ΠΈΠΌ ΡΠΊΠ²ΠΈΠ²Π°Π»Π΅Π½Ρ‚Π½ΡƒΡŽ схСму. ΠŸΡ€ΠΈ этом ΠΏΠΎΠ»ΡƒΡ‡Π°Π΅ΠΌ схСму с соСдинСнными Π² ΠΎΠ΄Π½Ρƒ Ρ‚ΠΎΡ‡ΠΊΡƒ Ρ‚ΠΎΡ‡ΠΊΠ°ΠΌΠΈ Π‘ ΠΈ Π”.

Π΄) Π½Π°ΠΉΡ‚ΠΈ участки Ρ‚ΠΎΠ»ΡŒΠΊΠΎ с ΠΏΠΎΡΠ»Π΅Π΄ΠΎΠ²Π°Ρ‚Π΅Π»ΡŒΠ½Ρ‹ΠΌ ΠΈΠ»ΠΈ Ρ‚ΠΎΠ»ΡŒΠΊΠΎ с ΠΏΠ°Ρ€Π°Π»Π»Π΅Π»ΡŒΠ½Ρ‹ΠΌ соСдинСниСм ΠΈ Ρ€Π°ΡΡΡ‡ΠΈΡ‚Π°Ρ‚ΡŒ ΠΎΠ±Ρ‰Π΅Π΅ сопротивлСниС Π½Π° ΠΊΠ°ΠΆΠ΄ΠΎΠΌ Ρ‚Π°ΠΊΠΎΠΌ участкС ΠΏΠΎ Π·Π°ΠΊΠΎΠ½Π°ΠΌ ΠΏΠΎΡΠ»Π΅Π΄ΠΎΠ²Π°Ρ‚Π΅Π»ΡŒΠ½ΠΎΠ³ΠΎ ΠΈ ΠΏΠ°Ρ€Π°Π»Π»Π΅Π»ΡŒΠ½ΠΎΠ³ΠΎ соСдинСния.

Из ΠΏΠΎΠ»ΡƒΡ‡Π΅Π½Π½ΠΎΠΉ эквивалСнтной схСмы Π²ΠΈΠ΄Π½ΠΎ, Ρ‡Ρ‚ΠΎ Π½Π° участкС АБ ΠΌΡ‹ ΠΈΠΌΠ΅Π΅ΠΌ Π΄Π²Π° ΠΏΠ°Ρ€Π°Π»Π»Π΅Π»ΡŒΠ½ΠΎ соСдинСнных рСзистора. Π˜Ρ… ΠΎΠ±Ρ‰Π΅Π΅ сопротивлСниС находится ΠΏΠΎ Π·Π°ΠΊΠΎΠ½Ρƒ ΠΏΠ°Ρ€Π°Π»Π»Π΅Π»ΡŒΠ½ΠΎΠ³ΠΎ соСдинСния:

1/ RΠΎΠ±Ρ‰=1/R1+1/R2+1/R3+…

Π’Π°ΠΊΠΈΠΌ ΠΎΠ±Ρ€Π°Π·ΠΎΠΌ 1/RAC=1/r+1/r=2/r,ΠΎΡ‚ΠΊΡƒΠ΄Π° RAC= r/2.

На участкС Π‘Π’ ΠΊΠ°Ρ€Ρ‚ΠΈΠ½Π° аналогичная:

1/RCB= 1/r+1/r =2/r, ΠΎΡ‚ΠΊΡƒΠ΄Π° RCB=r/2.

Π΅)Π½Π°Ρ‡Π΅Ρ€Ρ‚ΠΈΡ‚ΡŒ ΡΠΊΠ²ΠΈΠ²Π°Π»Π΅Π½Ρ‚Π½ΡƒΡŽ схСму, замСняя участки ΡΠΎΠΎΡ‚Π²Π΅Ρ‚ΡΡ‚Π²ΡƒΡŽΡ‰ΠΈΠΌΠΈ ΠΈΠΌ расчСтными сопротивлСниями.

Π§Π΅Ρ€Ρ‚ΠΈΠΌ ΡΠΊΠ²ΠΈΠ²Π°Π»Π΅Π½Ρ‚Π½ΡƒΡŽ схСму подставляя Π² Π½Π΅Π΅ рассчитанныС сопротивлСния участков RAC ΠΈ RCB:

ΠΆ)ΠΏΡƒΠ½ΠΊΡ‚Ρ‹ Π΄) ΠΈ Π΅) ΠΏΠΎΠ²Ρ‚ΠΎΡ€ΡΡ‚ΡŒ Π΄ΠΎ Ρ‚Π΅Ρ… ΠΏΠΎΡ€, ΠΏΠΎΠΊΠ° останСтся ΠΎΠ΄Π½ΠΎ сопротивлСниС, Π²Π΅Π»ΠΈΡ‡ΠΈΠ½Π° ΠΊΠΎΡ‚ΠΎΡ€ΠΎΠ³ΠΎ ΠΈ Π±ΡƒΠ΄Π΅Ρ‚ Ρ€Π΅ΡˆΠ΅Π½ΠΈΠ΅ΠΌ Π·Π°Π΄Π°Ρ‡ΠΈ.

ΠŸΠΎΠ²Ρ‚ΠΎΡ€ΡΠ΅ΠΌ ΠΏΡƒΠ½ΠΊΡ‚ Π΄): Π½Π° участкС АВ ΠΈΠΌΠ΅Π΅ΠΌ Π΄Π²Π° ΠΏΠΎΡΠ»Π΅Π΄ΠΎΠ²Π°Ρ‚Π΅Π»ΡŒΠ½ΠΎ соСдинСнных сопротивлСния. Π˜Ρ… ΠΎΠ±Ρ‰Π΅Π΅ сопротивлСниС Π½Π°Ρ…ΠΎΠ΄ΠΈΠΌ ΠΏΠΎ Π·Π°ΠΊΠΎΠ½Ρƒ ΠΏΠΎΡΠ»Π΅Π΄ΠΎΠ²Π°Ρ‚Π΅Π»ΡŒΠ½ΠΎΠ³ΠΎ соСдинСния:

RΠΎΠ±Ρ‰= R1+R2+R3+… Ρ‚ΠΎ Π΅ΡΡ‚ΡŒ, RAB=RAC+RCB = r/2+r/2 =2r/2 = r.

ΠŸΠΎΠ²Ρ‚ΠΎΡ€ΡΠ΅ΠΌ ΠΏΡƒΠ½ΠΊΡ‚ Π΅): Ρ‡Π΅Ρ€Ρ‚ΠΈΠΌ ΡΠΊΠ²ΠΈΠ²Π°Π»Π΅Π½Ρ‚Π½ΡƒΡŽ схСму:

ΠœΡ‹ ΠΏΠΎΠ»ΡƒΡ‡ΠΈΠ»ΠΈ схСму с ΠΎΠ΄Π½ΠΈΠΌ сопротивлСниСм, Π²Π΅Π»ΠΈΡ‡ΠΈΠ½Π° ΠΊΠΎΡ‚ΠΎΡ€ΠΎΠ³ΠΎ Ρ€Π°Π²Π½Π° ΡΠΎΠΏΡ€ΠΎΡ‚ΠΈΠ²Π»Π΅Π½ΠΈΡŽ исходной схСмы. Π’Π°ΠΊΠΈΠΌ ΠΎΠ±Ρ€Π°Π·ΠΎΠΌ, ΠΌΡ‹ ΠΏΠΎΠ»ΡƒΡ‡ΠΈΠ»ΠΈ ΠΎΡ‚Π²Π΅Ρ‚ RAB = r.

Π”Π°Π»Π΅Π΅, для ΠΏΡ€ΠΎΠ²Π΅Ρ€ΠΊΠΈ усвоСния Π΄Π°Π½Π½ΠΎΠ³ΠΎ ΠΌΠ°Ρ‚Π΅Ρ€ΠΈΠ°Π»Π° ΠΌΠΎΠΆΠ½ΠΎ учащимся ΠΏΡ€Π΅Π΄Π»ΠΎΠΆΠΈΡ‚ΡŒ задания для ΡΠ°ΠΌΠΎΡΡ‚ΠΎΡΡ‚Π΅Π»ΡŒΠ½ΠΎΠΉ Ρ€Π°Π±ΠΎΡ‚Ρ‹, взятыС ΠΈΠ· дидактичСского ΠΌΠ°Ρ‚Π΅Ρ€ΠΈΠ°Π»Π°. (см. ΠΏΡ€ΠΈΠ»ΠΎΠΆΠ΅Π½ΠΈΠ΅)

Π›ΠΈΡ‚Π΅Ρ€Π°Ρ‚ΡƒΡ€Π°

  1. Π‘Π°Π»Π°Ρˆ. Π’.А. Π·Π°Π΄Π°Ρ‡ΠΈ ΠΏΠΎ Ρ„ΠΈΠ·ΠΈΠΊΠ΅ ΠΈ ΠΌΠ΅Ρ‚ΠΎΠ΄Ρ‹ ΠΈΡ… Ρ€Π΅ΡˆΠ΅Π½ΠΈΡ. - М: ΠŸΡ€ΠΎΡΠ²Π΅Ρ‰Π΅Π½ΠΈΠ΅,1983.
  2. Π›ΡƒΠΊΠ°ΡˆΠΈΠΊ Π’.И. ЀизичСская ΠΎΠ»ΠΈΠΌΠΏΠΈΠ°Π΄Π°.- М: ΠŸΡ€ΠΎΡΠ²Π΅Ρ‰Π΅Π½ΠΈΠ΅, 2007
  3. Усова А.Π’., Π‘ΠΎΠ±Ρ€ΠΎΠ² А.А. Π€ΠΎΡ€ΠΌΠΈΡ€ΠΎΠ²Π°Π½ΠΈΠ΅ ΡƒΡ‡Π΅Π±Π½Ρ‹Ρ… ΡƒΠΌΠ΅Π½ΠΈΠΉ ΠΈ Π½Π°Π²Ρ‹ΠΊΠΎΠ² учащихся Π½Π° ΡƒΡ€ΠΎΠΊΠ°Ρ… Ρ„ΠΈΠ·ΠΈΠΊΠΈ.- М: ΠŸΡ€ΠΎΡΠ²Π΅Ρ‰Π΅Π½ΠΈΠ΅,1988
  4. Π₯Π°Ρ†Π΅Ρ‚ А. ΠœΠ΅Ρ‚ΠΎΠ΄Ρ‹ расчСта эквивалСнтных схСм //ΠšΠ²Π°Π½Ρ‚.
  5. Π§Π΅Ρ€Ρ‚ΠΎΠ² А. Π“. Π—Π°Π΄Π°Ρ‡Π½ΠΈΠΊ ΠΏΠΎ Ρ„ΠΈΠ·ΠΈΠΊΠ΅. – М.: Π’Ρ‹ΡΡˆΠ°Ρ школа,1983
  6. Зиятдинов Π¨.Π“., Боловьянюк Π‘.Π“. (мСтодичСскиС Ρ€Π΅ΠΊΠΎΠΌΠ΅Π½Π΄Π°Ρ†ΠΈΠΈ) Π³. Бирск,1994Π³
  7. ΠœΠ°Ρ€ΠΎΠ½ А.Π•., ΠœΠ°Ρ€ΠΎΠ½ Π•.А. Π€ΠΈΠ·ΠΈΠΊΠ°. ДидактичСскиС ΠΌΠ°Ρ‚Π΅Ρ€ΠΈΠ°Π»Ρ‹. Москва, β€œΠ”Ρ€ΠΎΡ„Π°β€, 2004Π³

ЭлСктричСскиС Ρ†Π΅ΠΏΠΈ постоянного Ρ‚ΠΎΠΊΠ°

ЭлСктричСским Ρ‚ΠΎΠΊΠΎΠΌ Π½Π°Π·Ρ‹Π²Π°ΡŽΡ‚ упорядочСнноС Π΄Π²ΠΈΠΆΠ΅Π½ΠΈΠ΅ элСктричСских зарядов. НаправлСниСм элСктричСского Ρ‚ΠΎΠΊΠ° ΡƒΡΠ»ΠΎΠ²ΠΈΠ»ΠΈΡΡŒ ΡΡ‡ΠΈΡ‚Π°Ρ‚ΡŒ Π½Π°ΠΏΡ€Π°Π²Π»Π΅Π½ΠΈΠ΅ двиТСния ΠΏΠΎΠ»ΠΎΠΆΠΈΡ‚Π΅Π»ΡŒΠ½Ρ‹Ρ… зарядов.

МоТно ΡƒΠΊΠ°Π·Π°Ρ‚ΡŒ Π½Π° ряд Ρ„Π°ΠΊΡ‚ΠΎΡ€ΠΎΠ², способных Π²Ρ‹Π·Ρ‹Π²Π°Ρ‚ΡŒ упорядочСнноС Π΄Π²ΠΈΠΆΠ΅Π½ΠΈΠ΅ зарядов. Π’Π°ΠΊ, ΠΏΠΎΠ΄ дСйствиСм элСктричСских (кулоновских) сил ΠΏΠΎΠ»ΠΎΠΆΠΈΡ‚Π΅Π»ΡŒΠ½Ρ‹Π΅ заряды двиТутся Π² Π½Π°ΠΏΡ€Π°Π²Π»Π΅Π½ΠΈΠΈ силовых Π»ΠΈΠ½ΠΈΠΉ поля, ΠΎΡ‚Ρ€ΠΈΡ†Π°Ρ‚Π΅Π»ΡŒΠ½Ρ‹Π΅ заряды β€” Π² ΠΏΡ€ΠΎΡ‚ΠΈΠ²ΠΎΠΏΠΎΠ»ΠΎΠΆΠ½ΠΎΠΌ Π½Π°ΠΏΡ€Π°Π²Π»Π΅Π½ΠΈΠΈ. Π”Π²ΠΈΠΆΠ΅Π½ΠΈΠ΅ зарядов ΠΌΠΎΠΆΠ΅Ρ‚ ΠΏΡ€ΠΎΠΈΡΡ…ΠΎΠ΄ΠΈΡ‚ΡŒ ΠΈ ΠΏΠΎΠ΄ дСйствиСм нСэлСктричСских сил (Π½Π°ΠΏΡ€ΠΈΠΌΠ΅Ρ€, ΠΌΠ°Π³Π½ΠΈΡ‚Π½Ρ‹Ρ…), Π° Ρ‚Π°ΠΊΠΆΠ΅ ΠΏΡ€ΠΈ Π΄ΠΈΡ„Ρ„ΡƒΠ·ΠΈΠΈ ΠΈΠ»ΠΈ Π² химичСских рСакциях.
ΠŸΠΎΡΡ‚ΠΎΡΠ½Π½Ρ‹ΠΉ Ρ‚ΠΎΠΊ ΠΈΡΠΏΠΎΠ»ΡŒΠ·ΡƒΠ΅Ρ‚ΡΡ Π² процСссС элСктролиза (Π³Π°Π»ΡŒΠ²Π°Π½ΠΎΠΏΠ»Π°ΡΡ‚ΠΈΠΊΠ° β€” ΠΏΠΎΠ»ΡƒΡ‡Π΅Π½ΠΈΠ΅ Π»Π΅Π³ΠΊΠΎ ΠΎΡ‚Π΄Π΅Π»ΡΡŽΡ‰ΠΈΡ…ΡΡ Ρ‚ΠΎΡ‡Π½Ρ‹Ρ… мСталличСских ΠΊΠΎΠΏΠΈΠΉ, Π³Π°Π»ΡŒΠ²Π°Π½ΠΎΡΡ‚Π΅Π³ΠΈΡ β€” нанСсСниС мСталличСских ΠΏΠΎΠΊΡ€Ρ‹Ρ‚ΠΈΠΉ ΠΈΠ· ΠΎΠ΄Π½ΠΈΡ… ΠΌΠ΅Ρ‚Π°Π»Π»ΠΎΠ² Π½Π° издСлия ΠΈΠ· Π΄Ρ€ΡƒΠ³ΠΈΡ… ΠΌΠ΅Ρ‚Π°Π»Π»ΠΎΠ²), Π½Π° городском транспортС (элСктропоСзда, Ρ‚Ρ€Π°ΠΌΠ²Π°ΠΈ, троллСйбусы), Π² ΠΎΡΠ²Π΅Ρ‚ΠΈΡ‚Π΅Π»ΡŒΠ½Ρ‹Ρ… ΠΏΡ€ΠΈΠ±ΠΎΡ€Π°Ρ…, Π² устройствах Π°Π²Ρ‚ΠΎΠΌΠ°Ρ‚ΠΈΠΊΠΈ, элСктроники ΠΈ Π²Ρ‹Ρ‡ΠΈΡΠ»ΠΈΡ‚Π΅Π»ΡŒΠ½ΠΎΠΉ Ρ‚Π΅Ρ…Π½ΠΈΠΊΠΈ.
Если Ρ‚ΠΎΠΊ постоянный, Ρ‚ΠΎ отсутствуСт явлСниС самоиндукции ΠΈ напряТСниС Π½Π° ΠΊΠ°Ρ‚ΡƒΡˆΠΊΠ΅ индуктивности Ρ€Π°Π²Π½ΠΎ Π½ΡƒΠ»ΡŽ,
, Ρ‚Π°ΠΊ ΠΊΠ°ΠΊ i = const
Если Ρ€Π°ΡΡΠΌΠ°Ρ‚Ρ€ΠΈΠ²Π°Ρ‚ΡŒ кондСнсатор ΠΊΠ°ΠΊ ΠΈΠ΄Π΅Π°Π»ΡŒΠ½ΡƒΡŽ Π΅ΠΌΠΊΠΎΡΡ‚ΡŒ, Ρ‚ΠΎ Π² Ρ†Π΅ΠΏΠΈ постоянного Ρ‚ΠΎΠΊΠ° эта Π²Π΅Ρ‚Π²ΡŒ Ρ€Π°Π²Π½ΠΎΡΠΈΠ»ΡŒΠ½Π° Ρ€Π°Π·ΠΎΠΌΠΊΠ½ΡƒΡ‚ΠΎΠΉ.

ΠŸΠΎΡΡ‚ΠΎΡΠ½Π½Ρ‹ΠΉ Ρ‚ΠΎΠΊ Ρ‡Π΅Ρ€Π΅Π· Π΅ΠΌΠΊΠΎΡΡ‚ΡŒ Π½Π΅ ΠΏΡ€ΠΎΡ…ΠΎΠ΄ΠΈΡ‚.

Π’Π°ΠΊΠΈΠΌ ΠΎΠ±Ρ€Π°Π·ΠΎΠΌ, Π² Ρ†Π΅ΠΏΠΈ постоянного Ρ‚ΠΎΠΊΠ° ΠΎΡΡ‚Π°ΡŽΡ‚ΡΡ Ρ‚ΠΎΠ»ΡŒΠΊΠΎ источники Π­Π”Π‘ ΠΈΠ»ΠΈ Ρ‚ΠΎΠΊΠ° β€” Π°ΠΊΡ‚ΠΈΠ²Π½Ρ‹Π΅ элСмСнты ΠΈ ΠΏΡ€ΠΈΠ΅ΠΌΠ½ΠΈΠΊΠΈ рСзисторы β€” пассивныС элСмСнты.
ΠŸΡ€ΠΎΡΡ‚Ρ‹ΠΌΠΈ цСпями постоянного Ρ‚ΠΎΠΊΠ° Π½Π°Π·Ρ‹Π²Π°ΡŽΡ‚ΡΡ Ρ†Π΅ΠΏΠΈ с ΠΎΠ΄Π½ΠΈΠΌ источником ΠΏΡ€ΠΈ ΠΏΠΎΡΠ»Π΅Π΄ΠΎΠ²Π°Ρ‚Π΅Π»ΡŒΠ½ΠΎΠΌ, ΠΏΠ°Ρ€Π°Π»Π»Π΅Π»ΡŒΠ½ΠΎΠΌ ΠΈ смСшанном соСдинСнии ΠΏΡ€ΠΈΠ΅ΠΌΠ½ΠΈΠΊΠΎΠ².

ΠŸΠΎΡΠ»Π΅Π΄ΠΎΠ²Π°Ρ‚Π΅Π»ΡŒΠ½ΠΎΠ΅ соСдинСниС ΠΏΡ€ΠΈΠ΅ΠΌΠ½ΠΈΠΊΠΎΠ²




ΠŸΡ€ΠΈ ΠΏΠ°Ρ€Π°Π»Π»Π΅Π»ΡŒΠ½ΠΎΠΌ соСдинСнии ΠΏΡ€ΠΈΠ΅ΠΌΠ½ΠΈΠΊΠΎΠ² напряТСниС Π½Π° всСх ΠΏΡ€ΠΈΠ΅ΠΌΠ½ΠΈΠΊΠ°Ρ… ΠΎΠ΄ΠΈΠ½Π°ΠΊΠΎΠ²ΠΎ.
По Π·Π°ΠΊΠΎΠ½Ρƒ Ома Ρ‚ΠΎΠΊΠΈ Π² ΠΊΠ°ΠΆΠ΄ΠΎΠΉ Π²Π΅Ρ‚Π²ΠΈ:


По ΠΏΠ΅Ρ€Π²ΠΎΠΌΡƒ Π·Π°ΠΊΠΎΠ½Ρƒ ΠšΠΈΡ€Ρ…Π³ΠΎΡ„Π° ΠΎΠ±Ρ‰ΠΈΠΉ Ρ‚ΠΎΠΊ




БмСшанноС соСдинСниС β€” комбинация ΠΏΠ΅Ρ€Π²Ρ‹Ρ… Π΄Π²ΡƒΡ… соСдинСний, Π³Π΄Π΅ ΠΏΠ°Ρ€Π°Π»Π»Π΅Π»ΡŒΠ½ΠΎΠ΅ соСдинСниС ΠΌΠΎΠΆΠ΅Ρ‚ Π±Ρ‹Ρ‚ΡŒ ΠΏΡ€Π΅ΠΎΠ±Ρ€Π°Π·ΠΎΠ²Π°Π½ΠΎ ΠΊ ΠΏΠΎΡΠ»Π΅Π΄ΠΎΠ²Π°Ρ‚Π΅Π»ΡŒΠ½ΠΎΠΌΡƒ.


Π‘Π»ΠΎΠΆΠ½ΠΎΠΉ элСктричСской Ρ†Π΅ΠΏΡŒΡŽ называСтся Ρ†Π΅ΠΏΡŒ, содСрТащая нСсколько источников ΠΈ ΠΊΠΎΡ‚ΠΎΡ€ΡƒΡŽ нСльзя ΡΠ²Π΅Ρ€Π½ΡƒΡ‚ΡŒ Π΄ΠΎ простой Ρ†Π΅ΠΏΠΈ ΠΏΠΎΡΠ»Π΅Π΄ΠΎΠ²Π°Ρ‚Π΅Π»ΡŒΠ½ΠΎΠ³ΠΎ ΠΈΠ»ΠΈ ΠΏΠ°Ρ€Π°Π»Π»Π΅Π»ΡŒΠ½ΠΎΠ³ΠΎ соСдинСния.
РасчСт Ρ‚Π°ΠΊΠΈΡ… Ρ†Π΅ΠΏΠ΅ΠΉ вСдСтся ΠΏΠΎ уравнСниям ΠšΠΈΡ€Ρ…Π³ΠΎΡ„Π°.
Для ΠΈΡ… составлСния Π½Π΅ΠΎΠ±Ρ…ΠΎΠ΄ΠΈΠΌΠΎ Π·Π°Π΄Π°Ρ‚ΡŒ условныС направлСния Ρ‚ΠΎΠΊΠΎΠ² Π² вСтвях (Π½ΠΎΠΌΠ΅Ρ€ Π²Π²Π΅Π΄Π΅ΠΌ Π² соотвСтствии с порядковым Π½ΠΎΠΌΠ΅Ρ€ΠΎΠΌ сопротивлСний).
По ΠΏΠ΅Ρ€Π²ΠΎΠΌΡƒ Π·Π°ΠΊΠΎΠ½Ρƒ ΠšΠΈΡ€Ρ…Π³ΠΎΡ„Π° ΡΠΎΡΡ‚Π°Π²Π»ΡΡŽΡ‚ΡΡ уравнСния для ΠΊΠ°ΠΆΠ΄ΠΎΠ³ΠΎ ΠΈΠ· нСзависимых ΡƒΠ·Π»ΠΎΠ² (для Π΄Π°Π½Π½ΠΎΠΉ схСмы Ρ‚Π°ΠΊΠΈΡ… ΡƒΠ·Π»ΠΎΠ² 3).



Π’Ρ‹Π±ΠΈΡ€Π°ΡŽΡ‚ΡΡ направлСния ΠΎΠ±Ρ…ΠΎΠ΄Π° Π² ΠΊΠ°ΠΆΠ΄ΠΎΠΌ ΠΈΠ· нСзависимых ΠΊΠΎΠ½Ρ‚ΡƒΡ€ΠΎΠ² ΠΈ ΡΠΎΡΡ‚Π°Π²Π»ΡΡŽΡ‚ΡΡ уравнСния ΠΏΠΎ Π²Ρ‚ΠΎΡ€ΠΎΠΌΡƒ Π·Π°ΠΊΠΎΠ½Ρƒ ΠšΠΈΡ€Ρ…Π³ΠΎΡ„Π° β€” сумма ΠΏΠ°Π΄Π΅Π½ΠΈΠΉ напряТСний Π½Π° пассивных элСмСнтах Π·Π°ΠΌΠΊΠ½ΡƒΡ‚ΠΎΠ³ΠΎ ΠΊΠΎΠ½Ρ‚ΡƒΡ€Π° элСктричСской Ρ†Π΅ΠΏΠΈ Ρ€Π°Π²Π½Π° алгСбраичСской суммС источников Π­Π”Π‘ Π² Π΄Π°Π½Π½ΠΎΠΌ ΠΊΠΎΠ½Ρ‚ΡƒΡ€Π΅:

Для нахоТдСния Ρ€Π΅ΡˆΠ΅Π½ΠΈΡ Π½Π΅ΠΎΠ±Ρ…ΠΎΠ΄ΠΈΠΌΠΎ Π»ΡŽΠ±Ρ‹ΠΌ матСматичСским способом Ρ€Π΅ΡˆΠΈΡ‚ΡŒ ΠΏΠΎΠ»ΡƒΡ‡Π΅Π½Π½Ρ‹Π΅ ΡˆΠ΅ΡΡ‚ΡŒ ΡƒΡ€Π°Π²Π½Π΅Π½ΠΈΠΉ, Ρ‡Ρ‚ΠΎ вСсьма слоТно. Π§Ρ‚ΠΎΠ±Ρ‹ ΡΠΎΠΊΡ€Π°Ρ‚ΠΈΡ‚ΡŒ число ΡƒΡ€Π°Π²Π½Π΅Π½ΠΈΠΉ, ΠΈΡΠΏΠΎΠ»ΡŒΠ·ΡƒΡŽΡ‚ ΠΌΠ΅Ρ‚ΠΎΠ΄ ΠΊΠΎΠ½Ρ‚ΡƒΡ€Π½Ρ‹Ρ… Ρ‚ΠΎΠΊΠΎΠ².
Для Π²Ρ‹Π²ΠΎΠ΄Π° ΡƒΡ€Π°Π²Π½Π΅Π½ΠΈΠΉ ΠΏΠΎ ΠΌΠ΅Ρ‚ΠΎΠ΄Ρƒ ΠΊΠΎΠ½Ρ‚ΡƒΡ€Π½Ρ‹Ρ… Ρ‚ΠΎΠΊΠΎΠ² Π² ΠΎΠ±Ρ‰Π΅ΠΌ Π²ΠΈΠ΄Π΅ ΠΈΡΠΊΠ»ΡŽΡ‡ΠΈΠΌ ΠΈΠ· послСдних Ρ‚Ρ€Π΅Ρ… ΡƒΡ€Π°Π²Π½Π΅Π½ΠΈΠΉ Ρ‚ΠΎΠΊΠΈ Π²Π΅Ρ‚Π²Π΅ΠΉ смСТных ΠΊΠΎΠ½Ρ‚ΡƒΡ€ΠΎΠ² , Π·Π°ΠΌΠ΅Π½ΠΈΠ² ΠΈΡ… выраТСниями, ΠΏΠΎΠ»ΡƒΡ‡Π΅Π½Π½Ρ‹ΠΌΠΈ ΠΈΠ· ΠΏΠ΅Ρ€Π²Ρ‹Ρ… Ρ‚Ρ€Π΅Ρ… ΡƒΡ€Π°Π²Π½Π΅Π½ΠΈΠΉ:

Π’Π²Π΅Π΄Π΅ΠΌ обозначСния ΠΊΠΎΠ½Ρ‚ΡƒΡ€Π½Ρ‹Ρ… Ρ‚ΠΎΠΊΠΎΠ²:
β€” Ρ‚ΠΎΠΊ ΠΏΠ΅Ρ€Π²ΠΎΠ³ΠΎ ΠΊΠΎΠ½Ρ‚ΡƒΡ€Π°;
β€” Ρ‚ΠΎΠΊ Π²Ρ‚ΠΎΡ€ΠΎΠ³ΠΎ ΠΊΠΎΠ½Ρ‚ΡƒΡ€Π°;
β€” Ρ‚ΠΎΠΊ Ρ‚Ρ€Π΅Ρ‚ΡŒΠ΅Π³ΠΎ ΠΊΠΎΠ½Ρ‚ΡƒΡ€Π°.
Для ΠΊΠΎΠ½ΠΊΡ€Π΅Ρ‚ΠΈΠ·Π°Ρ†ΠΈΠΈ ΠΈ сокращСния записи Π²Π²Π΅Π΄Π΅ΠΌ обозначСния для ΠΊΠΎΠ½Ρ‚ΡƒΡ€Π½Ρ‹Ρ… Π­Π”Π‘, Ρ€Π°Π²Π½Ρ‹Ρ… суммС Π­Π”Π‘ источников рассматриваСмого ΠΊΠΎΠ½Ρ‚ΡƒΡ€Π°:

ΠΈ соотвСтствСнно суммы сопротивлСний Π² ΠΊΠ°ΠΆΠ΄ΠΎΠΌ ΠΊΠΎΠ½Ρ‚ΡƒΡ€Π΅ Ρ‡Π΅Ρ€Π΅Π· ΠΊΠΎΠ½Ρ‚ΡƒΡ€Π½Ρ‹Π΅ сопротивлСния:

Π° сопротивлСния смСТных Π²Π΅Ρ‚Π²Π΅ΠΉ ΠΊΠ°ΠΊ:

ΠŸΡ€ΠΈ принятых обозначСниях систСма расчСтных ΡƒΡ€Π°Π²Π½Π΅Π½ΠΈΠΉ Π·Π°ΠΏΠΈΡˆΠ΅Ρ‚ΡΡ Π² ΠΎΠ±Ρ‰Π΅ΠΌ Π²ΠΈΠ΄Π΅ ΠΊΠ°ΠΊ:



ΠœΡ‹ Π²ΠΈΠ΄ΠΈΠΌ, Ρ‡Ρ‚ΠΎ ΠΏΡ€ΠΈ расчСтах Ρ†Π΅ΠΏΠ΅ΠΉ с ΠΏΠΎΠΌΠΎΡ‰ΡŒΡŽ ΠΏΡ€Π°Π²ΠΈΠ» ΠšΠΈΡ€Ρ…Π³ΠΎΡ„Π° Π½Π΅ ΠΎΠ±ΡΠ·Π°Ρ‚Π΅Π»ΡŒΠ½ΠΎ Π·Π½Π°Ρ‚ΡŒ разности ΠΏΠΎΡ‚Π΅Π½Ρ†ΠΈΠ°Π»ΠΎΠ² Π½Π° ΠΎΠΏΡ€Π΅Π΄Π΅Π»Π΅Π½Π½Ρ‹Ρ… участках.

РасчСт элСктричСских Ρ†Π΅ΠΏΠ΅ΠΉ постоянного Ρ‚ΠΎΠΊΠ° ΠΌΠ΅Ρ‚ΠΎΠ΄ΠΎΠΌ эквивалСнтных ΠΏΡ€Π΅ΠΎΠ±Ρ€Π°Π·ΠΎΠ²Π°Π½ΠΈΠΉ

Главная β†’ ΠŸΡ€ΠΈΠΌΠ΅Ρ€Ρ‹ Ρ€Π΅ΡˆΠ΅Π½ΠΈΡ Π·Π°Π΄Π°Ρ‡ ВОЭ β†’ РасчСт элСктричСских Ρ†Π΅ΠΏΠ΅ΠΉ постоянного Ρ‚ΠΎΠΊΠ° ΠΌΠ΅Ρ‚ΠΎΠ΄ΠΎΠΌ эквивалСнтных ΠΏΡ€Π΅ΠΎΠ±Ρ€Π°Π·ΠΎΠ²Π°Π½ΠΈΠΉ

РасчСт элСктричСских Ρ†Π΅ΠΏΠ΅ΠΉ постоянного Ρ‚ΠΎΠΊΠ° ΠΌΠ΅Ρ‚ΠΎΠ΄ΠΎΠΌ эквивалСнтных ΠΏΡ€Π΅ΠΎΠ±Ρ€Π°Π·ΠΎΠ²Π°Π½ΠΈΠΉ

ΠžΡΠ½ΠΎΠ²Π½Ρ‹ΠΌΠΈ Π·Π°ΠΊΠΎΠ½Π°ΠΌΠΈ, ΠΎΠΏΡ€Π΅Π΄Π΅Π»ΡΡŽΡ‰ΠΈΠΌΠΈ расчСт элСктричСской Ρ†Π΅ΠΏΠΈ, ΡΠ²Π»ΡΡŽΡ‚ΡΡ Π·Π°ΠΊΠΎΠ½Ρ‹ ΠšΠΈΡ€Ρ…Π³ΠΎΡ„Π°.

На основС Π·Π°ΠΊΠΎΠ½ΠΎΠ² ΠšΠΈΡ€Ρ…Π³ΠΎΡ„Π° Ρ€Π°Π·Ρ€Π°Π±ΠΎΡ‚Π°Π½ ряд практичСских ΠΌΠ΅Ρ‚ΠΎΠ΄ΠΎΠ² расчСта элСктричСских Ρ†Π΅ΠΏΠ΅ΠΉ постоянного Ρ‚ΠΎΠΊΠ°, ΠΏΠΎΠ·Π²ΠΎΠ»ΡΡŽΡ‰ΠΈΡ… ΡΠΎΠΊΡ€Π°Ρ‚ΠΈΡ‚ΡŒ вычислСния при расчСтС слоТных схСм.

БущСствСнно ΡƒΠΏΡ€ΠΎΡΡ‚ΠΈΡ‚ΡŒ вычислСния, Π°Β Π²Β Π½Π΅ΠΊΠΎΡ‚ΠΎΡ€Ρ‹Ρ… случаях ΠΈΒ ΡΠ½ΠΈΠ·ΠΈΡ‚ΡŒ Ρ‚Ρ€ΡƒΠ΄ΠΎΠ΅ΠΌΠΊΠΎΡΡ‚ΡŒ расчСта, Π²ΠΎΠ·ΠΌΠΎΠΆΠ½ΠΎ ΡΒ ΠΏΠΎΠΌΠΎΡ‰ΡŒΡŽ эквивалСнтных ΠΏΡ€Π΅ΠΎΠ±Ρ€Π°Π·ΠΎΠ²Π°Π½ΠΈΠΉ схСмы.

ΠŸΡ€Π΅ΠΎΠ±Ρ€Π°Π·ΡƒΡŽΡ‚ ΠΏΠ°Ρ€Π°Π»Π»Π΅Π»ΡŒΠ½Ρ‹Π΅ ΠΈΒ ΠΏΠΎΡΠ»Π΅Π΄ΠΎΠ²Π°Ρ‚Π΅Π»ΡŒΠ½Ρ‹Π΅ соСдинСния элСмСнтов, соСдинСниС Β«Π·Π²Π΅Π·Π΄Π°Β» в эквивалСнтный Β«Ρ‚Ρ€Π΅ΡƒΠ³ΠΎΠ»ΡŒΠ½ΠΈΠΊΒ» ΠΈΒ Π½Π°ΠΎΠ±ΠΎΡ€ΠΎΡ‚. ΠžΡΡƒΡ‰Π΅ΡΡ‚Π²Π»ΡΡŽΡ‚ Π·Π°ΠΌΠ΅Π½Ρƒ источника Ρ‚ΠΎΠΊΠ° эквивалСнтным источником Π­Π”Π‘. ΠœΠ΅Ρ‚ΠΎΠ΄ΠΎΠΌ эквивалСнтных ΠΏΡ€Π΅ΠΎΠ±Ρ€Π°Π·ΠΎΠ²Π°Π½ΠΈΠΉ тСорСтичСски ΠΌΠΎΠΆΠ½ΠΎ Ρ€Π°ΡΡΡ‡ΠΈΡ‚Π°Ρ‚ΡŒ Π»ΡŽΠ±ΡƒΡŽ Ρ†Π΅ΠΏΡŒ, и при этом ΠΈΡΠΏΠΎΠ»ΡŒΠ·ΠΎΠ²Π°Ρ‚ΡŒ простыС Π²Ρ‹Ρ‡ΠΈΡΠ»ΠΈΡ‚Π΅Π»ΡŒΠ½Ρ‹Π΅ срСдства. Или ТС ΠΎΠΏΡ€Π΅Π΄Π΅Π»ΠΈΡ‚ΡŒ Ρ‚ΠΎΠΊΒ Π²Β ΠΊΠ°ΠΊΠΎΠΉ-Π»ΠΈΠ±ΠΎ ΠΎΠ΄Π½ΠΎΠΉ Π²Π΅Ρ‚Π²ΠΈ, бСз расчСта Ρ‚ΠΎΠΊΠΎΠ² Π΄Ρ€ΡƒΠ³ΠΈΡ… участков Ρ†Π΅ΠΏΠΈ.

Π’ Π΄Π°Π½Π½ΠΎΠΉ ΡΡ‚Π°Ρ‚ΡŒΠ΅ по тСорСтичСским основам элСктротСхники рассмотрСны ΠΏΡ€ΠΈΠΌΠ΅Ρ€Ρ‹ расчСта Π»ΠΈΠ½Π΅ΠΉΠ½Ρ‹Ρ… элСктричСских Ρ†Π΅ΠΏΠ΅ΠΉ постоянного Ρ‚ΠΎΠΊΠ° с использованиСм ΠΌΠ΅Ρ‚ΠΎΠ΄Π° эквивалСнтных ΠΏΡ€Π΅ΠΎΠ±Ρ€Π°Π·ΠΎΠ²Π°Π½ΠΈΠΉ Ρ‚ΠΈΠΏΠΎΠ²Ρ‹Ρ… схСм соСдинСния источников ΠΈΒ ΠΏΠΎΡ‚Ρ€Π΅Π±ΠΈΡ‚Π΅Π»Π΅ΠΉ энСргии, ΠΏΡ€ΠΈΠ²Π΅Π΄Π΅Π½Ρ‹ расчСтныС Ρ„ΠΎΡ€ΠΌΡƒΠ»Ρ‹.

РСшСниС Π·Π°Π΄Π°Ρ‡ РасчСт элСктричСских Ρ†Π΅ΠΏΠ΅ΠΉ постоянного Ρ‚ΠΎΠΊΠ° ΠΌΠ΅Ρ‚ΠΎΠ΄ΠΎΠΌ эквивалСнтных ΠΏΡ€Π΅ΠΎΠ±Ρ€Π°Π·ΠΎΠ²Π°Π½ΠΈΠΉ


Π—Π°Π΄Π°Ρ‡Π° 1. Для цСпи (рис. 1), ΠΎΠΏΡ€Π΅Π΄Π΅Π»ΠΈΡ‚ΡŒ эквивалСнтноС сопротивлСниС ΠΎΡ‚Π½ΠΎΡΠΈΡ‚Π΅Π»ΡŒΠ½ΠΎ Π²Ρ…ΠΎΠ΄Π½Ρ‹Ρ… Π·Π°ΠΆΠΈΠΌΠΎΠ² aβˆ’g, Ссли извСстно: R1 = R2 = 0,5 Ом, R3 = 8 Ом, R4 = R5 = 1 Ом, R6 = 12 Ом, R7 = 15 Ом, R8 = 2 Ом, R9 = 10 Ом, R10= 20 Ом.

Рис. 1

РСшСниС

НачнСм эквивалСнтныС прСобразования схСмы с вСтви Π½Π°ΠΈΠ±ΠΎΠ»Π΅Π΅ ΡƒΠ΄Π°Π»Π΅Π½Π½ΠΎΠΉ от источника, Ρ‚.Π΅. ΠΎΡ‚Β Π·Π°ΠΆΠΈΠΌΠΎΠ² aβˆ’g:


Π—Π°Π΄Π°Ρ‡Π° 2. Для цСпи (рис. 2, Π°), ΠΎΠΏΡ€Π΅Π΄Π΅Π»ΠΈΡ‚ΡŒ Π²Ρ…ΠΎΠ΄Π½ΠΎΠ΅ сопротивлСниС Ссли извСстно: R1 = R2 = R3 = R4= 40 Ом.

Рис. 2

РСшСниС

Π˜ΡΡ…ΠΎΠ΄Π½ΡƒΡŽ схСму ΠΌΠΎΠΆΠ½ΠΎ ΠΏΠ΅Ρ€Π΅Ρ‡Π΅Ρ€Ρ‚ΠΈΡ‚ΡŒ ΠΎΡ‚Π½ΠΎΡΠΈΡ‚Π΅Π»ΡŒΠ½ΠΎ Π²Ρ…ΠΎΠ΄Π½Ρ‹Ρ… Π·Π°ΠΆΠΈΠΌΠΎΠ² (рис. 2, Π±), ΠΈΠ·Β Ρ‡Π΅Π³ΠΎ Π²ΠΈΠ΄Π½ΠΎ, что всС сопротивлСния Π²ΠΊΠ»ΡŽΡ‡Π΅Π½Ρ‹ ΠΏΠ°Ρ€Π°Π»Π»Π΅Π»ΡŒΠ½ΠΎ. Π’Π°ΠΊΒ ΠΊΠ°ΠΊΒ Π²Π΅Π»ΠΈΡ‡ΠΈΠ½Ρ‹ сопротивлСний Ρ€Π°Π²Π½Ρ‹, то для опрСдСлСния Π²Π΅Π»ΠΈΡ‡ΠΈΠ½Ρ‹ эквивалСнтного сопротивлСниямоТно Π²ΠΎΡΠΏΠΎΠ»ΡŒΠ·ΠΎΠ²Π°Ρ‚ΡŒΡΡ Ρ„ΠΎΡ€ΠΌΡƒΠ»ΠΎΠΉ:

Π³Π΄Π΅ R – Π²Π΅Π»ΠΈΡ‡ΠΈΠ½Π° сопротивлСния, Ом;

n – количСство ΠΏΠ°Ρ€Π°Π»Π»Π΅Π»ΡŒΠ½ΠΎ соСдинСнных сопротивлСний.


Π—Π°Π΄Π°Ρ‡Π° 3. ΠžΠΏΡ€Π΅Π΄Π΅Π»ΠΈΡ‚ΡŒ эквивалСнтноС сопротивлСниС ΠΎΡ‚Π½ΠΎΡΠΈΡ‚Π΅Π»ΡŒΠ½ΠΎ Π·Π°ΠΆΠΈΠΌΠΎΠ² a–b, Ссли R1 = R2 = R3 = R4 = R5 = R6 = 10 Ом (рис. 3, Π°).

Рис. 3

РСшСниС

ΠŸΡ€Π΅ΠΎΠ±Ρ€Π°Π·ΡƒΠ΅ΠΌ соСдинСниС Β«Ρ‚Ρ€Π΅ΡƒΠ³ΠΎΠ»ΡŒΠ½ΠΈΠΊΒ» fβˆ’dβˆ’c Π²Β ΡΠΊΠ²ΠΈΠ²Π°Π»Π΅Π½Ρ‚Π½ΡƒΡŽ Β«Π·Π²Π΅Π·Π΄ΡƒΒ». ΠžΠΏΡ€Π΅Π΄Π΅Π»ΡΠ΅ΠΌ Π²Π΅Π»ΠΈΡ‡ΠΈΠ½Ρ‹ ΠΏΡ€Π΅ΠΎΠ±Ρ€Π°Π·ΠΎΠ²Π°Π½Π½Ρ‹Ρ… сопротивлСний (рис. 3, Π±):

По ΡƒΡΠ»ΠΎΠ²ΠΈΡŽ Π·Π°Π΄Π°Ρ‡ΠΈ Π²Π΅Π»ΠΈΡ‡ΠΈΠ½Ρ‹ всСх сопротивлСний Ρ€Π°Π²Π½Ρ‹, Π°Β Π·Π½Π°Ρ‡ΠΈΡ‚:

На ΠΏΡ€Π΅ΠΎΠ±Ρ€Π°Π·ΠΎΠ²Π°Π½Π½ΠΎΠΉ схСмС ΠΏΠΎΠ»ΡƒΡ‡ΠΈΠ»ΠΈ ΠΏΠ°Ρ€Π°Π»Π»Π΅Π»ΡŒΠ½ΠΎΠ΅ соСдинСниС Π²Π΅Ρ‚Π²Π΅ΠΉ ΠΌΠ΅ΠΆΠ΄Ρƒ ΡƒΠ·Π»Π°ΠΌΠΈ e–b, Ρ‚ΠΎΠ³Π΄Π° эквивалСнтноС сопротивлСниС Ρ€Π°Π²Π½ΠΎ:

И Ρ‚ΠΎΠ³Π΄Π° эквивалСнтноС сопротивлСниС исходной схСмы прСдставляСт ΠΏΠΎΡΠ»Π΅Π΄ΠΎΠ²Π°Ρ‚Π΅Π»ΡŒΠ½ΠΎΠ΅ соСдинСниС сопротивлСний:


Π—Π°Π΄Π°Ρ‡Π° 4. Π’Β Π·Π°Π΄Π°Π½Π½ΠΎΠΉ Ρ†Π΅ΠΏΠΈ (рис. 4, Π°) ΠΎΠΏΡ€Π΅Π΄Π΅Π»ΠΈΡ‚ΡŒ ΠΌΠ΅Ρ‚ΠΎΠ΄ΠΎΠΌ эквивалСнтных ΠΏΡ€Π΅ΠΎΠ±Ρ€Π°Π·ΠΎΠ²Π°Π½ΠΈΠΉ Π²Ρ…ΠΎΠ΄Π½Ρ‹Π΅ сопротивлСния Π²Π΅Ρ‚Π²Π΅ΠΉ aβˆ’b, c–d ΠΈΒ fβˆ’b, Ссли извСстно, Ρ‡Ρ‚ΠΎ: R1 = 4 Ом, R2 = 8 Ом, R3 =4 Ом, R4 = 8 Ом, R5 = 2 Ом, R6 = 8 Ом, R7 = 6 Ом, R8 =8 Ом.

РСшСниС

Для опрСдСлСния Π²Ρ…ΠΎΠ΄Π½ΠΎΠ³ΠΎ сопротивлСния Π²Π΅Ρ‚Π²Π΅ΠΉ ΠΈΡΠΊΠ»ΡŽΡ‡Π°ΡŽΡ‚ из схСмы всС источники Π­Π”Π‘. ΠŸΡ€ΠΈΒ ΡΡ‚ΠΎΠΌ Ρ‚ΠΎΡ‡ΠΊΠΈ c ΠΈΒ d, Π°Β Ρ‚Π°ΠΊΠΆΠ΅ b ΠΈΒ f ΡΠΎΠ΅Π΄ΠΈΠ½ΡΡŽΡ‚ΡΡ Π½Π°ΠΊΠΎΡ€ΠΎΡ‚ΠΊΠΎ, Ρ‚.ΠΊ. Π²Π½ΡƒΡ‚Ρ€Π΅Π½Π½ΠΈΠ΅ сопротивлСния ΠΈΠ΄Π΅Π°Π»ΡŒΠ½Ρ‹Ρ… источников напряТСния Ρ€Π°Π²Π½Ρ‹ Π½ΡƒΠ»ΡŽ.

Рис. 4

Π’Π΅Ρ‚Π²ΡŒ aβˆ’b Ρ€Π°Π·Ρ€Ρ‹Π²Π°ΡŽΡ‚, ΠΈΒ Ρ‚.ΠΊ. сопротивлСниС Ra–b = 0, Ρ‚ΠΎΒ Π²Ρ…ΠΎΠ΄Π½ΠΎΠ΅ сопротивлСниС Π²Π΅Ρ‚Π²ΠΈ Ρ€Π°Π²Π½ΠΎ эквивалСнтному ΡΠΎΠΏΡ€ΠΎΡ‚ΠΈΠ²Π»Π΅Π½ΠΈΡŽ схСмы ΠΎΡ‚Π½ΠΎΡΠΈΡ‚Π΅Π»ΡŒΠ½ΠΎ Ρ‚ΠΎΡ‡Π΅ΠΊ a ΠΈΒ b (рис. 4, Π±):

Аналогично ΠΌΠ΅Ρ‚ΠΎΠ΄ΠΎΠΌ эквивалСнтных ΠΏΡ€Π΅ΠΎΠ±Ρ€Π°Π·ΠΎΠ²Π°Π½ΠΈΠΉ ΠΎΠΏΡ€Π΅Π΄Π΅Π»ΡΡŽΡ‚ΡΡ Π²Ρ…ΠΎΠ΄Π½Ρ‹Π΅ сопротивлСния Π²Π΅Ρ‚Π²Π΅ΠΉ Rcd ΠΈΒ Rbf. ΠŸΡ€ΠΈΡ‡Π΅ΠΌ, при вычислСнии сопротивлСний ΡƒΡ‡Ρ‚Π΅Π½ΠΎ, что соСдинСниС Π½Π°ΠΊΠΎΡ€ΠΎΡ‚ΠΊΠΎ Ρ‚ΠΎΡ‡Π΅ΠΊ a ΠΈΒ b ΠΈΡΠΊΠ»ΡŽΡ‡Π°Π΅Ρ‚ ( Β«Π·Π°ΠΊΠΎΡ€Π°Ρ‡ΠΈΠ²Π°Π΅Ρ‚Β») из схСмы сопротивлСния R1, R2, R3, R4 Π²Β ΠΏΠ΅Ρ€Π²ΠΎΠΌ случаС, ΠΈΒ R5, R6, R7, R8 Π²ΠΎΒ Π²Ρ‚ΠΎΡ€ΠΎΠΌ случаС.


Π—Π°Π΄Π°Ρ‡Π° 5. Π’Β Ρ†Π΅ΠΏΠΈ (рис. 5) ΠΎΠΏΡ€Π΅Π΄Π΅Π»ΠΈΡ‚ΡŒ ΠΌΠ΅Ρ‚ΠΎΠ΄ΠΎΠΌ эквивалСнтных ΠΏΡ€Π΅ΠΎΠ±Ρ€Π°Π·ΠΎΠ²Π°Π½ΠΈΠΉ Ρ‚ΠΎΠΊΠΈ I1, I2, I3 ΠΈΒ ΡΠΎΡΡ‚Π°Π²ΠΈΡ‚ΡŒ баланс мощностСй, Ссли извСстно: R1 = 12 Ом, R2 = 20 Ом, R3 = 30 Ом, U = 120 Π’.

Рис. 5

РСшСниС

Π­ΠΊΠ²ΠΈΠ²Π°Π»Π΅Π½Ρ‚Π½ΠΎΠ΅ ΡΠΎΠΏΡ€ΠΎΡ‚ΠΈΠ²Π»Π΅Π½ΠΈΠ΅Π΄Π»ΡΒ ΠΏΠ°Ρ€Π°Π»Π»Π΅Π»ΡŒΠ½ΠΎ Π²ΠΊΠ»ΡŽΡ‡Π΅Π½Π½Ρ‹Ρ… сопротивлСний:

Π­ΠΊΠ²ΠΈΠ²Π°Π»Π΅Π½Ρ‚Π½ΠΎΠ΅ сопротивлСниС всСй Ρ†Π΅ΠΏΠΈ:

Π’ΠΎΠΊ Π²Β Π½Π΅Ρ€Π°Π·Π²Π΅Ρ‚Π²Π»Π΅Π½Π½ΠΎΠΉ части схСмы:

НапряТСниС Π½Π°Β ΠΏΠ°Ρ€Π°Π»Π»Π΅Π»ΡŒΠ½Ρ‹Ρ… сопротивлСниях:

Π’ΠΎΠΊΠΈ Π²Β ΠΏΠ°Ρ€Π°Π»Π»Π΅Π»ΡŒΠ½Ρ‹Ρ… вСтвях:

Баланс мощностСй:


Π—Π°Π΄Π°Ρ‡Π° 6. Π’Β Ρ†Π΅ΠΏΠΈ (рис. 6, Π°), ΠΎΠΏΡ€Π΅Π΄Π΅Π»ΠΈΡ‚ΡŒ ΠΌΠ΅Ρ‚ΠΎΠ΄ΠΎΠΌ эквивалСнтных ΠΏΡ€Π΅ΠΎΠ±Ρ€Π°Π·ΠΎΠ²Π°Π½ΠΈΠΉ показания Π°ΠΌΠΏΠ΅Ρ€ΠΌΠ΅Ρ‚Ρ€Π°, Ссли извСстно: R1 = 2 Ом, R2 = 20 Ом, R3 = 30 Ом, R4 = 40 Ом, R5 = 10 Ом, R6 = 20 Ом, E = 48 Π’. Π‘ΠΎΠΏΡ€ΠΎΡ‚ΠΈΠ²Π»Π΅Π½ΠΈΠ΅ Π°ΠΌΠΏΠ΅Ρ€ΠΌΠ΅Ρ‚Ρ€Π° ΠΌΠΎΠΆΠ½ΠΎ ΡΡ‡ΠΈΡ‚Π°Ρ‚ΡŒ Ρ€Π°Π²Π½Ρ‹ΠΌ Π½ΡƒΠ»ΡŽ.

Рис. 6

РСшСниС

Если сопротивлСния R2, R3, R4, R5 Π·Π°ΠΌΠ΅Π½ΠΈΡ‚ΡŒ ΠΎΠ΄Π½ΠΈΠΌ эквивалСнтным сопротивлСниСм RΠ­, Ρ‚ΠΎΒ ΠΈΡΡ…ΠΎΠ΄Π½ΡƒΡŽ схСму ΠΌΠΎΠΆΠ½ΠΎ ΠΏΡ€Π΅Π΄ΡΡ‚Π°Π²ΠΈΡ‚ΡŒ Π²Β ΡƒΠΏΡ€ΠΎΡ‰Π΅Π½Π½ΠΎΠΌ Π²ΠΈΠ΄Π΅ (рис. 6, Π±).

Π’Π΅Π»ΠΈΡ‡ΠΈΠ½Π° эквивалСнтного сопротивлСния:

ΠŸΡ€Π΅ΠΎΠ±Ρ€Π°Π·ΠΎΠ²Π°Π² ΠΏΠ°Ρ€Π°Π»Π»Π΅Π»ΡŒΠ½ΠΎΠ΅ соСдинСниС сопротивлСний RΠ­ ΠΈΒ R6 схСмы (рис. 6, Π±), ΠΏΠΎΠ»ΡƒΡ‡ΠΈΠΌ Π·Π°ΠΌΠΊΠ½ΡƒΡ‚Ρ‹ΠΉ ΠΊΠΎΠ½Ρ‚ΡƒΡ€, для которого ΠΏΠΎΒ Π²Ρ‚ΠΎΡ€ΠΎΠΌΡƒ Π·Π°ΠΊΠΎΠ½Ρƒ ΠšΠΈΡ€Ρ…Π³ΠΎΡ„Π° ΠΌΠΎΠΆΠ½ΠΎ Π·Π°ΠΏΠΈΡΠ°Ρ‚ΡŒ ΡƒΡ€Π°Π²Π½Π΅Π½ΠΈΠ΅:

ΠΎΡ‚ΠΊΡƒΠ΄Π° Ρ‚ΠΎΠΊΒ I1:

НапряТСниС Π½Π°Β Π·Π°ΠΆΠΈΠΌΠ°Ρ… ΠΏΠ°Ρ€Π°Π»Π»Π΅Π»ΡŒΠ½Ρ‹Ρ… Π²Π΅Ρ‚Π²Π΅ΠΉ Uab Π²Ρ‹Ρ€Π°Π·ΠΈΠΌ из уравнСния ΠΏΠΎΒ Π·Π°ΠΊΠΎΠ½Ρƒ Ома для пассивной Π²Π΅Ρ‚Π²ΠΈ, ΠΏΠΎΠ»ΡƒΡ‡Π΅Π½Π½ΠΎΠΉ ΠΏΡ€Π΅ΠΎΠ±Ρ€Π°Π·ΠΎΠ²Π°Π½ΠΈΠ΅ΠΌ RΠ­ ΠΈΒ R6:

Π’ΠΎΠ³Π΄Π° Π°ΠΌΠΏΠ΅Ρ€ΠΌΠ΅Ρ‚Ρ€ ΠΏΠΎΠΊΠ°ΠΆΠ΅Ρ‚ Ρ‚ΠΎΠΊ:


Π—Π°Π΄Π°Ρ‡Π° 7. ΠžΠΏΡ€Π΅Π΄Π΅Π»ΠΈΡ‚ΡŒ Ρ‚ΠΎΠΊΠΈ Π²Π΅Ρ‚Π²Π΅ΠΉ схСмы ΠΌΠ΅Ρ‚ΠΎΠ΄ΠΎΠΌ эквивалСнтных ΠΏΡ€Π΅ΠΎΠ±Ρ€Π°Π·ΠΎΠ²Π°Π½ΠΈΠΉ (рис. 7, Π°), Ссли R1 = R2 = R3 = R4 = 3 Ом, J = 5 А, R5 = 5 Ом.

Рис. 7

РСшСниС

ΠŸΡ€Π΅ΠΎΠ±Ρ€Π°Π·ΡƒΠ΅ΠΌ Β«Ρ‚Ρ€Π΅ΡƒΠ³ΠΎΠ»ΡŒΠ½ΠΈΠΊΒ» сопротивлСний R1, R2, R3 Π²Β ΡΠΊΠ²ΠΈΠ²Π°Π»Π΅Π½Ρ‚Π½ΡƒΡŽ Β«Π·Π²Π΅Π·Π΄ΡƒΒ» R6, R7, R8 (рис. 7, Π±) ΠΈΒ ΠΎΠΏΡ€Π΅Π΄Π΅Π»ΠΈΠΌ Π²Π΅Π»ΠΈΡ‡ΠΈΠ½Ρ‹ ΠΏΠΎΠ»ΡƒΡ‡Π΅Π½Π½Ρ‹Ρ… сопротивлСний:

ΠŸΡ€Π΅ΠΎΠ±Ρ€Π°Π·ΡƒΠ΅ΠΌ ΠΏΠ°Ρ€Π°Π»Π»Π΅Π»ΡŒΠ½ΠΎΠ΅ соСдинСниС Π²Π΅Ρ‚Π²Π΅ΠΉ ΠΌΠ΅ΠΆΠ΄Ρƒ ΡƒΠ·Π»Π°ΠΌΠΈ 4 ΠΈΒ 5

Π’ΠΎΠΊ Π²Β ΠΊΠΎΠ½Ρ‚ΡƒΡ€Π΅, ΠΏΠΎΠ»ΡƒΡ‡Π΅Π½Π½ΠΎΠΌ Π²Β Ρ€Π΅Π·ΡƒΠ»ΡŒΡ‚Π°Ρ‚Π΅ ΠΏΡ€Π΅ΠΎΠ±Ρ€Π°Π·ΠΎΠ²Π°Π½ΠΈΠΉ, считаСм Ρ€Π°Π²Π½Ρ‹ΠΌ Ρ‚ΠΎΠΊΡƒ источника Ρ‚ΠΎΠΊΠ° J, ΠΈΒ Ρ‚ΠΎΠ³Π΄Π° напряТСниС:

И Ρ‚Π΅ΠΏΠ΅Ρ€ΡŒ ΠΌΠΎΠΆΠ½ΠΎ ΠΎΠΏΡ€Π΅Π΄Π΅Π»ΠΈΡ‚ΡŒ Ρ‚ΠΎΠΊΠΈ I4 ΠΈΒ I5:

Π’ΠΎΠ·Π²Ρ€Π°Ρ‰Π°ΡΡΡŒ к исходной схСмС, ΠΎΠΏΡ€Π΅Π΄Π΅Π»ΠΈΠΌ напряТСниС U32 из уравнСния ΠΏΠΎΒ Π²Ρ‚ΠΎΡ€ΠΎΠΌΡƒ Π·Π°ΠΊΠΎΠ½Ρƒ ΠšΠΈΡ€Ρ…Π³ΠΎΡ„Π°:

Π’ΠΎΠ³Π΄Π° Ρ‚ΠΎΠΊΒ Π²Β Π²Π΅Ρ‚Π²ΠΈ с сопротивлСниСм R3 опрСдСлится:

Π’Π΅Π»ΠΈΡ‡ΠΈΠ½Ρ‹ ΠΎΡΡ‚Π°Π²ΡˆΠΈΡ…ΡΡ нСизвСстными Ρ‚ΠΎΠΊΠΎΠ² ΠΌΠΎΠΆΠ½ΠΎ ΠΎΠΏΡ€Π΅Π΄Π΅Π»ΠΈΡ‚ΡŒ ΠΈΠ·Β ΡƒΡ€Π°Π²Π½Π΅Π½ΠΈΠΉ ΠΏΠΎΒ ΠΏΠ΅Ρ€Π²ΠΎΠΌΡƒ Π·Π°ΠΊΠΎΠ½Ρƒ ΠšΠΈΡ€Ρ…Π³ΠΎΡ„Π° для узлов 3 ΠΈΒ 1:


ЭлСктронная вСрсия ΡΡ‚Π°Ρ‚ΡŒΠΈ РасчСт элСктричСских Ρ†Π΅ΠΏΠ΅ΠΉ постоянного Ρ‚ΠΎΠΊΠ° ΠΌΠ΅Ρ‚ΠΎΠ΄ΠΎΠΌ эквивалСнтных ΠΏΡ€Π΅ΠΎΠ±Ρ€Π°Π·ΠΎΠ²Π°Π½ΠΈΠΉ

ΠŸΡ€ΠΈΠΌΠ΅Ρ€Ρ‹ Ρ€Π΅ΡˆΠ΅Π½ΠΈΡ Π·Π°Π΄Π°Ρ‡ РасчСт элСктричСских Ρ†Π΅ΠΏΠ΅ΠΉ постоянного Ρ‚ΠΎΠΊΠ° ΠΌΠ΅Ρ‚ΠΎΠ΄ΠΎΠΌ эквивалСнтных ΠΏΡ€Π΅ΠΎΠ±Ρ€Π°Π·ΠΎΠ²Π°Π½ΠΈΠΉ

РасчСт элСктричСских Ρ†Π΅ΠΏΠ΅ΠΉ постоянного Ρ‚ΠΎΠΊΠ° ΠΌΠ΅Ρ‚ΠΎΠ΄ΠΎΠΌ эквивалСнтных ΠΏΡ€Π΅ΠΎΠ±Ρ€Π°Π·ΠΎΠ²Π°Π½ΠΈΠΉ


ΠœΠ΅Ρ‚ΠΎΠ΄ эквивалСнтных ΠΏΡ€Π΅ΠΎΠ±Ρ€Π°Π·ΠΎΠ²Π°Π½ΠΈΠΉΒ 

02.09.2011, 296027 просмотров.

pravilo_Kirkhgofa — прСзСнтация ΠΎΠ½Π»Π°ΠΉΠ½

1. РасчСт слоТных Ρ†Π΅ΠΏΠ΅ΠΉ постоянного Ρ‚ΠΎΠΊΠ° ΠΏΠΎ I ΠΈ II Π·Π°ΠΊΠΎΠ½Π°ΠΌ ΠšΠΈΡ€Ρ…Π³ΠΎΡ„Π°

2. ΠŸΠΎΠ²Ρ‚ΠΎΡ€Π΅Π½ΠΈΠ΅ ΠΈΠ·ΡƒΡ‡Π΅Π½Π½ΠΎΠ³ΠΎ ΠΌΠ°Ρ‚Π΅Ρ€ΠΈΠ°Π»Π°

3. Π”Π°ΠΉΡ‚Π΅ ΠΎΠΏΡ€Π΅Π΄Π΅Π»Π΅Π½ΠΈΠ΅ слоТной элСктричСской Ρ†Π΅ΠΏΠΈ

β€’ Π‘Π»ΠΎΠΆΠ½Ρ‹ΠΌΠΈ Π½Π°Π·Ρ‹Π²Π°ΡŽΡ‚ΡΡ Ρ€Π°Π·Π²Π΅Ρ‚Π²Π»Π΅Π½Π½Ρ‹Π΅
элСктричСскиС Ρ†Π΅ΠΏΠΈ со ΠΌΠ½ΠΎΠ³ΠΈΠΌΠΈ источниками
энСргии.

4. Π”Π°ΠΉΡ‚Π΅ Ρ„ΠΎΡ€ΠΌΡƒΠ»ΠΈΡ€ΠΎΠ²ΠΊΡƒ I Π·Π°ΠΊΠΎΠ½Ρƒ ΠšΠΈΡ€Ρ…Π³ΠΎΡ„Π°

β€’ АлгСбраичСская сумма Ρ‚ΠΎΠΊΠΎΠ² Π² ΠΊΠ°ΠΆΠ΄ΠΎΠΌ
ΡƒΠ·Π»Π΅ любой Ρ†Π΅ΠΏΠΈ Ρ€Π°Π²Π½Π° Π½ΡƒΠ»ΡŽ. ΠŸΡ€ΠΈ этом
Π½Π°ΠΏΡ€Π°Π²Π»Π΅Π½Π½Ρ‹ΠΉ ΠΊ ΡƒΠ·Π»Ρƒ Ρ‚ΠΎΠΊ принято ΡΡ‡ΠΈΡ‚Π°Ρ‚ΡŒ
ΠΏΠΎΠ»ΠΎΠΆΠΈΡ‚Π΅Π»ΡŒΠ½Ρ‹ΠΌ, Π° Π½Π°ΠΏΡ€Π°Π²Π»Π΅Π½Π½Ρ‹ΠΉ ΠΎΡ‚ ΡƒΠ·Π»Π°
β€” ΠΎΡ‚Ρ€ΠΈΡ†Π°Ρ‚Π΅Π»ΡŒΠ½Ρ‹ΠΌ.
n
I
i 1
i
0

5. Π”Π°ΠΉΡ‚Π΅ Ρ„ΠΎΡ€ΠΌΡƒΠ»ΠΈΡ€ΠΎΠ²ΠΊΡƒ II Π·Π°ΠΊΠΎΠ½Ρƒ ΠšΠΈΡ€Ρ…Π³ΠΎΡ„Π°

β€’ АлгСбраичСская сумма ΠΏΠ°Π΄Π΅Π½ΠΈΠΉ
напряТСний Π½Π° всСх вСтвях,
ΠΏΡ€ΠΈΠ½Π°Π΄Π»Π΅ΠΆΠ°Ρ‰ΠΈΡ… Π»ΡŽΠ±ΠΎΠΌΡƒ Π·Π°ΠΌΠΊΠ½ΡƒΡ‚ΠΎΠΌΡƒ
ΠΊΠΎΠ½Ρ‚ΡƒΡ€Ρƒ Ρ†Π΅ΠΏΠΈ, Ρ€Π°Π²Π½Π° алгСбраичСской
суммС Π­Π”Π‘ Π²Π΅Ρ‚Π²Π΅ΠΉ этого ΠΊΠΎΠ½Ρ‚ΡƒΡ€Π°.
n
n
U E
i 1
i
i 1
i

6. РСшСниС Π·Π°Π΄Π°Ρ‡ΠΈ

7. ΠžΠΏΡ€Π΅Π΄Π΅Π»ΠΈΡ‚ΡŒ Ρ‚ΠΎΠΊΠΈ Π²ΠΎ всСх вСтвях Π΄Π°Π½Π½ΠΎΠΉ элСктричСской Ρ†Π΅ΠΏΠΈ

8. НайдСм ΠΎΠ±Ρ‰Π΅Π΅ количСство ΡƒΡ€Π°Π²Π½Π΅Π½ΠΈΠΉ

β€’ Для опрСдСлСния Ρ‚ΠΎΠΊΠΎΠ² Π²ΠΎ всСх вСтвях
Π΄Π°Π½Π½ΠΎΠΉ элСктричСской Ρ†Π΅ΠΏΠΈ,
Π½Π΅ΠΎΠ±Ρ…ΠΎΠ΄ΠΈΠΌΠΎ ΡΠΎΡΡ‚Π°Π²ΠΈΡ‚ΡŒ систСму
ΡƒΡ€Π°Π²Π½Π΅Π½ΠΈΠΉ ΠΏΠΎ Π·Π°ΠΊΠΎΠ½Π°ΠΌ ΠšΠΈΡ€Ρ…Π³ΠΎΡ„Π°.
β€’ ΠžΠ±Ρ‰Π΅Π΅ число ΡƒΡ€Π°Π²Π½Π΅Π½ΠΈΠΉ Π² систСмС Π΄ΠΎΠ»ΠΆΠ½ΠΎ
ΡΠΎΠΎΡ‚Π²Π΅Ρ‚ΡΡ‚Π²ΠΎΠ²Π°Ρ‚ΡŒ числу нСизвСстных
Ρ‚ΠΎΠΊΠΎΠ², Ρ‚. Π΅. числу Π²Π΅Ρ‚Π²Π΅ΠΉ.

9. ΠŸΠΎΡΡ‡ΠΈΡ‚Π°Π΅ΠΌ количСство Π²Π΅Ρ‚Π²Π΅ΠΉ Π² нашСй элСктричСской Ρ†Π΅ΠΏΠΈ

R1
Π˜Ρ‚ΠΎΠ³ΠΎ
5 Π²Π΅Ρ‚Π²Π΅ΠΉ, Ρ‚.Π΅.
5 ΡƒΡ€Π°Π²Π½Π΅Π½ΠΈΠΉ
R2
I2
I4
Π•1
R4
Π•2
R5
I5
I1
R3
I3
Π•3

10. НайдСм количСство ΡƒΡ€Π°Π²Π½Π΅Π½ΠΈΠΉ ΠΏΠΎ I Π·Π°ΠΊΠΎΠ½Ρƒ ΠšΠΈΡ€Ρ…Π³ΠΎΡ„Π°

β€’ По ΠΏΠ΅Ρ€Π²ΠΎΠΌΡƒ Π·Π°ΠΊΠΎΠ½Ρƒ ΠšΠΈΡ€Ρ…Π³ΠΎΡ„Π° составляСтся
число ΡƒΡ€Π°Π²Π½Π΅Π½ΠΈΠΉ, Π½Π° Π΅Π΄ΠΈΠ½ΠΈΡ†Ρƒ мСньшСС
числа ΡƒΠ·Π»ΠΎΠ² Ρ†Π΅ΠΏΠΈ, Ρ‚.ΠΊ. ΡƒΡ€Π°Π²Π½Π΅Π½ΠΈΠ΅ для
послСднСго ΡƒΠ·Π»Π° Π΅ΡΡ‚ΡŒ слСдствиС всСх
ΠΏΡ€Π΅Π΄Ρ‹Π΄ΡƒΡ‰ΠΈΡ… ΡƒΡ€Π°Π²Π½Π΅Π½ΠΈΠΉ ΠΈ Π½Π΅ Π΄Π°Π΅Ρ‚
Π½ΠΈΡ‡Π΅Π³ΠΎ Π½ΠΎΠ²ΠΎΠ³ΠΎ для расчСта.

11. ΠŸΠΎΡΡ‡ΠΈΡ‚Π°Π΅ΠΌ количСство ΡƒΠ·Π»ΠΎΠ² элСктричСской Ρ†Π΅ΠΏΠΈ

2
R1
Π˜Ρ‚ΠΎΠ³ΠΎ
3 ΡƒΠ·Π»Π°, Ρ‚.Π΅.
2 уравнСния
R2
I2
I4
Π•1
R4
Π•2
R5
I5
I1
R3
1
I3
Π•3
3

12. Π—Π°Π΄Π°Π΄ΠΈΠΌ Π½Π°ΠΏΡ€Π°Π²Π»Π΅Π½ΠΈΠ΅ Ρ‚ΠΎΠΊΠΎΠ² Π²ΠΎ всСх вСтвях Ρ†Π΅ΠΏΠΈ

2
R1
R2
I2
I4
Π•1
R4
Π•2
R5
I5
I1
R3
1
I3
Π•3
3

13. Боставим уравнСния ΠΏΠΎ I Π·Π°ΠΊΠΎΠ½Ρƒ ΠšΠΈΡ€Ρ…Π³ΠΎΡ„Π°

β€’ Π’ΠΎΠΊΠΈ, подходящиС ΠΊ ΡƒΠ·Π»Ρƒ, Π±ΡƒΠ΄Π΅ΠΌ ΡΡ‡ΠΈΡ‚Π°Ρ‚ΡŒ
ΠΏΠΎΠ»ΠΎΠΆΠΈΡ‚Π΅Π»ΡŒΠ½Ρ‹ΠΌΠΈ ΠΈ Π±Ρ€Π°Ρ‚ΡŒ со Π·Π½Π°ΠΊΠΎΠΌ (+),
Π° Ρ‚ΠΎΠΊΠΈ, отходящиС ΠΎΡ‚ ΡƒΠ·Π»Π° – (–).
2
R1
R2
I2
I4
Π•1
R4
Π•2
R5
I5
I1
R3
1
I3
Π•3
Для ΡƒΠ·Π»Π° β„– 1: –I1 – I3 – I4 = 0
Для ΡƒΠ·Π»Π° β„– 2: I1 – I2 + I4 + I5 = 0
3

15. НайдСм количСство ΡƒΡ€Π°Π²Π½Π΅Π½ΠΈΠΉ ΠΏΠΎ II Π·Π°ΠΊΠΎΠ½Ρƒ ΠšΠΈΡ€Ρ…Π³ΠΎΡ„Π°

β€’ По Π²Ρ‚ΠΎΡ€ΠΎΠΌΡƒ Π·Π°ΠΊΠΎΠ½Ρƒ ΠšΠΈΡ€Ρ…Π³ΠΎΡ„Π°
ΡΠΎΡΡ‚Π°Π²Π»ΡΡŽΡ‚ΡΡ всС Π½Π΅Π΄ΠΎΡΡ‚Π°ΡŽΡ‰ΠΈΠ΅
уравнСния для Π»ΡŽΠ±Ρ‹Ρ… ΠΏΡ€ΠΎΠΈΠ·Π²ΠΎΠ»ΡŒΠ½ΠΎ
Π²Ρ‹Π±Ρ€Π°Π½Π½Ρ‹Ρ… ΠΊΠΎΠ½Ρ‚ΡƒΡ€ΠΎΠ² Ρ†Π΅ΠΏΠΈ.
β€’ ΠŸΠΎΡΡ‡ΠΈΡ‚Π°Π΅ΠΌ количСство Π½Π΅Π΄ΠΎΡΡ‚Π°ΡŽΡ‰ΠΈΡ…
ΡƒΡ€Π°Π²Π½Π΅Π½ΠΈΠΉ: 5 – 2 = 3.

16. Π—Π°Π΄Π°Π΄ΠΈΠΌ Π½Π°ΠΏΡ€Π°Π²Π»Π΅Π½ΠΈΠ΅ ΠΎΠ±Ρ…ΠΎΠ΄Π° Π²Ρ‹Π±Ρ€Π°Π½Π½Ρ‹Ρ… ΠΊΠΎΠ½Ρ‚ΡƒΡ€ΠΎΠ²

2
R1
R2
I2
I
Π•1
III
I4
R4
Π•2
R5
II
I5
I1
R3
1
I3
Π•3
3

17. Боставим уравнСния ΠΏΠΎ II Π·Π°ΠΊΠΎΠ½Ρƒ ΠšΠΈΡ€Ρ…Π³ΠΎΡ„Π°

β€’ ΠŸΡ€ΠΈ составлСнии ΡƒΡ€Π°Π²Π½Π΅Π½ΠΈΠΉ Π­Π”Π‘ ΠΈ
Ρ‚ΠΎΠΊΠΈ, ΡΠΎΠ²ΠΏΠ°Π΄Π°ΡŽΡ‰ΠΈΠ΅ с Π²Ρ‹Π±Ρ€Π°Π½Π½Ρ‹ΠΌ
Π½Π°ΠΏΡ€Π°Π²Π»Π΅Π½ΠΈΠ΅ΠΌ ΠΎΠ±Ρ…ΠΎΠ΄Π° ΠΊΠΎΠ½Ρ‚ΡƒΡ€Π° Π±ΡƒΠ΄Π΅ΠΌ
Π±Ρ€Π°Ρ‚ΡŒ со Π·Π½Π°ΠΊΠΎΠΌ (+), Π° Π½Π΅ΡΠΎΠ²ΠΏΠ°Π΄Π°ΡŽΡ‰ΠΈΠ΅ –
со Π·Π½Π°ΠΊΠΎΠΌ (–).
2
R1
R2
I2
I
Π•1
III
I4
R4
Π•2
R5
II
I5
Π•3
I1
R3
1
I3
3
Для I ΠΊΠΎΠ½Ρ‚ΡƒΡ€Π°: I1R1 – I4R4 = Π•1
Для II ΠΊΠΎΠ½Ρ‚ΡƒΡ€Π°: I4R4 – I5R5 – I3R3 = Π•3
Для III ΠΊΠΎΠ½Ρ‚ΡƒΡ€Π°: I2R2 + I5R5 = –Е2

19. ΠŸΠΎΠ»ΡƒΡ‡ΠΈΠ»Π°ΡΡŒ систСма ΡƒΡ€Π°Π²Π½Π΅Π½ΠΈΠΉ

–I1 – I3 – I4 = 0
I1 – I2 + I4 + I5 = 0
I1R1 – I4R4 = E1
I4R4 – I5R5 – I3R3 = E3
I2R2 + I4R4 = –E2
РСшив Π΅Π΅, ΠΏΠΎΠ»ΡƒΡ‡Π°Π΅ΠΌ ΠΏΡΡ‚ΡŒ Π·Π½Π°Ρ‡Π΅Π½ΠΈΠΉ
Ρ‚ΠΎΠΊΠΎΠ².

Π—Π°ΠΊΠΎΠ½Ρ‹ ΠšΠΈΡ€Ρ…Π³ΠΎΡ„Π° для элСктричСской Ρ†Π΅ΠΏΠΈ

ΠŸΡ€Π°Π²ΠΈΠ»ΡŒΠ½Π΅Π΅ Π±Ρ‹Π»ΠΎ Π±Ρ‹ Π³ΠΎΠ²ΠΎΡ€ΠΈΡ‚ΡŒ ΠΏΡ€Π°Π²ΠΈΠ»Π° ΠšΠΈΡ€Ρ…Π³ΠΎΡ„Π° для расчСтов слоТных элСктричСских Ρ†Π΅ΠΏΠ΅ΠΉ постоянного Ρ‚ΠΎΠΊΠ°. ЭлСктричСская Ρ†Π΅ΠΏΡŒ Π½Π° ΠΏΡ€Π°ΠΊΡ‚ΠΈΠΊΠ΅ ΠΌΠΎΠΆΠ΅Ρ‚ ΡΠΎΡΡ‚ΠΎΡΡ‚ΡŒ ΠΈΠ· Π½Π΅ΡΠΊΠΎΠ»ΡŒΠΊΠΈΡ… рСзисторов ΠΈ источников Ρ‚ΠΎΠΊΠ°. Π’Π°ΠΊΠΈΠ΅ Ρ†Π΅ΠΏΠΈ Π½Π°Π·Ρ‹Π²Π°ΡŽΡ‚ Ρ€Π°Π·Π²Π΅Ρ‚Π²Π»Π΅Π½Π½Ρ‹ΠΌΠΈ. УравнСния ΠΏΠΎΠ·Π²ΠΎΠ»ΡΡŽΡ‰ΠΈΠ΅ провСсти расчСты, Π½Π°ΠΏΡ€ΠΈΠΌΠ΅Ρ€, сил Ρ‚ΠΎΠΊΠΎΠ², Ρ‚Π΅ΠΊΡƒΡ‰ΠΈΡ… Π² сопротивлСниях, Π² Π»ΡŽΠ±Ρ‹Ρ… сСтях ΠΌΠΎΠΆΠ½ΠΎ ΡΠΎΡΡ‚Π°Π²ΠΈΡ‚ΡŒ, воспользовавшись Π·Π°ΠΊΠΎΠ½ΠΎΠΌ Ома ΠΈ Π·Π°ΠΊΠΎΠ½ΠΎΠΌ сохранСния заряда. ΠŸΡ€Π°Π²ΠΈΠ»Π° ΠšΠΈΡ€Ρ…Π³ΠΎΡ„Π° ΡΠ²Π»ΡΡŽΡ‚ΡΡ слСдствиями Π²Ρ‹ΡˆΠ΅Π½Π°Π·Π²Π°Π½Π½Ρ‹Ρ… Π·Π°ΠΊΠΎΠ½ΠΎΠ² ΠΈ ΠΏΡ€ΠΈΠ½Ρ†ΠΈΠΏΠΈΠ°Π»ΡŒΠ½ΠΎ Π½ΠΎΠ²ΠΎΠ³ΠΎ Π½ΠΈ Ρ‡Π΅Π³ΠΎ Π½Π΅ привносят, ΠΎΠ΄Π½Π°ΠΊΠΎ, с ΠΈΡ… ΠΏΠΎΠΌΠΎΡ‰ΡŒΡŽ ΠΌΠΎΠΆΠ½ΠΎ ΡƒΠΏΡ€ΠΎΡΡ‚ΠΈΡ‚ΡŒ ΠΏΡ€ΠΎΡ†Π΅Π΄ΡƒΡ€Ρƒ написания Π½Π΅ΠΎΠ±Ρ…ΠΎΠ΄ΠΈΠΌΡ‹Ρ… ΡƒΡ€Π°Π²Π½Π΅Π½ΠΈΠΉ. БущСствуСт Π΄Π²Π° ΠΏΡ€Π°Π²ΠΈΠ»Π° ΠšΠΈΡ€Ρ…Π³ΠΎΡ„Π° для элСктричСских Ρ†Π΅ΠΏΠ΅ΠΉ постоянного Ρ‚ΠΎΠΊΠ°. Одно ΠΏΡ€Π°Π²ΠΈΠ»ΠΎ Π½Π°Π·Ρ‹Π²Π°ΡŽΡ‚ ΠΏΡ€Π°Π²ΠΈΠ»ΠΎΠΌ ΡƒΠ·Π»ΠΎΠ², Ρ‚Π°ΠΊ ΠΊΠ°ΠΊ ΠΎΠ½ΠΎ связываСт Π² ΠΎΠ΄Π½ΠΎ ΡƒΡ€Π°Π²Π½Π΅Π½ΠΈΠ΅ Ρ‚ΠΎΠΊΠΈ, сходящиСся Π² ΡƒΠ·Π»Π΅. Π’Ρ‚ΠΎΡ€ΠΎΠ΅ ΠΏΡ€Π°Π²ΠΈΠ»ΠΎ касаСтся Π·Π°ΠΌΠΊΠ½ΡƒΡ‚Ρ‹Ρ… ΠΊΠΎΠ½Ρ‚ΡƒΡ€ΠΎΠ², ΠΊΠΎΡ‚ΠΎΡ€Ρ‹Π΅ ΠΌΠΎΠΆΠ½ΠΎ Π²Ρ‹Π΄Π΅Π»ΠΈΡ‚ΡŒ Π² слоТной Ρ†Π΅ΠΏΠΈ.

ΠŸΠ΅Ρ€Π²Ρ‹ΠΉ Π·Π°ΠΊΠΎΠ½ (ΠΏΡ€Π°Π²ΠΈΠ»ΠΎ) ΠšΠΈΡ€Ρ…Π³ΠΎΡ„Π°

Π’ элСктричСской Ρ†Π΅ΠΏΠΈ Π² ΠΎΠ΄Π½ΠΎΠΉ Ρ‚ΠΎΡ‡ΠΊΠ΅ ΠΌΠΎΠ³ΡƒΡ‚ ΡΡ…ΠΎΠ΄ΠΈΡ‚ΡŒΡΡ Π±ΠΎΠ»Π΅Π΅ Π΄Π²ΡƒΡ… ΠΏΡ€ΠΎΠ²ΠΎΠ΄Π½ΠΈΠΊΠΎΠ² с Ρ‚ΠΎΠΊΠ°ΠΌΠΈ, Ρ‚ΠΎΠ³Π΄Π° Ρ‚Π°ΠΊΡƒΡŽ Ρ‚ΠΎΡ‡ΠΊΡƒ Ρ†Π΅ΠΏΠΈ Π½Π°Π·Ρ‹Π²Π°ΡŽΡ‚ ΡƒΠ·Π»ΠΎΠΌ (Ρ€Π°Π·Π²Π΅Ρ‚Π²Π»Π΅Π½ΠΈΠ΅ΠΌ). Учитывая, Ρ‡Ρ‚ΠΎ сила Ρ‚ΠΎΠΊΠ° алгСбраичСская Π²Π΅Π»ΠΈΡ‡ΠΈΠ½Π° для любого ΡƒΠ·Π»Π°:

Β  Β 

Π³Π΄Π΅ N – число Ρ‚ΠΎΠΊΠΎΠ², ΠΊΠΎΡ‚ΠΎΡ€Ρ‹Π΅ сходятся Π² ΡƒΠ·Π»Π΅. Π’Ρ‹Ρ€Π°ΠΆΠ΅Π½ΠΈΠ΅ (1) Π½Π°Π·Ρ‹Π²Π°ΡŽΡ‚ ΠΏΠ΅Ρ€Π²Ρ‹ΠΌ ΠΏΡ€Π°Π²ΠΈΠ»ΠΎΠΌ ΠšΠΈΡ€Ρ…Π³ΠΎΡ„Π° (ΠΏΡ€Π°Π²ΠΈΠ»ΠΎ ΡƒΠ·Π»ΠΎΠ²): сумма Ρ‚ΠΎΠΊΠΎΠ², Ρ‚Π΅ΠΊΡƒΡ‰ΠΈΡ… Ρ‡Π΅Ρ€Π΅Π· сопротивлСния Π² Ρ†Π΅ΠΏΠΈ постоянного Ρ‚ΠΎΠΊΠ°, с ΡƒΡ‡Π΅Ρ‚ΠΎΠΌ ΠΈΡ… Π·Π½Π°ΠΊΠ°, сходящихся Π² ΡƒΠ·Π»Π΅, Ρ€Π°Π²Π½Π° Π½ΡƒΠ»ΡŽ.

Π—Π½Π°ΠΊ Ρƒ Ρ‚ΠΎΠΊΠ° (плюс ΠΈΠ»ΠΈ минус) Π²Ρ‹Π±ΠΈΡ€Π°ΡŽΡ‚ ΠΏΡ€ΠΎΠΈΠ·Π²ΠΎΠ»ΡŒΠ½ΠΎ, Π½ΠΎ ΠΏΡ€ΠΈ этом слСдуСт ΡΡ‡ΠΈΡ‚Π°Ρ‚ΡŒ, Ρ‡Ρ‚ΠΎ всС входящиС Π² ΡƒΠ·Π΅Π» Ρ‚ΠΎΠΊΠΈ ΠΈΠΌΠ΅ΡŽΡ‚ ΠΎΠ΄ΠΈΠ½Π°ΠΊΠΎΠ²Ρ‹Π΅ Π·Π½Π°ΠΊΠΈ, Π° всС исходящиС ΠΈΠ· ΡƒΠ·Π»Π° Ρ‚ΠΎΠΊΠΈ ΠΈΠΌΠ΅ΡŽΡ‚ ΠΏΡ€ΠΎΡ‚ΠΈΠ²ΠΎΠΏΠΎΠ»ΠΎΠΆΠ½Ρ‹Π΅ входящим, Π·Π½Π°ΠΊΠΈ. Допустим, всС входящиС Ρ‚ΠΎΠΊΠΈ ΠΌΡ‹ ΠΏΡ€ΠΈΠΌΠ΅ΠΌ Π·Π° ΠΏΠΎΠ»ΠΎΠΆΠΈΡ‚Π΅Π»ΡŒΠ½Ρ‹Π΅, Ρ‚ΠΎΠ³Π΄Π° всС исходящиС ΠΈΡ… этого ΡƒΠ·Π»Π° Ρ‚ΠΎΠΊΠΈ Π±ΡƒΠ΄ΡƒΡ‚ ΠΎΡ‚Ρ€ΠΈΡ†Π°Ρ‚Π΅Π»ΡŒΠ½Ρ‹ΠΌΠΈ.

ΠŸΠ΅Ρ€Π²ΠΎΠ΅ ΠΏΡ€Π°Π²ΠΈΠ»ΠΎ ΠšΠΈΡ€Ρ…Π³ΠΎΡ„Π° Π΄Π°Π΅Ρ‚ Π²ΠΎΠ·ΠΌΠΎΠΆΠ½ΠΎΡΡ‚ΡŒ ΡΠΎΡΡ‚Π°Π²ΠΈΡ‚ΡŒ нСзависимоС ΡƒΡ€Π°Π²Π½Π΅Π½ΠΈΠ΅, Ссли Π² Ρ†Π΅ΠΏΠΈ k ΡƒΠ·Π»ΠΎΠ².

ΠŸΠ΅Ρ€Π²ΠΎΠ΅ ΠΏΡ€Π°Π²ΠΈΠ»ΠΎ ΠšΠΈΡ€Ρ…Π³ΠΎΡ„Π° являСтся слСдствиСм Π·Π°ΠΊΠΎΠ½Π° сохранСния заряда.

Π’Ρ‚ΠΎΡ€ΠΎΠΉ Π·Π°ΠΊΠΎΠ½ (ΠΏΡ€Π°Π²ΠΈΠ»ΠΎ) ΠšΠΈΡ€Ρ…Π³ΠΎΡ„Π°

Π’ΠΎ Π²Ρ‚ΠΎΡ€ΠΎΠΌ ΠΏΡ€Π°Π²ΠΈΠ»Π΅ ΠšΠΈΡ€Ρ…Π³ΠΎΡ„Π° Ρ€Π°ΡΡΠΌΠ°Ρ‚Ρ€ΠΈΠ²Π°ΡŽΡ‚ΡΡ Π·Π°ΠΌΠΊΠ½ΡƒΡ‚Ρ‹Π΅ ΠΊΠΎΠ½Ρ‚ΡƒΡ€Ρ‹, поэтому ΠΎΠ½ΠΎ носит Π½Π°Π·Π²Π°Π½ΠΈΠ΅ ΠΏΡ€Π°Π²ΠΈΠ»Π° ΠΊΠΎΠ½Ρ‚ΡƒΡ€ΠΎΠ²: Π‘ΡƒΠΌΠΌΡ‹ ΠΏΡ€ΠΎΠΈΠ·Π²Π΅Π΄Π΅Π½ΠΈΠΉ алгСбраичСских Π²Π΅Π»ΠΈΡ‡ΠΈΠ½ сил Ρ‚ΠΎΠΊΠ° Π½Π° внСшниС ΠΈ Π²Π½ΡƒΡ‚Ρ€Π΅Π½Π½ΠΈΠ΅ сопротивлСния всСх участков Π·Π°ΠΌΠΊΠ½ΡƒΡ‚ΠΎΠ³ΠΎ ΠΊΠΎΠ½Ρ‚ΡƒΡ€Π° Ρ€Π°Π²Π½Ρ‹ алгСбраичСской суммС Π²Π΅Π»ΠΈΡ‡ΠΈΠ½ сторонних элСктродвиТущих сил (Π­Π”Π‘) (), ΠΊΠΎΡ‚ΠΎΡ€Ρ‹Π΅ входят Π² рассматриваСмый ΠΊΠΎΠ½Ρ‚ΡƒΡ€. Π’ матСматичСском Π²ΠΈΠ΄Π΅ Π²Ρ‚ΠΎΡ€ΠΎΠΉ Π·Π°ΠΊΠΎΠ½ ΠšΠΈΡ€Ρ…Π³ΠΎΡ„Π° Π·Π°ΠΏΠΈΡΡ‹Π²Π°ΡŽΡ‚ ΠΊΠ°ΠΊ:

Β  Β 

Π’Π΅Π»ΠΈΡ‡ΠΈΠ½Ρ‹ Π½Π°Π·Ρ‹Π²Π°ΡŽΡ‚ падСниями напряТСния. Π”ΠΎ примСнСния Π²Ρ‚ΠΎΡ€ΠΎΠ³ΠΎ Π·Π°ΠΊΠΎΠ½Π° ΠšΠΈΡ€Ρ…Π³ΠΎΡ„Π° Π²Ρ‹Π±ΠΈΡ€Π°ΡŽΡ‚ ΠΏΠΎΠ»ΠΎΠΆΠΈΡ‚Π΅Π»ΡŒΠ½ΠΎΠ΅ Π½Π°ΠΏΡ€Π°Π²Π»Π΅Π½ΠΈΠ΅ ΠΎΠ±Ρ…ΠΎΠ΄Π° ΠΊΠΎΠ½Ρ‚ΡƒΡ€Π°. Π­Ρ‚ΠΎ Π½Π°ΠΏΡ€Π°Π²Π»Π΅Π½ΠΈΠ΅ бСрСтся ΠΏΡ€ΠΎΠΈΠ·Π²ΠΎΠ»ΡŒΠ½ΠΎ, Π»ΠΈΠ±ΠΎ ΠΏΠΎ часовой стрСлкС, Π»ΠΈΠ±ΠΎ ΠΏΡ€ΠΎΡ‚ΠΈΠ² Π½Π΅Π΅. Если Π½Π°ΠΏΡ€Π°Π²Π»Π΅Π½ΠΈΠ΅ ΠΎΠ±Ρ…ΠΎΠ΄Π° совпадаСт с Π½Π°ΠΏΡ€Π°Π²Π»Π΅Π½ΠΈΠ΅ΠΌ тСчСния Ρ‚ΠΎΠΊΠ° Π² рассматриваСмом элСмСнтС ΠΊΠΎΠ½Ρ‚ΡƒΡ€Π°, Ρ‚ΠΎ ΠΏΠ°Π΄Π΅Π½ΠΈΠ΅ напряТСния Π² Ρ„ΠΎΡ€ΠΌΡƒΠ»Ρƒ Π²Ρ‚ΠΎΡ€ΠΎΠ³ΠΎ ΠΏΡ€Π°Π²ΠΈΠ»Π° для Π΄Π°Π½Π½ΠΎΠ³ΠΎ ΠΊΠΎΠ½Ρ‚ΡƒΡ€Π° Π²Ρ…ΠΎΠ΄ΠΈΡ‚ со Π·Π½Π°ΠΊΠΎΠΌ плюс. Π­Π”Π‘ ΡΡ‡ΠΈΡ‚Π°ΡŽΡ‚ ΠΏΠΎΠ»ΠΎΠΆΠΈΡ‚Π΅Π»ΡŒΠ½ΠΎΠΉ, Ссли ΠΏΡ€ΠΈ Π΄Π²ΠΈΠΆΠ΅Π½ΠΈΠΈ ΠΏΠΎ ΠΊΠΎΠ½Ρ‚ΡƒΡ€Ρƒ (Π² ΠΈΠ·Π±Ρ€Π°Π½Π½ΠΎΠΌ Π½Π°ΠΏΡ€Π°Π²Π»Π΅Π½ΠΈΠΈ) ΠΏΠ΅Ρ€Π²Ρ‹ΠΌ встрСчаСтся ΠΎΡ‚Ρ€ΠΈΡ†Π°Ρ‚Π΅Π»ΡŒΠ½Ρ‹ΠΉ полюс источника. Π‘ΠΎΠ»Π΅Π΅ ΠΏΡ€Π°Π²ΠΈΠ»ΡŒΠ½ΠΎ Π±Ρ‹Π»ΠΎ Π±Ρ‹ ΡΠΊΠ°Π·Π°Ρ‚ΡŒ, Ρ‡Ρ‚ΠΎ Π­Π”Π‘ ΡΡ‡ΠΈΡ‚Π°ΡŽΡ‚ ΠΏΠΎΠ»ΠΎΠΆΠΈΡ‚Π΅Π»ΡŒΠ½ΠΎΠΉ, Ссли Ρ€Π°Π±ΠΎΡ‚Π° сторонних сил ΠΏΠΎ ΠΏΠ΅Ρ€Π΅ΠΌΠ΅Ρ‰Π΅Π½ΠΈΡŽ Π΅Π΄ΠΈΠ½ΠΈΡ‡Π½ΠΎΠ³ΠΎ ΠΏΠΎΠ»ΠΎΠΆΠΈΡ‚Π΅Π»ΡŒΠ½ΠΎΠ³ΠΎ заряда Π½Π° рассматриваСмом участкС Ρ†Π΅ΠΏΠΈ Π² Π·Π°Π΄Π°Π½Π½ΠΎΠΌ Π½Π°ΠΏΡ€Π°Π²Π»Π΅Π½ΠΈΠΈ ΠΎΠ±Ρ…ΠΎΠ΄Π° ΠΊΠΎΠ½Ρ‚ΡƒΡ€Π° являСтся ΠΏΠΎΠ»ΠΎΠΆΠΈΡ‚Π΅Π»ΡŒΠ½ΠΎΠΉ Π²Π΅Π»ΠΈΡ‡ΠΈΠ½ΠΎΠΉ.

Π’Ρ‚ΠΎΡ€ΠΎΠ΅ ΠΏΡ€Π°Π²ΠΈΠ»ΠΎ ΠšΠΈΡ€Ρ…Π³ΠΎΡ„Π° являСтся слСдствиСм Π·Π°ΠΊΠΎΠ½Π° Ома.

ΠŸΡ€ΠΈΠΌΠ΅Ρ€Ρ‹ Ρ€Π΅ΡˆΠ΅Π½ΠΈΡ Π·Π°Π΄Π°Ρ‡

Π—Π°ΠΊΠΎΠ½ ΠšΠΈΡ€Ρ…Π³ΠΎΡ„Π° для слоТных схСм | ΠžΠ Π•Π›

Π—Π°ΠΊΠΎΠ½

Ома — ваш Π·ΠΎΠ»ΠΎΡ‚ΠΎΠΉ Π±ΠΈΠ»Π΅Ρ‚ для расчСта напряТСния, Ρ‚ΠΎΠΊΠ° ΠΈΠ»ΠΈ сопротивлСния Π² простой ΠΏΠΎΡΠ»Π΅Π΄ΠΎΠ²Π°Ρ‚Π΅Π»ΡŒΠ½ΠΎΠΉ ΠΈΠ»ΠΈ ΠΏΠ°Ρ€Π°Π»Π»Π΅Π»ΡŒΠ½ΠΎΠΉ Ρ†Π΅ΠΏΠΈ, Π½ΠΎ Ρ‡Ρ‚ΠΎ происходит, ΠΊΠΎΠ³Π΄Π° ваша схСма Π±ΠΎΠ»Π΅Π΅ слоТная? Π’ΠΎΠ·ΠΌΠΎΠΆΠ½ΠΎ, Π²Ρ‹ Ρ€Π°Π·Ρ€Π°Π±Π°Ρ‚Ρ‹Π²Π°Π΅Ρ‚Π΅ элСктронику с ΠΏΠ°Ρ€Π°Π»Π»Π΅Π»ΡŒΠ½Ρ‹ΠΌ ΠΈ ΠΏΠΎΡΠ»Π΅Π΄ΠΎΠ²Π°Ρ‚Π΅Π»ΡŒΠ½Ρ‹ΠΌ сопротивлСниСм, ΠΈ Π·Π°ΠΊΠΎΠ½ Ома Π½Π°Ρ‡ΠΈΠ½Π°Π΅Ρ‚ ΠΏΠ°Π΄Π°Ρ‚ΡŒ. Или Ρ‡Ρ‚ΠΎ, Ссли Ρƒ вас Π½Π΅Ρ‚ источника постоянного Ρ‚ΠΎΠΊΠ°? Π’ Ρ‚Π°ΠΊΠΈΡ… ситуациях, ΠΊΠΎΠ³Π΄Π° нСльзя ΠΈΡΠΏΠΎΠ»ΡŒΠ·ΠΎΠ²Π°Ρ‚ΡŒ Ρ‚ΠΎΠ»ΡŒΠΊΠΎ V = IR, ΠΏΠΎΡ€Π° Π²ΡΡ‚Π°Ρ‚ΡŒ Π½Π° ΠΏΠ»Π΅Ρ‡ΠΈ Ома ΠΈ ΠΏΡ€ΠΈΠΌΠ΅Π½ΠΈΡ‚ΡŒ Π·Π°ΠΊΠΎΠ½ ΠšΠΈΡ€Ρ…Π³ΠΎΡ„Π°.Π—Π΄Π΅ΡΡŒ ΠΌΡ‹ рассмотрим, Ρ‡Ρ‚ΠΎ Ρ‚Π°ΠΊΠΎΠ΅ Π—Π°ΠΊΠΎΠ½ ΠšΠΈΡ€Ρ…Π³ΠΎΡ„Π° для Ρ†Π΅ΠΏΠ΅ΠΉ ΠΈ ΠΊΠ°ΠΊ Π΅Π³ΠΎ ΠΈΡΠΏΠΎΠ»ΡŒΠ·ΠΎΠ²Π°Ρ‚ΡŒ для Π°Π½Π°Π»ΠΈΠ·Π° напряТСния ΠΈ Ρ‚ΠΎΠΊΠ° слоТных элСктричСских Ρ†Π΅ΠΏΠ΅ΠΉ.

Π§Ρ‚ΠΎ Ρ‚Π°ΠΊΠΎΠ΅ ΠžΠΊΡ€ΡƒΠΆΠ½ΠΎΠΉ Π·Π°ΠΊΠΎΠ½ ΠšΠΈΡ€Ρ…Π³ΠΎΡ„Π°?

Когда Π²Ρ‹ строитС ΡΠ»ΠΎΠΆΠ½ΡƒΡŽ схСму, Π²ΠΊΠ»ΡŽΡ‡Π°ΡŽΡ‰ΡƒΡŽ мосты ΠΈΠ»ΠΈ Ρ‚Ρ€ΠΎΠΉΠ½ΠΈΠΊΠΈ, Π²Ρ‹ Π½Π΅ ΠΌΠΎΠΆΠ΅Ρ‚Π΅ ΠΏΠΎΠ»Π°Π³Π°Ρ‚ΡŒΡΡ Ρ‚ΠΎΠ»ΡŒΠΊΠΎ Π½Π° Π·Π°ΠΊΠΎΠ½ Ома, Ρ‡Ρ‚ΠΎΠ±Ρ‹ Π½Π°ΠΉΡ‚ΠΈ напряТСниС ΠΈΠ»ΠΈ Ρ‚ΠΎΠΊ. Π—Π΄Π΅ΡΡŒ пригодится Π·Π°ΠΊΠΎΠ½ ΠšΠΈΡ€Ρ…Π³ΠΎΡ„Π°, ΠΊΠΎΡ‚ΠΎΡ€Ρ‹ΠΉ позволяСт Ρ€Π°ΡΡΡ‡ΠΈΡ‚Ρ‹Π²Π°Ρ‚ΡŒ ΠΊΠ°ΠΊ Ρ‚ΠΎΠΊ, Ρ‚Π°ΠΊ ΠΈ напряТСниС для слоТных Ρ†Π΅ΠΏΠ΅ΠΉ с ΠΏΠΎΠΌΠΎΡ‰ΡŒΡŽ систСмы Π»ΠΈΠ½Π΅ΠΉΠ½Ρ‹Ρ… ΡƒΡ€Π°Π²Π½Π΅Π½ΠΈΠΉ.БущСствуСт Π΄Π²Π° Π²Π°Ρ€ΠΈΠ°Π½Ρ‚Π° Π·Π°ΠΊΠΎΠ½Π° ΠšΠΈΡ€Ρ…Π³ΠΎΡ„Π°, Π² Ρ‚ΠΎΠΌ числС:

  • Π—Π°ΠΊΠΎΠ½ ΠšΠΈΡ€Ρ…Π³ΠΎΡ„Π°: Для Π°Π½Π°Π»ΠΈΠ·Π° ΠΏΠΎΠ»Π½ΠΎΠ³ΠΎ Ρ‚ΠΎΠΊΠ° слоТной Ρ†Π΅ΠΏΠΈ
  • Π—Π°ΠΊΠΎΠ½ ΠšΠΈΡ€Ρ…Π³ΠΎΡ„Π° ΠΎ напряТСнии : для Π°Π½Π°Π»ΠΈΠ·Π° ΠΏΠΎΠ»Π½ΠΎΠ³ΠΎ напряТСния слоТной Ρ†Π΅ΠΏΠΈ
  • Когда Π²Ρ‹ ΠΎΠ±ΡŠΠ΅Π΄ΠΈΠ½ΡΠ΅Ρ‚Π΅ эти Π΄Π²Π° Π·Π°ΠΊΠΎΠ½Π°, Π²Ρ‹ ΠΏΠΎΠ»ΡƒΡ‡Π°Π΅Ρ‚Π΅ ΠžΠΊΡ€ΡƒΠΆΠ½ΠΎΠΉ Π·Π°ΠΊΠΎΠ½ ΠšΠΈΡ€Ρ…Π³ΠΎΡ„Π°

Как ΠΈ любой Π΄Ρ€ΡƒΠ³ΠΎΠΉ Π½Π°ΡƒΡ‡Π½Ρ‹ΠΉ ΠΈΠ»ΠΈ матСматичСский Π·Π°ΠΊΠΎΠ½, Π½Π°Π·Π²Π°Π½Π½Ρ‹ΠΉ Π² Ρ‡Π΅ΡΡ‚ΡŒ ΠΈΡ… создатСля, Π—Π°ΠΊΠΎΠ½ ΠšΠΈΡ€Ρ…Π³ΠΎΡ„Π° Π±Ρ‹Π» ΠΈΠ·ΠΎΠ±Ρ€Π΅Ρ‚Π΅Π½ Π½Π΅ΠΌΠ΅Ρ†ΠΊΠΈΠΌ Ρ„ΠΈΠ·ΠΈΠΊΠΎΠΌ Густавом ΠšΠΈΡ€Ρ…Π³ΠΎΡ„ΠΎΠΌ.Густав Π±Ρ‹Π» извСстСн ΠΌΠ½ΠΎΠ³ΠΈΠΌΠΈ достиТСниями ΠΏΡ€ΠΈ ΠΆΠΈΠ·Π½ΠΈ, Π²ΠΊΠ»ΡŽΡ‡Π°Ρ Ρ‚Π΅ΠΎΡ€ΠΈΡŽ ΡΠΏΠ΅ΠΊΡ‚Ρ€Π°Π»ΡŒΠ½ΠΎΠ³ΠΎ Π°Π½Π°Π»ΠΈΠ·Π°, которая Π΄ΠΎΠΊΠ°Π·Π°Π»Π°, Ρ‡Ρ‚ΠΎ элСмСнты ΠΈΠ·Π»ΡƒΡ‡Π°ΡŽΡ‚ ΡƒΠ½ΠΈΠΊΠ°Π»ΡŒΠ½Ρ‹ΠΉ свСтовой ΡƒΠ·ΠΎΡ€ ΠΏΡ€ΠΈ Π½Π°Π³Ρ€Π΅Π²Π°Π½ΠΈΠΈ. Когда ΠšΠΈΡ€Ρ…Π³ΠΎΡ„ ΠΈ Ρ…ΠΈΠΌΠΈΠΊ Π ΠΎΠ±Π΅Ρ€Ρ‚ Π‘ΡƒΠ½Π·Π΅Π½ ΠΏΡ€ΠΎΠ°Π½Π°Π»ΠΈΠ·ΠΈΡ€ΠΎΠ²Π°Π»ΠΈ эти свСтовыС ΡƒΠ·ΠΎΡ€Ρ‹ Ρ‡Π΅Ρ€Π΅Π· ΠΏΡ€ΠΈΠ·ΠΌΡƒ, ΠΎΠ½ΠΈ ΠΎΠ±Π½Π°Ρ€ΡƒΠΆΠΈΠ»ΠΈ, Ρ‡Ρ‚ΠΎ ΠΊΠ°ΠΆΠ΄Ρ‹ΠΉ элСмСнт пСриодичСской Ρ‚Π°Π±Π»ΠΈΡ†Ρ‹ ΠΈΠΌΠ΅Π΅Ρ‚ свою ΡƒΠ½ΠΈΠΊΠ°Π»ΡŒΠ½ΡƒΡŽ Π΄Π»ΠΈΠ½Ρƒ Π²ΠΎΠ»Π½Ρ‹. ΠžΡ‚ΠΊΡ€Ρ‹Ρ‚ΠΈΠ΅ этого ΠΏΠ°Ρ‚Ρ‚Π΅Ρ€Π½Π° ΠΏΠΎΠ·Π²ΠΎΠ»ΠΈΠ»ΠΎ дуэту ΠΎΡ‚ΠΊΡ€Ρ‹Ρ‚ΡŒ Π΄Π²Π° Π½ΠΎΠ²Ρ‹Ρ… элСмСнта, Ρ†Π΅Π·ΠΈΠΉ ΠΈ Ρ€ΡƒΠ±ΠΈΠ΄ΠΈΠΉ.

Густав ΠšΠΈΡ€Ρ…Π³ΠΎΡ„ (слСва) ΠΈ Π ΠΎΠ±Π΅Ρ€Ρ‚ Π‘ΡƒΠ½Π·Π΅Π½ (справа)

ΠšΠΈΡ€Ρ…Π³ΠΎΡ„ ΠΏΠΎΠ·ΠΆΠ΅ ΠΏΡ€ΠΈΠΌΠ΅Π½ΠΈΠ» свою Ρ‚Π΅ΠΎΡ€ΠΈΡŽ ΡΠΏΠ΅ΠΊΡ‚Ρ€Π°Π»ΡŒΠ½ΠΎΠ³ΠΎ Π°Π½Π°Π»ΠΈΠ·Π° для изучСния состава Π‘ΠΎΠ»Π½Ρ†Π°, Π³Π΄Π΅ ΠΎΠ½ ΠΎΠ±Π½Π°Ρ€ΡƒΠΆΠΈΠ» мноТСство Ρ‚Π΅ΠΌΠ½Ρ‹Ρ… Π»ΠΈΠ½ΠΈΠΉ Π² спСктрС Π΄Π»ΠΈΠ½ Π²ΠΎΠ»Π½ Π‘ΠΎΠ»Π½Ρ†Π°.Π­Ρ‚ΠΎ Π±Ρ‹Π»ΠΎ Π²Ρ‹Π·Π²Π°Π½ΠΎ Ρ‚Π΅ΠΌ, Ρ‡Ρ‚ΠΎ Π³Π°Π· Π‘ΠΎΠ»Π½Ρ†Π° ΠΏΠΎΠ³Π»ΠΎΡ‰Π°Π» свСтовыС Π²ΠΎΠ»Π½Ρ‹ ΠΎΠΏΡ€Π΅Π΄Π΅Π»Π΅Π½Π½ΠΎΠΉ Π΄Π»ΠΈΠ½Ρ‹, ΠΈ это ΠΎΡ‚ΠΊΡ€Ρ‹Ρ‚ΠΈΠ΅ ΠΎΠ·Π½Π°ΠΌΠ΅Π½ΠΎΠ²Π°Π»ΠΎ Π½Π°Ρ‡Π°Π»ΠΎ Π½ΠΎΠ²ΠΎΠΉ эры исслСдований Π² области астрономии.

НСмного Π±Π»ΠΈΠΆΠ΅ ΠΊ Π΄ΠΎΠΌΡƒ Π² ΠΌΠΈΡ€Π΅ элСктроники, ΠšΠΈΡ€Ρ…Π³ΠΎΡ„ объявил свой свод Π·Π°ΠΊΠΎΠ½ΠΎΠ² для Π°Π½Π°Π»ΠΈΠ·Π° Ρ‚ΠΎΠΊΠ° ΠΈ напряТСния Π² элСктричСских цСпях Π² 1845 Π³ΠΎΠ΄Ρƒ, извСстный сСгодня ΠΊΠ°ΠΊ Π—Π°ΠΊΠΎΠ½ ΠšΠΈΡ€Ρ…Π³ΠΎΡ„Π° ΠΎ цСпях. Π­Ρ‚Π° Ρ€Π°Π±ΠΎΡ‚Π° строится Π½Π° основС, ΠΈΠ·Π»ΠΎΠΆΠ΅Π½Π½ΠΎΠΉ Π² Π·Π°ΠΊΠΎΠ½Π΅ Ома, ΠΈ ΠΏΠΎΠΌΠΎΠ³Π»Π° ΠΏΡ€ΠΎΠ»ΠΎΠΆΠΈΡ‚ΡŒ ΠΏΡƒΡ‚ΡŒ для Π°Π½Π°Π»ΠΈΠ·Π° слоТных схСм, Π½Π° ΠΊΠΎΡ‚ΠΎΡ€Ρ‹ΠΉ ΠΌΡ‹ полагаСмся сСгодня.

ΠŸΠ΅Ρ€Π²Ρ‹ΠΉ Π·Π°ΠΊΠΎΠ½ — Π”Π΅ΠΉΡΡ‚Π²ΡƒΡŽΡ‰ΠΈΠΉ Π·Π°ΠΊΠΎΠ½ ΠšΠΈΡ€Ρ…Π³ΠΎΡ„Π°

Π—Π°ΠΊΠΎΠ½ ΠšΠΈΡ€Ρ…Π³ΠΎΡ„Π° ΠΏΠΎ Ρ‚ΠΎΠΊΡƒ гласит, Ρ‡Ρ‚ΠΎ Π²Π΅Π»ΠΈΡ‡ΠΈΠ½Π° Ρ‚ΠΎΠΊΠ°, входящСго Π² ΡƒΠ·Π΅Π», Ρ€Π°Π²Π½Π° Π²Π΅Π»ΠΈΡ‡ΠΈΠ½Π΅ Ρ‚ΠΎΠΊΠ°, выходящСго ΠΈΠ· ΡƒΠ·Π»Π°. ΠŸΠΎΡ‡Π΅ΠΌΡƒ? ΠŸΠΎΡ‚ΠΎΠΌΡƒ Ρ‡Ρ‚ΠΎ, ΠΊΠΎΠ³Π΄Π° Ρ‚ΠΎΠΊ Π²Ρ…ΠΎΠ΄ΠΈΡ‚ Π² ΡƒΠ·Π΅Π», Π΅ΠΌΡƒ Π½Π΅ΠΊΡƒΠ΄Π° ΠΈΠ΄Ρ‚ΠΈ, ΠΊΡ€ΠΎΠΌΠ΅ Π²Ρ‹Ρ…ΠΎΠ΄Π°. Π’ΠΎ, Ρ‡Ρ‚ΠΎ Π²Ρ…ΠΎΠ΄ΠΈΡ‚, Π΄ΠΎΠ»ΠΆΠ½ΠΎ Π²Ρ‹Ρ…ΠΎΠ΄ΠΈΡ‚ΡŒ Π½Π°Ρ€ΡƒΠΆΡƒ. Π’Ρ‹ ΠΌΠΎΠΆΠ΅Ρ‚Π΅ ΠΎΠΏΡ€Π΅Π΄Π΅Π»ΠΈΡ‚ΡŒ ΡƒΠ·Π΅Π», Π² ΠΊΠΎΡ‚ΠΎΡ€ΠΎΠΌ Π΄Π²Π° ΠΈΠ»ΠΈ Π±ΠΎΠ»Π΅Π΅ ΠΏΡƒΡ‚ΠΈ соСдинСны ΠΎΠ±Ρ‰Π΅ΠΉ Ρ‚ΠΎΡ‡ΠΊΠΎΠΉ. На схСмС это Π±ΡƒΠ΄Π΅Ρ‚ Ρ‚ΠΎΡ‡ΠΊΠ° соСдинСния, ΡΠΎΠ΅Π΄ΠΈΠ½ΡΡŽΡ‰Π°Ρ Π΄Π²Π΅ ΠΏΠ΅Ρ€Π΅ΡΠ΅ΠΊΠ°ΡŽΡ‰ΠΈΠ΅ΡΡ сСтСвыС соСдинСния.

ВзглянитС Π½Π° ΠΈΠ·ΠΎΠ±Ρ€Π°ΠΆΠ΅Π½ΠΈΠ΅ Π½ΠΈΠΆΠ΅, Ρ‡Ρ‚ΠΎΠ±Ρ‹ Π²ΠΈΠ·ΡƒΠ°Π»ΡŒΠ½ΠΎ ΠΏΠΎΠ½ΡΡ‚ΡŒ этот Π—Π°ΠΊΠΎΠ½.Π—Π΄Π΅ΡΡŒ Ρƒ нас Π΅ΡΡ‚ΡŒ Π΄Π²Π° Ρ‚ΠΎΠΊΠ°, входящиС Π² ΡƒΠ·Π΅Π», ΠΈ Ρ‚Ρ€ΠΈ Ρ‚ΠΎΠΊΠ°, выходящиС ΠΈΠ· ΡƒΠ·Π»Π°. Богласно Π—Π°ΠΊΠΎΠ½Ρƒ ΠšΠΈΡ€Ρ…Π³ΠΎΡ„Π°, взаимосвязь ΠΌΠ΅ΠΆΠ΄Ρƒ этими Ρ‚ΠΎΠΊΠ°ΠΌΠΈ, входящими Π² ΡƒΠ·Π΅Π» ΠΈ выходящими ΠΈΠ· Π½Π΅Π³ΠΎ, ΠΌΠΎΠΆΠ΅Ρ‚ Π±Ρ‹Ρ‚ΡŒ прСдставлСна ​​как I 1 + I 2 = I 3 + I 4 + I 5 .

Π’Π΅ΠΊΡƒΡ‰ΠΈΠΉ Π·Π°ΠΊΠΎΠ½ ΠšΠΈΡ€Ρ…Π³ΠΎΡ„Π°, Ρ‚ΠΎΠΊ Π½Π° Π²Ρ…ΠΎΠ΄Π΅ Π΄ΠΎΠ»ΠΆΠ΅Π½ Ρ€Π°Π²Π½ΡΡ‚ΡŒΡΡ Ρ‚ΠΎΠΊΡƒ Π½Π° Π²Ρ‹Ρ…ΠΎΠ΄Π΅. (Π˜ΡΡ‚ΠΎΡ‡Π½ΠΈΠΊ изобраТСния)

Когда Π²Ρ‹ ΡƒΡ€Π°Π²Π½ΠΎΠ²Π΅ΡˆΠΈΠ²Π°Π΅Ρ‚Π΅ это ΡƒΡ€Π°Π²Π½Π΅Π½ΠΈΠ΅ ΠΊΠ°ΠΊ алгСбраичСскоС Π²Ρ‹Ρ€Π°ΠΆΠ΅Π½ΠΈΠ΅, Π²Ρ‹ Π΄Π΅Π»Π°Π΅Ρ‚Π΅ Π²Ρ‹Π²ΠΎΠ΄, Ρ‡Ρ‚ΠΎ Ρ‚ΠΎΠΊ Π½Π° Π²Ρ…ΠΎΠ΄Π΅ ΠΈ Π²Ρ‹Ρ…ΠΎΠ΄Π΅ ΠΈΠ· ΡƒΠ·Π»Π° всСгда Π±ΡƒΠ΄Π΅Ρ‚ Ρ€Π°Π²Π΅Π½ 0, ΠΈΠ»ΠΈ I 1 + I 2 + (-I 3 + -I 4 + -I 5 ) = 0 ВсС Π΄ΠΎΠ»ΠΆΠ½ΠΎ ΡƒΡ€Π°Π²Π½ΠΎΠ²Π΅ΡˆΠΈΠ²Π°Ρ‚ΡŒΡΡ, ΠΈ ΠšΠΈΡ€Ρ…Π³ΠΎΡ„ Π½Π°Π·Π²Π°Π» этот ΠΏΡ€ΠΈΠ½Ρ†ΠΈΠΏ Π‘ΠΎΡ…Ρ€Π°Π½Π΅Π½ΠΈΠ΅ΠΌ заряда .

Π”Π°Π²Π°ΠΉΡ‚Π΅ посмотрим Π½Π° ΠΏΡ€ΠΈΠΌΠ΅Ρ€ схСмы, Ρ‡Ρ‚ΠΎΠ±Ρ‹ ΡƒΠ²ΠΈΠ΄Π΅Ρ‚ΡŒ, ΠΊΠ°ΠΊ это Ρ€Π°Π±ΠΎΡ‚Π°Π΅Ρ‚. НиТС прСдставлСна ​​схСма с Ρ‡Π΅Ρ‚Ρ‹Ρ€ΡŒΠΌΡ ΡƒΠ·Π»Π°ΠΌΠΈ: A, C, E ΠΈ F. Π‘Π½Π°Ρ‡Π°Π»Π° Ρ‚ΠΎΠΊ Ρ‚Π΅Ρ‡Π΅Ρ‚ ΠΎΡ‚ источника напряТСния ΠΈ отдСляСтся Π² ΡƒΠ·Π»Π΅ A, Π° Π·Π°Ρ‚Π΅ΠΌ ΠΏΡ€ΠΎΡ‚Π΅ΠΊΠ°Π΅Ρ‚ Ρ‡Π΅Ρ€Π΅Π· рСзисторы R1 ΠΈ R2. ΠžΡ‚Ρ‚ΡƒΠ΄Π° Ρ‚ΠΎΠΊ Ρ€Π΅ΠΊΠΎΠΌΠ±ΠΈΠ½ΠΈΡ€ΡƒΠ΅Ρ‚ Π² ΡƒΠ·Π»Π΅ C ΠΈ снова раздСляСтся, Ρ‡Ρ‚ΠΎΠ±Ρ‹ ΠΏΡ€ΠΎΡ‚Π΅ΠΊΠ°Ρ‚ΡŒ Ρ‡Π΅Ρ€Π΅Π· рСзисторы R3, R4 ΠΈ R5, Π³Π΄Π΅ ΠΎΠ½ встрСчаСтся с ΡƒΠ·Π»ΠΎΠΌ E ΠΈ ΡƒΠ·Π»ΠΎΠΌ F.

(Π˜ΡΡ‚ΠΎΡ‡Π½ΠΈΠΊ изобраТСния)

Π§Ρ‚ΠΎΠ±Ρ‹ ΠΏΠΎΠ΄Ρ‚Π²Π΅Ρ€Π΄ΠΈΡ‚ΡŒ Π·Π°ΠΊΠΎΠ½ ΠšΠΈΡ€Ρ…Π³ΠΎΡ„Π° Π² этой Ρ†Π΅ΠΏΠΈ, Π½Π°ΠΌ Π½ΡƒΠΆΠ½ΠΎ ΠΏΡ€Π΅Π΄ΠΏΡ€ΠΈΠ½ΡΡ‚ΡŒ ΡΠ»Π΅Π΄ΡƒΡŽΡ‰ΠΈΠ΅ шаги:

  1. Π Π°ΡΡΡ‡ΠΈΡ‚Π°Ρ‚ΡŒ ΠΏΠΎΠ»Π½Ρ‹ΠΉ Ρ‚ΠΎΠΊ Ρ†Π΅ΠΏΠΈ
  2. Π Π°ΡΡΡ‡ΠΈΡ‚Π°Ρ‚ΡŒ Ρ‚ΠΎΠΊ, ΠΏΡ€ΠΎΡ‚Π΅ΠΊΠ°ΡŽΡ‰ΠΈΠΉ Ρ‡Π΅Ρ€Π΅Π· ΠΊΠ°ΠΆΠ΄Ρ‹ΠΉ ΡƒΠ·Π΅Π»
  3. Π‘Ρ€Π°Π²Π½ΠΈΡ‚Π΅ Π²Ρ…ΠΎΠ΄Π½Ρ‹Π΅ ΠΈ Π²Ρ‹Ρ…ΠΎΠ΄Π½Ρ‹Π΅ Ρ‚ΠΎΠΊΠΈ Π² ΠΎΠΏΡ€Π΅Π΄Π΅Π»Π΅Π½Π½Ρ‹Ρ… ΡƒΠ·Π»Π°Ρ…, Ρ‡Ρ‚ΠΎΠ±Ρ‹ ΠΏΠΎΠ΄Ρ‚Π²Π΅Ρ€Π΄ΠΈΡ‚ΡŒ Ρ‚Π΅ΠΊΡƒΡ‰ΠΈΠΉ Π·Π°ΠΊΠΎΠ½ ΠšΠΈΡ€Ρ…Π³ΠΎΡ„Π°.

1. РассчитайтС ΠΎΠ±Ρ‰ΠΈΠΉ Ρ‚ΠΎΠΊ

Π—Π΄Π΅ΡΡŒ ΠΌΡ‹ ΠΈΡΠΏΠΎΠ»ΡŒΠ·ΡƒΠ΅ΠΌ Π·Π°ΠΊΠΎΠ½ Ома, Ρ‡Ρ‚ΠΎΠ±Ρ‹ ΠΏΠΎΠ»ΡƒΡ‡ΠΈΡ‚ΡŒ ΠΏΠΎΠ»Π½Ρ‹ΠΉ Ρ‚ΠΎΠΊ нашСй Ρ†Π΅ΠΏΠΈ с I = V / R . Π£ нас ΡƒΠΆΠ΅ Π΅ΡΡ‚ΡŒ ΠΎΠ±Ρ‰Π΅Π΅ напряТСниС 132 Π’, ΠΈ Ρ‚Π΅ΠΏΠ΅Ρ€ΡŒ Π½Π°ΠΌ просто Π½ΡƒΠΆΠ½ΠΎ Π½Π°ΠΉΡ‚ΠΈ ΠΎΠ±Ρ‰Π΅Π΅ сопротивлСниС Π²ΠΎ всСх Π½Π°ΡˆΠΈΡ… ΡƒΠ·Π»Π°Ρ…. Для этого трСбуСтся простой ΠΌΠ΅Ρ‚ΠΎΠ΄ расчСта ΠΎΠ±Ρ‰Π΅Π³ΠΎ сопротивлСния рСзисторов, ΠΏΠΎΠ΄ΠΊΠ»ΡŽΡ‡Π΅Π½Π½Ρ‹Ρ… ΠΏΠ°Ρ€Π°Π»Π»Π΅Π»ΡŒΠ½ΠΎ, ΠΊΠΎΡ‚ΠΎΡ€ΠΎΠ΅ составляСт:

Начиная с ΡƒΠ·Π»Π° AC, ΠΌΡ‹ ΠΏΠΎΠ»ΡƒΡ‡Π°Π΅ΠΌ ΡΠ»Π΅Π΄ΡƒΡŽΡ‰Π΅Π΅ сопротивлСниС для ΠΏΠ°Ρ€Π°Π»Π»Π΅Π»ΡŒΠ½Ρ‹Ρ… рСзисторов R1 ΠΈ R2:

И пСрСходя ΠΊ ΡƒΠ·Π»Ρƒ CEF, ΠΌΡ‹ ΠΏΠΎΠ»ΡƒΡ‡Π°Π΅ΠΌ ΡΠ»Π΅Π΄ΡƒΡŽΡ‰Π΅Π΅ сопротивлСниС для ΠΏΠ°Ρ€Π°Π»Π»Π΅Π»ΡŒΠ½Ρ‹Ρ… рСзисторов R3, R4 ΠΈ R5:

Π’Π΅ΠΏΠ΅Ρ€ΡŒ Ρƒ нас Π΅ΡΡ‚ΡŒ ΠΎΠ±Ρ‰Π΅Π΅ сопротивлСниС 11 Ом для всСй Ρ†Π΅ΠΏΠΈ, ΠΊΠΎΡ‚ΠΎΡ€ΠΎΠ΅ ΠΌΡ‹ ΠΌΠΎΠΆΠ΅ΠΌ Π·Π°Ρ‚Π΅ΠΌ ΠΏΠΎΠ΄ΠΊΠ»ΡŽΡ‡ΠΈΡ‚ΡŒ ΠΊ Π·Π°ΠΊΠΎΠ½Ρƒ Ома I = V / R , Ρ‡Ρ‚ΠΎΠ±Ρ‹ ΠΏΠΎΠ»ΡƒΡ‡ΠΈΡ‚ΡŒ ΠΎΠ±Ρ‰ΠΈΠΉ Ρ‚ΠΎΠΊ Π² нашСй Ρ†Π΅ΠΏΠΈ:

2.РасчСт ΡƒΠ·Π»ΠΎΠ²Ρ‹Ρ… Ρ‚ΠΎΠΊΠΎΠ²

Π’Π΅ΠΏΠ΅Ρ€ΡŒ, ΠΊΠΎΠ³Π΄Π° ΠΌΡ‹ Π·Π½Π°Π΅ΠΌ, Ρ‡Ρ‚ΠΎ ΠΈΠ· нашСй Ρ†Π΅ΠΏΠΈ Π²Ρ‹Ρ…ΠΎΠ΄ΠΈΡ‚ 12 Π°ΠΌΠΏΠ΅Ρ€, ΠΌΡ‹ ΠΌΠΎΠΆΠ΅ΠΌ Ρ€Π°ΡΡΡ‡ΠΈΡ‚Π°Ρ‚ΡŒ Ρ‚ΠΎΠΊ Π² ΠΊΠ°ΠΆΠ΄ΠΎΠΌ Π½Π°Π±ΠΎΡ€Π΅ ΡƒΠ·Π»ΠΎΠ². ΠœΡ‹ снова Π²ΠΎΡΠΏΠΎΠ»ΡŒΠ·ΡƒΠ΅ΠΌΡΡ ΠΏΠΎΠΌΠΎΡ‰ΡŒΡŽ Π·Π°ΠΊΠΎΠ½Π° Ома Π² Ρ„ΠΎΡ€ΠΌΠ΅ I = V / R , Ρ‡Ρ‚ΠΎΠ±Ρ‹ ΠΏΠΎΠ»ΡƒΡ‡ΠΈΡ‚ΡŒ Ρ‚ΠΎΠΊ для ΠΊΠ°ΠΆΠ΄ΠΎΠΉ Π²Π΅Ρ‚Π²ΠΈ ΡƒΠ·Π»Π°.

Для Π½Π°Ρ‡Π°Π»Π° Π½Π°ΠΌ Π½ΡƒΠΆΠ½Ρ‹ напряТСния для ΡƒΠ·Π»ΠΎΠ²Ρ‹Ρ… Π²Π΅Ρ‚Π²Π΅ΠΉ AC ΠΈ CF:

Π—Π°Ρ‚Π΅ΠΌ ΠΌΡ‹ ΠΌΠΎΠΆΠ΅ΠΌ Ρ€Π°ΡΡΡ‡ΠΈΡ‚Π°Ρ‚ΡŒ Ρ‚ΠΎΠΊ для ΠΊΠ°ΠΆΠ΄ΠΎΠΉ Π²Π΅Ρ‚Π²ΠΈ ΡƒΠ·Π»Π°:

3. ΠŸΠΎΠ΄Ρ‚Π²Π΅Ρ€Π΄ΠΈΡ‚Π΅ Π΄Π΅ΠΉΡΡ‚Π²ΡƒΡŽΡ‰ΠΈΠΉ Π·Π°ΠΊΠΎΠ½ ΠšΠΈΡ€Ρ…Π³ΠΎΡ„Π°

ПослС вычислСния Ρ‚ΠΎΠΊΠ° для ΠΊΠ°ΠΆΠ΄ΠΎΠΉ Π²Π΅Ρ‚Π²ΠΈ ΡƒΠ·Π»Π° Ρƒ нас Ρ‚Π΅ΠΏΠ΅Ρ€ΡŒ Π΅ΡΡ‚ΡŒ Π΄Π²Π΅ ΠΎΡ‚Π΄Π΅Π»ΡŒΠ½Ρ‹Π΅ ΠΊΠΎΠ½Ρ‚Ρ€ΠΎΠ»ΡŒΠ½Ρ‹Π΅ Ρ‚ΠΎΡ‡ΠΊΠΈ, ΠΊΠΎΡ‚ΠΎΡ€Ρ‹Π΅ ΠΌΡ‹ ΠΌΠΎΠΆΠ΅ΠΌ ΠΈΡΠΏΠΎΠ»ΡŒΠ·ΠΎΠ²Π°Ρ‚ΡŒ для сравнСния Π½Π°ΡˆΠΈΡ… Π²Ρ…ΠΎΠ΄Π½Ρ‹Ρ… ΠΈ Π²Ρ‹Ρ…ΠΎΠ΄Π½Ρ‹Ρ… Ρ‚ΠΎΠΊΠΎΠ².Π­Ρ‚ΠΎ ΠΏΠΎΠ·Π²ΠΎΠ»ΠΈΡ‚ Π½Π°ΠΌ ΠΏΡ€ΠΎΠ°Π½Π°Π»ΠΈΠ·ΠΈΡ€ΠΎΠ²Π°Ρ‚ΡŒ Π½Π°ΡˆΡƒ схСму ΠΈ ΠΏΠΎΠ΄Ρ‚Π²Π΅Ρ€Π΄ΠΈΡ‚ΡŒ Ρ‚Π΅ΠΊΡƒΡ‰ΠΈΠΉ Π·Π°ΠΊΠΎΠ½ ΠšΠΈΡ€Ρ…Π³ΠΎΡ„Π° ΡΠ»Π΅Π΄ΡƒΡŽΡ‰ΠΈΠΌ ΠΎΠ±Ρ€Π°Π·ΠΎΠΌ:

Π’Ρ‚ΠΎΡ€ΠΎΠΉ Π·Π°ΠΊΠΎΠ½ — Π—Π°ΠΊΠΎΠ½ ΠšΠΈΡ€Ρ…Π³ΠΎΡ„Π° ΠΎ напряТСнии

Π—Π°ΠΊΠΎΠ½ ΠšΠΈΡ€Ρ…Π³ΠΎΡ„Π° ΠΎ напряТСнии гласит, Ρ‡Ρ‚ΠΎ Π² любой Ρ†Π΅ΠΏΠΈ с Π·Π°ΠΌΠΊΠ½ΡƒΡ‚Ρ‹ΠΌ ΠΊΠΎΠ½Ρ‚ΡƒΡ€ΠΎΠΌ ΠΏΠΎΠ»Π½ΠΎΠ΅ напряТСниС всСгда Π±ΡƒΠ΄Π΅Ρ‚ Ρ€Π°Π²Π½ΠΎ суммС всСх ΠΏΠ°Π΄Π΅Π½ΠΈΠΉ напряТСния Π² ΠΊΠΎΠ½Ρ‚ΡƒΡ€Π΅. Π’Ρ‹ ΠΎΠ±Π½Π°Ρ€ΡƒΠΆΠΈΡ‚Π΅, Ρ‡Ρ‚ΠΎ ΠΏΠ°Π΄Π΅Π½ΠΈΠ΅ напряТСния происходит всякий Ρ€Π°Π·, ΠΊΠΎΠ³Π΄Π° Ρ‚ΠΎΠΊ ΠΏΡ€ΠΎΡ…ΠΎΠ΄ΠΈΡ‚ Ρ‡Π΅Ρ€Π΅Π· пассивный ΠΊΠΎΠΌΠΏΠΎΠ½Π΅Π½Ρ‚, Ρ‚Π°ΠΊΠΎΠΉ ΠΊΠ°ΠΊ рСзистор, ΠΈ ΠšΠΈΡ€Ρ…Π³ΠΎΡ„ Π½Π°Π·Π²Π°Π» этот Π·Π°ΠΊΠΎΠ½ Π‘ΠΎΡ…Ρ€Π°Π½Π΅Π½ΠΈΠ΅ΠΌ энСргии .ΠžΠΏΡΡ‚ΡŒ ΠΆΠ΅, Ρ‚ΠΎ, Ρ‡Ρ‚ΠΎ Π²Ρ…ΠΎΠ΄ΠΈΡ‚, Π΄ΠΎΠ»ΠΆΠ½ΠΎ Π²Ρ‹Ρ…ΠΎΠ΄ΠΈΡ‚ΡŒ Π½Π°Ρ€ΡƒΠΆΡƒ.

ВзглянитС Π½Π° ΠΈΠ·ΠΎΠ±Ρ€Π°ΠΆΠ΅Π½ΠΈΠ΅ Π½ΠΈΠΆΠ΅, Ρ‡Ρ‚ΠΎΠ±Ρ‹ ΠΏΠΎΠ½ΡΡ‚ΡŒ это Π²ΠΈΠ·ΡƒΠ°Π»ΡŒΠ½ΠΎ. Π’ этой схСмС Ρƒ нас Π΅ΡΡ‚ΡŒ источник напряТСния ΠΈ Ρ‡Π΅Ρ‚Ρ‹Ρ€Π΅ области Π² Ρ†Π΅ΠΏΠΈ, Π³Π΄Π΅ напряТСниС встрСтит пассивный ΠΊΠΎΠΌΠΏΠΎΠ½Π΅Π½Ρ‚, Ρ‡Ρ‚ΠΎ Π²Ρ‹Π·ΠΎΠ²Π΅Ρ‚ Π·Π°ΠΌΠ΅Ρ‚Π½ΠΎΠ΅ ΠΏΠ°Π΄Π΅Π½ΠΈΠ΅ напряТСния.

ΠŸΠΎΡΠΊΠΎΠ»ΡŒΠΊΡƒ эти пассивныС ΠΊΠΎΠΌΠΏΠΎΠ½Π΅Π½Ρ‚Ρ‹ соСдинСны ΠΏΠΎΡΠ»Π΅Π΄ΠΎΠ²Π°Ρ‚Π΅Π»ΡŒΠ½ΠΎ, Π²Ρ‹ ΠΌΠΎΠΆΠ΅Ρ‚Π΅ просто ΡΠ»ΠΎΠΆΠΈΡ‚ΡŒ ΠΎΠ±Ρ‰Π΅Π΅ ΠΏΠ°Π΄Π΅Π½ΠΈΠ΅ напряТСния ΠΈ ΡΡ€Π°Π²Π½ΠΈΡ‚ΡŒ Π΅Π³ΠΎ с ΠΎΠ±Ρ‰ΠΈΠΌ напряТСниСм, Ρ‡Ρ‚ΠΎΠ±Ρ‹ ΠΏΠΎΠ»ΡƒΡ‡ΠΈΡ‚ΡŒ ΡΠΎΠΎΡ‚Π½ΠΎΡˆΠ΅Π½ΠΈΠ΅, ΠΊΠΎΡ‚ΠΎΡ€ΠΎΠ΅ выглядит ΡΠ»Π΅Π΄ΡƒΡŽΡ‰ΠΈΠΌ ΠΎΠ±Ρ€Π°Π·ΠΎΠΌ:

Π”Π°Π²Π°ΠΉΡ‚Π΅ Π½Π°Ρ‡Π½Π΅ΠΌ с простой схСмы, Ρ‡Ρ‚ΠΎΠ±Ρ‹ ΠΏΡ€ΠΎΠ΄Π΅ΠΌΠΎΠ½ΡΡ‚Ρ€ΠΈΡ€ΠΎΠ²Π°Ρ‚ΡŒ, ΠΊΠ°ΠΊ это Ρ€Π°Π±ΠΎΡ‚Π°Π΅Ρ‚.Π’ ΠΏΡ€ΠΈΠ²Π΅Π΄Π΅Π½Π½ΠΎΠΌ Π½ΠΈΠΆΠ΅ ΠΏΡ€ΠΈΠΌΠ΅Ρ€Π΅ Ρƒ нас Π΅ΡΡ‚ΡŒ Π΄Π²Π΅ извСстныС ΠΏΠ΅Ρ€Π΅ΠΌΠ΅Π½Π½Ρ‹Π΅: ΠΏΠΎΠ»Π½ΠΎΠ΅ напряТСниС ΠΈ ΠΏΠ°Π΄Π΅Π½ΠΈΠ΅ напряТСния Π½Π° R1.

(Π˜ΡΡ‚ΠΎΡ‡Π½ΠΈΠΊ изобраТСния)

Π§Ρ‚ΠΎ Π½Π°ΠΌ Π½ΡƒΠΆΠ½ΠΎ Π²Ρ‹ΡΡΠ½ΠΈΡ‚ΡŒ, Ρ‚Π°ΠΊ это ΠΏΠ°Π΄Π΅Π½ΠΈΠ΅ напряТСния Π½Π° R2, ΠΈ ΠΌΡ‹ ΠΌΠΎΠΆΠ΅ΠΌ ΠΈΡΠΏΠΎΠ»ΡŒΠ·ΠΎΠ²Π°Ρ‚ΡŒ Π·Π°ΠΊΠΎΠ½ напряТСния ΠšΠΈΡ€Ρ…Π³ΠΎΡ„Π°, Ρ‡Ρ‚ΠΎΠ±Ρ‹ Π²Ρ‹ΡΡΠ½ΠΈΡ‚ΡŒ это со ΡΠ»Π΅Π΄ΡƒΡŽΡ‰Π΅ΠΉ Π·Π°Π²ΠΈΡΠΈΠΌΠΎΡΡ‚ΡŒΡŽ:

ΠŸΠΎΡΠΊΠΎΠ»ΡŒΠΊΡƒ ΠΏΠΎΠ»Π½ΠΎΠ΅ ΠΏΠ°Π΄Π΅Π½ΠΈΠ΅ напряТСния Π² Ρ†Π΅ΠΏΠΈ Π΄ΠΎΠ»ΠΆΠ½ΠΎ Ρ€Π°Π²Π½ΡΡ‚ΡŒΡΡ ΠΎΠ±Ρ‰Π΅ΠΌΡƒ Π½Π°ΠΏΡ€ΡΠΆΠ΅Π½ΠΈΡŽ источника, это обСспСчиваСт простой способ вычислСния нашСй Π½Π΅Π΄ΠΎΡΡ‚Π°ΡŽΡ‰Π΅ΠΉ ΠΏΠ΅Ρ€Π΅ΠΌΠ΅Π½Π½ΠΎΠΉ. Если Π±Ρ‹ Π²Ρ‹ Ρ…ΠΎΡ‚Π΅Π»ΠΈ Π²Ρ‹Ρ€Π°Π·ΠΈΡ‚ΡŒ это ΡΠΎΠΎΡ‚Π½ΠΎΡˆΠ΅Π½ΠΈΠ΅ Π² Π²ΠΈΠ΄Π΅ ΠΏΡ€Π°Π²ΠΈΠ»ΡŒΠ½ΠΎΠ³ΠΎ алгСбраичСского выраТСния, Π²Ρ‹ Π±Ρ‹ ΠΏΠΎΠ»ΡƒΡ‡ΠΈΠ»ΠΈ сумму всСх ΠΏΠ°Π΄Π΅Π½ΠΈΠΉ напряТСния ΠΈ ΠΎΠ±Ρ‰Π΅Π΅ напряТСниС, Ρ€Π°Π²Π½ΠΎΠ΅ Π½ΡƒΠ»ΡŽ, ΠΊΠ°ΠΊ ΠΏΠΎΠΊΠ°Π·Π°Π½ΠΎ здСсь:

Π”Π°Π²Π°ΠΉΡ‚Π΅ посмотрим Π½Π° Π΄Ρ€ΡƒΠ³ΠΎΠΉ ΠΏΡ€ΠΈΠΌΠ΅Ρ€.Π’ схСмС Π½ΠΈΠΆΠ΅ Ρƒ нас Π΅ΡΡ‚ΡŒ Ρ‚Ρ€ΠΈ рСзистора, ΠΏΠΎΠ΄ΠΊΠ»ΡŽΡ‡Π΅Π½Π½Ρ‹Ρ… ΠΏΠΎΡΠ»Π΅Π΄ΠΎΠ²Π°Ρ‚Π΅Π»ΡŒΠ½ΠΎ с Π±Π°Ρ‚Π°Ρ€Π΅Π΅ΠΉ Π½Π° 12 Π’.

Π§Ρ‚ΠΎΠ±Ρ‹ ΠΏΡ€ΠΎΠ²Π΅Ρ€ΠΈΡ‚ΡŒ Π·Π°ΠΊΠΎΠ½ напряТСния ΠšΠΈΡ€Ρ…Π³ΠΎΡ„Π° Π² этой Ρ†Π΅ΠΏΠΈ, Π½Π°ΠΌ Π½Π΅ΠΎΠ±Ρ…ΠΎΠ΄ΠΈΠΌΠΎ ΠΏΡ€Π΅Π΄ΠΏΡ€ΠΈΠ½ΡΡ‚ΡŒ ΡΠ»Π΅Π΄ΡƒΡŽΡ‰ΠΈΠ΅ шаги:

  1. Π’Ρ‹Ρ‡ΠΈΡΠ»ΠΈΡ‚ΡŒ ΠΎΠ±Ρ‰Π΅Π΅ сопротивлСниС Ρ†Π΅ΠΏΠΈ
  2. Π’Ρ‹Ρ‡ΠΈΡΠ»ΠΈΡ‚ΡŒ ΠΏΠΎΠ»Π½Ρ‹ΠΉ Ρ‚ΠΎΠΊ Ρ†Π΅ΠΏΠΈ
  3. РассчитайтС Ρ‚ΠΎΠΊ Ρ‡Π΅Ρ€Π΅Π· ΠΊΠ°ΠΆΠ΄ΠΎΠ³ΠΎ рСзистора
  4. РассчитайтС ΠΏΠ°Π΄Π΅Π½ΠΈΠ΅ напряТСния Π½Π° Π½Π° ΠΊΠ°ΠΆΠ΄ΠΎΠΌ рСзисторС

Π‘Ρ€Π°Π²Π½ΠΈΡ‚Π΅ источник напряТСния с ΠΎΠ±Ρ‰ΠΈΠΌ ΠΏΠ°Π΄Π΅Π½ΠΈΠ΅ΠΌ напряТСния , Ρ‡Ρ‚ΠΎΠ±Ρ‹ ΠΏΠΎΠ΄Ρ‚Π²Π΅Ρ€Π΄ΠΈΡ‚ΡŒ Π·Π°ΠΊΠΎΠ½ ΠšΠΈΡ€Ρ…Π³ΠΎΡ„Π° ΠΎ напряТСнии

1.РассчитайтС ΠΎΠ±Ρ‰Π΅Π΅ сопротивлСниС

ΠŸΠΎΡΠΊΠΎΠ»ΡŒΠΊΡƒ всС наши рСзисторы соСдинСны ΠΏΠΎΡΠ»Π΅Π΄ΠΎΠ²Π°Ρ‚Π΅Π»ΡŒΠ½ΠΎ, ΠΌΡ‹ ΠΌΠΎΠΆΠ΅ΠΌ Π»Π΅Π³ΠΊΠΎ Π½Π°ΠΉΡ‚ΠΈ ΠΎΠ±Ρ‰Π΅Π΅ сопротивлСниС, просто слоТив всС значСния сопротивлСния вмСстС:

2. РассчитайтС ΠΎΠ±Ρ‰ΠΈΠΉ Ρ‚ΠΎΠΊ

Π’Π΅ΠΏΠ΅Ρ€ΡŒ, ΠΊΠΎΠ³Π΄Π° ΠΌΡ‹ Π·Π½Π°Π΅ΠΌ нашС ΠΏΠΎΠ»Π½ΠΎΠ΅ сопротивлСниС, ΠΌΡ‹ снова ΠΌΠΎΠΆΠ΅ΠΌ ΠΈΡΠΏΠΎΠ»ΡŒΠ·ΠΎΠ²Π°Ρ‚ΡŒ Π·Π°ΠΊΠΎΠ½ Ома, Ρ‡Ρ‚ΠΎΠ±Ρ‹ ΠΏΠΎΠ»ΡƒΡ‡ΠΈΡ‚ΡŒ ΠΏΠΎΠ»Π½Ρ‹ΠΉ Ρ‚ΠΎΠΊ нашСй Ρ†Π΅ΠΏΠΈ Π² Π²ΠΈΠ΄Π΅ I = V / R, , ΠΊΠΎΡ‚ΠΎΡ€Ρ‹ΠΉ выглядит Ρ‚Π°ΠΊ:

3. РассчитайтС Ρ‚ΠΎΠΊ Ρ‡Π΅Ρ€Π΅Π· ΠΊΠ°ΠΆΠ΄Ρ‹ΠΉ рСзистор

ΠŸΠΎΡΠΊΠΎΠ»ΡŒΠΊΡƒ всС наши рСзисторы соСдинСны ΠΏΠΎΡΠ»Π΅Π΄ΠΎΠ²Π°Ρ‚Π΅Π»ΡŒΠ½ΠΎ, Ρ‡Π΅Ρ€Π΅Π· Π½ΠΈΡ… Π±ΡƒΠ΄Π΅Ρ‚ ΠΏΡ€ΠΎΡ‚Π΅ΠΊΠ°Ρ‚ΡŒ ΠΎΠ΄ΠΈΠ½Π°ΠΊΠΎΠ²Ρ‹ΠΉ Ρ‚ΠΎΠΊ, ΠΊΠΎΡ‚ΠΎΡ€Ρ‹ΠΉ ΠΌΡ‹ ΠΌΠΎΠΆΠ΅ΠΌ Π²Ρ‹Ρ€Π°Π·ΠΈΡ‚ΡŒ ΠΊΠ°ΠΊ:

4.РассчитайтС ΠΏΠ°Π΄Π΅Π½ΠΈΠ΅ напряТСния Π½Π° ΠΊΠ°ΠΆΠ΄ΠΎΠΌ рСзисторС

.

Π’ нашСм ΠΎΠΊΠΎΠ½Ρ‡Π°Ρ‚Π΅Π»ΡŒΠ½ΠΎΠΌ расчСтС ΠΌΡ‹ снова Π±ΡƒΠ΄Π΅ΠΌ ΠΈΡΠΏΠΎΠ»ΡŒΠ·ΠΎΠ²Π°Ρ‚ΡŒ Π·Π°ΠΊΠΎΠ½ Ома, Ρ‡Ρ‚ΠΎΠ±Ρ‹ ΠΏΠΎΠ»ΡƒΡ‡ΠΈΡ‚ΡŒ ΠΏΠΎΠ»Π½ΠΎΠ΅ ΠΏΠ°Π΄Π΅Π½ΠΈΠ΅ напряТСния для ΠΊΠ°ΠΆΠ΄ΠΎΠ³ΠΎ рСзистора Π² Π²ΠΈΠ΄Π΅ Π’ = IR , ΠΊΠΎΡ‚ΠΎΡ€ΠΎΠ΅ выглядит ΡΠ»Π΅Π΄ΡƒΡŽΡ‰ΠΈΠΌ ΠΎΠ±Ρ€Π°Π·ΠΎΠΌ:

5. ΠŸΠΎΠ΄Ρ‚Π²Π΅Ρ€Π΄ΠΈΡ‚Π΅ Π·Π°ΠΊΠΎΠ½ ΠšΠΈΡ€Ρ…Π³ΠΎΡ„Π° ΠΎ напряТСнии

Π’Π΅ΠΏΠ΅Ρ€ΡŒ Ρƒ нас Π΅ΡΡ‚ΡŒ всС Π½Π΅ΠΎΠ±Ρ…ΠΎΠ΄ΠΈΠΌΡ‹Π΅ Π΄Π°Π½Π½Ρ‹Π΅, Π²ΠΊΠ»ΡŽΡ‡Π°Ρ ΠΎΠ±Ρ‰Π΅Π΅ напряТСниС нашСй Ρ†Π΅ΠΏΠΈ, Π° Ρ‚Π°ΠΊΠΆΠ΅ ΠΊΠ°ΠΆΠ΄ΠΎΠ΅ ΠΏΠ°Π΄Π΅Π½ΠΈΠ΅ напряТСния Π½Π° ΠΊΠ°ΠΆΠ΄ΠΎΠΌ ΠΈΠ· Π½Π°ΡˆΠΈΡ… рСзисторов. Бобирая всС это вмСстС, ΠΌΡ‹ ΠΌΠΎΠΆΠ΅ΠΌ Π»Π΅Π³ΠΊΠΎ ΠΏΡ€ΠΎΠ²Π΅Ρ€ΠΈΡ‚ΡŒ Π·Π°ΠΊΠΎΠ½ напряТСния ΠšΠΈΡ€Ρ…Π³ΠΎΡ„Π° с ΠΏΠΎΠΌΠΎΡ‰ΡŒΡŽ ΡΠ»Π΅Π΄ΡƒΡŽΡ‰Π΅Π³ΠΎ ΡΠΎΠΎΡ‚Π½ΠΎΡˆΠ΅Π½ΠΈΡ:

Π­Ρ‚ΠΎ Ρ‚Π°ΠΊΠΆΠ΅ ΠΌΠΎΠΆΠ½ΠΎ Π²Ρ‹Ρ€Π°Π·ΠΈΡ‚ΡŒ ΠΊΠ°ΠΊ:

Как Π²ΠΈΠ΄ΠΈΡ‚Π΅, ΠΏΠΎΠ»Π½ΠΎΠ΅ напряТСниС Ρ€Π°Π²Π½ΠΎ ΠΎΠ±Ρ‰Π΅ΠΌΡƒ падСнию напряТСния Π² нашСй Ρ†Π΅ΠΏΠΈ.Π’ΠΎ, Ρ‡Ρ‚ΠΎ Π²Ρ…ΠΎΠ΄ΠΈΡ‚, Π΄ΠΎΠ»ΠΆΠ½ΠΎ Π²Ρ‹ΠΉΡ‚ΠΈ Π½Π°Ρ€ΡƒΠΆΡƒ, ΠΈ Π·Π°ΠΊΠΎΠ½ ΠšΠΈΡ€Ρ…Π³ΠΎΡ„Π° снова Ρ€Π°Π±ΠΎΡ‚Π°Π΅Ρ‚!

ΠŸΡ€ΠΎΡ†Π΅ΡΡ использования Π·Π°ΠΊΠΎΠ½Π° ΠšΠΈΡ€Ρ…Π³ΠΎΡ„Π° ΠΎ схСмах

Понимая, ΠΊΠ°ΠΊ Ρ€Π°Π±ΠΎΡ‚Π°Π΅Ρ‚ Π·Π°ΠΊΠΎΠ½ ΠšΠΈΡ€Ρ…Π³ΠΎΡ„Π°, Π² вашСм Π½Π°Π±ΠΎΡ€Π΅ инструмСнтов Ρ‚Π΅ΠΏΠ΅Ρ€ΡŒ Π΅ΡΡ‚ΡŒ Π½ΠΎΠ²Ρ‹ΠΉ инструмСнт для Π°Π½Π°Π»ΠΈΠ·Π° напряТСния ΠΈ Ρ‚ΠΎΠΊΠ° Π² ΠΏΠΎΠ»Π½Ρ‹Ρ… цСпях. ΠŸΡ€ΠΈ использовании этих Π—Π°ΠΊΠΎΠ½ΠΎΠ² Π² Π΄ΠΈΠΊΠΎΠΉ ΠΏΡ€ΠΈΡ€ΠΎΠ΄Π΅ рассмотритС Π²ΠΎΠ·ΠΌΠΎΠΆΠ½ΠΎΡΡ‚ΡŒ использования ΡΠ»Π΅Π΄ΡƒΡŽΡ‰Π΅Π³ΠΎ пошагового процСсса:

  1. Π’ΠΎ-ΠΏΠ΅Ρ€Π²Ρ‹Ρ…, Π½Π°Ρ‡Π½ΠΈΡ‚Π΅ с ΠΌΠ°Ρ€ΠΊΠΈΡ€ΠΎΠ²ΠΊΠΈ всСх извСстных напряТСний ΠΈ сопротивлСний Π½Π° вашСй Ρ†Π΅ΠΏΠΈ.
  2. Π—Π°Ρ‚Π΅ΠΌ Π½Π°Π·ΠΎΠ²ΠΈΡ‚Π΅ ΠΊΠ°ΠΆΠ΄ΡƒΡŽ Π²Π΅Ρ‚Π²ΡŒ Π² вашСй Ρ†Π΅ΠΏΠΈ Ρ‚Π΅ΠΊΡƒΡ‰Π΅ΠΉ ΠΌΠ΅Ρ‚ΠΊΠΎΠΉ, Π½Π°ΠΏΡ€ΠΈΠΌΠ΅Ρ€ I1, I2, I3 ΠΈ Ρ‚. Π”.Π’Π΅Ρ‚Π²ΡŒ — это ΠΎΠ΄ΠΈΠ½ ΠΈΠ»ΠΈ Π³Ρ€ΡƒΠΏΠΏΠ° ΠΊΠΎΠΌΠΏΠΎΠ½Π΅Π½Ρ‚ΠΎΠ², соСдинСнных ΠΌΠ΅ΠΆΠ΄Ρƒ двумя ΡƒΠ·Π»Π°ΠΌΠΈ.
  3. Π—Π°Ρ‚Π΅ΠΌ Π½Π°ΠΉΠ΄ΠΈΡ‚Π΅ Ρ‚Π΅ΠΊΡƒΡ‰ΠΈΠΉ Π·Π°ΠΊΠΎΠ½ ΠšΠΈΡ€Ρ…Π³ΠΎΡ„Π° для ΠΊΠ°ΠΆΠ΄ΠΎΠ³ΠΎ ΡƒΠ·Π»Π° Π² вашСй Ρ†Π΅ΠΏΠΈ.
  4. Π—Π°Ρ‚Π΅ΠΌ Π½Π°ΠΉΠ΄ΠΈΡ‚Π΅ Π·Π°ΠΊΠΎΠ½ напряТСния ΠšΠΈΡ€Ρ…Π³ΠΎΡ„Π° для ΠΊΠ°ΠΆΠ΄ΠΎΠ³ΠΎ ΠΈΠ· нСзависимых ΠΊΠΎΠ½Ρ‚ΡƒΡ€ΠΎΠ² Π² вашСй Ρ†Π΅ΠΏΠΈ.

ПослС Ρ‚ΠΎΠ³ΠΎ, ΠΊΠ°ΠΊ Π²Ρ‹ рассчитали Π·Π°ΠΊΠΎΠ½Ρ‹ ΠšΠΈΡ€Ρ…Π³ΠΎΡ„Π° ΠΏΠΎ Ρ‚ΠΎΠΊΡƒ ΠΈ Π½Π°ΠΏΡ€ΡΠΆΠ΅Π½ΠΈΡŽ, Π²Ρ‹ ΠΌΠΎΠΆΠ΅Ρ‚Π΅ ΠΈΡΠΏΠΎΠ»ΡŒΠ·ΠΎΠ²Π°Ρ‚ΡŒ свои уравнСния, Ρ‡Ρ‚ΠΎΠ±Ρ‹ Π½Π°ΠΉΡ‚ΠΈ Π½Π΅Π΄ΠΎΡΡ‚Π°ΡŽΡ‰ΠΈΠ΅ Ρ‚ΠΎΠΊΠΈ. Π“ΠΎΡ‚ΠΎΠ²Ρ‹ ΠΏΠΎΠΏΡ€ΠΎΠ±ΠΎΠ²Π°Ρ‚ΡŒ это ΡΠ°ΠΌΠΎΡΡ‚ΠΎΡΡ‚Π΅Π»ΡŒΠ½ΠΎ? ВзглянитС Π½Π° схСму Π½ΠΈΠΆΠ΅ ΠΈ посмотритС, смоТСтС Π»ΠΈ Π²Ρ‹ ΠΏΡ€ΠΎΠ²Π΅Ρ€ΠΈΡ‚ΡŒ Π·Π°ΠΊΠΎΠ½ Ρ‚ΠΎΠΊΠ° ΠšΠΈΡ€Ρ…Π³ΠΎΡ„Π° ΠΈ Π·Π°ΠΊΠΎΠ½ напряТСния с нСбольшой ΠΏΠΎΠΌΠΎΡ‰ΡŒΡŽ Ома!

ΠžΡΡ‚Π°Π²ΡŒΡ‚Π΅ свои ΠΎΡ‚Π²Π΅Ρ‚Ρ‹ Π² коммСнтариях Π½ΠΈΠΆΠ΅!

Бтоя Π½Π° ΠΏΠ»Π΅Ρ‡Π°Ρ… Ома

ИмСя Π² Ρ€ΡƒΠΊΠ°Ρ… Π—Π°ΠΊΠΎΠ½ ΠšΠΈΡ€Ρ…Π³ΠΎΡ„Π° ΠΎ цСпях, Ρ‚Π΅ΠΏΠ΅Ρ€ΡŒ Ρƒ вас Π΅ΡΡ‚ΡŒ всС инструмСнты, Π½Π΅ΠΎΠ±Ρ…ΠΎΠ΄ΠΈΠΌΡ‹Π΅ для Π°Π½Π°Π»ΠΈΠ·Π° напряТСния ΠΈ Ρ‚ΠΎΠΊΠ° для слоТных Ρ†Π΅ΠΏΠ΅ΠΉ.Как ΠΈ ΠΌΠ½ΠΎΠ³ΠΈΠ΅ Π΄Ρ€ΡƒΠ³ΠΈΠ΅ Π½Π°ΡƒΡ‡Π½Ρ‹Π΅ ΠΈ матСматичСскиС ΠΏΡ€ΠΈΠ½Ρ†ΠΈΠΏΡ‹, Π·Π°ΠΊΠΎΠ½ ΠšΠΈΡ€Ρ…Π³ΠΎΡ„Π° стоит Π½Π° ΠΏΠ»Π΅Ρ‡Π°Ρ… Ρ‚ΠΎΠ³ΠΎ, Ρ‡Ρ‚ΠΎ Π±Ρ‹Π»ΠΎ Π΄ΠΎ Π½Π΅Π³ΠΎ — Π·Π°ΠΊΠΎΠ½Π° Ома. Π’Ρ‹ ΠΎΠ±Π½Π°Ρ€ΡƒΠΆΠΈΡ‚Π΅, Ρ‡Ρ‚ΠΎ ΠΈΡΠΏΠΎΠ»ΡŒΠ·ΡƒΠ΅Ρ‚Π΅ Π·Π°ΠΊΠΎΠ½ Ома для расчСта ΠΎΡ‚Π΄Π΅Π»ΡŒΠ½Ρ‹Ρ… сопротивлСний, напряТСний ΠΈΠ»ΠΈ Ρ‚ΠΎΠΊΠΎΠ², Π° Π·Π°Ρ‚Π΅ΠΌ, ΠΎΡΠ½ΠΎΠ²Ρ‹Π²Π°ΡΡΡŒ Π½Π° этих расчСтах с Π·Π°ΠΊΠΎΠ½ΠΎΠΌ ΠšΠΈΡ€Ρ…Π³ΠΎΡ„Π°, ΡƒΠ²ΠΈΠ΄ΠΈΡ‚Π΅, соотвСтствуСт Π»ΠΈ ваша схСма этим ΠΏΡ€ΠΈΠ½Ρ†ΠΈΠΏΠ°ΠΌ Ρ‚ΠΎΠΊΠ° ΠΈ напряТСния.

Π“ΠΎΡ‚ΠΎΠ²Ρ‹ ΠΏΡ€ΠΈΠΌΠ΅Π½ΠΈΡ‚ΡŒ Π·Π°ΠΊΠΎΠ½ ΠšΠΈΡ€Ρ…Π³ΠΎΡ„Π° Π² своСм собствСнном ΠΏΡ€ΠΎΠ΅ΠΊΡ‚Π΅ ΠΏΠΎ Ρ€Π°Π·Ρ€Π°Π±ΠΎΡ‚ΠΊΠ΅ элСктроники? ΠŸΠΎΠΏΡ€ΠΎΠ±ΡƒΠΉΡ‚Π΅ Autodesk EAGLE бСсплатно сСгодня!

ΠŸΡ€ΠΎΠ΄ΡƒΠΊΡ†ΠΈΡ BMPRO β€’ РСшСниС для слоТных элСктричСских Ρ†Π΅ΠΏΠ΅ΠΉ

НСсмотря Π½Π° Ρ‚ΠΎ, Ρ‡Ρ‚ΠΎ ΠΌΠΎΠΆΠ½ΠΎ Π±Ρ‹Π»ΠΎ Π±Ρ‹ Π½Π°ΡΡ‚Ρ€ΠΎΠΈΡ‚ΡŒ систСму контроля аккумуляторов, зарядки, ΠΏΡ€Π΅Π΄ΠΎΡ…Ρ€Π°Π½ΠΈΡ‚Π΅Π»ΡŒΠ½Ρ‹Ρ… Ρ†Π΅ΠΏΠ΅ΠΉ, ΠΈΠ·ΠΌΠ΅Ρ€Π΅Π½ΠΈΠΉ напряТСния ΠΈ управлСния устройствами / устройствами с использованиСм дискрСтных устройств, Π±Ρ‹Π»ΠΎ Π±Ρ‹ Π½Π°ΠΌΠ½ΠΎΠ³ΠΎ ΠΏΡ€ΠΎΡ‰Π΅, особСнно для ΠΊΠΎΠ½Π΅Ρ‡Π½ΠΎΠ³ΠΎ ΠΏΠΎΠ»ΡŒΠ·ΠΎΠ²Π°Ρ‚Π΅Π»Ρ, Ссли Π±Ρ‹ всС эти Ρ„ΡƒΠ½ΠΊΡ†ΠΈΠΈ ΠΊΠΎΠ½Ρ‚Ρ€ΠΎΠ»ΠΈΡ€ΠΎΠ²Π°Π»ΠΈΡΡŒ ΠΎΠ΄Π½ΠΎ устройство.

Π’Π°ΠΊΠΈΠΌ ΠΎΠ±Ρ€Π°Π·ΠΎΠΌ, Π±Ρ‹Π»ΠΈ Ρ€Π°Π·Ρ€Π°Π±ΠΎΡ‚Π°Π½Ρ‹ ΠΈΠ½Ρ‚Π΅Π³Ρ€ΠΈΡ€ΠΎΠ²Π°Π½Π½Ρ‹Π΅ систСмы управлСния батарСями (IBMS) J35 ΠΈ BatteryPlus35.

Π’Π°ΠΆΠ½ΠΎΠΉ ΠΎΡΠΎΠ±Π΅Π½Π½ΠΎΡΡ‚ΡŒΡŽ J35 ΠΈ BatteryPlus35 являСтся Ρ‚ΠΎ, Ρ‡Ρ‚ΠΎ Π±Π°Ρ‚Π°Ρ€Π΅ΠΈ, Π³Π΅Π½Π΅Ρ€ΠΈΡ€ΡƒΡŽΡ‰ΠΈΠ΅ устройства ΠΈ всС Π½Π°Π³Ρ€ΡƒΠ·ΠΊΠΈ 12 Π’ / 24 Π’ соСдинСны вмСстС, Π½ΠΎ ΠΎΡ‚Π΄Π΅Π»ΡŒΠ½ΠΎ ΡƒΠΏΡ€Π°Π²Π»ΡΡŽΡ‚ΡΡ ΠΈ ΠΊΠΎΠ½Ρ‚Ρ€ΠΎΠ»ΠΈΡ€ΡƒΡŽΡ‚ΡΡ.

Он Ρ‚Π°ΠΊΠΆΠ΅ ΠΈΠΌΠ΅Π΅Ρ‚ прСимущСство

  • ВсС напряТСния ΠΈ Ρ‚ΠΎΠΊΠΈ ΠΌΠΎΠΆΠ½ΠΎ Π»Π΅Π³ΠΊΠΎ ΠΈΠ·ΠΌΠ΅Ρ€ΠΈΡ‚ΡŒ ΠΈ ΡΡ€Π°Π²Π½ΠΈΡ‚ΡŒ Π΄Ρ€ΡƒΠ³ с Π΄Ρ€ΡƒΠ³ΠΎΠΌ. Π§Ρ‚ΠΎΠ±Ρ‹ максимально ΠΏΠΎΠ²Ρ‹ΡΠΈΡ‚ΡŒ ΡΡ„Ρ„Π΅ΠΊΡ‚ΠΈΠ²Π½ΠΎΡΡ‚ΡŒ зарядки всСх систСмных Π±Π°Ρ‚Π°Ρ€Π΅ΠΉ, BMPRO ΠΏΡ€ΠΎΠ²Π΅Π»Π° всСсторонниС испытания всСх Ρ‚ΠΈΠΏΠΎΠ² Π±Π°Ρ‚Π°Ρ€Π΅ΠΉ, Π²ΠΊΠ»ΡŽΡ‡Π°Ρ новСйшиС конструкции Π»ΠΈΡ‚ΠΈΠ΅Π²Ρ‹Ρ… ΠΈ свинцовых кристаллов.Π’Π°ΠΊΠΈΠΌ ΠΎΠ±Ρ€Π°Π·ΠΎΠΌ, всС характСристики Π±Π°Ρ‚Π°Ρ€Π΅ΠΈ ΠΌΠΎΠ³ΡƒΡ‚ Π±Ρ‹Ρ‚ΡŒ Π·Π°Π³Ρ€ΡƒΠΆΠ΅Π½Ρ‹ Π² ΠΏΡ€ΠΎΠ³Ρ€Π°ΠΌΠΌΠ½ΠΎΠ΅ обСспСчСниС IBMS, ΠΈ ΠΎΠΏΡ‚ΠΈΠΌΠ°Π»ΡŒΠ½Ρ‹Π΅ условия зарядки ΠΌΠΎΠ³ΡƒΡ‚ ΠΏΠΎΠ΄Π΄Π΅Ρ€ΠΆΠΈΠ²Π°Ρ‚ΡŒΡΡ Π² любоС врСмя. ΠžΡ‡Π΅Π²ΠΈΠ΄Π½Ρ‹ΠΌ прСимущСством этого являСтся Ρ‚ΠΎ, Ρ‡Ρ‚ΠΎ срок слуТбы Π±Π°Ρ‚Π°Ρ€Π΅ΠΈ увСличиваСтся, Π½Π°ΠΏΡ€ΠΈΠΌΠ΅Ρ€, Π±Π°Ρ‚Π°Ρ€Π΅ΠΈ Π½Π΅ Ρ€Π°Π·Ρ€ΡΠΆΠ°ΡŽΡ‚ΡΡ Π΄ΠΎ Π½Π΅ΠΏΡ€ΠΈΠ΅ΠΌΠ»Π΅ΠΌΠΎΠ³ΠΎ уровня. Когда доступно нСсколько Π²Ρ…ΠΎΠ΄ΠΎΠ² для зарядки, Π½Π°ΠΏΡ€ΠΈΠΌΠ΅Ρ€, , ΠΈ сСтСвоС зарядноС устройство, ΠΈ солнСчныС ΠΏΠ°Π½Π΅Π»ΠΈ, Ρ‚ΠΎΠ³Π΄Π° ΠΎΠ½ΠΈ ΠΌΠΎΠ³ΡƒΡ‚ Ρ€Π΅Π³ΡƒΠ»ΠΈΡ€ΠΎΠ²Π°Ρ‚ΡŒΡΡ для Π½Π΅ Ρ‚ΠΎΠ»ΡŒΠΊΠΎ ΠΏΡ€Π°Π²ΠΈΠ»ΡŒΠ½ΠΎΠΉ ΡΠΊΠΎΡ€ΠΎΡΡ‚ΡŒΡŽ заряда, Π½ΠΎ ΠΈ Π²ΠΎ ΠΈΠ·Π±Π΅ΠΆΠ°Π½ΠΈΠ΅ скачков напряТСния ΠΈ условий ΠΏΠ΅Ρ€Π΅Π³Ρ€ΡƒΠ·ΠΊΠΈ ΠΏΠΎ Ρ‚ΠΎΠΊΡƒ .
  • ВсС Ρ†Π΅ΠΏΠΈ Π½Π°Π³Ρ€ΡƒΠ·ΠΊΠΈ ΠΏΠΎΠ΄ΠΊΠ»ΡŽΡ‡Π°ΡŽΡ‚ΡΡ ΠΎΡ‚Π΄Π΅Π»ΡŒΠ½ΠΎ ΠΈ ΠΌΠΎΠ³ΡƒΡ‚ ΠΎΡ‚ΠΊΠ»ΡŽΡ‡Π°Ρ‚ΡŒΡΡ ΠΏΠΎ ΠΎΡ‚Π΄Π΅Π»ΡŒΠ½ΠΎΡΡ‚ΠΈ Π² случаС нСисправности Ρ†Π΅ΠΏΠΈ. Π’ ΠΎΡ‚Π»ΠΈΡ‡ΠΈΠ΅ ΠΎΡ‚ прСдохранитСля, ΠΊΠΎΡ‚ΠΎΡ€Ρ‹ΠΉ просто ΠΏΠ΅Ρ€Π΅Π³ΠΎΡ€Π°Π΅Ρ‚, Ρ†Π΅ΠΏΡŒ контролируСтся Ρ‚Π°ΠΊΠΈΠΌ ΠΎΠ±Ρ€Π°Π·ΠΎΠΌ, Ρ‡Ρ‚ΠΎ Ссли Π½Π΅ΠΈΡΠΏΡ€Π°Π²Π½ΠΎΡΡ‚ΡŒ устраняСтся, Ρ‚. Π•. ΠŸΠ΅Ρ€Π΅Π³ΠΎΡ€Π΅Π²ΡˆΠΈΠΉ ΡˆΠ°Ρ€ ΠΈΠ»ΠΈ врСмСнная ΠΏΠ΅Ρ€Π΅Π³Ρ€ΡƒΠ·ΠΊΠ° устранСны, Ρ†Π΅ΠΏΡŒ автоматичСски ΠΏΠΎΠ΄ΠΊΠ»ΡŽΡ‡Π°Π΅Ρ‚ΡΡ снова, Ρ‚ΠΎ Π΅ΡΡ‚ΡŒ Π½Π΅Ρ‚ нСобходимости Π·Π°ΠΌΠ΅Π½ΡΡ‚ΡŒ ΠΏΡ€Π΅Π΄ΠΎΡ…Ρ€Π°Π½ΠΈΡ‚Π΅Π»ΡŒ ΠΈΠ»ΠΈ Π²ΠΊΠ»ΡŽΡ‡Π°Ρ‚ΡŒ Ρ†Π΅ΠΏΡŒ. Π²Ρ‹ΠΊΠ»ΡŽΡ‡Π°Ρ‚Π΅Π»ΡŒ снова Π²ΠΊΠ»ΡŽΡ‡Π΅Π½. Если Π½Π΅ΠΈΡΠΏΡ€Π°Π²Π½ΠΎΡΡ‚ΡŒ Π½Π΅ исчСзнСт, β€‹β€‹Ρ†Π΅ΠΏΡŒ останСтся ΠΎΡ‚ΠΊΠ»ΡŽΡ‡Π΅Π½Π½ΠΎΠΉ.
  • Π’ случаС Ρ‡Ρ€Π΅Π·ΠΌΠ΅Ρ€Π½ΠΎΠ³ΠΎ разряда Π±Π°Ρ‚Π°Ρ€Π΅ΠΉ IBMS ΠΌΠΎΠΆΠ΅Ρ‚ Π±Ρ‹Ρ‚ΡŒ Π·Π°ΠΏΡ€ΠΎΠ³Ρ€Π°ΠΌΠΌΠΈΡ€ΠΎΠ²Π°Π½Π° Π½Π° управляСмоС ΠΎΡ‚ΠΊΠ»ΡŽΡ‡Π΅Π½ΠΈΠ΅ .НСсущСствСнныС Ρ†Π΅ΠΏΠΈ, Ρ‚Π°ΠΊΠΈΠ΅ ΠΊΠ°ΠΊ, скаТСм, водяной насос ΠΈ элСктричСский навСс, Π±ΡƒΠ΄ΡƒΡ‚ ΠΎΡ‚ΠΊΠ»ΡŽΡ‡Π΅Π½Ρ‹ Π² ΠΏΠ΅Ρ€Π²ΡƒΡŽ ΠΎΡ‡Π΅Ρ€Π΅Π΄ΡŒ, Ρ‡Ρ‚ΠΎΠ±Ρ‹ ΠΎΡΡ‚Π°Π²ΠΈΡ‚ΡŒ Π² Ρ€Π°Π±ΠΎΡ‡Π΅ΠΌ состоянии Ρ…ΠΎΠ»ΠΎΠ΄ΠΈΠ»ΡŒΠ½ΠΈΠΊ ΠΈΠ»ΠΈ мСдицинскиС устройства, Ρ‚Π°ΠΊΠΈΠ΅ ΠΊΠ°ΠΊ Π°ΠΏΠΏΠ°Ρ€Π°Ρ‚ для апноэ Π²ΠΎ снС.
  • Π’ состав ΠΊΠΎΠΌΠΏΠ»Π΅ΠΊΡ‚Π° J35 Π²Ρ…ΠΎΠ΄ΠΈΡ‚ ΠΏΠ»Π°Π½ΡˆΠ΅Ρ‚, ΠΊΠΎΡ‚ΠΎΡ€Ρ‹ΠΉ ΠΌΠΎΠΆΠ½ΠΎ ΠΈΡΠΏΠΎΠ»ΡŒΠ·ΠΎΠ²Π°Ρ‚ΡŒ для управлСния, контроля ΠΈ управлСния всСм, Π²ΠΊΠ»ΡŽΡ‡Π°Ρ Ρ‚Π°ΠΊΠΈΠ΅ элСмСнты, ΠΊΠ°ΠΊ Π²ΠΊΠ»ΡŽΡ‡Π΅Π½ΠΈΠ΅ ΠΈ Π²Ρ‹ΠΊΠ»ΡŽΡ‡Π΅Π½ΠΈΠ΅ свСта ΠΈ ΡƒΠΏΡ€Π°Π²Π»Π΅Π½ΠΈΠ΅ систСмой горячСго водоснабТСния. Π‘ΠΎΠ΅Π΄ΠΈΠ½Π΅Π½ΠΈΠ΅ Bluetooth ΠΈΡΠΏΠΎΠ»ΡŒΠ·ΡƒΠ΅Ρ‚ΡΡ для соСдинСния ΠΏΠ»Π°Π½ΡˆΠ΅Ρ‚Π° с J35, Ρ‡Ρ‚ΠΎ ΠΎΠ·Π½Π°Ρ‡Π°Π΅Ρ‚, Ρ‡Ρ‚ΠΎ Π²ΠΎΠ·ΠΌΠΎΠΆΠ½ΠΎΡΡ‚ΡŒ Wi-Fi Π½Π΅ трСбуСтся.Π₯ΠΎΡ€ΠΎΡˆΠ°Ρ Π½ΠΎΠ²ΠΎΡΡ‚ΡŒ Π·Π°ΠΊΠ»ΡŽΡ‡Π°Π΅Ρ‚ΡΡ Π² Ρ‚ΠΎΠΌ, Ρ‡Ρ‚ΠΎ хотя Π² систСмС IBMS настроСн для отслСТивания довольно простым способом, тСхничСскиС Ρ€ΡƒΠΊΠΎΠ²ΠΎΠ΄ΠΈΡ‚Π΅Π»ΠΈ ΠΌΠΎΠ³ΡƒΡ‚ Π½Π°ΡΡ‚Ρ€ΠΎΠΈΡ‚ΡŒ всС Π² соотвСтствии со своими ТСланиями ΠΈ интСрСсами.

J35 ΠΈ BatteryPlus35 ΠΏΡ€Π΅Π΄Π»Π°Π³Π°ΡŽΡ‚ комплСксный ΠΏΠ°ΠΊΠ΅Ρ‚, ΠΊΠΎΡ‚ΠΎΡ€Ρ‹ΠΉ Π½Π΅ Ρ‚ΠΎΠ»ΡŒΠΊΠΎ управляСт всСй систСмой ΠΏΡ€ΠΎΠ³ΡƒΠ»ΠΎΡ‡Π½ΠΎΠ³ΠΎ транспорта 12 Π’ / 24 Π’ с максимальной ΡΡ„Ρ„Π΅ΠΊΡ‚ΠΈΠ²Π½ΠΎΡΡ‚ΡŒΡŽ, Π½ΠΎ ΠΈ обСспСчиваСт Π½Π°ΠΈΠ±ΠΎΠ»Π΅Π΅ экономичноС использованиС всСй систСмы, Π²ΠΊΠ»ΡŽΡ‡Π°Ρ всС Π²Π°ΠΆΠ½Ρ‹Π΅ Π±Π°Ρ‚Π°Ρ€Π΅ΠΈ.

Ρ€Π΅ΡˆΠ΅Π½ΠΈΠ΅ элСктричСской Ρ†Π΅ΠΏΠΈ

Π“Π΄Π΅ два… Π—Π°ΠΏΠΈΡˆΠΈΡ‚Π΅ трСбования ΠΊ элСктричСской Π½Π°Π³Ρ€ΡƒΠ·ΠΊΠ΅ ΠΊΠ°ΠΆΠ΄ΠΎΠ³ΠΎ устройства, ΠΏΠΎΠ΄ΠΊΠ»ΡŽΡ‡Π΅Π½Π½ΠΎΠ³ΠΎ ΠΊ Ρ†Π΅ΠΏΠΈ.Аудитория ΠšΠ½ΠΈΠ³Ρƒ ΠΌΠΎΠ³ΡƒΡ‚ ΠΈΡΠΏΠΎΠ»ΡŒΠ·ΠΎΠ²Π°Ρ‚ΡŒ студСнты, ΠΏΡ€ΠΎΡ„Π΅ΡΡΠΈΠΎΠ½Π°Π»ΡŒΠ½Ρ‹Π΅ ΠΈΠ½ΠΆΠ΅Π½Π΅Ρ€Ρ‹ ΠΈ Ρ‚Π΅Ρ…Π½ΠΈΠΊΠΈ. Π£Π½ΠΈΠ²Π΅Ρ€ΡΠ°Π»ΡŒΠ½Π°Ρ рабочая станция. Π Π΅ΡˆΠ΅Π½Π½Ρ‹Π΅ ΠΏΡ€ΠΈΠΌΠ΅Ρ€Ρ‹ слоТных схСм Π˜Π»Π»ΡŽΡΡ‚Ρ€Π°Ρ†ΠΈΡ: Π”Π°Π²Π°ΠΉΡ‚Π΅ ΠΏΡ€ΠΎΠ°Π½Π°Π»ΠΈΠ·ΠΈΡ€ΡƒΠ΅ΠΌ ΠΏΡ€ΠΎΡΡ‚ΡƒΡŽ схСму, ΠΏΠΎΠΊΠ°Π·Π°Π½Π½ΡƒΡŽ Π½Π° рисункС рядом. Π¨Π°Π³ 2: ΠŸΡ€Π΅Π΄ΡΡ‚Π°Π²ΡŒΡ‚Π΅ всС, Ρ‡Ρ‚ΠΎ Π²Ρ‹ Π·Π½Π°Π΅Ρ‚Π΅ ΠΎΠ± этой схСмС. Β«ΠžΡΠ½ΠΎΠ²Ρ‹ элСктричСских схСм», 4-Π΅ ΠΈΠ·Π΄Π°Π½ΠΈΠ΅, Π°Π²Ρ‚ΠΎΡ€ АлСксандр ΠΈ М. Π‘Π°Π΄ΠΈΠΊΡƒ Π­Ρ‚ΠΎ руководство ΠΏΠΎ Ρ€Π΅ΡˆΠ΅Π½ΠΈΡŽ «ЭлСктричСскиС схСмы». Π­Ρ‚ΠΎΡ‚ Π½Π°Π±ΠΎΡ€ практичСских тСстов 1 ΠΏΡ€ΠΎΠ²Π΅Ρ€ΠΈΡ‚ Π²Π°ΡˆΡƒ ΠΊΠΎΠΌΠΏΠ΅Ρ‚Π΅Π½Ρ‚Π½ΠΎΡΡ‚ΡŒ Π² тСориях, концСпциях ΠΈ Ρ€Π΅ΡˆΠ΅Π½ΠΈΠΈ ΠΏΡ€ΠΎΠ±Π»Π΅ΠΌ, связанных с элСктричСскими цСпями.БСсплатныС ΠΈΠ½Ρ‚Π΅Ρ€Π°ΠΊΡ‚ΠΈΠ²Π½Ρ‹Π΅ упраТнСния для ΠΏΡ€Π°ΠΊΡ‚ΠΈΠΊΠΈ ΠΎΠ½Π»Π°ΠΉΠ½ ΠΈΠ»ΠΈ ΡΠΊΠ°Ρ‡Π°Ρ‚ΡŒ Π² Ρ„ΠΎΡ€ΠΌΠ°Ρ‚Π΅ pdf для ΠΏΠ΅Ρ‡Π°Ρ‚ΠΈ. ЭлСктричСскиС Ρ†Π΅ΠΏΠΈ — это энСргия. АлСксандр Π§Π°Ρ€Π»ΡŒΠ· К. ΠΈ Π‘Π°Π΄ΠΈΠΊΡƒ ΠœΡΡ‚ΡŒΡŽ Н.О., ΠžΡΠ½ΠΎΠ²Ρ‹ элСктричСских Ρ†Π΅ΠΏΠ΅ΠΉ, 5-Π΅ ΠΈΠ·Π΄Π°Π½ΠΈΠ΅, McGraw-Hill, 2013. Если ΠΏΠΎΠ΄ΠΊΠ»ΡŽΡ‡Π΅Π½ΠΈΠ΅ Π½Π΅ΠΊΠΎΡ‚ΠΎΡ€Ρ‹Ρ… устройств ΠΊ Ρ€ΠΎΠ·Π΅Ρ‚ΠΊΠ°ΠΌ Π² Π΄Ρ€ΡƒΠ³ΠΈΡ… цСпях Π½Π΅ Ρ€Π΅ΡˆΠ°Π΅Ρ‚ ΠΏΡ€ΠΎΠ±Π»Π΅ΠΌΡƒ, Π²ΠΎΠ·ΠΌΠΎΠΆΠ½ΠΎ, Π²Π°ΠΌ придСтся ΠΎΠ±Π½ΠΎΠ²ΠΈΡ‚ΡŒ ΡΠ»Π΅ΠΊΡ‚Ρ€ΠΈΡ‡Π΅ΡΠΊΡƒΡŽ панСль Π² вашСм Π΄ΠΎΠΌΠ΅. ΠœΡ‹ Ρ‚Π°ΠΊΠΆΠ΅ ΠΎΠ±Π½Π°Ρ€ΡƒΠΆΠΈΠ»ΠΈ различия Π² Ρ‚ΠΎΠΌ, ΠΊΠ°ΠΊ ΠΎΠ½ΠΈ Ρ€Π΅ΡˆΠ°ΡŽΡ‚ ΠΏΡ€ΠΎΠ±Π»Π΅ΠΌΡ‹. Π˜Π·Π²Π΅ΡΡ‚Π½Π°Ρ своСй Ρ‡Π΅Ρ‚ΠΊΠΎΠΉ ΠΌΠ΅Ρ‚ΠΎΠ΄ΠΎΠ»ΠΎΠ³ΠΈΠ΅ΠΉ Ρ€Π΅ΡˆΠ΅Π½ΠΈΡ ΠΏΡ€ΠΎΠ±Π»Π΅ΠΌ ΠΈ ΡƒΠΏΠΎΡ€ΠΎΠΌ Π½Π° Π΄ΠΈΠ·Π°ΠΉΠ½, Π° Ρ‚Π°ΠΊΠΆΠ΅ Π½Π° качСство ΠΈ количСство Π½Π°Π±ΠΎΡ€ΠΎΠ² Π·Π°Π΄Π°Ρ‡, Β«Π’Π²Π΅Π΄Π΅Π½ΠΈΠ΅ Π² элСктричСскиС схСмы», дСвятоС ΠΈΠ·Π΄Π°Π½ΠΈΠ΅, написанноС Π”ΠΎΡ€Ρ„ΠΎΠΌ ΠΈ Π‘Π²ΠΎΠ±ΠΎΠ΄ΠΎΠΉ, ΠΏΠΎΠΌΠΎΠΆΠ΅Ρ‚ читатСлям ΠΌΡ‹ΡΠ»ΠΈΡ‚ΡŒ ΠΊΠ°ΠΊ ΠΈΠ½ΠΆΠ΅Π½Π΅Ρ€Ρ‹.Π—Π°ΠΊΠΎΠ½Ρ‹ ΠšΠΈΡ€Ρ…Π³ΠΎΡ„Π° ΠΈ Ома ΠΈΡΠΏΠΎΠ»ΡŒΠ·ΡƒΡŽΡ‚ΡΡ для Ρ€Π΅ΡˆΠ΅Π½ΠΈΡ ΠΏΡ€ΠΎΠ±Π»Π΅ΠΌ Ρ†Π΅ΠΏΠ΅ΠΉ постоянного Ρ‚ΠΎΠΊΠ°. Π’ ΠΏΡ€ΠΈΠ²Π΅Π΄Π΅Π½Π½ΠΎΠΉ Π²Ρ‹ΡˆΠ΅ схСмС Π΅ΡΡ‚ΡŒ Π΄Π²Π° Π·Π°ΠΌΠΊΠ½ΡƒΡ‚Ρ‹Ρ… ΠΊΠΎΠ½Ρ‚ΡƒΡ€Π°. Анализ элСктричСских Ρ†Π΅ΠΏΠ΅ΠΉ: источник «Анализ элСктричСских Ρ†Π΅ΠΏΠ΅ΠΉΒ» — ΠΎΡ‡Π΅Π½ΡŒ Ρ„ΡƒΠ½Π΄Π°ΠΌΠ΅Π½Ρ‚Π°Π»ΡŒΠ½Ρ‹ΠΉ курс для элСктротСхники. ЭлСктричСская Ρ†Π΅ΠΏΡŒ замыкаСтся Ρ‚ΠΎΠ»ΡŒΠΊΠΎ ΠΏΡ€ΠΈ Π½Π°Π»ΠΈΡ‡ΠΈΠΈ хотя Π±Ρ‹ ΠΎΠ΄Π½ΠΎΠ³ΠΎ Π·Π°ΠΌΠΊΠ½ΡƒΡ‚ΠΎΠ³ΠΎ ΠΊΠΎΠ½Ρ‚ΡƒΡ€Π° ΠΎΡ‚ ΠΏΠΎΠ»ΠΎΠΆΠΈΡ‚Π΅Π»ΡŒΠ½ΠΎΠ³ΠΎ Π΄ΠΎ ΠΎΡ‚Ρ€ΠΈΡ†Π°Ρ‚Π΅Π»ΡŒΠ½ΠΎΠ³ΠΎ ΠΊΠΎΠ½Ρ†Π°. Π—Π°ΠΊΠΎΠ½ ΠšΠΈΡ€Ρ…Π³ΠΎΡ„Π° ΠΏΠΎ Π½Π°ΠΏΡ€ΡΠΆΠ΅Π½ΠΈΡŽ гласит, Ρ‡Ρ‚ΠΎ направлСнная сумма напряТСний Π² Ρ†Π΅ΠΏΠΈ Π΄ΠΎΠ»ΠΆΠ½Π° Π±Ρ‹Ρ‚ΡŒ Ρ€Π°Π²Π½Π° Π½ΡƒΠ»ΡŽ. Устали Ρ€Π΅ΡˆΠ°Ρ‚ΡŒ слоТныС схСмы с Ρ‚ΠΎΠ½Π½Π°ΠΌΠΈ мостов ΠΈ столбов ΠΈΠ»ΠΈ слишком Π»Π΅Π½ΠΈΠ², Ρ‡Ρ‚ΠΎΠ±Ρ‹ Ρ€Π΅ΡˆΠ°Ρ‚ΡŒ Π΄Π°ΠΆΠ΅ ΠΏΡ€ΠΎΡΡ‚ΡƒΡŽ схСму.ΠŸΡ€ΠΈ Ρ€Π΅ΡˆΠ΅Π½ΠΈΠΈ Π·Π°Π΄Π°Ρ‡ элСктричСских Ρ†Π΅ΠΏΠ΅ΠΉ Π½Π΅ΠΎΠ±Ρ…ΠΎΠ΄ΠΈΠΌ расчСт Π±Π°Π·ΠΎΠ²ΠΎΠΉ схСмы. РСшСниС Ρ†Π΅ΠΏΠ΅ΠΉ Π½Π°ΠΏΡ€ΡΠΌΡƒΡŽ с использованиСм Лапласа ΠœΠ΅Ρ‚ΠΎΠ΄ Лапласа каТСтся ΠΏΠΎΠ»Π΅Π·Π½Ρ‹ΠΌ для Ρ€Π΅ΡˆΠ΅Π½ΠΈΡ Π΄ΠΈΡ„Ρ„Π΅Ρ€Π΅Π½Ρ†ΠΈΠ°Π»ΡŒΠ½Ρ‹Ρ… ΡƒΡ€Π°Π²Π½Π΅Π½ΠΈΠΉ, Π²ΠΎΠ·Π½ΠΈΠΊΠ°ΡŽΡ‰ΠΈΡ… Π² цСпях с кондСнсаторами, ΠΊΠ°Ρ‚ΡƒΡˆΠΊΠ°ΠΌΠΈ индуктивности ΠΈ источниками, ΠΊΠΎΡ‚ΠΎΡ€Ρ‹Π΅ ΠΌΠ΅Π½ΡΡŽΡ‚ΡΡ Π²ΠΎ Π²Ρ€Π΅ΠΌΠ΅Π½ΠΈ (шаги ΠΈ синусоиды). Π’ΠΎΡ‚ схСма Ρ†Π΅ΠΏΠΈ RL. Science Class 10 Physics (India) ЭлСктричСство РСшСниС схСмы с ΠΏΠΎΡΠ»Π΅Π΄ΠΎΠ²Π°Ρ‚Π΅Π»ΡŒΠ½Ρ‹ΠΌΠΈ ΠΈ ΠΏΠ°Ρ€Π°Π»Π»Π΅Π»ΡŒΠ½Ρ‹ΠΌΠΈ рСзисторами.Π˜ΡΠΏΠΎΠ»ΡŒΠ·ΡƒΠΉΡ‚Π΅ Π²ΠΎΠ»ΠΎΠΊΠΎΠ½Π½ΠΎ-оптичСскиС ΠΏΡƒΡ‚ΠΈ для ΠΊΠ°Π½Π°Π»ΠΎΠ² ΠΏΠ΅Ρ€Π΅Π΄Π°Ρ‡ΠΈ Π΄Π°Π½Π½Ρ‹Ρ…. Π‘ΠΎΠ·Π΄Π°Π²Π°ΠΉΡ‚Π΅ простыС конструкции с доступными ΠΊΠΎΠΌΠΏΠΎΠ½Π΅Π½Ρ‚Π°ΠΌΠΈ, добавляйтС ΠΊ Π½ΠΈΠΌ значСния ΠΈ Π½Π°Ρ…ΠΎΠ΄ΠΈΡ‚Π΅ ΠΏΡ€Π°Π²ΠΈΠ»ΡŒΠ½ΠΎΠ΅ Π·Π½Π°Ρ‡Π΅Π½ΠΈΠ΅ для устройства с нСизвСстным Π·Π½Π°Ρ‡Π΅Π½ΠΈΠ΅ΠΌ. Π›ΡƒΡ‡ΡˆΠ΅Π΅, Π½ΠΎ Ρ‚Π°ΠΊΠΆΠ΅ ΠΈ самоС Π΄ΠΎΡ€ΠΎΠ³ΠΎΠ΅ Ρ€Π΅ΡˆΠ΅Π½ΠΈΠ΅ — ΠΈΡΠΏΠΎΠ»ΡŒΠ·ΠΎΠ²Π°Ρ‚ΡŒ Π²ΠΎΠ»ΠΎΠΊΠΎΠ½Π½ΠΎ-оптичСскиС ΠΊΠ°Π±Π΅Π»ΠΈ для всСх Ρ†Π΅ΠΏΠ΅ΠΉ ΠΏΠ΅Ρ€Π΅Π΄Π°Ρ‡ΠΈ Π΄Π°Π½Π½Ρ‹Ρ…, ΠΏΠΎΡΠΊΠΎΠ»ΡŒΠΊΡƒ Π² цСпях Ρ‚Π°ΠΊΠΎΠ³ΠΎ Ρ‚ΠΈΠΏΠ° Π½Π΅ ΠΌΠΎΠΆΠ΅Ρ‚ Π±Ρ‹Ρ‚ΡŒ ΠΊΠΎΠ½Ρ‚ΡƒΡ€ΠΎΠ² зазСмлСния (ΠΈΠ»ΠΈ ΠΏΡ€ΠΎΠ±Π»Π΅ΠΌ с ΠΈΠΌΠΏΡƒΠ»ΡŒΡΠ½Ρ‹ΠΌΠΈ Ρ‚ΠΎΠΊΠ°ΠΌΠΈ). Он опрСдСляСт ΠΏΠΎΠ»ΠΎΠΆΠΈΡ‚Π΅Π»ΡŒΠ½Ρ‹ΠΉ ΠΈΠ»ΠΈ ΠΎΡ‚Ρ€ΠΈΡ†Π°Ρ‚Π΅Π»ΡŒΠ½Ρ‹ΠΉ Π·Π½Π°ΠΊ для Ρ€Π°Π·Π»ΠΈΡ‡Π½Ρ‹Ρ… элСктричСских ΠΏΠ°Ρ€Π°ΠΌΠ΅Ρ‚Ρ€ΠΎΠ², Π° ΠΈΠΌΠ΅Π½Π½ΠΎ. Если Π²Ρ‹ Π±ΡƒΠ΄Π΅Ρ‚Π΅ ΡΠ»Π΅Π΄ΠΎΠ²Π°Ρ‚ΡŒ ΠΏΡ€ΠΎΡ†Π΅Π΄ΡƒΡ€Π΅ Ρ€Π΅ΡˆΠ΅Π½ΠΈΡ схСмы, это Π±ΡƒΠ΄Π΅Ρ‚ Π»Π΅Π³ΠΊΠΎ.ΠžΠ·Π½Π°ΠΊΠΎΠΌΡŒΡ‚Π΅ΡΡŒ с нашим супСр-ΠΊΡ€ΡƒΡ‚Ρ‹ΠΌ Ρ€Π΅ΡˆΠ°Ρ‚Π΅Π»Π΅ΠΌ Ρ†Π΅ΠΏΠ΅ΠΉ, ΠΊΠΎΡ‚ΠΎΡ€Ρ‹ΠΉ ΠΌΠΎΠΆΠ΅Ρ‚ Ρ€Π΅ΡˆΠΈΡ‚ΡŒ Π»ΡŽΠ±Ρ‹Π΅ рСзистивныС Ρ†Π΅ΠΏΠΈ. 2. ΠšΠΎΠ³Π½ΠΈΡ‚ΠΈΠ²Π½Π°Ρ ΡΠΏΠΎΡΠΎΠ±Π½ΠΎΡΡ‚ΡŒ оцСниваСтся с использованиСм схСмы отобраТСния прСдставлСний, Ρ€Π°Π·Ρ€Π°Π±ΠΎΡ‚Π°Π½Π½ΠΎΠΉ Π₯Π°Π½ΠΎΠΌ ΠΈ Π§Π΅ΠΉΡ‚Π΅Ρ€ΠΎΠΌ [15]. Π Π΅ΡˆΠΈΡ‚Π΅ ΡΠ»ΠΎΠΆΠ½ΡƒΡŽ ΠΌΠ°Π³Π½ΠΈΡ‚Π½ΡƒΡŽ Ρ†Π΅ΠΏΡŒ, ΠΈΡΠΏΠΎΠ»ΡŒΠ·ΡƒΡ матСматичСскиС ΡΠΎΠΎΡ‚Π½ΠΎΡˆΠ΅Π½ΠΈΡ ΠΌΠ°Π³Π½ΠΈΡ‚Π½Ρ‹Ρ… Ρ†Π΅ΠΏΠ΅ΠΉ. Π Π°Ρ…ΡƒΠ» ΠŸΠ°Π½Ρ‡ΠΎΠ»ΠΈ. НС Π²ΠΎΠ»Π½ΡƒΠΉΡ‚Π΅ΡΡŒ! Π­Ρ‚ΠΎ Ρ‚Π΅ΠΊΡƒΡ‰ΠΈΠΉ Π²Ρ‹Π±Ρ€Π°Π½Π½Ρ‹ΠΉ элСмСнт. ΠŸΡ€Π°Π²ΠΈΠ»ΠΎ ΠšΡ€Π°ΠΌΠ΅Ρ€Π° для Π°Π½Π°Π»ΠΈΠ·Π° Π»ΠΈΠ½Π΅ΠΉΠ½Ρ‹Ρ… Ρ†Π΅ΠΏΠ΅ΠΉ | Π Π΅ΡˆΠ΅Π½Π½Ρ‹ΠΉ ΠΏΡ€ΠΈΠΌΠ΅Ρ€ с ΠΏΠΎΠΌΠΎΡ‰ΡŒΡŽ ΠΊΠ°Π»ΡŒΠΊΡƒΠ»ΡΡ‚ΠΎΡ€Π° ΠΏΡ€Π°Π²ΠΈΠ» ΠšΡ€Π°ΠΌΠ΅Ρ€Π° БСгодня ΠΌΡ‹ собираСмся ΠΏΠΎΠ΄Π΅Π»ΠΈΡ‚ΡŒΡΡ Π΅Ρ‰Π΅ ΠΎΠ΄Π½ΠΈΠΌ простым, Π½ΠΎ ΠΌΠΎΡ‰Π½Ρ‹ΠΌ ΠΌΠ΅Ρ‚ΠΎΠ΄ΠΎΠΌ Π°Π½Π°Π»ΠΈΠ·Π° Ρ†Π΅ΠΏΠ΅ΠΉ, извСстным ΠΊΠ°ΠΊ Β«ΠΏΡ€Π°Π²ΠΈΠ»ΠΎ ΠšΡ€Π°ΠΌΠ΅Ρ€Π°Β».Π‘ΠΎΠΊΡ€Π°Ρ‚ΠΈΡ‚Π΅ ΠΊΠΎΠΌΠΏΠ»Π΅ΠΊΡ‚Ρ‹ рСзисторов Π΄ΠΎ ΠΎΠ΄Π½ΠΎΠ³ΠΎ эквивалСнтного рСзистора. Solve Elec — Анализ ΠΈ Ρ€Π°Π·Ρ€Π΅ΡˆΠ΅Π½ΠΈΠ΅ элСктричСских Ρ†Π΅ΠΏΠ΅ΠΉ. ΠŸΡ€ΠΎΠ²ΠΎΠ΄Π° Ρ‚ΠΎΡ€Ρ‡Π°Ρ‚ ΠΈΠ· Ρ€ΠΎΠ·Π΅Ρ‚ΠΎΠΊ. Π˜ΡΠΏΠΎΠ»ΡŒΠ·ΡƒΠΉΡ‚Π΅ этот ΠΌΠ΅Ρ‚ΠΎΠ΄ для обнаруТСния ΠΏΡ€ΠΎΠ±Π»Π΅ΠΌ с элСктричСскими цСпями. 5 июля 2020 Π³. β€’ 1Ρ‡. Π‘Ρ‚ΡƒΠ΄Π΅Π½Ρ‚Ρ‹ написали Ρ„ΡƒΠ½ΠΊΡ†ΠΈΠΈ, ΠΊΠΎΡ‚ΠΎΡ€Ρ‹Ρ… Π½Π΅ Π±Ρ‹Π»ΠΎ Π² MATLAB, ΠΈ использованиС ΠΊΠΎΡ‚ΠΎΡ€Ρ‹Ρ… каТСтся ΠΎΡ‡Π΅Π½ΡŒ ΠΏΠΎΠ»Π΅Π·Π½Ρ‹ΠΌ для Ρ€Π΅ΡˆΠ΅Π½ΠΈΡ ΠΏΡ€ΠΎΠ±Π»Π΅ΠΌ с элСктричСскими цСпями. ЭлСктричСскиС Ρ†Π΅ΠΏΠΈ — это энСргия. Π‘ΠΎΠ»ΡŒΡˆΠΎΠ΅ Π²Π½ΠΈΠΌΠ°Π½ΠΈΠ΅ Π±Ρ‹Π»ΠΎ ΡƒΠ΄Π΅Π»Π΅Π½ΠΎ использованию PSpice ΠΈ MATLAB для Ρ€Π΅ΡˆΠ΅Π½ΠΈΡ ΠΏΡ€ΠΎΠ±Π»Π΅ΠΌ со схСмами. ЭнСргия Π²ΠΊΠ»ΡŽΡ‡Π°Π΅Ρ‚ΡΡ Π² Ρ†Π΅ΠΏΡŒ аккумулятором ΠΈΠ»ΠΈ коммСрчСским поставщиком элСктроэнСргии.ΠŸΠΎΠ»ΠΎΠΆΠΈΡ‚Π΅Π»ΡŒΠ½ΠΎΠ΅ соглашСниС ΠΎ Π·Π½Π°ΠΊΠ°Ρ… ΠΈΡΠΏΠΎΠ»ΡŒΠ·ΡƒΠ΅Ρ‚ΡΡ ΠΏΡ€ΠΈ ΠΎΠΏΡ€Π΅Π΄Π΅Π»Π΅Π½ΠΈΠΈ мощности элСмСнта. Π—Π°ΠΏΠΈΡˆΠΈΡ‚Π΅ всС Π΄Π°Π½Π½Ρ‹Π΅, систСматизируйтС ΠΈΠ½Ρ„ΠΎΡ€ΠΌΠ°Ρ†ΠΈΡŽ ΠΈ ΠΏΠΎΠ΄ΡƒΠΌΠ°ΠΉΡ‚Π΅ ΠΎ Π»ΡƒΡ‡ΡˆΠ΅ΠΌ способС Ρ€Π΅ΡˆΠ΅Π½ΠΈΡ ΠΏΡ€ΠΎΠ±Π»Π΅ΠΌΡ‹. Анализ Ρ†Π΅ΠΏΠΈ с использованиСм ΠΌΠ΅Ρ‚ΠΎΠ΄Π° Ρ‚ΠΎΠΊΠ° отвСтвлСния Π²ΠΊΠ»ΡŽΡ‡Π°Π΅Ρ‚ Ρ‚Ρ€ΠΈ шага: ΠžΠ±ΠΎΠ·Π½Π°Ρ‡ΡŒΡ‚Π΅ Ρ‚ΠΎΠΊ ΠΈ Π½Π°ΠΏΡ€Π°Π²Π»Π΅Π½ΠΈΠ΅ Ρ‚ΠΎΠΊΠ° Π² ΠΊΠ°ΠΆΠ΄ΠΎΠΉ Π²Π΅Ρ‚Π²ΠΈ. РаньшС я Π΄Π°Π²Π°Π» совСты ΠΏΠΎ элСктронной ΠΏΠΎΡ‡Ρ‚Π΅ ΠΈ Ρ‚Π΅Π»Π΅Ρ„ΠΎΠ½Ρƒ ΠΎΡ‚ΡΡŽΠ΄Π°, Π½ΠΎ Ρ‚Π΅ΠΏΠ΅Ρ€ΡŒ, ΠΊΠΎΠ³Π΄Π° я ΡƒΡˆΠ΅Π» Π½Π° пСнсию, Π²Π°ΠΌ придСтся ΠΏΠΎΠ»Π°Π³Π°Ρ‚ΡŒΡΡ Π½Π° этот ΡΠ°ΠΌΠΎΠ΄Π΅Π»ΡŒΠ½Ρ‹ΠΉ Π²Π΅Π±-сайт, Ρ‡Ρ‚ΠΎΠ±Ρ‹ Ρ€Π΅ΡˆΠΈΡ‚ΡŒ Π²Π°ΡˆΡƒ ΠΏΡ€ΠΎΠ±Π»Π΅ΠΌΡƒ, ΠΊΠ°ΠΊ это сдСлали Π±ΠΎΠ»Π΅Π΅ 13 ΠΌΠΈΠ»Π»ΠΈΠΎΠ½ΠΎΠ² с 2004 Π³ΠΎΠ΄Π°. 5. ΠžΡ‚Π²Π΅Ρ‚Ρ‹ Π½Π° ΠŸΡ€ΠΈΠΌΠ΅Ρ€Ρ‹ ΠΏΡ€ΠΎΠ±Π»Π΅ΠΌ с элСктричСским Ρ‚ΠΎΠΊΠΎΠΌ ΠΈ цСпями: ΠΏΡ€ΠΈΠΌΠ΅Ρ€Ρ‹ ΠΏΡ€ΠΎΠ±Π»Π΅ΠΌ с Ρ€Π΅ΡˆΠ΅Π½ΠΈΡΠΌΠΈ.Overlamping Π§Ρ‚ΠΎ это ΠΎΠ·Π½Π°Ρ‡Π°Π΅Ρ‚: ΡΠ²Π΅Ρ‚ΠΈΠ»ΡŒΠ½ΠΈΠΊ ΠΈΠΌΠ΅Π΅Ρ‚ Π»Π°ΠΌΠΏΠΎΡ‡ΠΊΡƒ с Π±ΠΎΠ»Π΅Π΅ высокой ΠΌΠΎΡ‰Π½ΠΎΡΡ‚ΡŒΡŽ, Ρ‡Π΅ΠΌ это приспособлСниС ΠΏΡ€Π΅Π΄Π½Π°Π·Π½Π°Ρ‡Π΅Π½ΠΎ. ΠΠ°Ρ€ΡƒΡˆΠ΅Π½ΠΈΠ΅ кодСкса? Π”Π° … А Ρ‚Π΅ΠΏΠ΅Ρ€ΡŒ Π΄Π°Π²Π°ΠΉΡ‚Π΅ Π΅Ρ‰Π΅ Π½Π΅ΠΌΠ½ΠΎΠ³ΠΎ Ρ€Π°ΡΡˆΠΈΡ€ΠΈΠΌ схСму. Π Π°Π±ΠΎΡ‡ΠΈΠ΅ листы ΠΏΠΎ элСктричСским схСмам ΠΈ ΠΎΠ½Π»Π°ΠΉΠ½-Π΄Π΅ΡΡ‚Π΅Π»ΡŒΠ½ΠΎΡΡ‚ΡŒ. Π‘Ρ…Π΅ΠΌΠ° Π²Ρ‚ΠΎΡ€ΠΎΠ³ΠΎ порядка состоит ΠΈΠ· рСзисторов ΠΈ Π²ΠΎΠ·Π±ΡƒΠΆΠ΄Π°Π΅Ρ‚ это ΠΏΡ€ΠΎΠ²Π΅Ρ€ΠΈΡ‚ΡŒ !! НиТС ΠΏΡ€ΠΈΠ²Π΅Π΄Π΅Π½ΠΎ пошаговоС руководство с Ρ€Π΅ΡˆΠ΅Π½Π½Ρ‹ΠΌΠΈ ΠΏΡ€ΠΈΠΌΠ΅Ρ€Π°ΠΌΠΈ, Π² ΠΊΠΎΡ‚ΠΎΡ€ΠΎΠΌ ΠΏΠΎΠ΄Ρ€ΠΎΠ±Π½ΠΎ рассказываСтся, ΠΊΠ°ΠΊ Ρ€Π΅ΡˆΠΈΡ‚ΡŒ ΡΠ»ΠΎΠΆΠ½ΡƒΡŽ ΡΠ»Π΅ΠΊΡ‚Ρ€ΠΈΡ‡Π΅ΡΠΊΡƒΡŽ Ρ†Π΅ΠΏΡŒ ΠΈ ΡΠ΅Ρ‚ΡŒ ΠΏΠΎ ΠΏΡ€Π°Π²ΠΈΠ»Ρƒ ΠšΡ€Π°ΠΌΠ΅Ρ€Π°. Π§Ρ‚ΠΎΠ±Ρ‹ Π²Ρ‹Ρ‡ΠΈΡΠ»ΠΈΡ‚ΡŒ ΠΏΠ°Π΄Π΅Π½ΠΈΠ΅ напряТСния, Π²Ρ‹ Π΄ΠΎΠ»ΠΆΠ½Ρ‹: (1) НайдитС рСзистор, эквивалСнтный ΠΏΠ°Ρ€Π΅, рСзистор 20 Ом, ΠΏΠΎΠΊΠ°Π·Π°Π½Π½Ρ‹ΠΉ Π½Π° срСднСй Π΄ΠΈΠ°Π³Ρ€Π°ΠΌΠΌΠ΅.Π˜Π½ΠΆΠ΅Π½Π΅Ρ€Π½Ρ‹ΠΉ Π°Π½Π°Π»ΠΈΠ· Ρ†Π΅ΠΏΠ΅ΠΉ, Ρ‚Π°ΠΊΠΆΠ΅ извСстный ΠΊΠ°ΠΊ тСория Ρ†Π΅ΠΏΠ΅ΠΉ, Ρ‚Ρ€ΡƒΠ΄Π΅Π½ для Π±ΠΎΠ»ΡŒΡˆΠΈΠ½ΡΡ‚Π²Π° студСнтов Π² ΠΏΠ΅Ρ€Π²ΡƒΡŽ ΠΎΡ‡Π΅Ρ€Π΅Π΄ΡŒ ΠΏΠΎΡ‚ΠΎΠΌΡƒ, Ρ‡Ρ‚ΠΎ основныС Π·Π°ΠΊΠΎΠ½Ρ‹ каТутся ΠΎΡ‡Π΅Π½ΡŒ простыми. e1 ΠΈ e2 — источники напряТСния. НапряТСниС Π² Ρ†Π΅ΠΏΠΈ отобраТаСтся символом Π½Π° рис. Π’Ρ‹ΠΏΠΎΠ»Π½ΠΈΡ‚Π΅ ΡΠ»Π΅Π΄ΡƒΡŽΡ‰ΠΈΠ΅ шаги, устраняя элСктричСскиС Ρ†Π΅ΠΏΠΈ. Π Π΅ΡˆΠ°Π΅Ρ‚ΡΡ простая схСма ΠΈ опрСдСляСтся ΠΌΠΎΡ‰Π½ΠΎΡΡ‚ΡŒ, потрСбляСмая ΠΈΠ»ΠΈ подаваСмая ΠΊΠ°ΠΆΠ΄Ρ‹ΠΌ элСмСнтом. ΠŸΠΎΠ΄Ρ…ΠΎΠ΄ ΠΊ ΠΎΠ±ΡƒΡ‡Π΅Π½ΠΈΡŽ Π² руководствС ΠΏΠΎ элСктричСским схСмам Π’ этой сСрии ΠΌΡ‹ ΠΏΡ€ΠΎΠ΄ΠΎΠ»ΠΆΠ°Π΅ΠΌ ΠΈΠ·ΡƒΡ‡Π΅Π½ΠΈΠ΅ Ρ‚ΠΎΠ³ΠΎ, Ρ‡Ρ‚ΠΎ Π±Ρ‹Π»ΠΎ ΠΈΠ·ΡƒΡ‡Π΅Π½ΠΎ Π² 10 классС. Когда Π΄ΠΎΠΌΠ° старыС,… Π£ΠΊΠ°ΠΆΠΈΡ‚Π΅ стрСлкой Π½Π°ΠΏΡ€Π°Π²Π»Π΅Π½ΠΈΠ΅ ΠΏΠΎΡ‚ΠΎΠΊΠ°.Π—Π°Π΄Π°Ρ‚ΡŒ вопрос Π·Π°Π΄Π°Π½ 8 Π»Π΅Ρ‚ Π½Π°Π·Π°Π΄. Π­Ρ‚ΠΎΡ‚ Π½Π°Π±ΠΎΡ€ Π²ΠΊΠ»ΡŽΡ‡Π°Π΅Ρ‚ 3 практичСских Ρ€Π°Π±ΠΎΡ‡ΠΈΡ… листа ΠΏΠΎ символам ΠΏΡ€ΠΈΠ½Ρ†ΠΈΠΏΠΈΠ°Π»ΡŒΠ½ΠΎΠΉ схСмы, Π·Π°ΠΊΠΎΠ½ Ома ΠΈ Ρ€Π΅ΡˆΠ΅Π½ΠΈΠ΅ для напряТСния, Ρ‚ΠΎΠΊΠ° ΠΈ сопротивлСния с использованиСм ΠΏΡ€ΠΈΠ½Ρ†ΠΈΠΏΠΈΠ°Π»ΡŒΠ½ΠΎΠΉ схСмы. Π’Π΅Ρ€ΠΎΡΡ‚Π½ΠΎΡΡ‚ΡŒ возникновСния Ρ‚Π°ΠΊΠΈΡ… ΠΏΡ€ΠΎΠ±Π»Π΅ΠΌ с элСктричСством Π² старых Π΄ΠΎΠΌΠ°Ρ… Π±Ρ‹Π»Π° довольно высокой, ΠΏΠΎΡ‚ΠΎΠΌΡƒ Ρ‡Ρ‚ΠΎ элСктричСскиС Ρ†Π΅ΠΏΠΈ ΡƒΡΡ‚Π°Π½Π°Π²Π»ΠΈΠ²Π°Π»ΠΈΡΡŒ Π² старыС Π²Ρ€Π΅ΠΌΠ΅Π½Π°. ΠžΡ‡Π΅Π½ΡŒ Π²Π°ΠΆΠ½ΠΎ ΠΏΡ€Π°ΠΊΡ‚ΠΈΠΊΠΎΠ²Π°Ρ‚ΡŒ ΠΈ ΠΏΠΎΠΌΠ½ΠΈΡ‚ΡŒ этот ΠΎΠ±Ρ‰ΠΈΠΉ процСсс: ΠΌΠ°Ρ€ΠΊΠΈΡ€ΠΎΠ²Π°Ρ‚ΡŒ напряТСниС ΡƒΠ·Π»ΠΎΠ². ΠžΠ±ΠΎΠ·Π½Π°Ρ‡ΡŒΡ‚Π΅ Ρ‚ΠΎΠΊΠΈ отвСтвлСния. Π¨Π°Π³ 1. ΠžΠΏΡ€Π΅Π΄Π΅Π»ΠΈΡ‚Π΅ ΠΏΡ€ΠΎΠ±Π»Π΅ΠΌΡƒ. Для этого Π±ΠΈΠ±Π»ΠΈΠΎΡ‚Π΅ΠΊΠ° pyswarms Π±ΡƒΠ΄Π΅Ρ‚ ΠΈΡΠΏΠΎΠ»ΡŒΠ·ΠΎΠ²Π°Ρ‚ΡŒΡΡ для Ρ€Π΅ΡˆΠ΅Π½ΠΈΡ Π½Π΅Π»ΠΈΠ½Π΅ΠΉΠ½ΠΎΠ³ΠΎ уравнСния ΠΏΡƒΡ‚Π΅ΠΌ прСобразования Π΅Π³ΠΎ Π² Π·Π°Π΄Π°Ρ‡Ρƒ ΠΎΠΏΡ‚ΠΈΠΌΠΈΠ·Π°Ρ†ΠΈΠΈ.Π’ ΡΡ‚Π°Ρ‚ΡŒΠ΅ прСдставлСна ​​идСя Ρ€Π΅ΡˆΠ΅Π½ΠΈΡ Π‘ΠšΠ’ Π΄ΠΈΡ„Ρ„Π΅Ρ€Π΅Π½Ρ†ΠΈΠ°Π»ΡŒΠ½Ρ‹ΠΌΠΈ уравнСниями для элСктричСских Ρ†Π΅ΠΏΠ΅ΠΉ. АвтоматичСский Π²Ρ‹ΠΊΠ»ΡŽΡ‡Π°Ρ‚Π΅Π»ΡŒ — ваТная Ρ‡Π°ΡΡ‚ΡŒ систСмы элСктропроводки вашСго Π΄ΠΎΠΌΠ°. ВычислитС ΠΈΠ½Π΄ΡƒΠΊΡ‚ΠΈΠ²Π½ΠΎΡΡ‚ΡŒ ΠΊΠ°Ρ‚ΡƒΡˆΠΊΠΈ. Π­Π»Π΅ΠΊΡ‚Ρ€ΠΎΠ±Π΅Π·ΠΎΠΏΠ°ΡΠ½ΠΎΡΡ‚ΡŒ: ΠΎΠΏΡ€Π΅Π΄Π΅Π»ΠΈΡ‚Π΅ Ρ†Π΅ΠΏΡŒ, Π²Ρ‹ΠΊΠ»ΡŽΡ‡ΠΈΡ‚Π΅ Π΅Π΅, Π° Π·Π°Ρ‚Π΅ΠΌ ΠΏΠΎΠΌΠ΅Ρ‚ΡŒΡ‚Π΅ Π΅Π΅ ΠΏΡ€ΠΈΠΌΠ΅Ρ‡Π°Π½ΠΈΠ΅ΠΌ ΠΏΠ΅Ρ€Π΅Π΄ Π²Ρ‹ΠΏΠΎΠ»Π½Π΅Π½ΠΈΠ΅ΠΌ Π»ΡŽΠ±Ρ‹Ρ… ΠΏΠΎΠ΄ΠΊΠ»ΡŽΡ‡Π΅Π½ΠΈΠΉ ΠΈΠ»ΠΈ ΠΏΡ€ΠΎΠ²Π΅Ρ€ΠΎΠΊ. ΠžΠ±Π΅ΡΠΏΠ΅Ρ‡ΡŒΡ‚Π΅ своих ΡƒΡ‡Π΅Π½ΠΈΠΊΠΎΠ² усилСниСм элСктричСских Ρ†Π΅ΠΏΠ΅ΠΉ с ΠΏΠΎΠΌΠΎΡ‰ΡŒΡŽ этого ΠΠΠ‘ΠžΠ Π« Π­Π›Π•ΠšΠ’Π Π˜Π§Π•Π‘ΠšΠ˜Π₯ Π¦Π•ΠŸΠ•Π™! Π Π°Π·ΠΌΠ΅Ρ€ домашнСй элСктричСской сСрвисной ΠΏΠ°Π½Π΅Π»ΠΈ рассчитываСтся ΠΏΡƒΡ‚Π΅ΠΌ расчСта ΠΏΠ»ΠΎΡ‰Π°Π΄ΠΈ Π΄ΠΎΠΌΠ° Π² ΠΊΠ²Π°Π΄Ρ€Π°Ρ‚Π½Ρ‹Ρ… Ρ„ΡƒΡ‚Π°Ρ… с ΡƒΡ‡Π΅Ρ‚ΠΎΠΌ Ρ‚Ρ€Π΅Π±ΠΎΠ²Π°Π½ΠΈΠΉ ΠΊΠΎΠ΄ΠΎΠ² для Ρ‚Ρ€Π΅Π±ΡƒΠ΅ΠΌΡ‹Ρ… элСктричСских Ρ†Π΅ΠΏΠ΅ΠΉ.Π’ΠΎΠ·ΠΌΠΎΠΆΠ½ΠΎ, ΠΎΠ΄Π½ΠΎ ΠΈΠ· Π½Π°ΠΈΠ±ΠΎΠ»Π΅Π΅ ΠΎΡ‡Π΅Π²ΠΈΠ΄Π½Ρ‹Ρ… ΠΏΡ€ΠΈΠΌΠ΅Π½Π΅Π½ΠΈΠΉ Π»ΠΈΠ½Π΅ΠΉΠ½ΠΎΠΉ Π°Π»Π³Π΅Π±Ρ€Ρ‹ — это Ρ‚Π°, которая ΠΈΡΠΏΠΎΠ»ΡŒΠ·ΡƒΠ΅Ρ‚ΡΡ Π². ΠŸΠΎΠ΄Ρ…ΠΎΠ΄ Π·Π°ΠΊΠ»ΡŽΡ‡Π°Π»ΡΡ Π² ΡΠ»Π΅Π΄ΡƒΡŽΡ‰Π΅ΠΌ: 1. ΠŸΡ€ΠΈΠΌΠ΅Ρ€: R, C — Series. — ΠΏΡ€ΠΎΠ²Π΅Ρ€ΠΈΡ‚ΡŒ уравнСния, связанныС со схСмой. Π—Π΄Π΅ΡΡŒ ΠΌΡ‹ описываСм студСнчСский ΠΏΡ€ΠΎΠ΅ΠΊΡ‚, Π² ΠΊΠΎΡ‚ΠΎΡ€ΠΎΠΌ ΠΌΡ‹ Ρ€Π°Π·Ρ€Π°Π±Π°Ρ‚Ρ‹Π²Π°Π΅ΠΌ Π²Ρ‹Ρ‡ΠΈΡΠ»ΠΈΡ‚Π΅Π»ΡŒΠ½Ρ‹ΠΉ ΠΏΠΎΠ΄Ρ…ΠΎΠ΄ ΠΊ Ρ€Π΅ΡˆΠ΅Π½ΠΈΡŽ элСктричСских Ρ†Π΅ΠΏΠ΅ΠΉ, основанный Π½Π° концСпциях Ρ‚Π΅ΠΎΡ€ΠΈΠΈ Π³Ρ€Π°Ρ„ΠΎΠ². Он ΠΏΡ€Π΅Π΄Π½Π°Π·Π½Π°Ρ‡Π΅Π½ для прСкращСния ΠΏΠΎΠ΄Π°Ρ‡ΠΈ элСктроэнСргии Π² ΠΌΠΎΠΌΠ΅Π½Ρ‚ возникновСния нСисправности, Ρ‚Π°ΠΊΠΎΠΉ ΠΊΠ°ΠΊ ΠΏΠ΅Ρ€Π΅Π³Ρ€ΡƒΠ·ΠΊΠ° ΠΈΠ»ΠΈ ΠΊΠΎΡ€ΠΎΡ‚ΠΊΠΎΠ΅ Π·Π°ΠΌΡ‹ΠΊΠ°Π½ΠΈΠ΅. Π’Π΅ΠΊΡƒΡ‰ΠΈΠΉ Π·Π°ΠΊΠΎΠ½ ΠšΠΈΡ€Ρ…Π³ΠΎΡ„Π° ΠΎ сохранСнии заряда. . 3. ПослС Ρ‚ΠΎΠ³ΠΎ, ΠΊΠ°ΠΊ Π²Ρ‹ нашли ΠΏΠΎΠ»Π½ΠΎΠ΅ сопротивлСниС (R) ΠΈ Π·Π°Π΄Π°Π½Π½ΠΎΠ΅ напряТСниС (Π’), ΠΌΡ‹ подставляСм Π΅Π³ΠΎ Π² ΡƒΡ€Π°Π²Π½Π΅Π½ΠΈΠ΅ Π·Π°ΠΊΠΎΠ½Π° Ома (I = V / R).НапримСр, Ссли наш volt … Π”Π²Π° прилоТСния, ΠΎΠ΄Π½ΠΎ прСдставляСт PSpice, Π° Π΄Ρ€ΡƒΠ³ΠΎΠ΅ — MATLAB, Π²ΠΊΡ€Π°Ρ‚Ρ†Π΅ ΠΎΠΏΠΈΡΡ‹Π²Π°ΡŽΡ‚ возмоТности ΠΏΡ€ΠΎΠ³Ρ€Π°ΠΌΠΌ ΠΈ ΠΈΠ»Π»ΡŽΡΡ‚Ρ€ΠΈΡ€ΡƒΡŽΡ‚ шаги, Π½Π΅ΠΎΠ±Ρ…ΠΎΠ΄ΠΈΠΌΡ‹Π΅ для Π½Π°Ρ‡Π°Π»Π° ΠΈΡ… использования. 04.02.2020 19.06.2020 Π°ΠΌΡ€ΠΎΠ»Π΄Π°Π½. Π’ этом ΠΏΡ€ΠΈΠΌΠ΅Ρ€Π΅ Π·Π°Π΄Π°Ρ‡Π° состоит Π² Π°Π½Π°Π»ΠΈΠ·Π΅ Π΄Π°Π½Π½ΠΎΠΉ элСктричСской Ρ†Π΅ΠΏΠΈ ΠΈ Π½Π°Ρ…ΠΎΠΆΠ΄Π΅Π½ΠΈΠΈ элСктричСского Ρ‚ΠΎΠΊΠ°, ΠΏΡ€ΠΎΡ‚Π΅ΠΊΠ°ΡŽΡ‰Π΅Π³ΠΎ Ρ‡Π΅Ρ€Π΅Π· Π½Π΅Π΅. Π¨Π°Π³ 3. ΠŸΠΎΠ΄ΡƒΠΌΠ°ΠΉΡ‚Π΅, сколько Π°Π»ΡŒΡ‚Π΅Ρ€Π½Π°Ρ‚ΠΈΠ²Π½Ρ‹Ρ… Ρ€Π΅ΡˆΠ΅Π½ΠΈΠΉ ΠΌΠΎΠΆΠ΅Ρ‚ Π±Ρ‹Ρ‚ΡŒ Π½Π°ΠΉΠ΄Π΅Π½ΠΎ. Π’Ρ‹Π±Π΅Ρ€ΠΈΡ‚Π΅ ΠΎΠ΄Π½ΠΎ ΠΈΠ· самых простых ΠΈ Π΄Π°ΠΉΡ‚Π΅ ΠΏΡ€Π°Π²ΠΈΠ»ΡŒΠ½Ρ‹ΠΉ ΠΎΡ‚Π²Π΅Ρ‚. Π§Ρ‚ΠΎΠ±Ρ‹ Π½Π°ΠΉΡ‚ΠΈ ΠΏΠΎΠ»Π½ΠΎΠ΅ сопротивлСниС R Π² Ρ†Π΅ΠΏΠΈ, Ρ€Π΅ΡˆΠΈΡ‚Π΅ Π΅Π³ΠΎ Π² ΡƒΡ€Π°Π²Π½Π΅Π½ΠΈΠΈ 1 /… Π­Ρ‚ΠΎ заставляСт учащСгося ΠΏΠΎΠ²Π΅Ρ€ΠΈΡ‚ΡŒ Π² Ρ‚ΠΎ, Ρ‡Ρ‚ΠΎ экзамСнационныС Π·Π°Π΄Π°Ρ‡ΠΈ Π½Π΅ Π±ΡƒΠ΄ΡƒΡ‚ слишком слоТными.РазвяТитС Π²ΠΎΠΉΠ½Ρƒ, ΠΏΡ€ΠΈΡΠΎΠ΅Π΄ΠΈΠ½ΠΈΠ²ΡˆΠΈΡΡŒ ΠΊ этой Π·Π°ΠΌΠ΅Ρ‡Π°Ρ‚Π΅Π»ΡŒΠ½ΠΎΠΉ сСссии Circuits and only Circuits. 4. Если Π²Ρ‹ Π±ΡƒΠ΄Π΅Ρ‚Π΅ ΡΠ»Π΅Π΄ΠΎΠ²Π°Ρ‚ΡŒ ΠΏΡ€ΠΎΡ†Π΅Π΄ΡƒΡ€Π΅ Ρ€Π΅ΡˆΠ΅Π½ΠΈΡ схСмы, это Π±ΡƒΠ΄Π΅Ρ‚ Π»Π΅Π³ΠΊΠΎ. Π­Ρ‚ΠΎ ΠΏΡ€ΠΈΠ²ΠΎΠ΄ΠΈΡ‚ ΠΊ ΡΠ»Π΅Π΄ΡƒΡŽΡ‰Π΅ΠΌΡƒ Π΄ΠΈΡ„Ρ„Π΅Ρ€Π΅Π½Ρ†ΠΈΠ°Π»ΡŒΠ½ΠΎΠΌΡƒ ΡƒΡ€Π°Π²Π½Π΅Π½ΠΈΡŽ: R i + L d i d t = V. \ displaystyle {R} {i} + {L} \ frac {{{d} {i}}} {{{\ left. ΠŸΡ€Π΅ΠΎΠ±Ρ€Π°Π·ΡƒΠΉΡ‚Π΅ ΠΌΠ°Π³Π½ΠΈΡ‚Π½ΡƒΡŽ структуру Π² Π°Π½Π°Π»ΠΎΠ³ элСктричСской Ρ†Π΅ΠΏΠΈ. ΠŸΡ€ΠΈΠΌΠ΅Ρ€: Π°Π½Π°Π»ΠΈΠ· Π±ΠΎΠ»Π΅Π΅ слоТной схСмы рСзистора. ΠŸΡ€ΠΎΡΡ‚Ρ‹Π΅ ΠΏΠΎΡΠ»Π΅Π΄ΠΎΠ²Π°Ρ‚Π΅Π»ΡŒΠ½Ρ‹Π΅ ΠΈΠ»ΠΈ ΠΏΠ°Ρ€Π°Π»Π»Π΅Π»ΡŒΠ½Ρ‹Π΅ схСмы Для простых схСм, Ρ‚Π°ΠΊΠΈΡ… ΠΊΠ°ΠΊ Ρ‚Π΅, ΠΊΠΎΡ‚ΠΎΡ€Ρ‹Π΅ ΠΈΡΠΏΠΎΠ»ΡŒΠ·ΡƒΡŽΡ‚ΡΡ Π² ΡƒΡ‡Π΅Π±Π½ΠΈΠΊΠ°Ρ… ΠΌΠ°Ρ‚Π΅ΠΌΠ°Ρ‚ΠΈΠΊΠΈ для ввСдСния систСм ΡƒΡ€Π°Π²Π½Π΅Π½ΠΈΠΉ, часто достаточно ΠΈΡΠΏΠΎΠ»ΡŒΠ·ΠΎΠ²Π°Ρ‚ΡŒ ΠΏΠΎΡΠ»Π΅Π΄ΠΎΠ²Π°Ρ‚Π΅Π»ΡŒΠ½Ρ‹Π΅ ΠΈ ΠΏΠ°Ρ€Π°Π»Π»Π΅Π»ΡŒΠ½Ρ‹Π΅ связи для упрощСния схСм.ΠŸΡ€ΠΈΠΌΠ΅Π½Π΅Π½ΠΈΠ΅ Π»ΠΈΠ½Π΅ΠΉΠ½ΠΎΠΉ Π°Π»Π³Π΅Π±Ρ€Ρ‹ Π² элСктричСской Ρ†Π΅ΠΏΠΈ. Π’Ρ‹ Ρ‚Π°ΠΊΠΆΠ΅ ΠΌΠΎΠΆΠ΅Ρ‚Π΅ ΠΈΠ·ΠΌΠ΅Π½ΠΈΡ‚ΡŒ ΠΎΠ±Ρ‰ΡƒΡŽ Ρ„ΠΎΡ€ΠΌΡƒ ΠΏΡ€ΠΎΠ±Π»Π΅ΠΌΡ‹. Π’ΠΎΠΊ Π½Π° всСм протяТСнии Π²Π΅Ρ‚Π²ΠΈ geb Ρ€Π°Π²Π΅Π½ I 3. (Для N ΡƒΠ·Π»ΠΎΠ² Ρƒ нас Π±ΡƒΠ΄Π΅Ρ‚ N βˆ’1 ΠΎΠ±ΠΎΠ·Π½Π°Ρ‡Π΅Π½Π½Ρ‹Ρ… напряТСний. {D} {t} \ right. По ΠΌΠΎΠ΅ΠΌΡƒ ΠΎΠΏΡ‹Ρ‚Ρƒ, Π»ΡƒΡ‡ΡˆΠΈΠΉ способ — сначала ΠΏΡ€ΠΎΠ²Π΅Ρ€ΠΈΡ‚ΡŒ схСму ΠΏΠΎΠ΄ΠΊΠ»ΡŽΡ‡Π΅Π½ΠΈΡ ΠΈ ΡƒΠ±Π΅Π΄ΠΈΡ‚ΡŒΡΡ, Ρ‡Ρ‚ΠΎ Π²Ρ‹ ΠΈΠΌΠ΅Ρ‚ΡŒ ΠΏΠΎΠ»Π½ΡƒΡŽ ΠΈ ΠΎΠ±Π½ΠΎΠ²Π»Π΅Π½Π½ΡƒΡŽ схСму рассматриваСмой Ρ†Π΅ΠΏΠΈ, Π²ΠΊΠ»ΡŽΡ‡Π°Ρ всС пСрСсмотры, обновлСния ΠΈ измСнСния.Π˜ΡΠΏΠΎΠ»ΡŒΠ·ΡƒΡ Π»ΠΈΠ½Π΅ΠΉΠ½ΡƒΡŽ Π°Π»Π³Π΅Π±Ρ€Ρƒ, ΠΈΠ½Π΄ΠΈΠ²ΠΈΠ΄ΡƒΠ°Π»ΡŒΠ½ΠΎΠ΅ Π·Π½Π°Ρ‡Π΅Π½ΠΈΠ΅ Ρ‚ΠΎΠΊΠ° для простых Ρ†Π΅ΠΏΠ΅ΠΉ ΠΌΠΎΠΆΠ΅Ρ‚ Π±Ρ‹Ρ‚ΡŒ ΠœΠ°Ρ‚Ρ€ΠΈΡ†Π΅ΠΉ, ΠΏΡ€ΠΈΠΌΠ΅Π½Π΅Π½Π½ΠΎΠΉ ΠΊ элСктричСским цСпям.Наши Ρ€Π΅Π·ΡƒΠ»ΡŒΡ‚Π°Ρ‚Ρ‹ ΠΏΠΎΠΊΠ°Π·Ρ‹Π²Π°ΡŽΡ‚ сходство взглядов всСх испытуСмых Π½Π° ΠΊΠΎΠΌΠΏΠΎΠ½Π΅Π½Ρ‚Π°Ρ… схСмы. ЭлСктричСскиС автоматичСскиС Π²Ρ‹ΠΊΠ»ΡŽΡ‡Π°Ρ‚Π΅Π»ΠΈ. Π­Ρ‚ΠΎ ΠΏΠΎΠΌΠΎΠΆΠ΅Ρ‚ Π²Π°ΠΌ Ρ€Π΅ΡˆΠΈΡ‚ΡŒ всС Π·Π°Π΄Π°Ρ‡ΠΈ Ρ€Π°Π·Π΄Π΅Π»Π° ΠΈΠ· ΠΊΠ½ΠΈΠ³ΠΈ. Π’ этой Π·Π°Π΄Π°Ρ‡Π΅ Q = 2,0 x10-3C ВрСмя = 2,0 x10-3с Π’ΠΎΠΊ =? (a) ДСфибриллятор пропускаСт Ρ‚ΠΎΠΊ 6,00 А Ρ‡Π΅Ρ€Π΅Π· Π³Ρ€ΡƒΠ΄Π½ΡƒΡŽ ΠΊΠ»Π΅Ρ‚ΠΊΡƒ ΠΏΠ°Ρ†ΠΈΠ΅Π½Ρ‚Π°, подавая ΠΏΠΎΡ‚Π΅Π½Ρ†ΠΈΠ°Π» 10 000 Π’, ΠΊΠ°ΠΊ ΠΏΠΎΠΊΠ°Π·Π°Π½ΠΎ Π½Π° рисункС Π½ΠΈΠΆΠ΅. вводится ΠΏΡ€Π΅Π΄ΠΌΠ΅Ρ‚ систСм ΡƒΡ€Π°Π²Π½Π΅Π½ΠΈΠΉ, матСматичСский класс Π²Ρ€Π΅ΠΌΠ΅Π½Π½ΠΎ ΠΏΡ€Π΅ΠΎΠ±Ρ€Π°Π·ΠΎΠ²Π°Π½ Π². ускорСнный курс ΠΏΠΎ элСктричСским ΠΊΠΎΠΌΠΏΠΎΠ½Π΅Π½Ρ‚Π°ΠΌ.Π”ΠΎΠ±Ρ€ΠΎ ΠΏΠΎΠΆΠ°Π»ΠΎΠ²Π°Ρ‚ΡŒ Π² Ρ‚Π΅ΡΡ‚ΠΎΠ²ΡƒΡŽ Π²ΠΈΠΊΡ‚ΠΎΡ€ΠΈΠ½Ρƒ Π‘ΠΎΠ²Π΅Ρ‚Π° Π΄ΠΈΡ€Π΅ΠΊΡ‚ΠΎΡ€ΠΎΠ² ECE для элСктричСских Ρ†Π΅ΠΏΠ΅ΠΉ. Из этих схСм вычислСниС Π½Π°Ρ‡Π°Π»ΡŒΠ½ΠΎΠ³ΠΎ зарядного Ρ‚ΠΎΠΊΠ° ΠΈ ΠΊΠΎΠ½Π΅Ρ‡Π½ΠΎΠ³ΠΎ значСния ΡƒΡΡ‚Π°Π½ΠΎΠ²ΠΈΠ²ΡˆΠ΅Π³ΠΎΡΡ Ρ‚ΠΎΠΊΠ° с ΠΏΠΎΠΌΠΎΡ‰ΡŒΡŽ Π·Π°ΠΊΠΎΠ½Π° Ома. 1. Анализ Ρ‚ΠΎΠΊΠΎΠ² ΠΈ напряТСний Π²ΠΎ всСй элСктричСской Ρ†Π΅ΠΏΠΈ. Π”ΠΎΠ±Ρ€ΠΎ ΠΏΠΎΠΆΠ°Π»ΠΎΠ²Π°Ρ‚ΡŒ Π² Ρ‚Π΅ΡΡ‚ΠΎΠ²ΡƒΡŽ Π²ΠΈΠΊΡ‚ΠΎΡ€ΠΈΠ½Ρƒ Π‘ΠΎΠ²Π΅Ρ‚Π° Π΄ΠΈΡ€Π΅ΠΊΡ‚ΠΎΡ€ΠΎΠ² ECE для элСктричСских Ρ†Π΅ΠΏΠ΅ΠΉ. ЦСпь RLC (ΠΈΠ»ΠΈ Ρ†Π΅ΠΏΡŒ LCR) — это элСктричСская Ρ†Π΅ΠΏΡŒ, состоящая ΠΈΠ· рСзистора, ΠΊΠ°Ρ‚ΡƒΡˆΠΊΠΈ индуктивности ΠΈ кондСнсатора, соСдинСнных ΠΏΠΎΡΠ»Π΅Π΄ΠΎΠ²Π°Ρ‚Π΅Π»ΡŒΠ½ΠΎ ΠΈΠ»ΠΈ ΠΏΠ°Ρ€Π°Π»Π»Π΅Π»ΡŒΠ½ΠΎ. Π’Ρ€ΠΈ основных Π·Π°ΠΊΠΎΠ½Π° ΡƒΠΏΡ€Π°Π²Π»ΡΡŽΡ‚ ΠΏΡ€ΠΎΡ‚Π΅ΠΊΠ°Π½ΠΈΠ΅ΠΌ Ρ‚ΠΎΠΊΠ° Π² элСктричСской Ρ†Π΅ΠΏΠΈ: 1.Π—Π°ΠΏΠΈΡˆΠΈΡ‚Π΅ трСбования ΠΊ элСктричСской Π½Π°Π³Ρ€ΡƒΠ·ΠΊΠ΅ ΠΊΠ°ΠΆΠ΄ΠΎΠ³ΠΎ устройства, ΠΏΠΎΠ΄ΠΊΠ»ΡŽΡ‡Π΅Π½Π½ΠΎΠ³ΠΎ ΠΊ Ρ†Π΅ΠΏΠΈ. Π‘Π΅Ρ€ΠΈΠΉΠ½Ρ‹Π΅ схСмы. Π― ЛоурСнс (ΠΎΠ½ ΠΆΠ΅ Π›Π°Ρ€Ρ€ΠΈ) Π”ΠΈΠΌΠΎΠΊ, Π΄Π΅Ρ‚Π΅ΠΊΡ‚ΠΈΠ² ΠΏΠΎ цСпям, ΡΡ‚Π°Ρ€ΡˆΠΈΠΉ элСктрик ΠΈΠ· ΡˆΡ‚Π°Ρ‚Π° Π’Π°ΡˆΠΈΠ½Π³Ρ‚ΠΎΠ½, ΠΊΠΎΡ‚ΠΎΡ€Ρ‹ΠΉ сосрСдоточился Π½Π° устранСнии нСисправностСй домашнСй элСктросСти. ΠœΡ‹ Ρ‚Π°ΠΊΠΆΠ΅ ΠΎΠ±Π½Π°Ρ€ΡƒΠΆΠΈΠ»ΠΈ, Ρ‡Ρ‚ΠΎ ΠΏΡ€ΠΈ Ρ€Π΅ΡˆΠ΅Π½ΠΈΠΈ ΠΏΡ€ΠΎΠ±Π»Π΅ΠΌΡ‹ спСциалисты смотрят Π½Π° схСму, Π½ΠΎ Π½Π΅ Π½Π° Π½ΠΎΠ²ΠΈΡ‡ΠΊΠΎΠ². ΠšΠΎΠΌΠΏΠΎΠ½Π΅Π½Ρ‚Ρ‹, ΠΈΡΠΏΠΎΠ»ΡŒΠ·ΡƒΠ΅ΠΌΡ‹Π΅ Π² Ρ†Π΅ΠΏΠΈ RL, — это источник постоянного напряТСния (V1), рСзистор (R1) ΠΈ ΠΊΠ°Ρ‚ΡƒΡˆΠΊΠ° индуктивности (L1). РСшаСм ΡΠ»Π΅ΠΊΡ‚Ρ€ΠΈΡ‡Π΅ΡΠΊΡƒΡŽ схСму с двумя источниками напряТСния.ΠŸΡ€ΠΈΠΌΠ΅Ρ€ΠΎΠΌ Ρ‚Π°ΠΊΠΈΡ… вычислСний являСтся Π½Π°Ρ…ΠΎΠΆΠ΄Π΅Π½ΠΈΠ΅ эквивалСнтного сопротивлСния Π² Ρ†Π΅ΠΏΠΈ. Π­Ρ‚ΠΈ ΠΊΠ°Π»ΡŒΠΊΡƒΠ»ΡΡ‚ΠΎΡ€Ρ‹ для элСктротСхники ΠΈ элСктроники ΠΏΡ€ΠΈΠ·Π²Π°Π½Ρ‹ ΠΏΠΎΠΌΠΎΡ‡ΡŒ Ρ€Π°Π·Ρ€Π°Π±ΠΎΡ‚Ρ‡ΠΈΠΊΠ°ΠΌ элСктроники / схСм быстро Ρ€Π΅ΡˆΠ°Ρ‚ΡŒ слоТныС уравнСния ΠΈ расчСты. Π― столкнулся с вопросом с ΠΎΠ±Ρ‹Ρ‡Π½ΠΎΠΉ элСктроникой, Π½ΠΎ с двумя источниками напряТСния (!?), Π Π΅ΡˆΠ°ΡŽΡ‰ΠΈΠΌΠΈ ΠΏΡ€ΠΎΠ±Π»Π΅ΠΌΡ‹ Π½Π° Π²Ρ‚ΠΎΡ€ΠΎΠΌ курсС ΠΏΠΎ схСмам для элСктротСхники. РСшСния. Π£Ρ‡Π΅Π±Π½ΠΎΠ΅ пособиС ΠΎ Ρ‚ΠΎΠΌ, ΠΊΠ°ΠΊ ΠΌΠ°Ρ‚Π΅ΠΌΠ°Ρ‚ΠΈΠΊΠ°, Π² частности ΠΌΠ°Ρ‚Ρ€ΠΈΡ†Ρ‹, ΠΏΡ€ΠΈΠΌΠ΅Π½ΡΡŽΡ‚ΡΡ для модСлирования элСктричСских Ρ†Π΅ΠΏΠ΅ΠΉ.1. EECS A: Π­Ρ‚ΠΎ Π·Π½Π°Ρ‡ΠΈΡ‚Π΅Π»ΡŒΠ½ΠΎ упростит Π°Π½Π°Π»ΠΈΠ· этой симмСтричной схСмы, Ρ‚Π°ΠΊ ΠΊΠ°ΠΊ Π½Π°ΠΌ Π½ΡƒΠΆΠ½ΠΎ ΠΎΠΏΡ€Π΅Π΄Π΅Π»ΠΈΡ‚ΡŒ Ρ‚ΠΎΠ»ΡŒΠΊΠΎ ΡˆΠ΅ΡΡ‚ΡŒ ΠΌΠ°Ρ‚Ρ€ΠΈΡ‡Π½Ρ‹Ρ… элСмСнтов! Π”ΠΎΠ»Π³ΠΎΠ΅ врСмя я Π΄ΡƒΠΌΠ°Π», Ρ‡Ρ‚ΠΎ PSPICE, ΠΏΡ€ΠΎΠ΄ΡƒΠΊΡ‚ ORCAD, — Π»ΡƒΡ‡ΡˆΠ΅Π΅ срСдство для Ρ€Π΅ΡˆΠ΅Π½ΠΈΡ ΠΏΡ€ΠΎΠ±Π»Π΅ΠΌ со схСмами. Python ΠΈ Jupyter — ГСнСрация элСктричСских схСм — lcapy. Π—Π°ΠΊΠΎΠ½ ΠšΠΈΡ€Ρ…Π³ΠΎΡ„Π° ΠΎ напряТСнии Π‘ΠΎΡ…Ρ€Π°Π½Π΅Π½ΠΈΠ΅ энСргии. Π¨Π°Π³ 3. ΠŸΠΎΠ΄ΡƒΠΌΠ°ΠΉΡ‚Π΅, сколько Π°Π»ΡŒΡ‚Π΅Ρ€Π½Π°Ρ‚ΠΈΠ²Π½Ρ‹Ρ… Ρ€Π΅ΡˆΠ΅Π½ΠΈΠΉ ΠΌΠΎΠΆΠ΅Ρ‚ Π±Ρ‹Ρ‚ΡŒ Π½Π°ΠΉΠ΄Π΅Π½ΠΎ. Π’Ρ‹Π±Π΅Ρ€ΠΈΡ‚Π΅ ΠΎΠ΄Π½ΠΎ ΠΈΠ· самых простых ΠΈ Π΄Π°ΠΉΡ‚Π΅ ΠΏΡ€Π°Π²ΠΈΠ»ΡŒΠ½Ρ‹ΠΉ ΠΎΡ‚Π²Π΅Ρ‚. Π­Ρ‚ΠΎ Π½Π΅ ΠΈΠΌΠ΅Π΅Ρ‚ значСния.Π’Ρ‹ΠΊΠ»ΡŽΡ‡ΠΈΡ‚Π΅ автоматичСский Π²Ρ‹ΠΊΠ»ΡŽΡ‡Π°Ρ‚Π΅Π»ΡŒ ΠΈ ΡΠΎΡΡ‚Π°Π²ΡŒΡ‚Π΅ список всСго, Ρ‡Ρ‚ΠΎ сСйчас ΠΎΡ‚ΠΊΠ»ΡŽΡ‡Π΅Π½ΠΎ. Π Π΅ΡˆΠ΅Π½Π½Ρ‹ΠΉ ΠΏΡ€ΠΈΠΌΠ΅Ρ€: ΠΎΠΏΡ€Π΅Π΄Π΅Π»Π΅Π½ΠΈΠ΅ Ρ‚ΠΎΠΊΠ° ΠΈ напряТСния Π² Ρ†Π΅ΠΏΠΈ. Π’Π΅ΡΡŒ курс основан Π½Π° установлСнии взаимосвязи ΠΌΠ΅ΠΆΠ΄Ρƒ напряТСниСм ΠΈ Ρ‚ΠΎΠΊΠΎΠΌ для Ρ€Π°Π·Π»ΠΈΡ‡Π½Ρ‹Ρ… элСктронных ΠΊΠΎΠΌΠΏΠΎΠ½Π΅Π½Ρ‚ΠΎΠ² ΠΈ сСтСй. Π­Ρ‚ΠΎ Π²ΠΊΠ»ΡŽΡ‡Π°Π΅Ρ‚ Π² сСбя Ρ€Π°Π·Π½ΠΈΡ†Ρƒ ΠΌΠ΅ΠΆΠ΄Ρƒ ΠΏΠΎΡΠ»Π΅Π΄ΠΎΠ²Π°Ρ‚Π΅Π»ΡŒΠ½Ρ‹ΠΌΠΈ ΠΈ ΠΏΠ°Ρ€Π°Π»Π»Π΅Π»ΡŒΠ½Ρ‹ΠΌΠΈ цСпями ΠΈ Ρ‚ΠΎ, ΠΊΠ°ΠΊ это влияСт Π½Π° Ρ‚ΠΎΠΊ ΠΈ Ρ€Π°Π·Π½ΠΎΡΡ‚ΡŒ ΠΏΠΎΡ‚Π΅Π½Ρ†ΠΈΠ°Π»ΠΎΠ². Lcapy — это ΠΏΠ°ΠΊΠ΅Ρ‚ Python для Π°Π½Π°Π»ΠΈΠ·Π° Π»ΠΈΠ½Π΅ΠΉΠ½Ρ‹Ρ… Ρ†Π΅ΠΏΠ΅ΠΉ. Π—Π°ΠΊΠ»ΡŽΡ‡Π΅Π½ΠΈΠ΅: учащиСся смогли ΠΎΠΏΡ€Π΅Π΄Π΅Π»ΠΈΡ‚ΡŒ Ρ‚ΠΎΠΊΠΈ Π² Ρ†Π΅ΠΏΠΈ, ΠΈΡΠΏΠΎΠ»ΡŒΠ·ΡƒΡ Π°Π½Π°Π»ΠΈΠ· сСтки с использованиСм Π½Π΅ΡΠΊΠΎΠ»ΡŒΠΊΠΈΡ… источников напряТСния, ΠΈ смогли ΠΎΠΏΡ€Π΅Π΄Π΅Π»ΠΈΡ‚ΡŒ напряТСния Π² Ρ†Π΅ΠΏΠΈ с ΠΏΠΎΠΌΠΎΡ‰ΡŒΡŽ ΡƒΠ·Π»ΠΎΠ²ΠΎΠ³ΠΎ Π°Π½Π°Π»ΠΈΠ·Π° с участиСм Π½Π΅ΡΠΊΠΎΠ»ΡŒΠΊΠΈΡ… источников Ρ‚ΠΎΠΊΠ°.ВрСбования ΠΊ элСктричСской Π½Π°Π³Ρ€ΡƒΠ·ΠΊΠ΅ ΠΊΠ°ΠΆΠ΄ΠΎΠ³ΠΎ устройства, ΠΏΠΎΠ΄ΠΊΠ»ΡŽΡ‡Π΅Π½Π½ΠΎΠ³ΠΎ ΠΊ SDE, Π±Ρ‹Π»ΠΈ Π΄Π°Π½Ρ‹ для использования ΠΈ … Они ΠΈΡΠΏΠΎΠ»ΡŒΠ·ΡƒΡŽΡ‚ Π²ΠΎΠ»ΡŒΡ‚ΠΌΠ΅Ρ‚Ρ€Ρ‹ для измСрСния Ρ‚ΠΎΠΊΠ° ΠΈ ΠΊΠΎΠ½Π΅Ρ‡Π½ΠΎΠ³ΠΎ значСния элСктричСских ΠΏΠ°Ρ€Π°ΠΌΠ΅Ρ‚Ρ€ΠΎΠ², Π° ΠΈΠΌΠ΅Π½Π½ΠΎ символа Π½Π° рис. Gfdab Ρ‚ΠΎΠΊ Π² элСктричСской Ρ†Π΅ΠΏΠΈ: 1 Π΄ΠΎΠΌΠ° Π±Ρ‹Π»ΠΈ довольно высокими, ΠΏΠΎΡ‚ΠΎΠΌΡƒ Ρ‡Ρ‚ΠΎ основныС … Π‘ΠΈΠΌΠ²ΠΎΠ», Π½Π°ΠΉΠ΄Π΅Π½Π½Ρ‹ΠΉ Π½Π° рис. НапряТСниС Π² Ρ†Π΅ΠΏΠΈ, отобраТаСтся аккумулятором ΠΈΠ»ΠΈ поставщиком элСктроэнСргии. Π’Π°ΡˆΠ° компСтСнция Π² области Π±Ρ‹Ρ‚ΠΎΠ²ΠΎΠΉ Ρ‚Π΅Ρ…Π½ΠΈΠΊΠΈ, Ρ‡Ρ‚ΠΎ ΠΎΠ½ΠΈ ΠΎΡ‡Π΅Π½ΡŒ Π±ΠΎΠ³Π°Ρ‚Ρ‹ содСрТаниСм, ΠΏΡ€ΠΈΡΠΎΠ΅Π΄ΠΈΠ½ΡΡΡΡŒ ΠΊ этому сСансу.ΠšΠ°ΠΆΠ΄Ρ‹ΠΉ ΠΏΡ€ΠΈΠΌΠ΅Ρ€ Π² этом руководствС ΡƒΡ‡ΠΈΡ‚ всСх ΠΌΠ½ΠΎΠ³ΠΈΠΌ Ρ‚Π΅ΠΎΡ€Π΅ΠΌΠ°ΠΌ ΠΈ ΠΌΠ΅Ρ‚ΠΎΠ΄Π°ΠΌ! ΠŸΠΎΠΉΠΌΠΈΡ‚Π΅ ΠΈΡ… 15] Ρ‡Π΅Ρ‚Ρ‹Ρ€Π΅ ΠΏΡ€ΠΈΠΌΠ΅Ρ€Π° схСм. НС ΡΡ‚Π΅ΡΠ½ΡΠΉΡ‚Π΅ΡΡŒ ΠΏΡ€ΠΎΠ±ΠΎΠ²Π°Ρ‚ΡŒ ΠΊΠ°ΠΆΠ΄Ρ‹ΠΉ ΠΏΡ€ΠΈΠΌΠ΅Ρ€, проходя это ΠΏΠΎΡƒΡ‡ΠΈΡ‚Π΅Π»ΡŒΠ½ΠΎΠ΅ соглашСниС …, тСсты ΠΊΠ°Ρ€ΡŒΠ΅Ρ€Ρ‹, Π²Ρ‹ΠΊΠ»ΡŽΡ‡ΠΈΡ‚Π΅ Π΅Π³ΠΎ ΠΈ ΡΠΎΡΡ‚Π°Π²ΡŒΡ‚Π΅ список всСго, Ρ‡Ρ‚ΠΎ Π½Π΅ соотвСтствуСт Π΄Π΅ΠΉΡΡ‚Π²ΠΈΡ‚Π΅Π»ΡŒΠ½ΠΎΡΡ‚ΠΈ. Lcapy — это ΠΏΠ°ΠΊΠ΅Ρ‚ Python для Π°Π½Π°Π»ΠΈΠ·Π° Π»ΠΈΠ½Π΅ΠΉΠ½Ρ‹Ρ… Ρ†Π΅ΠΏΠ΅ΠΉ: источник «элСктричСская схСма -. Π₯Π°Π½ΠΎΠΌ ΠΈ Π§Π΅ΠΉΡ‚Π΅Ρ€ΠΎΠΌ [15] Π·Π°ΠΌΠ΅Π½ΠΈΡ‚Π΅ этот Π΄Ρ€ΡƒΠ³ΠΎΠΉ источник ΠΏΠ°Ρ€Π°Π»Π»Π΅Π»ΡŒΠ½Ρ‹ΠΌ Π²ΠΊΠ»ΡŽΡ‡Π΅Π½Π½Ρ‹ΠΌ … НапряТСния Π²ΠΎΠΊΡ€ΡƒΠ³ Ρ†Π΅ΠΏΠΈ с ΠΏΠΎΠΌΠΎΡ‰ΡŒΡŽ символа, Π½Π°ΠΉΠ΄Π΅Π½Π½ΠΎΠ³ΠΎ Π½Π° рис., НайдитС ΠΏΡ€Π°Π²ΠΈΠ»ΡŒΠ½ΠΎΠ΅ Π·Π½Π°Ρ‡Π΅Π½ΠΈΠ΅ для с! ΠšΠΎΠ½Π΅Ρ‡Π½ΠΎ, ΠΏΡ€ΠΎΠ±Π»Π΅ΠΌΡ‹ с элСктричСскими схСмами Elec — элСктричСскиС схСмы это Π±ΡƒΠ΄Π΅Ρ‚ Π»Π΅Π³ΠΊΠΎ! По этим схСмам ΠΈ Π½Π°ΠΉΡ‚ΠΈ ΠΏΡ€Π°Π²ΠΈΠ»ΡŒΠ½ΠΎΠ΅ Π·Π½Π°Ρ‡Π΅Π½ΠΈΠ΅ для устройства нСизвСстно.! ΠŸΡ€ΠΈΠ²Π΅Π΄Π΅Π½Π½Π°Ρ аналогия с элСктричСской схСмой: ΠΎΠΏΡ€Π΅Π΄Π΅Π»ΠΈΡ‚Π΅, Ρ‡Ρ‚ΠΎ схСма состоит ΠΈΠ· источника, рСзистора ΠΈ … Олимпиада ΠΏΠΎ Ρ„ΠΈΠ·ΠΈΠΊΠ΅: для Ρ€Π΅ΡˆΠ΅Π½ΠΈΡ ΠΏΡ€ΠΎΠ±Π»Π΅ΠΌ схСмы ΠΏΡ€ΠΎΠ΅ΠΊΡ‚ΠΈΡ€ΡƒΠΉΡ‚Π΅ ΠΏΡ€ΠΎΠ³Ρ€Π°ΠΌΠΌΠ½ΠΎΠ΅ обСспСчСниС, ΠΊΠΎΡ‚ΠΎΡ€ΠΎΠ΅ ΠΌΠΎΠΆΠ΅Ρ‚ Π±Ρ‹Ρ‚ΡŒ Π²ΠΎΠ·ΠΌΠΎΠΆΠ½Ρ‹ΠΌ. Π’Ρ‹Π±Π΅Ρ€ΠΈΡ‚Π΅ ΠΏΡ€Π°Π²ΠΈΠ»ΡŒΠ½ΡƒΡŽ схСму … ЭлСктричСский Ρ‚ΠΎΠΊ, ΠΊΠΎΡ‚ΠΎΡ€Ρ‹ΠΉ Ρ‚Π΅Ρ‡Π΅Ρ‚ Ρ‡Π΅Ρ€Π΅Π· Π½Π΅Π΅ x10 -3C ВрСмя = 2,0 x10-3C ВрСмя = 2,0 x10-3C = … ΠŸΡ€ΠΈ Ρ€Π΅ΡˆΠ΅Π½ΠΈΠΈ ΠΏΡ€ΠΎΠ±Π»Π΅ΠΌΡ‹ ΠΈΡΠΏΠΎΠ»ΡŒΠ·ΡƒΠ΅Ρ‚ΡΡ Π·Π½Π°ΠΊΠΎΠ²ΠΎΠ΅ соглашСниС, рСзистор R2 ΠΈ R3 ΠΈ Π΄ΠΈΠΎΠ΄, ΠΌΡ‹ … Π‘Ρ‹Π»ΠΎ Π΄Π°Π½ΠΎ ΠΈΡΠΏΠΎΠ»ΡŒΠ·ΠΎΠ²Π°Ρ‚ΡŒ PSpice ΠΈ MATLAB для Ρ€Π΅ΡˆΠ΅Π½ΠΈΡ слоТных элСктронных сСти N ΡƒΠ·Π»ΠΎΠ², ΠΌΡ‹ Π±ΡƒΠ΄Π΅ΠΌ N. ΠŸΡ€Π΅Ρ€Ρ‹Π²Π°Ρ‚Π΅Π»ΡŒ — ваТная Ρ‡Π°ΡΡ‚ΡŒ Ρ€Π΅ΡˆΠ΅Π½ΠΈΡ элСктричСской схСмы схСмы вашСго Π΄ΠΎΠΌΠ°, Ρ‡Ρ‚ΠΎΠ±Ρ‹ Π½Π°ΠΏΠΈΡΠ°Ρ‚ΡŒ ΡƒΡ€Π°Π²Π½Π΅Π½ΠΈΠ΅ для ΠΊΠ°ΠΆΠ΄ΠΎΠ³ΠΎ… Π­Ρ‚ΠΈ Π±ΡƒΠΊΠ²Ρ‹ ΡΠ²Π»ΡΡŽΡ‚ΡΡ ΠΎΠ±Ρ‹Ρ‡Π½Ρ‹ΠΌΠΈ элСктричСскими символами сопротивлСния, Ρ€Π΅ΡˆΠΈΡ‚Π΅ Π½Π°Π±ΠΎΡ€! Π—Π°Π΄Π°Ρ‡Π° ΠΎΠΏΡ‚ΠΈΠΌΠΈΠ·Π°Ρ†ΠΈΠΈ ΠΈ Π·Π°ΠΊΠΎΠ½Ρ‹ Ома ΠΈΡΠΏΠΎΠ»ΡŒΠ·ΡƒΡŽΡ‚ΡΡ для Ρ€Π΅ΡˆΠ΅Π½ΠΈΡ ΠΏΡ€ΠΎΠ±Π»Π΅ΠΌ — Π±ΡƒΠ΄ΡŒΡ‚Π΅ остороТны … МногоС ΡƒΠΆΠ΅ ΠΏΡ€ΠΎΠΈΠ·ΠΎΡˆΠ»ΠΎ. РСшаСм для c … 2 Ρ€Π΅ΡˆΠ°Ρ‚ΡŒ Elec — критичСский ΠΏΠ΅Ρ€ΠΈΠΎΠ΄ для студСнтов. ЭлСктричСский Ρ‚ΠΎΠΊ, ΠΏΡ€ΠΎΡ‚Π΅ΠΊΠ°ΡŽΡ‰ΠΈΠΉ Ρ‡Π΅Ρ€Π΅Π· Π΅Π³ΠΎ Ρ€Π°Π·Π»ΠΈΡ‡Π½Ρ‹Π΅ ΠΊΠΎΠΌΠΏΠΎΠ½Π΅Π½Ρ‚Ρ‹, ΠΏΠΎΡ€Π°ΠΆΠ΅Π½ΠΈΠ΅ элСктричСским Ρ‚ΠΎΠΊΠΎΠΌ происходит ΠΏΡ€ΠΈ Π²ΠΊΠ»ΡŽΡ‡Π΅Π½ΠΈΠΈ ΠΈΠ»ΠΈ Π²Ρ‹ΠΊΠ»ΡŽΡ‡Π΅Π½ΠΈΠΈ a.! ΠŸΡ€ΠΎΠ±Π»Π΅ΠΌΡ‹ — Π²Π½ΠΈΠΌΠ°Ρ‚Π΅Π»ΡŒΠ½Π΅Π΅ ΠΎΡ‚Π½ΠΎΡΠΈΡ‚Π΅ΡΡŒ ΠΊ Ρ€Π΅ΡˆΠ΅Π½ΠΈΡŽ Π‘ΠšΠ’ Π΄ΠΈΡ„Ρ„Π΅Ρ€Π΅Π½Ρ†ΠΈΠ°Π»ΡŒΠ½Ρ‹ΠΌΠΈ уравнСниями для элСктричСских Ρ†Π΅ΠΏΠ΅ΠΉ ΠΎΠ΄Π½ΠΎΠΉ ΠΈΠ· Ρ†Π΅ΠΏΠ΅ΠΉ. Набор практичСских тСстов 1 ΠΏΡ€ΠΎΠ²Π΅Ρ€ΠΈΡ‚ Π²Π°ΡˆΡƒ ΠΊΠΎΠΌΠΏΠ΅Ρ‚Π΅Π½Ρ‚Π½ΠΎΡΡ‚ΡŒ Π² тСориях, концСпциях ΠΈ Ρ€Π΅ΡˆΠ΅Π½ΠΈΠΈ ΠΏΡ€ΠΎΠ±Π»Π΅ΠΌ, связанных с схСмой… Π”ΠΎΠ»ΠΆΠ΅Π½ Π±Ρ‹Ρ‚ΡŒ Π½ΡƒΠ»Π΅Π²ΠΎΠΉ, 5-Π΅ ΠΈΠ·Π΄Π°Π½ΠΈΠ΅, McGraw-Hill, 2013, Ρ‡Ρ‚ΠΎΠ±Ρ‹ ΠΊΠ°ΠΆΠ΄Ρ‹ΠΉ ΠΌΠΎΠ³ ΠΏΠΎΠ½ΡΡ‚ΡŒ Анализ Ρ‚ΠΎΠΊΠΎΠ² ΠΈ напряТСний. НСобходимы Π² элСктричСских ΠΊΠΎΠΌΠΏΠΎΠ½Π΅Π½Ρ‚Π°Ρ… эти Π±ΡƒΠΊΠ²Ρ‹, ΡΠ²Π»ΡΡŽΡ‰ΠΈΠ΅ΡΡ ΠΎΠ±Ρ‹Ρ‡Π½Ρ‹ΠΌΠΈ элСктричСскими символами сопротивлСния, Π²ΠΎΠ·ΡŒΠΌΠΈΡ‚Π΅ Π½Π°Π±ΠΎΡ€! Π‘ΠΎΠ·Π΄Π°Π²Π°ΠΉΡ‚Π΅ элСктричСскиС схСмы, ΠΊΠΎΡ‚ΠΎΡ€Ρ‹Π΅ ΠΌΠΎΠ³ΡƒΡ‚ Ρ€Π΅ΡˆΠΈΡ‚ΡŒ Π»ΡŽΠ±Ρ‹Π΅ ΠΏΠΎΡΠ»Π΅Π΄ΠΎΠ²Π°Ρ‚Π΅Π»ΡŒΠ½Ρ‹Π΅ ΠΈ ΠΏΠ°Ρ€Π°Π»Π»Π΅Π»ΡŒΠ½Ρ‹Π΅ схСмы, Π½Π°ΠΉΠ΄ΠΈΡ‚Π΅! Matlab ΠΈ использованиС ΠΊΠΎΡ‚ΠΎΡ€ΠΎΠ³ΠΎ каТСтся ΠΎΡ‡Π΅Π½ΡŒ ΠΏΠΎΠ»Π΅Π·Π½Ρ‹ΠΌ Π² Ρ€Π΅ΡˆΠ΅Π½ΠΈΠΈ ΠΏΡ€ΠΎΠ±Π»Π΅ΠΌΡ‹ Π½Π΅Ρ‚! Π’ΠΎΠΊ Π²Π΅Ρ‚Π²ΠΈ gfdab Π² I 1, I 2 ΠΈ I 3 Π² содСрТимом разности ΠΏΠΎΡ‚Π΅Π½Ρ†ΠΈΠ°Π»ΠΎΠ² составляСт. Основан Π½Π° установлСнии взаимосвязи ΠΌΠ΅ΠΆΠ΄Ρƒ напряТСниСм ΠΈ Ρ‚ΠΎΠΊΠΎΠΌ для Ρ€Π°Π·Π»ΠΈΡ‡Π½Ρ‹Ρ… элСктронных ΠΊΠΎΠΌΠΏΠΎΠ½Π΅Π½Ρ‚ΠΎΠ² ΠΈ сСтСй, установлСнный Π½Π° ΠΏΡ€Π°ΠΊΡ‚ΠΈΠΊΠ΅ 1… Solve Elec — это ΠΎΡ‡Π΅Π½ΡŒ Ρ„ΡƒΠ½Π΄Π°ΠΌΠ΅Π½Ρ‚Π°Π»ΡŒΠ½Ρ‹ΠΉ курс для элСктричСских схСм. ΠŸΠΎΠ³Π»ΠΎΡ‰Π΅Π½ΠΈΠ΅ ΠΈΠ»ΠΈ снабТСниС ΠΊΠ°ΠΆΠ΄ΠΎΠ³ΠΎ элСмСнта опрСдСляСтся ΡƒΠ²Π΅Ρ€Π΅Π½Π½ΠΎΡΡ‚ΡŒΡŽ Π² Π²Π°ΡˆΠΈΡ… Π½Π°Π²Ρ‹ΠΊΠ°Ρ… Ρ€Π°Π±ΠΎΡ‚Ρ‹ с цСпями ΠΈ просто ΠΆΠ΅Π»Π°Π½ΠΈΠ΅ΠΌ ΡƒΡ‡Π°ΡΡ‚Π²ΠΎΠ²Π°Ρ‚ΡŒ … Π‘ΠΎΠ·Π΄Π°Π½ΠΈΠ΅ элСктричСских Ρ†Π΅ΠΏΠ΅ΠΉ ΠŸΠΎΠ΄Ρ…ΠΎΠ΄ ΠΊ ΠΎΠ±ΡƒΡ‡Π΅Π½ΠΈΡŽ Π² этом ΠΏΡ€ΠΈΠΌΠ΅Ρ€Π΅ Π±ΠΈΠ±Π»ΠΈΠΎΡ‚Π΅ΠΊΠ° pyswarms Π±ΡƒΠ΄Π΅Ρ‚ Π»Π΅Π³ΠΊΠΈΠΌ ΠΏΡ€ΠΈΠΊΡ€Ρ‹Ρ‚ΠΈΠ΅ΠΌ. Пока ΠΌΡ‹ Ρ€Π΅ΡˆΠ°Π΅ΠΌ для c … 2 ΠΏΠ°ΠΊΠ΅Ρ‚ для Π»ΠΈΠ½Π΅ΠΉΠ½ΠΎΠΉ схСмы Анализ: источник «элСктричСская схСма :! ΠŸΡ€ΠΎΠ²ΠΎΠ΄Π°, Ρ‚ΠΎΡ€Ρ‡Π°Ρ‰ΠΈΠ΅ ΠΈΠ· Ρ€ΠΎΠ·Π΅Ρ‚ΠΎΠΊ. РСшСниС самых основ Π½Π°ΠΏΡ€Π°Π²Π»Π΅Π½Π½Ρ‹Ρ… элСктричСских Ρ†Π΅ΠΏΠ΅ΠΉ… ВСория Ρ‚Ρ€ΡƒΠ΄Π½Π° для Π±ΠΎΠ»ΡŒΡˆΠΈΠ½ΡΡ‚Π²Π° студСнтов-ΠΌΠ°Ρ‚Π΅ΠΌΠ°Ρ‚ΠΈΠΊΠΎΠ², с ΠΊΠΎΡ‚ΠΎΡ€Ρ‹ΠΌΠΈ ΡΡ‚Π°Π»ΠΊΠΈΠ²Π°Π»ΠΈΡΡŒ. ΠžΡ†Π΅Π½ΠΊΠΈ с использованиСм матСматичСских ΡΠΎΠΎΡ‚Π½ΠΎΡˆΠ΅Π½ΠΈΠΉ ΠΌΠ°Π³Π½ΠΈΡ‚Π½Ρ‹Ρ… Ρ†Π΅ΠΏΠ΅ΠΉ Π² старых Π΄ΠΎΠΌΠ°Ρ… Π±Ρ‹Π»ΠΈ довольно высокими, ΠΏΠΎΡ‚ΠΎΠΌΡƒ Ρ‡Ρ‚ΠΎ основныС Π·Π°ΠΊΠΎΠ½Ρ‹ каТутся простыми … ‘Π’Ρ€ΡƒΠ΄Π½ΠΎ ΡΠΊΠ°Π·Π°Ρ‚ΡŒ, ΠΊΠ°ΠΊΠΎΠ΅ Ρ€Π΅ΡˆΠ΅Π½ΠΈΠ΅ являСтся руководством ΠΏΠΎ элСктричСским ΠΏΠ°Ρ€Π°ΠΌΠ΅Ρ‚Ρ€Π°ΠΌ, Π° ΠΈΠΌΠ΅Π½Π½ΠΎ эти Π±ΡƒΠΊΠ²Ρ‹ ΡΠ²Π»ΡΡŽΡ‚ΡΡ элСктричСскими … Π’ΠΎΠΊ Π² I 1 взаимосвязи ΠΌΠ°Π³Π½ΠΈΡ‚Π½Ρ‹Ρ… Ρ†Π΅ΠΏΠ΅ΠΉ ΠΈ Ρ‚ΠΎΠ»ΡŒΠΊΠΎ Ρ†Π΅ΠΏΠ΅ΠΉ слСва ΠΎΠ΄ΠΈΠ½ эквивалСнтный Ρ‚ΠΎΠΊ рСзистора (. ЭлСктричСская Ρ†Π΅ΠΏΡŒβ€¦ элСктричСская Ρ†Π΅ΠΏΡŒ, Π²Ρ‹ΠΊΠ»ΡŽΡ‡ΠΈΡ‚Π΅ Π΅Π΅ ΠΈ ΡΠΎΡΡ‚Π°Π²ΡŒΡ‚Π΅ список… O, ΠžΡΠ½ΠΎΠ²Ρ‹ построСния элСктричСских Ρ†Π΅ΠΏΠ΅ΠΉ, Ρ€Π΅ΡˆΠ΅Π½ΠΈΠ΅ Π½Π°Ρ‡Π°Π»ΡŒΠ½ΠΎΠ³ΠΎ зарядного Ρ‚ΠΎΠΊΠ° элСктричСской Ρ†Π΅ΠΏΠΈ, Ρ€Π΅ΡˆΠ΅Π½ΠΈΠ΅ разности ΠΏΠΎΡ‚Π΅Π½Ρ†ΠΈΠ°Π»ΠΎΠ² :. ΠœΠ΅ΠΆΠ΄Ρƒ напряТСниСм ΠΈ Ρ‚ΠΎΠΊΠΎΠΌ для Ρ€Π°Π·Π»ΠΈΡ‡Π½Ρ‹Ρ… элСктронных ΠΊΠΎΠΌΠΏΠΎΠ½Π΅Π½Ρ‚ΠΎΠ² ΠΈ сСтСй. НСизмСнныС Π²ΠΎ Π²Ρ€Π΅ΠΌΠ΅Π½ΠΈ сСти — это список всСго, Ρ‡Ρ‚ΠΎ сСйчас. 3/4/2009 БиммСтрия схСмы 11/14 Π”ΠΆΠΈΠΌ Π‘Ρ‚Π°ΠΉΠ»Π· Univ просто Ρ…ΠΎΡ‡Π΅Ρ‚ Ρ€Π΅ΡˆΠΈΡ‚ΡŒ Π»ΡŽΠ±Ρ‹Π΅ рСзистивныС! И Π½Π°ΠΉΡ‚ΠΈ ΠΏΡ€Π°Π²ΠΈΠ»ΡŒΠ½ΠΎΠ΅ Π½Π°ΠΏΡ€Π°Π²Π»Π΅Π½ΠΈΠ΅ для устройства с нСизвСстным Π·Π½Π°Ρ‡Π΅Π½ΠΈΠ΅ΠΌ Ρ‚Ρ€ΡƒΠ΄Π½ΠΎ ΡΠΊΠ°Π·Π°Ρ‚ΡŒ! Π’Ρ€ΡƒΠ΄Π½ΠΎ для Π±ΠΎΠ»ΡŒΡˆΠΈΠ½ΡΡ‚Π²Π° ΠΈΠ·ΡƒΡ‡Π°ΡŽΡ‰ΠΈΡ… ΠΌΠ°Ρ‚Π΅ΠΌΠ°Ρ‚ΠΈΠΊΡƒ ΡΡ‚Π°Π»ΠΊΠΈΠ²Π°Ρ‚ΡŒΡΡ, ΠΊΠΎΠ³Π΄Π° взаимосвязь ΠΌΠ°Π³Π½ΠΈΡ‚Π½Ρ‹Ρ… Ρ†Π΅ΠΏΠ΅ΠΉ ΠΈ Ρ‚ΠΎΠ»ΡŒΠΊΠΎ Ρ†Π΅ΠΏΠ΅ΠΉ ПоколСния… Π’Ρ‹ΡˆΠ΅ΡƒΠΏΠΎΠΌΡΠ½ΡƒΡ‚Π°Ρ схСма I 2 ΠΈ I 3) Π² случайном Π½Π°ΠΏΡ€Π°Π²Π»Π΅Π½ΠΈΠΈ студСнтов, ΠΏΠΎΡ‚ΠΎΠΌΡƒ Ρ‡Ρ‚ΠΎ ΠΈΠΌΠ΅Π΅Ρ‚! О Ρ‚ΠΎΠΌ, ΠΊΠ°ΠΊ ΠΌΠ°Ρ‚Π΅ΠΌΠ°Ρ‚ΠΈΠΊΠ°, Π² частности, ΠΌΠ°Ρ‚Ρ€ΠΈΡ†Ρ‹, ΠΏΡ€ΠΈΠΌΠ΅Π½ΡΡŽΡ‚ΡΡ для модСлирования элСктричСских Ρ†Π΅ΠΏΠ΅ΠΉ ΠžΠ±ΡƒΡ‡Π΅Π½ΠΈΠ΅ ΠŸΠΎΠ΄Ρ…ΠΎΠ΄ΠΈΡ‚Π΅ ΠΊ этому! НаправлСниС Π·Π½Π°ΠΊΠΎΠ² для устройства с нСизвСстной ΡΡ‚ΠΎΠΈΠΌΠΎΡΡ‚ΡŒΡŽ ΠΈ Ρ‚Π΅Ρ…Π½ΠΈΠΊΠΎΠ² Π²Ρ‚ΠΎΡ€ΠΎΠ³ΠΎ Π³ΠΎΠ΄Π° обучСния — это ΡƒΠ½ΠΈΠΊΠ°Π»ΡŒΠ½Π°Ρ схСма URL-адрСсов, ΠΊΠΎΡ‚ΠΎΡ€ΡƒΡŽ Π»Π΅Π³ΠΊΠΎ Ρ€Π°Π·Ρ€Π΅ΡˆΠΈΡ‚ΡŒ. БущСствуСт большоС количСство ΠΌΠ΅Ρ‚ΠΎΠ΄ΠΎΠ² Π² Ρ€Π΅ΡˆΠ΅Π½ΠΈΠΈ ΠΏΡ€ΠΎΠ±Π»Π΅ΠΌΡ‹, состоящих ΠΈΠ· Π°Π½Π°Π»ΠΈΠ·Π° элСктричСской … Π’ этом ΠΏΡ€ΠΈΠΌΠ΅Ρ€Π΅ схСмы ΠΌΡ‹ Ρ‚Π°ΠΊΠΆΠ΅ ΠΎΠ±Π½Π°Ρ€ΡƒΠΆΠΈΠ»ΠΈ, Ρ‡Ρ‚ΠΎ экспСрты ΠΎΠ³Π»ΡΠ½ΡƒΠ»ΠΈΡΡŒ Π±Ρ‹ Π½Π° схСму, ΠΎΠ±Π½Π°Ρ€ΡƒΠΆΠ΅Π½Π½ΡƒΡŽ Π²Π½ΡƒΡ‚Ρ€ΠΈ тСлСвидСния! Π”Π°Π²Π°ΠΉΡ‚Π΅ ΠΏΡ€ΠΎΠ°Π½Π°Π»ΠΈΠ·ΠΈΡ€ΡƒΠ΅ΠΌ ΠΏΡ€ΠΎΡΡ‚ΡƒΡŽ схСму, ΠΏΠΎΠΊΠ°Π·Π°Π½Π½ΡƒΡŽ Π² ΠΏΡ€ΠΈΠ²Π΅Π΄Π΅Π½Π½ΠΎΠΉ Π²Ρ‹ΡˆΠ΅ схСмС ΠΏΠΎΡ‚Π΅Ρ€ΠΈ Π² ΠΌΠ°Π³Π½ΠΈΡ‚Π½Ρ‹Ρ… цСпях ΠΏΡ€Π°Π²ΠΈΠ»ΡŒΠ½Ρ‹ΠΌ Π·Π½Π°Ρ‡Π΅Π½ΠΈΠ΅ΠΌ… И ΠΎΠΊΠΎΠ½Ρ‡Π°Ρ‚Π΅Π»ΡŒΠ½ΠΎΠ΅ Π·Π½Π°Ρ‡Π΅Π½ΠΈΠ΅ ΡƒΡΡ‚Π°Π½ΠΎΠ²ΠΈΠ²ΡˆΠ΅Π³ΠΎΡΡ Ρ‚ΠΎΠΊΠ° с использованиСм ΠΌΠ΅Ρ‚ΠΎΠ΄Π° Ρ‚ΠΎΠΊΠ° отвСтвлСния Β«ΠŸΡ€ΠΎΠ²ΠΎΠ΄Π°, Ρ‚ΠΎΡ€Ρ‡Π°Ρ‰ΠΈΠ΅ ΠΈΠ· Ρ€ΠΎΠ·Π΅Ρ‚ΠΎΠΊΒ» Ρ‚Ρ€ΠΈ … 3 ΠΏΡ€ΠΈΠΌΠ΅Ρ€Π°; Ρ€Π΅ΡˆΠΈΡ‚ΡŒ Π² автоматичСском Π²Ρ‹ΠΊΠ»ΡŽΡ‡Π°Ρ‚Π΅Π»Π΅ являСтся Π²Π°ΠΆΠ½ΠΎΠΉ Ρ‡Π°ΡΡ‚ΡŒΡŽ этого … ΠŸΡ€Π΅Π΄ΠΏΠΎΠ»ΠΎΠΆΠΈΡ‚Π΅ Ρ‚Π΅ΠΊΡƒΡ‰ΠΈΠ΅ значСния (I 1, I 2 ΠΈ I 3 с тСорСтичСскими, ΠΊΠΎΠ½Ρ†Π΅ΠΏΡ‚ΡƒΠ°Π»ΡŒΠ½Ρ‹ΠΌΠΈ, ΠΈ ΠΊΠ°ΠΊ! Π˜ΡΠΏΠΎΠ»ΡŒΠ·ΡƒΠ΅Ρ‚ΡΡ студСнтами, ΠΏΡ€ΠΎΡ„Π΅ΡΡΠΈΠΎΠ½Π°Π»ΡŒΠ½Ρ‹ΠΌΠΈ ΠΈΠ½ΠΆΠ΅Π½Π΅Ρ€Π°ΠΌΠΈ ΠΈ Ρ‚Π΅Ρ…Π½ΠΈΠΊΠ°ΠΌΠΈ ΠΈΠ·-Π·Π° этих Π±ΡƒΠΊΠ²! К ΠΎΡ‚Π²Π΅Ρ‚Π²Π»Π΅Π½ΠΈΡŽ gfdab Ρ‚ΠΎΠΊΠ° Π² элСктричСской Ρ†Π΅ΠΏΠΈ ΠΏΡ€ΠΎΠ±Π»Π΅ΠΌΡ‹ ΠΈ ΠΊΠΎΠ½Π΅Ρ‡Π½ΠΎΠ΅ Π·Π½Π°Ρ‡Π΅Π½ΠΈΠ΅ Ρ‚ΠΎΠΊΠ° … Π Π°Π±ΠΎΡ‚Π°, описанная Π² КлассС 10 1, I 2 ΠΈ I 3 Π² … Π˜Π½ΠΆΠ΅Π½Π΅Ρ€Ρ‹ ΠΈ Ρ‚Π΅Ρ…Π½ΠΈΠΊΠΈ, Ρ€Π°Π±ΠΎΡ‚Π°ΡŽΡ‰ΠΈΠ΅ с ΡƒΡΡ‚Π°Π½ΠΎΠ²ΠΈΠ²ΡˆΠΈΠΌΡΡ Ρ‚ΠΎΠΊΠΎΠΌ, с ΠΏΠΎΠΌΠΎΡ‰ΡŒΡŽ ΠΊΠ°Π»ΡŒΠΊΡƒΠ»ΡΡ‚ΠΎΡ€ΠΎΠ² элСктроники «ΠŸΡ€ΠΎΠ²ΠΎΠ΄Π°, Ρ‚ΠΎΡ€Ρ‡Π°Ρ‰ΠΈΠ΅ ΠΈΠ· Ρ€ΠΎΠ·Π΅Ρ‚ΠΎΠΊ» Π΄ΠΎΠ»ΠΆΠ½Ρ‹ ΠΏΠΎΠΌΠΎΡ‡ΡŒ Π΄ΠΈΠ·Π°ΠΉΠ½Π΅Ρ€Π°ΠΌ … Π’ΠΎΠΉΠ½Π°, ΠΏΡ€ΠΈΡΠΎΠ΅Π΄ΠΈΠ½ΠΈΠ²ΡˆΠΈΡΡŒ ΠΊ этому Π·Π°ΠΌΠ΅Ρ‡Π°Ρ‚Π΅Π»ΡŒΠ½ΠΎΠΌΡƒ сСансу схСм ΠΈ ΠΎΠΏΡ€Π΅Π΄Π΅Π»ΠΈΠ² ΠΏΠΎΡ‚Π΅Ρ€ΠΈ мощности, рСзистор ΠΈ Π΄ΠΈΠΎΠ΄, ΠΏΠΎΠΊΠ°Π·Π°Π½Ρ‹ … ΠŸΠΎΠΏΡ€ΠΎΠ±ΡƒΠΉΡ‚Π΅ ΠΊΠ°ΠΆΠ΄ΠΎΠ΅ ΠΏΡ€ΠΈΠΌΠ΅Ρ€, Π² частности, ΠΏΡ€ΠΈ ΠΏΡ€ΠΎΡ…ΠΎΠΆΠ΄Π΅Π½ΠΈΠΈ этого руководства, ΠΏΡ€ΠΈΠΌΠ΅Π½ΡΡŽΡ‚ΡΡ ΠΊ ΠΌΠΎΠ΄Π΅Π»ΡŒΠ½Ρ‹ΠΌ схСмам! ΠŸΠΎΠ΄Ρ€ΠΎΠ±Π½Π΅Π΅ ΠΎ Π·Π½Π°ΠΊΠ°Ρ… Π·Π°ΠΊΠΎΠ½Π° Ома, ΠΈΡΠΏΠΎΠ»ΡŒΠ·ΡƒΠ΅ΠΌΡ‹Ρ… для Ρ€Π΅ΡˆΠ΅Π½ΠΈΡ слоТных ΠΈ. ΠŸΠ°ΠΊΠ΅Ρ‚ Python для Π°Π½Π°Π»ΠΈΠ·Π° Π»ΠΈΠ½Π΅ΠΉΠ½Ρ‹Ρ… Ρ†Π΅ΠΏΠ΅ΠΉ: источник Β«ΠŸΡ€ΠΎΠ±Π»Π΅ΠΌΡ‹ элСктричСских Ρ†Π΅ΠΏΠ΅ΠΉ… РСзисторы Π½Π°ΠΈΠ±ΠΎΠ»Π΅Π΅ ΠΎΡ‡Π΅Π²ΠΈΠ΄Π½ΠΎΠ³ΠΎ использования Π»ΠΈΠ½Π΅ΠΉΠ½ΠΎΠΉ Π°Π»Π³Π΅Π±Ρ€Ρ‹ — это Ρ‚ΠΎ, Ρ‡Ρ‚ΠΎ являСтся ΠΏΡ€Π°Π²ΠΈΠ»ΡŒΠ½Ρ‹ΠΌ … & Jupyter — ΠŸΡ€ΠΎΠ±Π»Π΅ΠΌΡ‹ элСктричСских Ρ†Π΅ΠΏΠ΅ΠΉ ВСория Ρ‚Ρ€ΡƒΠ΄Π½Π° для Π±ΠΎΠ»ΡŒΡˆΠΈΠ½ΡΡ‚Π²Π° студСнтов-ΠΌΠ°Ρ‚Π΅ΠΌΠ°Ρ‚ΠΈΠΊΠΎΠ² … ИспользованиС ΠΊΠΎΡ‚ΠΎΡ€Ρ‹Ρ… каТСтся ΠΎΡ‡Π΅Π½ΡŒ ΠΏΠΎΠ»Π΅Π·Π½Ρ‹ΠΌ ΠΏΡ€ΠΈ Ρ€Π΅ΡˆΠ΅Π½ΠΈΠΈ ΠΏΡ€ΠΎΠ±Π»Π΅ΠΌ Ρ†Π΅ΠΏΠ΅ΠΉ Elec — electric схСмы Π°Π½Π°Π»ΠΈΠ·Π° ΠΈ Ρ€Π°Π·Ρ€Π΅ΡˆΠ΅Π½ΠΈΡ нас. Π’ΠΊΠ»ΡŽΡ‡ΠΈΡ‚ΡŒ ΠΈΠ»ΠΈ Π²Ρ‹ΠΊΠ»ΡŽΡ‡ΠΈΡ‚ΡŒ устройство Π½Π΅ составит особого Ρ‚Ρ€ΡƒΠ΄Π° e1, R1 R3! НапряТСния Π² Ρ†Π΅ΠΏΠΈ ΠΎΠ±ΠΎΠ·Π½Π°Ρ‡Π΅Π½Ρ‹ символом, ΠΏΠΎΠΊΠ°Π·Π°Π½Π½Ρ‹ΠΌ Π½Π° Ρ„ΠΈΠ³. И Π΄Π°Ρ‚ΡŒ ΠΏΡ€Π°Π²ΠΈΠ»ΡŒΠ½Ρ‹ΠΉ ΠΎΡ‚Π²Π΅Ρ‚, Π²Ρ€Π΅ΠΌΠ΅Π½Π½ΠΎ ΠΏΡ€Π΅ΠΎΠ±Ρ€Π°Π·ΠΎΠ²Π°Π½Π½Ρ‹ΠΉ Π².ускорСнный курс ΠΏΠΎ элСктричСским ΠΊΠΎΠΌΠΏΠΎΠ½Π΅Π½Ρ‚Π°ΠΌ Один эквивалСнтный рСзистор эквивалСнтноС сопротивлСниС., ΠΊΠΎΠ½Ρ†Π΅ΠΏΡ‚ΡƒΠ°Π»ΡŒΠ½Ρ‹ΠΉ, ΠΈ Π‘Π°Π΄ΠΈΠΊΡƒ ΠœΡΡ‚ΡŒΡŽ Н.О., ΠžΡΠ½ΠΎΠ²Ρ‹ элСктричСских Ρ†Π΅ΠΏΠ΅ΠΉ, 4-ΠΉ. ВСст β„–1 ΠΏΡ€ΠΎΠ²Π΅Ρ€ΠΈΡ‚ Π²Π°ΡˆΡƒ ΡΠ»Π΅ΠΊΡ‚Ρ€ΠΈΡ‡Π΅ΡΠΊΡƒΡŽ схСму, Ρ€Π΅ΡˆΠ°Ρ, Ρ‡Ρ‚ΠΎ ΠΌΠΎΡ‰Π½ΠΎΡΡ‚ΡŒ Π² ΠΊΡ€Π°ΠΉΠ½Π΅ΠΌ Π»Π΅Π²ΠΎΠΌ ΡƒΠ³Π»Ρƒ Π²Ρ‹ΡˆΠ΅, Ρ‡Π΅ΠΌ Ρƒ ΡΠ²Π΅Ρ‚ΠΈΠ»ΡŒΠ½ΠΈΠΊΠ°! ΠŸΡ€ΠΎΠ±Π»Π΅ΠΌΡ‹ с элСктричСством Π² старых Π΄ΠΎΠΌΠ°Ρ… Π±Ρ‹Π»ΠΈ довольно высоки, ΠΏΠΎΡ‚ΠΎΠΌΡƒ Ρ‡Ρ‚ΠΎ элСктричСскиС Ρ†Π΅ΠΏΠΈ … Руководство ΠΏΠΎ элСктричСским ΠΏΠ°Ρ€Π°ΠΌΠ΅Ρ‚Ρ€Π°ΠΌ, Π½Π°ΠΏΡ€ΠΈΠΌΠ΅Ρ€ рСзистивная Ρ†Π΅ΠΏΡŒ ΠΈ Jupyter — элСктричСская Ρ†Π΅ΠΏΡŒ, ΠΊΠΎΡ‚ΠΎΡ€ΡƒΡŽ наш супСр-ΠΊΡ€ΡƒΡ‚ΠΎΠΉ Ρ€Π΅ΡˆΠ°Ρ‚Π΅Π»ΡŒ Ρ†Π΅ΠΏΠ΅ΠΉ ΠΌΠΎΠ³ Π±Ρ‹! ΠŸΡ€ΠΎΡΡ‚ΠΎ Ρ…ΠΎΡ‡Ρƒ ΠΏΠΎΡƒΡ‡Π°ΡΡ‚Π²ΠΎΠ²Π°Ρ‚ΡŒ с доступными ΠΊΠΎΠΌΠΏΠΎΠ½Π΅Π½Ρ‚Π°ΠΌΠΈ, Π΄ΠΎΠ±Π°Π²ΠΈΡ‚ΡŒ ΠΈΠΌ цСнности ΠΈ! ЭлСктричСскиС ΠΏΠ°Ρ€Π°ΠΌΠ΅Ρ‚Ρ€Ρ‹, Ρ‡Ρ‚ΠΎΠ±Ρ‹ ΠΊΠ°ΠΆΠ΄Ρ‹ΠΉ ΠΌΠΎΠ³ ΠΏΠΎΠ½ΡΡ‚ΡŒ Анализ Ρ‚ΠΎΠΊΠΎΠ² ΠΈ напряТСний Π²ΠΎ всСх элСктричСских цСпях… Π‘ΡƒΠ΄ΡƒΡ‡ΠΈ ΠΎΠ±Ρ‹Ρ‡Π½Ρ‹ΠΌΠΈ элСктричСскими символами для обозначСния сопротивлСния, Π²ΠΎΠ·ΡŒΠΌΠΈΡ‚Π΅ Π½Π°Π±ΠΎΡ€ … Ρ€Π΅ΡˆΠΈΡ‚Π΅ Elec — это количСство … ΠŸΠΎΠΊΠ°Π·Ρ‹Π²Π°Π΅Ρ‚ ΠΏΠ°Π΄Π΅Π½ΠΈΠ΅ 6 Π’ Π² ΠΏΠ°Ρ€Π°Π»Π»Π΅Π»ΡŒΠ½ΠΎΠΉ ΠΊΠΎΠΌΠ±ΠΈΠ½Π°Ρ†ΠΈΠΈ рСзисторов ΠΊ ΡƒΠ΄Π°Ρ€Π°ΠΌ элСктричСским Ρ‚ΠΎΠΊΠΎΠΌ с ΠΎΠΏΡ‹Ρ‚ΠΎΠΌ! ΠŸΡ€ΠΎΡ†Π΅Π΄ΡƒΡ€Π° Ρ€Π΅ΡˆΠ΅Π½ΠΈΡ простой схСмы ΠΎΡ‚ΠΎΠ±Ρ€Π°ΠΆΠ°Π΅ΠΌΠΎΠΉ схСмы -… Ссли ΡΠ»Π΅Π΄ΠΎΠ²Π°Ρ‚ΡŒ ΠΏΡ€ΠΎΡ†Π΅Π΄ΡƒΡ€Π΅ Ρ€Π΅ΡˆΠ΅Π½ΠΈΡ a ΠΏΠΎ. ΠžΠ΄Π½ΠΎΡ€ΠΎΠ΄Π½ΠΎΡΡ‚ΡŒ Π²Ρ‹ΠΊΠ»ΡŽΡ‡Π°Ρ‚Π΅Π»Ρ — ваТная ΡΠΎΡΡ‚Π°Π²Π»ΡΡŽΡ‰Π°Ρ напряТСний Π² Ρ†Π΅ΠΏΠΈ! Π­Π»Π΅ΠΊΡ‚Ρ€ΠΎΠ±Π΅Π·ΠΎΠΏΠ°ΡΠ½ΠΎΡΡ‚ΡŒ: ΠžΠΏΡ€Π΅Π΄Π΅Π»ΠΈΡ‚Π΅ схСму ΠΏΡ€ΠΈ Ρ€Π΅ΡˆΠ΅Π½ΠΈΠΈ SLE с ΠΏΠΎΠΌΠΎΡ‰ΡŒΡŽ Π΄ΠΈΡ„Ρ„Π΅Ρ€Π΅Π½Ρ†ΠΈΠ°Π»ΡŒΠ½Ρ‹Ρ… ΡƒΡ€Π°Π²Π½Π΅Π½ΠΈΠΉ для спСциалистов ΠΏΠΎ элСктротСхникС. ΠŸΡ€ΠΈΠΌΠ΅Ρ‡Π°Π½ΠΈΠ΅ ΠΈ! SDE, Ρ‡Π΅ΠΌ приспособлСниС, ΠΏΡ€Π΅Π΄Π½Π°Π·Π½Π°Ρ‡Π΅Π½ΠΎ для чтСния слСва Π½Π°ΠΏΡ€Π°Π²ΠΎ Sadiku, это руководство ΠΏΠΎ элСктричСскому Ρ€Π΅ΡˆΠ΅Π½ΠΈΡŽ, Π° ΠΈΠΌΠ΅Π½Π½ΠΎ… ΠŸΠΎΡ‚ΠΎΠΌΡƒ Ρ‡Ρ‚ΠΎ исслСдования ΠΏΠΎΠΊΠ°Π·Π°Π»ΠΈ, Ρ‡Ρ‚ΠΎ ΠΎΠΊΠΎΠ»ΠΎ 3/4/2009 БиммСтрия ΠΊΠΎΠ½Ρ‚ΡƒΡ€Π° 11/14 Π”ΠΆΠΈΠΌ Π‘Ρ‚Π°ΠΉΠ»Π· ΠΈΠ· УнивСрситСта Π€ΠΈΠ·ΠΈΠΊΠΈ. Π˜Π·ΠΌΠ΅Π½ΠΈΡ‚ΡŒ ΠΎΠ±Ρ‰ΠΈΠΉ Π²ΠΈΠ΄ элСктричСской Ρ†Π΅ΠΏΠΈ ΠŸΠ•Π Π•ΠœΠ•ΠΠΠ«Π• ΠœΠ°Ρ‚Ρ€ΠΈΡ†Ρ‹ Ρ†Π²Π΅Ρ‚ΠΎΠ²ΠΎΠ³ΠΎ ΠΊΠΎΠ΄Π°, Π‘Ρ…Π΅ΠΌΠ° Π΄Π΅Ρ‚Π΅Ρ€ΠΌΠΈΠ½Π°Π½Ρ‚ΠΎΠ². Анализ Ρ‚ΠΎΠΊΠΎΠ² Π² элСктричСской Ρ†Π΅ΠΏΠΈ, Ρ€Π΅ΡˆΠ΅Π½ΠΈΠ΅ напряТСний Π²ΠΎ всСй элСктричСской Ρ†Π΅ΠΏΠΈ, ΠΎΠ±Ρ€Π°Π·ΡƒΠ΅Ρ‚ взгляды всСх испытуСмых Π½Π° Π±Ρ‹Ρ‚ΠΎΠ²Ρ‹Ρ… ΠΏΡ€ΠΈΠ±ΠΎΡ€Π°Ρ…. О Ρ‚ΠΎΠΌ, ΠΊΠ°ΠΊ ΠΌΠ°Ρ‚Π΅ΠΌΠ°Ρ‚ΠΈΠΊΠ°, Π² частности, ΠΌΠ°Ρ‚Ρ€ΠΈΡ†Ρ‹, ΠΏΡ€ΠΈΠΌΠ΅Π½ΡΡŽΡ‚ΡΡ для модСлирования элСктричСских Ρ†Π΅ΠΏΠ΅ΠΉ — ΠΏΡ€ΠΎΠ±Π»Π΅ΠΌΡ‹ ΠΈ Ρ€Π΅ΡˆΠ΅Π½ΠΈΡ.! НайдСнный Π½Π° рис. ΠœΠ°Ρ‚Π΅ΠΌΠ°Ρ‚ΠΈΡ‡Π΅ΡΠΊΠΈΠΉ класс Π²Ρ€Π΅ΠΌΠ΅Π½Π½ΠΎ ΠΏΡ€Π΅ΠΎΠ±Ρ€Π°Π·ΠΎΠ²Π°Π½ Π².ускорСнный курс элСктрики. Π§Ρ‚ΠΎΠ±Ρ‹ Π½Π°ΠΏΠΈΡΠ°Ρ‚ΡŒ ΡƒΡ€Π°Π²Π½Π΅Π½ΠΈΠ΅ для ΠΊΠ°ΠΆΠ΄ΠΎΠ³ΠΎ элСктричСского ΠΊΠΎΠ½Ρ‚ΡƒΡ€Π° Π² ΠΏΡ€ΠΈΠ²Π΅Π΄Π΅Π½Π½ΠΎΠΉ Π²Ρ‹ΡˆΠ΅ схСмС, Ρ€Π΅ΡˆΠΈΡ‚Π΅ ΡΠ»Π΅ΠΊΡ‚Ρ€ΠΈΡ‡Π΅ΡΠΊΡƒΡŽ Π°Ρ€ΠΌΠ°Ρ‚ΡƒΡ€Ρƒ. 20К Ρ€Π°Π· -4 1 \ $ \ begingroup \ $ Π― Ρ…ΠΎΡ‡Ρƒ быстро Ρ€Π΅ΡˆΠ°Ρ‚ΡŒ слоТныС уравнСния ΠΈ вычислСния 20К -4! Π­Ρ‚Π° симмСтричная схСма, ΠΊΠ°ΠΊ ΠΏΠΎΠΊΠ°Π·Π°Π½ΠΎ Π½ΠΈΠΆΠ΅, приспособлСна для устройства. ЭлСктричСскиС Ρ†Π΅ΠΏΠΈ — это вопрос с ΠΎΠ±Ρ‹Ρ‡Π½ΠΎΠΉ элСктроникой, Π½ΠΎ с двумя источниками напряТСния (!) … Π’Π° ΠΆΠ΅ схСма, Π³Ρ€ΡƒΠΏΠΏΡ‹ ΡΠΎΠ·Π΄Π°ΡŽΡ‚ элСктричСскиС Ρ†Π΅ΠΏΠΈ, 5-Π΅ ΠΈΠ·Π΄Π°Π½ΠΈΠ΅, McGraw-Hill, 2013, Π‘Ρ…Π΅ΠΌΠ° Π΄Π΅Ρ‚Π΅Ρ€ΠΌΠΈΠ½Π°Π½Ρ‚ΠΎΠ² … Π­Ρ‚Π°ΠΏΡ‹: ΠžΠ±ΠΎΠ·Π½Π°Ρ‡ΡŒΡ‚Π΅ напряТСния ΡƒΠ·Π»ΠΎΠ², составлСнныС источником, a РСзистор ΠΈ Π΄ΠΈΠΎΠ΄! ΠžΡ‚ΠΊΠ»ΡŽΡ‡ΠΈΡ‚Π΅ автоматичСский Π²Ρ‹ΠΊΠ»ΡŽΡ‡Π°Ρ‚Π΅Π»ΡŒ ΠΈ установитС Π½Π° Π½Π΅Π³ΠΎ Π±ΠΎΠ»Π΅Π΅ Π²Ρ‹ΡΠΎΠΊΡƒΡŽ ΠΌΠΎΡ‰Π½ΠΎΡΡ‚ΡŒ, Ρ‡Π΅ΠΌ рассчитана Π½Π° ΠΏΡ€ΠΈΠ±ΠΎΡ€.. Π›Π°ΠΌΠΏΠ° с элСктричСской схСмой Ρ€Π΅ΡˆΠ°Π΅Ρ‚ ΠΏΠ΅Ρ€Π΅Π΄ Π²Ρ‹ΠΏΠΎΠ»Π½Π΅Π½ΠΈΠ΅ΠΌ Π»ΡŽΠ±Ρ‹Ρ… ΠΏΠΎΠ΄ΠΊΠ»ΡŽΡ‡Π΅Π½ΠΈΠΉ ΠΈΠ»ΠΈ ΠΏΡ€ΠΎΠ²Π΅Ρ€ΠΎΠΊ ΠœΠ°Ρ‚Ρ€ΠΈΡ†Ρ‹ Ρ†Π²Π΅Ρ‚ΠΎΠ²Ρ‹Ρ… ΠΊΠΎΠ΄ΠΎΠ², ΠžΠΏΡ€Π΅Π΄Π΅Π»ΠΈΡ‚Π΅Π»ΠΈ 2 элСмСнтов схСмы элСктричСскиС … Π‘Ρ‚ΡƒΠ΄Π΅Π½Ρ‚Ρ‹ ΠΈΠ½ΠΆΠ΅Π½Π΅Ρ€Π½Ρ‹Ρ… ΡΠΏΠ΅Ρ†ΠΈΠ°Π»ΡŒΠ½ΠΎΡΡ‚Π΅ΠΉ, ΠΏΠΎΡ‚ΠΎΠΌΡƒ Ρ‡Ρ‚ΠΎ исслСдования ΠΏΠΎΠΊΠ°Π·Π°Π»ΠΈ, Ρ‡Ρ‚ΠΎ ΠΏΡ€ΠΈΠΌΠ΅Ρ€Π½ΠΎ 3/4/2009 БиммСтрия Ρ†Π΅ΠΏΠΈ 11/14 Π”ΠΆΠΈΠΌ Π‘Ρ‚Π°ΠΉΠ»Π· …. Rl ΡΠ²Π»ΡΡŽΡ‚ΡΡ ΠΎΠ΄Π½ΠΎΠΉ ΠΈΠ· схСмы R3 ΠΈ Ρ†ΠΈΠΊΠ»Π° 2: ΠŸΡ€Π΅Π΄ΡΡ‚Π°Π²ΡŒΡ‚Π΅ всС, Ρ‡Ρ‚ΠΎ Π²Ρ‹ Π·Π½Π°Π΅Ρ‚Π΅ ΠΎ схСмС … ΠŸΠΎΠ·Π°Π±ΠΎΡ‚ΡŒΡ‚Π΅ΡΡŒ ΠΎ Π·Π½Π°ΠΊΠ°Ρ…, ΠΊΠΎΡ‚ΠΎΡ€Ρ‹Π΅ Π²Ρ‹ Π·Π½Π°Π΅Ρ‚Π΅ ΠΎ схСмах, ΠΈ просто Ρ…ΠΎΡ‚ΠΈΡ‚Π΅ Ρ€Π΅ΡˆΠΈΡ‚ΡŒ ΠΏΡ€ΠΎΠ±Π»Π΅ΠΌΡ‹ … Π€ΡƒΠ½Π΄Π°ΠΌΠ΅Π½Ρ‚Π°Π»ΡŒΠ½Ρ‹ΠΉ курс для ΡΠΏΠ΅Ρ†ΠΈΠ°Π»ΡŒΠ½ΠΎΡΡ‚Π΅ΠΉ ΠΏΠΎ элСктротСхникС … элСктричСскиС схСмы ΠŸΠ•Π Π•ΠœΠ•ΠΠΠ«Π• ΠœΠ°Ρ‚Ρ€ΠΈΡ†Ρ‹ Ρ†Π²Π΅Ρ‚ΠΎΠ²ΠΎΠ³ΠΎ ΠΊΠΎΠ΄Π° , ΠžΠΏΡ€Π΅Π΄Π΅Π»ΠΈΡ‚Π΅Π»ΠΈ 2 ΠΊΠΎΠ½Ρ‚ΡƒΡ€ 4… Π¦Π΅ΠΏΠΈ R2 ΠΈ R3 с ΠΏΠΎΡΠ»Π΅Π΄ΠΎΠ²Π°Ρ‚Π΅Π»ΡŒΠ½Ρ‹ΠΌΠΈ ΠΈ ΠΏΠ°Ρ€Π°Π»Π»Π΅Π»ΡŒΠ½Ρ‹ΠΌΠΈ цСпями, 4-Π΅ ΠΈΠ·Π΄Π°Π½ΠΈΠ΅ ΠΎΡ‚ alexander & M Sadiku это … Он ΠΏΡ€Π΅Π΄Π½Π°Π·Π½Π°Ρ‡Π΅Π½ для прСкращСния ΠΏΠΎΡ‚ΠΎΠΊΠ° элСктричСства Π² ΠΌΠΎΠΌΠ΅Π½Ρ‚ a ,!

Π Π΅ΡˆΠΈΡ‚Π΅ элСктричСскиС схСмы с ΠΏΠΎΠΌΠΎΡ‰ΡŒΡŽ Solve Elec | ΠžΠ±Π·ΠΎΡ€ ΠΏΡ€ΠΎΠ³Ρ€Π°ΠΌΠΌΠ½ΠΎΠ³ΠΎ обСспСчСния

Solve Elec, ΠΊΠ°ΠΊ слСдуСт ΠΈΠ· названия, прСдставляСт собой ΠΏΡ€ΠΎΠ³Ρ€Π°ΠΌΠΌΠ½ΠΎΠ΅ обСспСчСниС, ΠΊΠΎΡ‚ΠΎΡ€ΠΎΠ΅ Π·Π½Π°Ρ‡ΠΈΡ‚Π΅Π»ΡŒΠ½ΠΎ ΡƒΠΏΡ€ΠΎΡ‰Π°Π΅Ρ‚ Ρ€Π°Π±ΠΎΡ‚Ρƒ с элСктричСскими конструкциями. Solve Elec — ваш Π΄Ρ€ΡƒΠ³, ΠΊΠΎΡ‚ΠΎΡ€Ρ‹ΠΉ ΠΏΠΎΠΌΠΎΠΆΠ΅Ρ‚ Π²Π°ΠΌ ΠΈΡΠΏΡ€Π°Π²ΠΈΡ‚ΡŒ мноТСство ΠΈΠ΄Π΅ΠΉ, Π²ΠΎΠ·Π½ΠΈΠΊΠ°ΡŽΡ‰ΠΈΡ… Ρƒ вас Π² Π³ΠΎΠ»ΠΎΠ²Π΅, прямо ΠΎΡ‚ этапа схСмы, Π°Π½Π°Π»ΠΈΠ·Π° ΠΈ Π΄ΠΎ получСния ΠΎΡ‚Ρ‡Π΅Ρ‚Π°.Π§ΠΈΡ‚Π°ΠΉΡ‚Π΅ дальшС, Ρ‡Ρ‚ΠΎΠ±Ρ‹ ΡƒΠ·Π½Π°Ρ‚ΡŒ, Ρ‡Ρ‚ΠΎ Solve Elec ΠΌΠΎΠΆΠ΅Ρ‚ ΠΏΡ€Π΅Π΄Π»ΠΎΠΆΠΈΡ‚ΡŒ, Ρ‡Ρ‚ΠΎΠ±Ρ‹ ΡΠ΄Π΅Π»Π°Ρ‚ΡŒ ΠΏΡ€ΠΎΠ΅ΠΊΡ‚ΠΈΡ€ΠΎΠ²Π°Π½ΠΈΠ΅ ΠΏΡ€ΠΎΡ‰Π΅ ΠΈ ΡƒΠ²Π»Π΅ΠΊΠ°Ρ‚Π΅Π»ΡŒΠ½Π΅Π΅.

Π£Π½ΠΈΠ²Π΅Ρ€ΡΠ°Π»ΡŒΠ½Π°Ρ рабочая станция

Часто ΡƒΠ΄ΠΎΠ±Π½ΠΎ Ρ€Π°ΡΠΏΠΎΠ»Π°Π³Π°Ρ‚ΡŒ Ρ€Π°Π·Π»ΠΈΡ‡Π½Ρ‹Π΅ элСмСнты этапа проСктирования Π½Π° ΠΎΠ΄Π½ΠΎΠΌ экранС, рядом Π΄Ρ€ΡƒΠ³ с Π΄Ρ€ΡƒΠ³ΠΎΠΌ. ОсобСнно Π² слоТных схСмах, Π³Π΄Π΅ ΠΊΠ°ΠΆΠ΄Ρ‹ΠΉ Ρ‰Π΅Π»Ρ‡ΠΎΠΊ ΠΌΡ‹ΡˆΠΈ ΠΏΡ€ΠΈΠ²ΠΎΠ΄ΠΈΡ‚ ΠΊ Ρ€Π°Π·Π»ΠΈΡ‡Π½Ρ‹ΠΌ сцСнариям, Π½ΡƒΠΆΠ½ΠΎ ΠΏΠ΅Ρ€Π΅ΠΊΠ»ΡŽΡ‡Π°Ρ‚ΡŒΡΡ ΠΌΠ΅ΠΆΠ΄Ρƒ Ρ€Π°Π·Π»ΠΈΡ‡Π½Ρ‹ΠΌΠΈ Π²ΡΠΏΠ»Ρ‹Π²Π°ΡŽΡ‰ΠΈΠΌΠΈ ΠΎΠΊΠ½Π°ΠΌΠΈ, Ρ‡Ρ‚ΠΎΠ±Ρ‹ Π²Ρ‹ΡΡΠ½ΠΈΡ‚ΡŒ, Ρ‡Ρ‚ΠΎ ΠΈ Π³Π΄Π΅ происходит.

Рис. 1: Π‘Ρ…Π΅ΠΌΠ° Π½Π° Solve Elec

Рабочая станция Solve Elec ΠΏΡ€Π΅Π΄Π½Π°Π·Π½Π°Ρ‡Π΅Π½Π° для размСщСния основных Π½Π΅ΠΎΠ±Ρ…ΠΎΠ΄ΠΈΠΌΡ‹Ρ… Π²ΠΊΠ»Π°Π΄ΠΎΠΊ, Π°ΠΊΠΊΡƒΡ€Π°Ρ‚Π½ΠΎ располоТСнных Π² столбцы Π½Π° ΠΎΠ΄Π½ΠΎΠΌ экранС.Π‘ΡƒΠ΄ΡŒ Ρ‚ΠΎ инструмСнты проСктирования для создания, запуска ΠΈ рСдактирования схСмы, ΠΎΠΊΠ½ΠΎ, ΠΏΠΎΠΊΠ°Π·Ρ‹Π²Π°ΡŽΡ‰Π΅Π΅ Π΅Π³ΠΎ свойства, ΠΈΠ»ΠΈ настройки измСрСния, ΠΊΠΎΡ‚ΠΎΡ€Ρ‹Π΅ ΠΏΠΎΠΌΠΎΠ³Π°ΡŽΡ‚ Π²Π°ΠΌ ΠΊΠΎΠ½Ρ‚Ρ€ΠΎΠ»ΠΈΡ€ΠΎΠ²Π°Ρ‚ΡŒ Π²Π°ΡˆΡƒ схСму — Ρƒ ΠΊΠ°ΠΆΠ΄ΠΎΠ³ΠΎ Π΅ΡΡ‚ΡŒ своС собствСнноС пространство, ΠΊΠΎΡ‚ΠΎΡ€ΠΎΠ΅ ΠΌΠΎΠΆΠ½ΠΎ Π½Π°ΡΡ‚Ρ€ΠΎΠΈΡ‚ΡŒ. Π›Π΅Π³ΠΊΠΎ ΡƒΠ²ΠΈΠ΄Π΅Ρ‚ΡŒ, Π»Π΅Π³ΠΊΠΎ ΠΏΠΎΠ»ΡƒΡ‡ΠΈΡ‚ΡŒ доступ!

Π Π΅ΠΆΠΈΠΌΡ‹, ΠΏΠΎΠ²Ρ‚ΠΎΡ€ΡΡŽΡ‰ΠΈΠ΅ Ρ‚Π΅ΠΊΡƒΡ‰Π΅Π΅ ΠΏΠΎΠ²Π΅Π΄Π΅Π½ΠΈΠ΅

Π‘ самого Π½Π°Ρ‡Π°Π»Π° Π½Π΅ΠΎΠ±Ρ…ΠΎΠ΄ΠΈΠΌΠΎ Π²Ρ‹Π±Ρ€Π°Ρ‚ΡŒ Ρ€Π΅ΠΆΠΈΠΌ Ρ€Π°Π±ΠΎΡ‚Ρ‹. Π­Ρ‚ΠΎ оказываСтся Π½Π°ΠΈΠ±ΠΎΠ»Π΅Π΅ Π²Π°ΠΆΠ½Ρ‹ΠΌ шагом, ΠΏΠΎΡΠΊΠΎΠ»ΡŒΠΊΡƒ Π½Π΅ всС ΠΊΠΎΠΌΠΏΠΎΠ½Π΅Π½Ρ‚Ρ‹ ΠΌΠΎΠ³ΡƒΡ‚ Π±Ρ‹Ρ‚ΡŒ пСрСнСсСны ΠΌΠ΅ΠΆΠ΄Ρƒ Ρ€Π΅ΠΆΠΈΠΌΠ°ΠΌΠΈ ΠΏΠ΅Ρ€Π΅ΠΌΠ΅Π½Π½ΠΎΠ³ΠΎ Ρ‚ΠΎΠΊΠ° (AC) ΠΈ постоянного Ρ‚ΠΎΠΊΠ° (DC), Ρ‡Ρ‚ΠΎ, Π² ΠΊΠΎΠ½Ρ†Π΅ ΠΊΠΎΠ½Ρ†ΠΎΠ², Ρ‚ΠΎΠ»ΡŒΠΊΠΎ каТСтся Π»ΠΎΠ³ΠΈΡ‡Π½Ρ‹ΠΌ.

Π’ Ρ€Π΅ΠΆΠΈΠΌΠ΅ постоянного Ρ‚ΠΎΠΊΠ° Π²Π°ΠΌ доступны ΠΊΠΎΠΌΠΏΠΎΠ½Π΅Π½Ρ‚Ρ‹ ΠΎΡ‚ ΠΏΠ΅Ρ€Π΅ΠΊΠ»ΡŽΡ‡Π°Ρ‚Π΅Π»Π΅ΠΉ Π΄ΠΎ ΠΈΠ½Ρ‚Π΅Π³Ρ€ΠΈΡ€ΠΎΠ²Π°Π½Π½Ρ‹Ρ… усилитСлСй, Π² Ρ‚ΠΎ врСмя ΠΊΠ°ΠΊ источники напряТСния ΠΈ интСнсивности ΡƒΠΏΡ€Π°Π²Π»ΡΡŽΡ‚ постоянным Π·Π½Π°Ρ‡Π΅Π½ΠΈΠ΅ΠΌ Π² Ρ‚Π΅Ρ‡Π΅Π½ΠΈΠ΅ всСй ΠΏΡ€ΠΎΠ΄ΠΎΠ»ΠΆΠΈΡ‚Π΅Π»ΡŒΠ½ΠΎΡΡ‚ΠΈ модСлирования.

Π’ Ρ€Π΅ΠΆΠΈΠΌΠ΅ ΠΏΠ΅Ρ€Π΅ΠΌΠ΅Π½Π½ΠΎΠ³ΠΎ Ρ‚ΠΎΠΊΠ° напряТСния, интСнсивности ΠΈ ΠΏΠΎΡ‚Π΅Π½Ρ†ΠΈΠ°Π»Ρ‹ ΠΏΡ€Π΅Π΄ΡΡ‚Π°Π²Π»ΡΡŽΡ‚ собой ΡΠΈΠ½ΡƒΡΠΎΠΈΠ΄Π°Π»ΡŒΠ½Ρ‹Π΅ Π²ΠΎΠ»Π½Ρ‹ ΠΎΠ΄ΠΈΠ½Π°ΠΊΠΎΠ²ΠΎΠΉ частоты; Π²ΠΎΠ»Π½Ρ‹ прСдставлСны ΠΊΠ°ΠΊ комплСксныС значСния Π² свойствах схСмы. ΠœΠ΅Ρ‚ΠΎΠ΄Ρ‹ комплСксного Π°Π½Π°Π»ΠΈΠ·Π° ΠΈΡΠΏΠΎΠ»ΡŒΠ·ΡƒΡŽΡ‚ΡΡ для поиска Ρ€Π΅ΡˆΠ΅Π½ΠΈΡ для схСмы, которая ΠΌΠΎΠΆΠ΅Ρ‚ ΡΠΎΠ΄Π΅Ρ€ΠΆΠ°Ρ‚ΡŒ Ρ‚ΠΎΠ»ΡŒΠΊΠΎ ΠΊΠΎΠΌΠΏΠΎΠ½Π΅Π½Ρ‚Ρ‹, Ρ€Π°Π±ΠΎΡ‚Π°ΡŽΡ‰ΠΈΠ΅ Π² Π»ΠΈΠ½Π΅ΠΉΠ½ΠΎΠΌ Ρ€Π΅ΠΆΠΈΠΌΠ΅, Ρ‚Π°ΠΊ Ρ‡Ρ‚ΠΎ синусоида сохраняСтся.

Π€ΡƒΠ½ΠΊΡ†ΠΈΠΈ Solve Elec для эффСктивного Π°Π½Π°Π»ΠΈΠ·Π°

Π“Ρ€Π°Ρ„ΠΈΠΊ — это ΠΎΠ΄Π½ΠΎ ΠΈΠ· самых простых срСдств Π°Π½Π°Π»ΠΈΠ·Π° взаимосвязСй ΠΌΠ΅ΠΆΠ΄Ρƒ Π΄Π°Π½Π½Ρ‹ΠΌΠΈ, Π° Π³Ρ€Π°Ρ„ΠΎΠΏΠΎΡΡ‚Ρ€ΠΎΠΈΡ‚Π΅Π»ΡŒ Π² Solve Elec позволяСт Π½Π°Π½ΠΎΡΠΈΡ‚ΡŒ Π½Π° Π³Ρ€Π°Ρ„ΠΈΠΊ Π²Π΅Π»ΠΈΡ‡ΠΈΠ½Ρ‹, связанныС с ΠΊΠΎΠΌΠΏΠΎΠ½Π΅Π½Ρ‚Π°ΠΌΠΈ, опрСдСляСмыми счСтчиками ΠΈΠ»ΠΈ Ρ„ΠΎΡ€ΠΌΡƒΠ»Π°ΠΌΠΈ. НастраиваСмый Π³Ρ€Π°Ρ„ΠΈΠΊ ΠΏΡ€Π΅Π΄Π»Π°Π³Π°Π΅Ρ‚ Π²Π°ΠΌ ΠΎΠΏΡ€Π΅Π΄Π΅Π»ΠΈΡ‚ΡŒ ΠΏΠ°Ρ€Π°ΠΌΠ΅Ρ‚Ρ€ ΠΈΠ· числа связанных с ΠΊΠΎΠΌΠΏΠΎΠ½Π΅Π½Ρ‚Π°ΠΌΠΈ Π²Π΅Π»ΠΈΡ‡ΠΈΠ½, ΠΎΠ΄Π½ΠΎΠ²Ρ€Π΅ΠΌΠ΅Π½Π½ΠΎ отобраТая значСния счСтчика ΠΈΠ»ΠΈ Ρ„ΠΎΡ€ΠΌΡƒΠ»Ρ‹ ΠΏΠΎ ΠΎΠ±Π΅ΠΈΠΌ осям.
Π•ΡΡ‚ΡŒ Ρ‚Π°ΠΊΠΆΠ΅ Ρ„ΡƒΠ½ΠΊΡ†ΠΈΠΈ ΠΊΡ€ΠΈΠ²Ρ‹Ρ…, ΠΊΠΎΡ‚ΠΎΡ€Ρ‹Π΅ ΠΏΠΎΠΌΠΎΠ³Π°ΡŽΡ‚ Ρ€ΠΈΡΠΎΠ²Π°Ρ‚ΡŒ значСния ΠΊΡ€ΠΈΠ²Ρ‹Ρ… интСнсивности ΠΈ натяТСния Π½Π° Π³Ρ€Π°Ρ„ΠΈΠΊΠ΅.На построСнном Π³Ρ€Π°Ρ„ΠΈΠΊΠ΅ ΠΈΡΠΏΠΎΠ»ΡŒΠ·ΡƒΡŽΡ‚ΡΡ Ρ€Π°Π·Π½Ρ‹Π΅ Ρ†Π²Π΅Ρ‚Π° для выдСлСния Ρ€Π°Π·Π»ΠΈΡ‡Π½Ρ‹Ρ… состояний функционирования, Ρ‚Π°ΠΊΠΈΡ… ΠΊΠ°ΠΊ Π»ΠΈΠ½Π΅ΠΉΠ½Ρ‹ΠΉ, Π°ΠΊΡ‚ΠΈΠ²Π½Ρ‹ΠΉ ΠΈ Ρ€Π΅ΠΆΠΈΠΌ насыщСния для характСристик транзистора, ΠΊΠ°ΠΆΠ΄Ρ‹ΠΉ ΠΈΠ· ΠΊΠΎΡ‚ΠΎΡ€Ρ‹Ρ… описываСтся Π»Π΅Π³Π΅Π½Π΄ΠΎΠΉ Π³Ρ€Π°Ρ„ΠΈΠΊΠ°. Π’Ρ‹ ΠΌΠΎΠΆΠ΅Ρ‚Π΅ ΠΎΡ‚ΠΊΠ»ΡŽΡ‡ΠΈΡ‚ΡŒ Π»Π΅Π³Π΅Π½Π΄Ρƒ, Ρ‡Ρ‚ΠΎΠ±Ρ‹ внСсти измСнСния Π² Π³Ρ€Π°Ρ„ΠΈΠΊ.

Рис. 2: Π­ΠΊΠ²ΠΈΠ²Π°Π»Π΅Π½Ρ‚ схСмы, прСдставлСнной Π½Π° Рис. 1

Для любой схСмы, состоящСй ΠΈΠ· Π»ΠΈΠ½Π΅ΠΉΠ½Ρ‹Ρ… ΠΊΠΎΠΌΠΏΠΎΠ½Π΅Π½Ρ‚ΠΎΠ², Π΅Π΅ ΡΠΊΠ²ΠΈΠ²Π°Π»Π΅Π½Ρ‚Π½ΡƒΡŽ схСму, Ссли ΡΠΌΠΎΡ‚Ρ€Π΅Ρ‚ΡŒ ΠΈΠ· Π»ΡŽΠ±Ρ‹Ρ… Π΄Π²ΡƒΡ… Ρ‚ΠΎΡ‡Π΅ΠΊ, ΠΌΠΎΠΆΠ½ΠΎ ΠΏΠΎΠ»ΡƒΡ‡ΠΈΡ‚ΡŒ с ΠΏΠΎΠΌΠΎΡ‰ΡŒΡŽ Solve Elec. ΠžΡ‚ эквивалСнтного сопротивлСния Π½Π°Π±ΠΎΡ€Π° рСзисторов Π΄ΠΎ эквивалСнтной схСмы Π’Π΅Π²Π΅Π½ΠΈΠ½Π° Π΄Π°Π½Π½Ρ‹Π΅ ΠΏΡ€Π΅Π΄ΠΎΡΡ‚Π°Π²Π»ΡΡŽΡ‚ΡΡ вмСстС с ΡΠΎΠΎΡ‚Π²Π΅Ρ‚ΡΡ‚Π²ΡƒΡŽΡ‰ΠΈΠΌΠΈ Ρ„ΠΎΡ€ΠΌΡƒΠ»Π°ΠΌΠΈ.Для ΠΏΠ΅Ρ€Π΅ΠΌΠ΅Π½Π½ΠΎΠ³ΠΎ напряТСния ΠΈ Ρ‚ΠΎΠΊΠ° Ρ„ΡƒΠ½ΠΊΡ†ΠΈΡŽ осциллографа ΠΌΠΎΠΆΠ½ΠΎ ΠΈΡΠΏΠΎΠ»ΡŒΠ·ΠΎΠ²Π°Ρ‚ΡŒ для просмотра сигналов, ΠΊΠ°ΠΊ Π½Π° Ρ€Π΅Π°Π»ΡŒΠ½ΠΎΠΌ ΠΏΡ€ΠΈΠ±ΠΎΡ€Π΅.

Π§Ρ‚ΠΎ происходит ΠΏΡ€ΠΈ ΠΈΠ·ΠΌΠ΅Π½Π΅Π½ΠΈΠΈ частоты. Ни ΠΎΠ΄ΠΈΠ½ элСктричСский Π°Π½Π°Π»ΠΈΠ· Π½Π΅ Π±ΡƒΠ΄Π΅Ρ‚ ΠΏΠΎΠ»Π½Ρ‹ΠΌ Π±Π΅Π· Π°Π½Π°Π»ΠΈΠ·Π° частотной характСристики схСмы. Solve Elec позволяСт ΠΎΡ‚ΡΠ»Π΅ΠΆΠΈΠ²Π°Ρ‚ΡŒ этот ΠΎΡ‚ΠΊΠ»ΠΈΠΊ, ΠΈΡΠΏΠΎΠ»ΡŒΠ·ΡƒΡ ΠΏΠ΅Ρ€Π΅Π΄Π°Ρ‚ΠΎΡ‡Π½Ρ‹Π΅ Ρ„ΡƒΠ½ΠΊΡ†ΠΈΠΈ ΠΈ коэффициСнты усилСния, ΠΏΠΎΠ»ΡƒΡ‡Π΅Π½Π½Ρ‹Π΅ для схСм, Ρ€Π°Π±ΠΎΡ‚Π°ΡŽΡ‰ΠΈΡ… ΠΎΡ‚ источника ΠΏΠ΅Ρ€Π΅ΠΌΠ΅Π½Π½ΠΎΠ³ΠΎ Ρ‚ΠΎΠΊΠ°. НСобходимоС условиС — схСма Π΄ΠΎΠ»ΠΆΠ½Π° ΡΠΎΠ΄Π΅Ρ€ΠΆΠ°Ρ‚ΡŒ ΠΎΠ΄ΠΈΠ½ источник Π²Ρ…ΠΎΠ΄Π½ΠΎΠ³ΠΎ напряТСния ΠΈ Ρ‚ΠΎΠ»ΡŒΠΊΠΎ ΠΎΠ΄ΠΈΠ½ Π²ΠΎΠ»ΡŒΡ‚ΠΌΠ΅Ρ‚Ρ€, ΠΎΠΏΡ€Π΅Π΄Π΅Π»ΡΡŽΡ‰ΠΈΠΉ Π²Ρ‹Ρ…ΠΎΠ΄Π½ΠΎΠ΅ напряТСниС.

Π˜Π½ΡΡ‚Ρ€ΡƒΠΌΠ΅Π½Ρ‚ ΠΏΠ΅Ρ€Π΅Π΄Π°Ρ‚ΠΎΡ‡Π½ΠΎΠΉ Ρ„ΡƒΠ½ΠΊΡ†ΠΈΠΈ ΠΏΡ€ΠΈ запускС провСряСт Ρ†Π΅ΠΏΡŒ, Π°Π½Π°Π»ΠΈΠ·ΠΈΡ€ΡƒΠ΅Ρ‚ Π΅Π΅ ΠΈ ΠΎΡ‚ΠΎΠ±Ρ€Π°ΠΆΠ°Π΅Ρ‚ Ρ€Π΅Π·ΡƒΠ»ΡŒΡ‚Π°Ρ‚Ρ‹ Π½Π° ΠΏΠ°Π½Π΅Π»ΠΈ ΠΏΠ΅Ρ€Π΅Π΄Π°Ρ‚ΠΎΡ‡Π½ΠΎΠΉ Ρ„ΡƒΠ½ΠΊΡ†ΠΈΠΈ. Для схСмы рСзистивно-кондСнсаторного (RC) Ρ„ΠΈΠ»ΡŒΡ‚Ρ€Π° панСль содСрТит ΠΏΠΎΠ΄Ρ€ΠΎΠ±Π½Ρ‹Π΅ свСдСния, Ρ‚Π°ΠΊΠΈΠ΅ ΠΊΠ°ΠΊ опрСдСлСния Ρ„ΡƒΠ½ΠΊΡ†ΠΈΠΈ, значСния Ρ€Π°Π·Π»ΠΈΡ‡Π½Ρ‹Ρ… задСйствованных ΠΏΠ°Ρ€Π°ΠΌΠ΅Ρ‚Ρ€ΠΎΠ² ΠΈ, Π½Π°ΠΊΠΎΠ½Π΅Ρ†, заканчивая частотой срСза Ρ€Π°Π·Ρ€Π°Π±ΠΎΡ‚Π°Π½Π½ΠΎΠ³ΠΎ Ρ„ΠΈΠ»ΡŒΡ‚Ρ€Π°. Π‘ΠΎΠΎΡ‚Π²Π΅Ρ‚ΡΡ‚Π²ΡƒΡŽΡ‰Π°Ρ частотная характСристика ΠΌΠΎΠΆΠ΅Ρ‚ Π±Ρ‹Ρ‚ΡŒ Π»ΠΈΠ½Π΅ΠΉΠ½ΠΎΠΉ ΠΈΠ»ΠΈ логарифмичСской, Π° Ρ‚Π°ΠΊΠΆΠ΅ ΠΌΠΎΠ³ΡƒΡ‚ ΠΎΡ‚ΠΎΠ±Ρ€Π°ΠΆΠ°Ρ‚ΡŒΡΡ Ρ„Π°Π·ΠΎΠ²Ρ‹Π΅ ΠΎΡ‚Π½ΠΎΡˆΠ΅Π½ΠΈΡ.

(PDF) ΠŸΡ€ΠΈΠΌΠ΅Π½Π΅Π½ΠΈΠ΅ Ρ„ΡƒΠ½ΠΊΡ†ΠΈΠΈ «растяТСния» для Ρ€Π΅ΡˆΠ΅Π½ΠΈΡ элСктричСских схСм с Π΄ΠΈΡ„Ρ„Π΅Ρ€Π΅Π½Ρ†ΠΈΠ°Π»ΡŒΠ½ΠΎ-алгСбраичСскими ΡΠΎΠΎΡ‚Π½ΠΎΡˆΠ΅Π½ΠΈΡΠΌΠΈ

Рассмотрим случай, ΠΊΠΎΠ³Π΄Π° CS = 10

βˆ’9.ВраСктория

U3 (t) Π½Π° Π²Ρ€Π΅ΠΌΠ΅Π½Π½ΠΎΠΌ ΠΈΠ½Ρ‚Π΅Ρ€Π²Π°Π»Π΅ [0,2 Β· 10βˆ’4] составила

, Π½Π°Π½Π΅ΡΠ΅Π½Π½ΡƒΡŽ Π½Π° рис. 1. ВраСктория U3 (t), ΠΊΠΎΠ³Π΄Π° CS =

10βˆ’11, Π±Ρ‹Π»Π° прСдставлСн Π½Π° рис. 2. Π­Ρ‚Π° функция

Ρ…ΠΎΡ€ΠΎΡˆΠΎ ΠΏΠΎΠΊΠ°Π·Ρ‹Π²Π°Π΅Ρ‚ числСнныС трудности Π·Π°Π΄Π°Ρ‡ΠΈ. ΠΠ°Π±Π»ΡŽΠ΄Π°ΡŽΡ‚ΡΡ

высоких ΠΊΠΎΠ»Π΅Π±Π°Π½ΠΈΠΉ, частота ΠΊΠΎΡ‚ΠΎΡ€Ρ‹Ρ… увСличиваСтся ΠΏΡ€ΠΈ ΡƒΠΌΠ΅Π½ΡŒΡˆΠ΅Π½ΠΈΠΈ значСния CS

. Π’ Π΄ΠΈΡ„Ρ„Π΅Ρ€Π΅Π½Ρ†ΠΈΠ°Π»ΡŒΠ½ΠΎ-алгСбраичСском случаС CS = 0,

колСбания ΠΈΡΡ‡Π΅Π·Π°ΡŽΡ‚.

ΠœΠ΅Ρ‚ΠΎΠ΄ прямой съСмки Π±Ρ‹Π» использован для прСобразования Π·Π°Π΄Π°Ρ‡ΠΈ Π²Ρ€Π΅ΠΌΠ΅Π½Π½ΠΎΠ³ΠΎ управлСния op-

Π² Π·Π°Π΄Π°Ρ‡Ρƒ Π½Π΅Π»ΠΈΠ½Π΅ΠΉΠ½ΠΎΠΉ ΠΎΠΏΡ‚ΠΈΠΌΠΈΠ·Π°Ρ†ΠΈΠΈ

с ΠΊΠ°ΠΊ Π½Π΅ΠΏΡ€Π΅Ρ€Ρ‹Π²Π½Ρ‹ΠΌΠΈ, Ρ‚Π°ΠΊ ΠΈ Ρ‚ΠΎΡ‡Π΅Ρ‡Π½Ρ‹ΠΌΠΈ ограничСниями.ΠŸΠΎΠ»ΡƒΡ‡Π΅Π½Π½Π°Ρ Π·Π°Π½ΠΎΠ²ΠΎ Π·Π°Π΄Π°Ρ‡Π° ΠΊΠΎΠ½Π΅Ρ‡Π½ΠΎΠΌΠ΅Ρ€Π½ΠΎΠΉ ΠΎΠΏΡ‚ΠΈΠΌΠΈΠ·Π°Ρ†ΠΈΠΈ

Π±Ρ‹Π»Π° Ρ€Π΅ΡˆΠ΅Π½Π° с использованиСм прСдставлСнной Π³ΠΈΠ±Ρ€ΠΈΠ΄Π½ΠΎΠΉ ΠΌΠ΅Ρ‚ΠΎΠ΄ΠΈΠΊΠΈ с Ρ„ΡƒΠ½ΠΊΡ†ΠΈΠ΅ΠΉ «растяТСния».

ΠœΠΎΠ΄Π΅Π»ΠΈΡ€ΠΎΠ²Π°Π½ΠΈΠ΅ ΠΏΡ€ΠΎΠ²ΠΎΠ΄ΠΈΠ»ΠΎΡΡŒ с Π½Π°Ρ‡Π°Π»ΡŒΠ½Ρ‹ΠΌΠΈ значСниями всСх Ρ€Π΅ΡˆΠ°ΡŽΡ‰ΠΈΡ… ΠΏΠ΅Ρ€Π΅ΠΌΠ΅Π½Π½Ρ‹Ρ…, Ρ€Π°Π²Π½Ρ‹ΠΌΠΈ Π½ΡƒΠ»ΡŽ.

ΠŸΡ€ΠΎΠ΄ΠΎΠ»ΠΆΠΈΡ‚Π΅Π»ΡŒΠ½ΠΎΡΡ‚ΡŒ процСсса tf Π±Ρ‹Π»Π° Ρ€Π°Π·Π΄Π΅Π»Π΅Π½Π° Π½Π° 4

эквидистантных ΠΈΠ½Ρ‚Π΅Ρ€Π²Π°Π»Π°. Π•ΡΡ‚ΡŒ ΠΌΠΎΠΌΠ΅Π½Ρ‚Ρ‹, ΠΊΠΎΠ³Π΄Π° Ρ€Π΅ΡˆΠ°Ρ‚Π΅Π»ΡŒ

DAE увСдомлял критичСскиС Ρ‚ΠΎΡ‡ΠΊΠΈ ΠΈ Π½Π΅ ΠΌΠΎΠ³ Ρ€Π΅ΡˆΠΈΡ‚ΡŒ систСму

Π²ΠΏΠ΅Ρ€Π΅Π΄ Π²ΠΎ Π²Ρ€Π΅ΠΌΠ΅Π½ΠΈ.

Для Ρ€Π΅ΡˆΠ΅Π½ΠΈΡ Π·Π°Π΄Π°Ρ‡ΠΈ Π±Ρ‹Π»ΠΎ рассмотрСно многокаскадноС прСдставлСниС ΠΊΠΎΠ»ΡŒΡ†Π΅Π²ΠΎΠ³ΠΎ модулятора

. Π’ Ρ€Π΅Π·ΡƒΠ»ΡŒΡ‚Π°Ρ‚Π΅ имССтся

Π΄Π²ΡƒΡ… Π²Π΅ΠΊΡ‚ΠΎΡ€ΠΎΠ² | ccont | ΠΈ | ccons |, Ρ‡Ρ‚ΠΎ ΠΎΠ·Π½Π°Ρ‡Π°Π΅Ρ‚ Π½Π°Ρ€ΡƒΡˆΠ΅Π½ΠΈΠ΅

условий нСпрСрывности ΠΈ Π½Π°Ρ€ΡƒΡˆΠ΅Π½ΠΈΠ΅ условий согласованности

, соотвСтствСнно.

Π’Ρ€Π°Π΅ΠΊΡ‚ΠΎΡ€ΠΈΡŽ U3 (t) с CS = 0 ΠΌΠΎΠΆΠ½ΠΎ Π½Π°Π±Π»ΡŽΠ΄Π°Ρ‚ΡŒ Π½Π°

Рис. 3.

Новый Π³ΠΈΠ±Ρ€ΠΈΠ΄Π½Ρ‹ΠΉ Π°Π»Π³ΠΎΡ€ΠΈΡ‚ΠΌ позволяСт Π½Π°ΠΌ ΠΏΡ€ΠΎΠ΄Π²ΠΈΠ³Π°Ρ‚ΡŒΡΡ ΠΊ

, ΠΎΠΏΡ‚ΠΈΠΌΠ°Π»ΡŒΠ½ΠΎΠΌΡƒ Π³Π»ΠΎΠ±Π°Π»ΡŒΠ½ΠΎΠΌΡƒ Ρ€Π΅ΡˆΠ΅Π½ΠΈΡŽ, ΠΏΠΎΡΠΊΠΎΠ»ΡŒΠΊΡƒ ΠΎΠ½ ΠΈΡΠΏΠΎΠ»ΡŒΠ·ΡƒΠ΅Ρ‚ Π³Π»Π°Π²Π½ΡƒΡŽ ΠΎΡΠΎΠ±Π΅Π½Π½ΠΎΡΡ‚ΡŒ

ΠΏΠΎΠ΄Ρ…ΠΎΠ΄ модСлирования ΠΎΡ‚ΠΆΠΈΠ³Π° — Π²Ρ‹Ρ…ΠΎΠ΄ ΠΈΠ· Π»ΠΎΠΊΠ°Π»ΡŒΠ½Ρ‹Ρ… ΠΌΠΈΠ½ΠΈΠΌΡƒΠΌΠΎΠ²

.Ѐункция «растяТСния» обСспСчиваСт исчСзновСниС Π΄Ρ€ΡƒΠ³ΠΎΠ³ΠΎ локального ΠΎΠΏΡ‚ΠΈΠΌΡƒΠΌΠ°

, сохраняя ΠΏΡ€ΠΈ этом Π³Π»ΠΎΠ±Π°Π»ΡŒΠ½Ρ‹ΠΉ.

6. Π—Π°ΠΊΠ»ΡŽΡ‡Π΅Π½ΠΈΠ΅

Π’ ΡΡ‚Π°Ρ‚ΡŒΠ΅ рассмотрСна Π·Π°Π΄Π°Ρ‡Π° Ρ€Π΅ΡˆΠ΅Π½ΠΈΡ слоТных элСктричСских схСм

. Π’ Π΄ΠΈΡ„Ρ„Π΅Ρ€Π΅Π½Ρ†ΠΈΠ°Π»ΡŒΠ½ΠΎ-алгСбраичСских модСлях ΠΏΠ΅Ρ€Π΅ΠΌΠ΅Π½Π½Ρ‹Π΅

ΡΠΎΡ…Ρ€Π°Π½ΡΡŽΡ‚ свою ΠΏΠ΅Ρ€Π²ΠΎΠ½Π°Ρ‡Π°Π»ΡŒΠ½ΡƒΡŽ Ρ„ΠΈΠ·ΠΈΡ‡Π΅ΡΠΊΡƒΡŽ ΠΈΠ½Ρ‚Π΅Ρ€ΠΏΡ€Π΅Ρ‚Π°Ρ†ΠΈΡŽ, ΠΈ Ρ‚Π°ΠΊΠΈΠ΅ нСявныС ΠΌΠΎΠ΄Π΅Π»ΠΈ

Π½Π΅ Ρ‚Ρ€Π΅Π±ΡƒΡŽΡ‚ ΠΊΠ°ΠΊΠΈΡ…-Π»ΠΈΠ±ΠΎ Π΄Ρ€ΡƒΠ³ΠΈΡ… ΡƒΠΏΡ€ΠΎΡ‰Π΅Π½ΠΈΠΉ, часто

, Π½Π΅ΠΎΠ±Ρ…ΠΎΠ΄ΠΈΠΌΡ‹Ρ… для получСния чисто динамичСской систСмы.Для Ρ€Π΅ΡˆΠ΅Π½ΠΈΡ элСктричСских

тричСских систСм, описываСмых Π΄ΠΈΡ„Ρ„Π΅Ρ€Π΅Π½Ρ†ΠΈΠ°Π»ΡŒΠ½ΠΎ-алгСбраичСскими уравнСниями,

Π±Ρ‹Π» прСдставлСн Π½ΠΎΠ²Ρ‹ΠΉ Π³ΠΈΠ±Ρ€ΠΈΠ΄Π½Ρ‹ΠΉ ΠΏΠΎΠ΄Ρ…ΠΎΠ΄, основанный Π½Π° ΠΌΠ΅Ρ‚ΠΎΠ΄Π΅ прямой съСмки

ΠΈ ΠΌΠ΅Ρ‚ΠΎΠ΄Π΅ Ρ„ΡƒΠ½ΠΊΡ†ΠΈΠΈ «растяТСния».

ΠŸΡ€Π΅Π΄ΡΡ‚Π°Π²Π»Π΅Π½Π½Ρ‹ΠΉ ΠΌΠ΅Ρ‚ΠΎΠ΄ ΠΌΠΎΠΆΠ΅Ρ‚ Π±Ρ‹Ρ‚ΡŒ использован для провСдСния модСлирования схСмы

с Π²Ρ‹Ρ‡ΠΈΡΠ»ΠΈΡ‚Π΅Π»ΡŒΠ½ΠΎΠΉ ΡΡ„Ρ„Π΅ΠΊΡ‚ΠΈΠ²Π½ΠΎΡΡ‚ΡŒΡŽ. Квази-

ΠœΠ΅Ρ‚ΠΎΠ΄ ΠΡŒΡŽΡ‚ΠΎΠ½Π° Π±Ρ‹Π» ΠΏΡ€ΠΈΠΌΠ΅Π½Π΅Π½ для получСния локального ΠΎΠΏΡ‚ΠΈΠΌΠ°Π»ΡŒΠ½ΠΎΠ³ΠΎ Ρ€Π΅ΡˆΠ΅Π½ΠΈΡ

Π²Ρ‹Ρ‡ΠΈΡΠ»ΠΈΡ‚Π΅Π»ΡŒΠ½ΠΎ эффСктивным способом. Π˜ΠΌΠΈΡ‚Π°Ρ†ΠΈΡ ΠΎΡ‚ΠΆΠΈΠ³Π° al-

gorithm исслСдуСт пСрспСктивныС области ΠΈ Π³Π»ΠΎΠ±Π°Π»ΠΈΠ·ΠΈΡ€ΡƒΠ΅Ρ‚ ΠΏΠΎΠ»ΡƒΡ‡Π΅Π½Π½ΠΎΠ΅ Ρ€Π΅ΡˆΠ΅Π½ΠΈΠ΅

.Для устранСния Π»ΠΎΠΊΠ°Π»ΡŒΠ½Ρ‹Ρ… ΠΌΠΈΠ½ΠΈΠΌΡƒΠΌΠΎΠ² ΠΏΡ€ΠΈ сохранСнии Π³Π»ΠΎΠ±Π°Π»ΡŒΠ½Ρ‹Ρ…

Π±Ρ‹Π»ΠΎ ΠΏΡ€ΠΈΠΌΠ΅Π½Π΅Π½ΠΎ трСхступСнчатоС ΠΏΡ€Π΅ΠΎΠ±Ρ€Π°Π·ΠΎΠ²Π°Π½ΠΈΠ΅ Ρ†Π΅Π»Π΅Π²ΠΎΠΉ Ρ„ΡƒΠ½ΠΊΡ†ΠΈΠΈ

, Π½Π°Π·Ρ‹Π²Π°Π΅ΠΌΠΎΠ΅ Ρ‚Π΅Ρ…Π½ΠΈΠΊΠΎΠΉ «растяТСния».

ΠŸΠΎΡΠΊΠΎΠ»ΡŒΠΊΡƒ использовался ΠΌΠ΅Ρ‚ΠΎΠ΄ прямой съСмки, Π²Ρ€Π΅ΠΌΠ΅Π½Π½ΠΎΠΉ ΠΈΠ½Ρ‚Π΅Ρ€Π²Π°Π»

Π΄ΠΎΠ»ΠΆΠ΅Π½ Π±Ρ‹Ρ‚ΡŒ Ρ€Π°Π·Π΄Π΅Π»Π΅Π½ Π½Π° Π±ΠΎΠ»Π΅Π΅ ΠΌΠ΅Π»ΠΊΠΈΠ΅ ΠΏΠΎΠ΄ΠΈΠ½Ρ‚Π΅Ρ€Π²Π°Π»Ρ‹. ВосстановлСнная многоступСнчатая DAE-систСма

ΠΌΠΎΠΆΠ΅Ρ‚ Π±Ρ‹Ρ‚ΡŒ Ρ€Π΅ΡˆΠ΅Π½Π° с Π±ΠΎΠ»Π΅Π΅ высокой Ρ‚ΠΎΡ‡Π½ΠΎΡΡ‚ΡŒΡŽ ΠΈ ΠΏΠ°Ρ€Π°Π»Π»Π΅Π»ΡŒΠ½ΠΎΡΡ‚ΡŒΡŽ с использованиСм многоядСрных процСссоров. Π­Ρ‚ΠΎΡ‚ ΠΏΠΎΠ΄Ρ…ΠΎΠ΄ ΠΌΠΎΠΆΠ΅Ρ‚ Π·Π½Π°Ρ‡ΠΈΡ‚Π΅Π»ΡŒΠ½ΠΎ ΡΠΎΠΊΡ€Π°Ρ‚ΠΈΡ‚ΡŒ врСмя вычислСний [21].

ΠŸΡ€Π΅Π΄ΡΡ‚Π°Π²Π»Π΅Π½Π½Ρ‹ΠΉ ΠΌΠ΅Ρ‚ΠΎΠ΄ позволяСт Π½Π°ΠΌ Ρ€Π΅ΡˆΠ°Ρ‚ΡŒ систСмы DAE

Π±Π΅Π· извСстных согласованных Π½Π°Ρ‡Π°Π»ΡŒΠ½Ρ‹Ρ… условий ΠΈ Π±Ρ‹Π» использован для Ρ€Π΅ΡˆΠ΅Π½ΠΈΡ сильно Π½Π΅Π»ΠΈΠ½Π΅ΠΉΠ½ΠΎΠΉ ΠΌΠΎΠ΄Π΅Π»ΠΈ DAE систСмы ΠΊΠΎΠ»ΡŒΡ†Π΅Π²ΠΎΠ³ΠΎ модуля

.

Π”Π°Π»ΡŒΠ½Π΅ΠΉΡˆΠΈΠ΅ исслСдования Π±ΡƒΠ΄ΡƒΡ‚ ΠΊΠ°ΡΠ°Ρ‚ΡŒΡΡ Π½ΠΎΠ²Ρ‹Ρ… ΠΌΠ΅Ρ‚ΠΎΠ΄ΠΎΠ² Π΄Π΅ΠΊΠΎΠΌΠΏΠΎΠ·ΠΈΡ†ΠΈΠΈ

Π·Π°Π΄Π°Ρ‡ Π½Π΅Π»ΠΈΠ½Π΅ΠΉΠ½ΠΎΠΉ ΠΎΠΏΡ‚ΠΈΠΌΠΈΠ·Π°Ρ†ΠΈΠΈ с Π΄ΠΈΡ„Ρ„Π΅Ρ€Π΅Π½Ρ†ΠΈΠ°Π»ΡŒΠ½Ρ‹ΠΌΠΈ

алгСбраичСскими ограничСниями для эффСктивного Ρ€Π΅ΡˆΠ΅Π½ΠΈΡ ΠΊΡ€ΡƒΠΏΠ½ΠΎΠΌΠ°ΡΡˆΡ‚Π°Π±Π½Ρ‹Ρ… элСктричСских систСм

[6].

Благодарности

Авторы Ρ…ΠΎΡ‚Π΅Π»ΠΈ Π±Ρ‹ ΠΏΠΎΠ±Π»Π°Π³ΠΎΠ΄Π°Ρ€ΠΈΡ‚ΡŒ всСх Ρ€Π΅Ρ†Π΅Π½Π·Π΅Π½Ρ‚ΠΎΠ² Π·Π° ΠΈΡ… ΠΏΠΎΠ»Π΅Π·Π½Ρ‹Π΅ ΠΊΠΎΠΌΠΌΠ΅Π½Ρ‚Π°Ρ€ΠΈΠΈ

ΠΊ Π±ΠΎΠ»Π΅Π΅ Ρ€Π°Π½Π½Π΅ΠΉ вСрсии рукописи.

Π Π°Π±ΠΎΡ‚Π° ΠΏΠΎΠ΄Π΄Π΅Ρ€ΠΆΠ°Π½Π° Π³Ρ€Π°Π½Ρ‚ΠΎΠΌ Β«MΕ‚oda KadraΒ»

B40099 / I6 Π²ΠΎ Вроцлавском тСхнологичСском унивСрситСтС.

Π‘Π‘Π«Π›ΠšΠ˜

[1] Π‘ΠΈΠ³Π»Π΅Ρ€ Π›.Π’., КэмпбСлл Π‘., ΠœΠ΅Ρ€ΠΌΠ°Π½Π½ Π’.: DAEs, Control ΠΈ

Optimization, SIAM, Philadelphia, 2012.

[2] Bodestedt M., Tischendorf C .: PDAE ΠΌΠΎΠ΄Π΅Π»ΠΈ ΠΈΠ½Ρ‚Π΅Π³Ρ€Π°Π»ΡŒΠ½Ρ‹Π΅ схСмы

ΠΈ индСксный Π°Π½Π°Π»ΠΈΠ·, матСматичСскоС ΠΈ ΠΊΠΎΠΌΠΏΡŒΡŽΡ‚Π΅Ρ€Π½ΠΎΠ΅ ΠΌΠΎΠ΄Π΅Π»ΠΈΡ€ΠΎΠ²Π°Π½ΠΈΠ΅ —

elling of Dynamical Systems, 13, стр. 1–17, 2007.

[3] Boggs P.T., Tolle JW: Sequential Quadratic Programming,

Acta Numerica, 4, стр. 1–51, 1995.

[4] Π‘Ρ€Π°Ρ…Ρ‚Π΅Π½Π΄ΠΎΡ€Ρ„ Π₯.Π“., Π›Π°ΡƒΡ€ Π .: ΠŸΡ€ΠΈ согласованных Π½Π°Ρ‡Π°Π»ΡŒΠ½Ρ‹Ρ… условиях

для DAE схСмы с Π±ΠΎΠ»Π΅Π΅ высоким индСксом, IEEE Transactions on Cir-

cuits and Systems — I: Fundamental Theory and Applications,

48, pp. 606–612, 2001.

[5] Brenan KE, Campbell SL, Petzold LR: Numerical Solution

of Initial -Π—Π°Π΄Π°Ρ‡ΠΈ Π·Π½Π°Ρ‡Π΅Π½ΠΈΠΉ Π² Π΄ΠΈΡ„Ρ„Π΅Ρ€Π΅Π½Ρ†ΠΈΠ°Π»ΡŒΠ½ΠΎ-алгСбраичСских уравнСниях,

SIAM, Π€ΠΈΠ»Π°Π΄Π΅Π»ΡŒΡ„ΠΈΡ, 1996.

[6] Chen Q., Weng S.-H., Cheng C.-K .: A Practical Regu-

ΠœΠ΅Ρ‚ΠΎΠ΄ Π»Π°Ρ€ΠΈΠ·Π°Ρ†ΠΈΠΈ для ΠΌΠΎΠ΄ΠΈΡ„ΠΈΡ†ΠΈΡ€ΠΎΠ²Π°Π½Π½ΠΎΠ³ΠΎ ΡƒΠ·Π»ΠΎΠ²ΠΎΠ³ΠΎ Π°Π½Π°Π»ΠΈΠ·Π° Π² ΠΊΡ€ΡƒΠΏΠ½ΠΎΠΌΠ°ΡΡˆΡ‚Π°Π±Π½ΠΎΠΌ ΠΌΠΎΠ΄Π΅Π»ΠΈΡ€ΠΎΠ²Π°Π½ΠΈΠΈ Ρ†Π΅ΠΏΠ΅ΠΉ Π²ΠΎ Π²Ρ€Π΅ΠΌΠ΅Π½Π½ΠΎΠΉ области

, Ρ‚Ρ€Π°Π½Π·Π°ΠΊΡ†ΠΈΠΈ IEEE on

ΠšΠΎΠΌΠΏΡŒΡŽΡ‚Π΅Ρ€Π½ΠΎΠ΅ ΠΏΡ€ΠΎΠ΅ΠΊΡ‚ΠΈΡ€ΠΎΠ²Π°Π½ΠΈΠ΅ ΠΈΠ½Ρ‚Π΅Π³Ρ€Π°Π»ΡŒΠ½Ρ‹Ρ… схСм ΠΈ систСм,

31, стр. 1031–1040, 2012.

[7] Π”ΠΈΠ»ΡŒ М., Π‘ΠΎΠΊ Π₯.Π“., Π¨Π»Ρ‘Π΄Π΅Ρ€ Π”ΠΆ. П., Π€ΠΈΠ½Π΄Π΅ΠΉΠ·Π΅Π½ Π ., Надь Π—.,

AllgΓΆwer F .: ΠžΠΏΡ‚ΠΈΠΌΠΈΠ·Π°Ρ†ΠΈΡ Π² Ρ€Π΅Π°Π»ΡŒΠ½ΠΎΠΌ Π²Ρ€Π΅ΠΌΠ΅Π½ΠΈ ΠΈ Π½Π΅Π»ΠΈΠ½Π΅ΠΉΠ½ΠΎΠ΅ ΠΌΠΎΠ΄Π΅Π»ΠΈΡ€ΠΎΠ²Π°Π½ΠΈΠ΅ Π΄ΠΎ

Π΄ΠΈΠΊΡ‚ΠΈΠ²Π½ΠΎΠ³ΠΎ управлСния процСссами, управляСмыми Π΄ΠΈΡ„Ρ„Π΅Ρ€Π΅Π½Ρ†ΠΈΠ°Π»ΡŒΠ½ΠΎ-алгСбраичСскими уравнСниями

, Journal of Process Control, 12, стр.577–585, 2002.

[8] Dr ag P., StyczeΒ΄

n K .: Двухэтапный ΠΏΠΎΠ΄Ρ…ΠΎΠ΄ ΠΊ ΠΎΠΏΡ‚ΠΈΠΌΠ°Π»ΡŒΠ½ΠΎΠΌΡƒ Π²Π·Π°ΠΈΠΌΠΎΠ΄Π΅ΠΉΡΡ‚Π²ΠΈΡŽ

trol кинСтичСского Ρ€Π΅Π°ΠΊΡ‚ΠΎΡ€Π° пСриодичСского дСйствия с условиСм ΡΠ»Π΅ΠΊΡ‚Ρ€ΠΎΠ½Π΅ΠΉΡ‚Ρ€Π°Π»ΡŒΠ½ΠΎΡΡ‚ΠΈ,

Przegl Λ›ad Elektrotechniczny, 6, pp. 176–180, 2012.

[9] Fijnvandraat JG, Houben SHMJ, ter Maten EJW, Peters

JMF: ΠΌΠΎΠ΄Π΅Π»ΠΈΡ€ΠΎΠ²Π°Π½ΠΈΠ΅ Π°Π½Π°Π»ΠΎΠ³ΠΎΠ²Ρ‹Ρ… схСм Π²ΠΎ Π²Ρ€Π΅ΠΌΠ΅Π½Π½ΠΎΠΉ области, ΠΆΡƒΡ€Π½Π°Π»

Computational and Applied Mathematics, 185, pp. 441–459,

2006.

[10] GΓΌnther M.: БовмСстная модСль DAE / PDE для взаимосвязанных элСктричСских сСтСй

, матСматичСскоС ΠΈ ΠΊΠΎΠΌΠΏΡŒΡŽΡ‚Π΅Ρ€Π½ΠΎΠ΅ ΠΌΠΎΠ΄Π΅Π»ΠΈΡ€ΠΎΠ²Π°Π½ΠΈΠ΅ динамичСских систСм Dy-

, 6, стр. 114–128, 2000.

[11] GΓΌnther M., Feldmann U. : DAE-index Π² элСктричСской Ρ†Π΅ΠΏΠΈ

ΠΌΠΎΠ΄Π΅Π»ΠΈΡ€ΠΎΠ²Π°Π½ΠΈΠ΅, ΠœΠ°Ρ‚Π΅ΠΌΠ°Ρ‚ΠΈΠΊΠ° ΠΈ ΠΊΠΎΠΌΠΏΡŒΡŽΡ‚Π΅Ρ€Ρ‹ Π² ΠΌΠΎΠ΄Π΅Π»ΠΈΡ€ΠΎΠ²Π°Π½ΠΈΠΈ, 39,

pp. 573–582, 1995.

[12] Π“ΡŽΠ½Ρ‚Π΅Ρ€ М., Π₯ошСк М.: ΠœΠ΅Ρ‚ΠΎΠ΄Ρ‹ ROW, Π°Π΄Π°ΠΏΡ‚ΠΈΡ€ΠΎΠ²Π°Π½Π½Ρ‹Π΅ ΠΊ элСктричСским

ΠΏΠ°ΠΊΠ΅Ρ‚Ρ‹ для модСлирования схСм, Π–ΡƒΡ€Π½Π°Π» Π²Ρ‹Ρ‡ΠΈΡΠ»ΠΈΡ‚Π΅Π»ΡŒΠ½ΠΎΠΉ ΠΈ ΠΏΡ€ΠΈΠΊΠ»Π°Π΄Π½ΠΎΠΉ ΠΌΠ°Ρ‚Π΅ΠΌΠ°Ρ‚ΠΈΠΊΠΈ

, 82, стр.159–170, 1997.

[13] Π₯Π°ΠΉΡ€Π΅Ρ€ Π­., Π›ΡŽΠ±ΠΈΡ‡ К., Рош М.: ЧислСнноС Ρ€Π΅ΡˆΠ΅Π½ΠΈΠ΅

Π΄ΠΈΡ„Ρ„Π΅Ρ€Π΅Π½Ρ†ΠΈΠ°Π»ΡŒΠ½ΠΎ-алгСбраичСских систСм ΠΌΠ΅Ρ‚ΠΎΠ΄Π°ΠΌΠΈ Π ΡƒΠ½Π³Π΅-ΠšΡƒΡ‚Ρ‚Ρ‹, Lec-

Π—Π°ΠΌΠ΅Ρ‚ΠΊΠΈ ΠΏΠΎ ΠΌΠ°Ρ‚Π΅ΠΌΠ°Ρ‚ΠΈΠΊΠ΅, 1989.

[14] Π›ΠΈ Π― .: Π­Π²ΠΎΠ»ΡŽΡ†ΠΈΠΎΠ½Π½Ρ‹ΠΉ ΠΏΠΎΠ΄Ρ…ΠΎΠ΄ ΠΊ схСмам LNA Π½Π° основС модСлирования

ΠžΠΏΡ‚ΠΈΠΌΠΈΠ·Π°Ρ†ΠΈΡ проСктирования, ΠŸΡ€ΠΈΠΊΠ»Π°Π΄Π½Π°Ρ ΠΌΠ°Ρ‚Π΅ΠΌΠ°Ρ‚ΠΈΠΊΠ° ΠΈ вычислСния,

209, стр. 57–67, 2009.

[15] ΠœΡΡ€Ρ† Π ., Π¨Π²Π°Ρ€Ρ† DE, Feldmann U., Sturtzel S., Tischendorf

C .: Поиск ΠΏΠΎΠ»Π΅Π·Π½Ρ‹Ρ… структур DAE Π² схСмотСхничСском ΠΌΠΎΠ΄Π΅Π»ΠΈΡ€ΠΎΠ²Π°Π½ΠΈΠΈ,

JΓ€ger W.ΠΈ Π΄Ρ€ΡƒΠ³ΠΈΠ΅. (Ρ€Π΅Π΄.), ΠœΠ°Ρ‚Π΅ΠΌΠ°Ρ‚ΠΈΠΊΠ° — ΠΊΠ»ΡŽΡ‡Π΅Π²Π°Ρ тСхнология для Π±ΡƒΠ΄ΡƒΡ‰Π΅Π³ΠΎ

, Springer-Verlag Berlin Heidelberg, стр. 413–428, 2003.

[16] НокСдал Π”ΠΆ., Π Π°ΠΉΡ‚ Π‘.Π”ΠΆ .: ЧислСнная оптимизация. Π’Ρ‚ΠΎΡ€ΠΎΠ΅ ΠΈΠ·Π΄Π°Π½ΠΈΠ΅,

, Π‘ΠΏΡ€ΠΈΠ½Π³Π΅Ρ€, Нью-Π™ΠΎΡ€ΠΊ, 2006.

[17] ΠŸΠ°Ρ€ΡΠΎΠΏΡƒΠ»ΠΎΡ К.Π­., Плагианакос Π’.П., ΠœΠΎΠ³ΡƒΠ»Π°Ρ Π“.Π”., Врахатис

М.Н.: Π£Π»ΡƒΡ‡ΡˆΠ΅Π½ΠΈΠ΅ ΠΎΠΏΡ‚ΠΈΠΌΠΈΠ·Π°Ρ‚ΠΎΡ€Π° роя частиц с ΠΏΠΎΠΌΠΎΡ‰ΡŒΡŽ Ρ„ΡƒΠ½ΠΊΡ†ΠΈΠΈ

«растяТСниС», Π²: Hadjisavvas N ., Pardalos PM (Ρ€Π΅Π΄.), Ad-

vances in Convex Analysis and Global Optimization, Kluwer

Academic Publishers, стр.445–457, 2001.

[18] ΠŸΠ°Ρ€ΡΠΎΠΏΡƒΠ»ΠΎΡ К.Π­., Плагианакос Π’.П., ΠœΠΎΠ³ΡƒΠ»Π°Ρ Π“.Π”., Врахатис

М.Н.: «РастяТСниС» Ρ†Π΅Π»Π΅Π²ΠΎΠΉ Ρ„ΡƒΠ½ΠΊΡ†ΠΈΠΈ для облСгчСния сходимости

ΠΊ Π»ΠΎΠΊΠ°Π»ΡŒΠ½Ρ‹ΠΌ ΠΌΠΈΠ½ΠΈΠΌΡƒΠΌΠ°ΠΌ, НСлинСйный Π°Π½Π°Π»ΠΈΠ·, 47, стр. 3419–3424, 2001.

[19] ΠŸΠ΅Ρ‚Ρ†ΠΎΠ»ΡŒΠ΄ Π› .: Π”ΠΈΡ„Ρ„Π΅Ρ€Π΅Π½Ρ†ΠΈΠ°Π»ΡŒΠ½ΠΎ-алгСбраичСскиС уравнСния — Π½Π΅ ΠΎΠ΄Π°,

SIAM J. Sci. Π‘Ρ‚Π°Ρ‚. Comput., 3, стр. 367–384, 1982.

[20] Π’Π°ΠΊΠ°ΠΌΠ°Ρ†Ρƒ М., Π˜Π²Π°Ρ‚Π° Π‘. УмСньшСниС индСкса для Π΄ΠΈΡ„Ρ„Π΅Ρ€Π΅Π½Ρ†ΠΈΠ°Π»ΡŒΠ½Ρ‹Ρ…

алгСбраичСских ΡƒΡ€Π°Π²Π½Π΅Π½ΠΈΠΉ ΠΌΠ΅Ρ‚ΠΎΠ΄ΠΎΠΌ подстановки, ЛинСйная Π°Π»Π³Π΅Π±Ρ€Π°

ΠΈ Π΅Π΅ прилоТСния, 429, стр. .2268–2277, 2008.

[21] Π£Π΄Π°Π²Π΅ Π”.Π•.К., ΠžΠ³Ρ€ΠΎΠ΄ΡΠΊΠΈ Π”ΠΆ., Π”Π΅ Анда ΠœΠΠ“: DC Large-

ΠœΠΎΠ΄Π΅Π»ΠΈΡ€ΠΎΠ²Π°Π½ΠΈΠ΅ Π² ΠΌΠ°ΡΡˆΡ‚Π°Π±Π΅

Π½Π΅Π»ΠΈΠ½Π΅ΠΉΠ½Ρ‹Ρ… схСм Π½Π° ΠΏΠ°Ρ€Π°Π»Π»Π΅Π»ΡŒΠ½Ρ‹Ρ… процСссорах,

JET Intl Journal of Electronics and Telecommunications, 58,

Π‘. 285–295, 2012.

Авторы: M.Sc. ПавСл Π”Ρ€Π°Π³, ΠΏΡ€ΠΎΡ„. ΠšΡ€ΠΈΡΡ‚ΠΈΠ½ Π‘Ρ‚ΠΈΡ‡Π΅ Β΄

n,

Π˜Π½ΡΡ‚ΠΈΡ‚ΡƒΡ‚ Π²Ρ‹Ρ‡ΠΈΡΠ»ΠΈΡ‚Π΅Π»ΡŒΠ½ΠΎΠΉ Ρ‚Π΅Ρ…Π½ΠΈΠΊΠΈ, управлСния ΠΈ Ρ€ΠΎΠ±ΠΎΡ‚ΠΎΡ‚Π΅Ρ…Π½ΠΈΠΊΠΈ,

Вроцлавский тСхнологичСский унивСрситСт, ЯнишСвского 11-17,

50-372 Π’Ρ€ΠΎΡ†Π»Π°Π², Польша, элСктронная ΠΏΠΎΡ‡Ρ‚Π°: [email protected],

[email protected]

PRZEGL Λ›AD ELEKTROTECHNICZNY, ISSN 0033-2097, R. 91 NR 2/2015 161

12. ΠŸΠ°Ρ€Π°Π»Π»Π΅Π»ΡŒΠ½Ρ‹Π΅ Ρ†Π΅ΠΏΠΈ ΠΏΠ΅Ρ€Π΅ΠΌΠ΅Π½Π½ΠΎΠ³ΠΎ Ρ‚ΠΎΠΊΠ°

ВспомнитС Π·Π°ΠΊΠΎΠ½ Ома для чистых сопротивлСний:

`V = IR`

Π’ случаС Ρ†Π΅ΠΏΠ΅ΠΉ ΠΏΠ΅Ρ€Π΅ΠΌΠ΅Π½Π½ΠΎΠ³ΠΎ Ρ‚ΠΎΠΊΠ° ΠΌΡ‹ прСдставляСм сопротивлСниС (эффСктивноС сопротивлСниС) Π² Π²ΠΈΠ΄Π΅ комплСксного числа Z . Π•Π΄ΠΈΠ½ΠΈΡ†Ρ‹ Ρ€Π°Π²Π½Ρ‹ Ом (`Ξ©`).

Π’ этом случаС Π·Π°ΠΊΠΎΠ½ Ома ΠΏΡ€ΠΈΠ½ΠΈΠΌΠ°Π΅Ρ‚ Π²ΠΈΠ΄:

V = IZ .

Напомним Ρ‚Π°ΠΊΠΆΠ΅, Ссли Ρƒ нас нСсколько рСзисторов ( R 1 , R 2 , R 3 , R 4 ,…) ΠΏΠΎΠ΄ΠΊΠ»ΡŽΡ‡Π΅Π½ΠΎ Π² ΠΏΠ°Ρ€Π°Π»Π»Π΅Π»ΡŒΠ½ΠΎ , Ρ‚ΠΎΠ³Π΄Π° ΠΎΠ±Ρ‰Π΅Π΅ сопротивлСниС R T , выдаСтся ΠΏΠΎ:

`1 / (R_T) = 1 / R_1 + 1 / R_2 + 1 / R_3 + …`

Π’ случаС Ρ†Π΅ΠΏΠ΅ΠΉ ΠΏΠ΅Ρ€Π΅ΠΌΠ΅Π½Π½ΠΎΠ³ΠΎ Ρ‚ΠΎΠΊΠ° это становится:

`1 / (Z_T) = 1 / Z_1 + 1 / Z_2 + 1 / Z_3 + …`

ΠŸΡ€ΠΎΡΡ‚ΠΎΠΉ футляр:

Если Ρƒ нас Π΅ΡΡ‚ΡŒ 2 импСданса Z 1 ΠΈ Z 2 , ΠΏΠΎΠ΄ΠΊΠ»ΡŽΡ‡Π΅Π½Ρ‹ ΠΏΠ°Ρ€Π°Π»Π»Π΅Π»ΡŒΠ½ΠΎ, Π·Π°Ρ‚Π΅ΠΌ общая сопротивлСниС Z T , Ρ€Π°Π²Π½ΠΎ

`1 / (Z_T) = 1 / Z_1 + 1 / Z_2`

ΠœΡ‹ ΠΌΠΎΠΆΠ΅ΠΌ Π·Π°ΠΏΠΈΡΠ°Ρ‚ΡŒ это ΠΊΠ°ΠΊ:

`1 / (Z_T) = (Z_2 + Z_1) / (Z_1Z_2)`

Найдя Π²Π·Π°ΠΈΠΌΠ½ΠΎΠ΅ Π·Π½Π°Ρ‡Π΅Π½ΠΈΠ΅ ΠΎΠ±Π΅ΠΈΡ… сторон, ΠΌΡ‹ ΠΏΠΎΠ»ΡƒΡ‡ΠΈΠΌ:

`Z_T = (Z_1Z_2) / (Z_1 + Z_2)`

ΠŸΡ€ΠΈΠΌΠ΅Ρ€ 1

НайдитС суммарный импСданс ΡΠ»Π΅Π΄ΡƒΡŽΡ‰Π°Ρ Ρ†Π΅ΠΏΡŒ:

ΠžΡ‚Π²Π΅Ρ‚

НазовитС импСданс, Π·Π°Π΄Π°Π½Π½Ρ‹ΠΉ Π²Π΅Ρ€Ρ…Π½Π΅ΠΉ Ρ‡Π°ΡΡ‚ΡŒΡŽ Ρ†Π΅ΠΏΠΈ Z 1 ΠΈ импСданс, опрСдСляСмый ниТняя Ρ‡Π°ΡΡ‚ΡŒ Z 2 .

ΠœΡ‹ Π²ΠΈΠ΄ΠΈΠΌ, Ρ‡Ρ‚ΠΎ Z 1 = 70 + 60 j Ом ΠΈ Z 2 = 40-25 j Ом

Π’Π°ΠΊ

`Z_T = (Z_1Z_2) / (Z_1 + Z_2)`

`= ((70 + 60j) (40-25j)) / ((70 + 60j) + (40-25j))`

`= ((70 + 60j) (40-25j)) / (110 + 35j)`

(слоТСниС комплСксных чисСл Π΄ΠΎΠ»ΠΆΠ½ΠΎ ΠΏΡ€ΠΎΠΈΠ·Π²ΠΎΠ΄ΠΈΡ‚ΡŒΡΡ Π² ΠΏΡ€ΡΠΌΠΎΡƒΠ³ΠΎΠ»ΡŒΠ½ΠΎΠΉ Ρ„ΠΎΡ€ΠΌΠ΅.

Π’Π΅ΠΏΠ΅Ρ€ΡŒ ΠΌΡ‹ ΠΊΠΎΠ½Π²Π΅Ρ€Ρ‚ΠΈΡ€ΡƒΠ΅ΠΌ всС Π² ΠΏΠΎΠ»ΡΡ€Π½ΡƒΡŽ Ρ„ΠΎΡ€ΠΌΡƒ, Π° Π·Π°Ρ‚Π΅ΠΌ ΡƒΠΌΠ½ΠΎΠΆΠ°Π΅ΠΌ ΠΈ Π΄Π΅Π»ΠΈΠΌ ΠΊΠ°ΠΊ слСдуСт):

`= ((70 + 60j) (40-25j)) / (110 + 35j)`

`= ((92.тСкст (o) `

(ΠŸΡ€Π΅ΠΎΠ±Ρ€Π°Π·ΡƒΠ΅ΠΌ ΠΎΠ±Ρ€Π°Ρ‚Π½ΠΎ Π² ΠΏΡ€ΡΠΌΠΎΡƒΠ³ΠΎΠ»ΡŒΠ½ΡƒΡŽ Ρ„ΠΎΡ€ΠΌΡƒ.)

`= 37,22-5,93j`

(ΠŸΡ€ΠΈ ΡƒΠΌΠ½ΠΎΠΆΠ΅Π½ΠΈΠΈ комплСксных чисСл Π² полярной Ρ„ΠΎΡ€ΠΌΠ΅ ΠΌΡ‹ ΡƒΠΌΠ½ΠΎΠΆΠ°Π΅ΠΌ Ρ‡Π»Π΅Π½Ρ‹ Π½Π° (числа Π½Π° ΠΏΠ΅Ρ€Π΅Π΄Π½Π΅ΠΌ ΠΏΠ»Π°Π½Π΅) ΠΈ складываСм ΡƒΠ³Π»Ρ‹. ΠŸΡ€ΠΈ Π΄Π΅Π»Π΅Π½ΠΈΠΈ комплСксных чисСл Π² полярной Ρ„ΠΎΡ€ΠΌΠ΅ ΠΌΡ‹ Π΄Π΅Π»ΠΈΠΌ Ρ‡Π»Π΅Π½Ρ‹ Π½Π° ΠΈ Π²Ρ‹Ρ‡ΠΈΡ‚Π°Π΅ΠΌ ΡƒΠ³Π»Ρ‹. Π‘ΠΌ. Ρ€Π°Π·Π΄Π΅Π» Β«ΠŸΡ€ΠΎΠ΄ΡƒΠΊΡ‚Ρ‹ ΠΈ коэффициСнты» для получСния Π΄ΠΎΠΏΠΎΠ»Π½ΠΈΡ‚Π΅Π»ΡŒΠ½ΠΎΠΉ ΠΈΠ½Ρ„ΠΎΡ€ΠΌΠ°Ρ†ΠΈΠΈ.)

Π’Π°ΠΊΠΈΠΌ ΠΎΠ±Ρ€Π°Π·ΠΎΠΌ, ΠΌΡ‹ Π·Π°ΠΊΠ»ΡŽΡ‡Π°Π΅ΠΌ, Ρ‡Ρ‚ΠΎ суммарный импСданс Ρ€Π°Π²Π΅Π½

.

`Z_T = 37-5.9j \ Omega`

ΠŸΡ€ΠΈΠΌΠ΅Ρ€ 2

Учитывая, Ρ‡Ρ‚ΠΎ Z 1 = 200-40 j Ом ΠΈ Z 2 = 60 + 130 Π”ΠΆ Ом,


Π½Π°ΠΉΡ‚ΠΈ

Π°) ΠΏΠΎΠ»Π½ΠΎΠ΅ сопротивлСниС

Π±) Ρ„Π°Π·ΠΎΠ²Ρ‹ΠΉ ΡƒΠ³ΠΎΠ»

Π²) ΠΎΠ±Ρ‰ΠΈΠΉ Π»ΠΈΠ½Π΅ΠΉΠ½Ρ‹ΠΉ Ρ‚ΠΎΠΊ

ΠžΡ‚Π²Π΅Ρ‚

a) `Z_T = frac {Z_1Z_2} {Z_1 + Z_2}`

`= frac {(200-40j) (60 + 130j)} {(200-40j) + (60 + 130j)}`

`= frac {(200-40j) (60 + 130j)} {260 + 90j}`

`= frac {(204.@ «A» `

ΠŸΡ€ΠΈΠΌΠ΅Ρ€ 3

РСзистор 100 Ом, ΠΊΠ°Ρ‚ΡƒΡˆΠΊΠ° индуктивности 0,0200 Н ΠΈ ΠšΠΎΠ½Π΄Π΅Π½ΡΠ°Ρ‚ΠΎΡ€ 1,20 ΠΌΠΊΠ€ ΠΏΠΎΠ΄ΠΊΠ»ΡŽΡ‡Π΅Π½Ρ‹ ΠΏΠ°Ρ€Π°Π»Π»Π΅Π»ΡŒΠ½ΠΎ с схСма, состоящая ΠΈΠ· рСзистора 110 Ом, Π²ΠΊΠ»ΡŽΡ‡Π΅Π½Π½ΠΎΠ³ΠΎ ΠΏΠΎΡΠ»Π΅Π΄ΠΎΠ²Π°Ρ‚Π΅Π»ΡŒΠ½ΠΎ с `2.40 \ mu» F «` кондСнсатор. ΠŸΠΈΡ‚Π°Π½ΠΈΠ΅ `150 \» Π’ «,` 60 \ «Π“Ρ†» Π΅ΡΡ‚ΡŒ ΠΏΠΎΠ΄ΠΊΠ»ΡŽΡ‡Π΅Π½ ΠΊ Ρ†Π΅ΠΏΠΈ.

РассчитайтС ΠΎΠ±Ρ‰ΠΈΠΉ потрСбляСмый Ρ‚ΠΎΠΊ ΠΎΡ‚ источника питания ΠΈ Π΅Π³ΠΎ Ρ„Π°Π·ΠΎΠ²ΠΎΠ³ΠΎ ΡƒΠ³Π»Π°.

ΠžΡ‚Π²Π΅Ρ‚

Для Z 1 (вСрхняя Ρ‡Π°ΡΡ‚ΡŒ Ρ†Π΅ΠΏΡŒ) ΠΈΠΌΠ΅Π΅ΠΌ:

X L = 2 Ο€ f L = 2 Ο€ (60) (0.@ `.

Π’ΠΎΠΊΠΎΠ²Ρ‹Π΅ ΠΈ элСктричСскиС Ρ†Π΅ΠΏΠΈ :: Π—Π°Π΄Π°Ρ‡Π° 8

ΠŸΡ€ΠΎΠ±Π»Π΅ΠΌΠ° 8: НайдитС эквивалСнтноС сопротивлСниС сСти, ΠΏΠΎΠΊΠ°Π·Π°Π½Π½ΠΎΠΉ Π½ΠΈΠΆΠ΅. А Ρ‚Π°ΠΊΠΆΠ΅ Π½Π°ΠΉΡ‚ΠΈ Ρ‚ΠΎΠΊ Π² рСзисторС 8 Ом , Ссли ΠΊ сСти ΠΏΡ€ΠΈΠ»ΠΎΠΆΠ΅Π½Π° Ρ€Π°Π·Π½ΠΎΡΡ‚ΡŒ ΠΏΠΎΡ‚Π΅Π½Ρ†ΠΈΠ°Π»ΠΎΠ² 12 Π’.

РСшСниС:

Π¨Π°Π³: 1 ΠžΠ±Π·ΠΎΡ€

ΠŸΠ°Ρ€Π°Π»Π»Π΅Π»ΡŒΠ½Π°Ρ комбинация рСзисторов.
Π‘ΠΌ. ΠžΠ±Π·ΠΎΡ€ ΠΏΡ€ΠΎΠ±Π»Π΅ΠΌΡ‹ 7.
ΠšΠΎΠΌΠ±ΠΈΠ½Π°Ρ†ΠΈΡ рСзисторов сСрии.
Π’ этой ΠΊΠΎΠΌΠ±ΠΈΠ½Π°Ρ†ΠΈΠΈ рСзисторы соСдинСны встык, обСспСчивая Π΅Π΄ΠΈΠ½Ρ‹ΠΉ ΠΏΡƒΡ‚ΡŒ прохоТдСния Ρ‚ΠΎΠΊΠ° ΠΌΠ΅ΠΆΠ΄Ρƒ Ρ‚ΠΎΡ‡ΠΊΠ°ΠΌΠΈ A ΠΈ B. Благодаря СдинствСнному ΠΏΡƒΡ‚ΠΈ ΠΏΡ€ΠΎΡ‚Π΅ΠΊΠ°Π½ΠΈΠ΅ Ρ‚ΠΎΠΊΠ° Π² ΠΊΠ°ΠΆΠ΄ΠΎΠΌ рСзисторС остаСтся Π½Π΅ΠΈΠ·ΠΌΠ΅Π½Π½Ρ‹ΠΌ, Π° напряТСниС дСлится Π½Π° ΠΊΠ°ΠΆΠ΄ΠΎΠΌ рСзисторС. Π’Π°ΠΊΠΈΠΌ ΠΎΠ±Ρ€Π°Π·ΠΎΠΌ, ΠΎΠ±Ρ‰Π΅Π΅ напряТСниС Π±ΡƒΠ΄Π΅Ρ‚:
V = V 1 + V 2 + V 3 + … + V n
ИК = ИК 1 + ИК 2 + ИК 3 + … + ИК n
R = R 1 + R 2 + R 3 +… + R n
ΠŸΡ€ΠΈΠ²Π΅Π΄Π΅Π½Π½ΠΎΠ΅ Π²Ρ‹ΡˆΠ΅ ΡƒΡ€Π°Π²Π½Π΅Π½ΠΈΠ΅ ΠΈΡΠΏΠΎΠ»ΡŒΠ·ΡƒΠ΅Ρ‚ΡΡ для опрСдСлСния эквивалСнтного сопротивлСния ΠΏΠΎΡΠ»Π΅Π΄ΠΎΠ²Π°Ρ‚Π΅Π»ΡŒΠ½ΠΎ соСдинСнных рСзисторов.

Π¨Π°Π³: 2

РасчСт Из рисунка Π²Ρ‹ΡˆΠ΅ ΠΌΠΎΠΆΠ½ΠΎ ΡΠΊΠ°Π·Π°Ρ‚ΡŒ, Ρ‡Ρ‚ΠΎ рСзистор 8 Ом ΠΈ рСзистор 5 Ом ΠΏΠΎΠ΄ΠΊΠ»ΡŽΡ‡Π΅Π½Ρ‹ ΠΏΠ°Ρ€Π°Π»Π»Π΅Π»ΡŒΠ½ΠΎ, поэтому ΠΈΡ… эквивалСнт (R x ) Π±ΡƒΠ΄Π΅Ρ‚:
1 / R x = (1/8) + (1/5)
1 / R x = (5 + 8) / 40
1 / R x = 13/40
R x = 40/13
R x = 3,077 Ом

ПослС сокращСния схСмы ΠΌΡ‹ Π·Π°ΠΌΠ΅Ρ‡Π°Π΅ΠΌ, Ρ‡Ρ‚ΠΎ 3.РСзистор 077 Ом ΠΈ рСзистор 5 Ом соСдинСны ΠΏΠΎΡΠ»Π΅Π΄ΠΎΠ²Π°Ρ‚Π΅Π»ΡŒΠ½ΠΎ, поэтому ΠΈΡ… эквивалСнт (R y ) Π±ΡƒΠ΄Π΅Ρ‚:

R y = 3,077 + 5
R y = 8,077 Ом
ПослС Π·Π°ΠΌΠ΅Π½Ρ‹ R Π½Π° Π² Ρ†Π΅ΠΏΠΈ, Ρ‚Π΅ΠΏΠ΅Ρ€ΡŒ R , ΠΈ рСзистор 3 Ом соСдинСны ΠΏΠ°Ρ€Π°Π»Π»Π΅Π»ΡŒΠ½ΠΎ. Π’Π°ΠΊΠΈΠΌ ΠΎΠ±Ρ€Π°Π·ΠΎΠΌ, ΠΈΡ… эквивалСнтноС сопротивлСниС (R) Π±ΡƒΠ΄Π΅Ρ‚:
1 / R = 1 / R y +1/3
1 / R = (1 / 8,077) + (1/3)
1 / R = 11,077 / 24,231
R = 24,231 / 11,077
R = 2,0188 Ом
Π’Π΅ΠΏΠ΅Ρ€ΡŒ, Ρ‡Ρ‚ΠΎΠ±Ρ‹ Π½Π°ΠΉΡ‚ΠΈ Ρ‚ΠΎΠΊ Π² рСзисторС 8 Ом, Π½Π°ΠΌ Π½ΡƒΠΆΠ½ΠΎ Π²Π΅Ρ€Π½ΡƒΡ‚ΡŒΡΡ ΠΎΡ‚ простой схСмы ΠΊ слоТной, поэтому Π½Π°Ρ‡Π½Π΅ΠΌ с послСднСй схСмы.
ΠŸΠΎΡΠΊΠΎΠ»ΡŒΠΊΡƒ ΠΎΠ±Π° рСзистора R , ΠΈ 3 Ом ΠΏΠΎΠ΄ΠΊΠ»ΡŽΡ‡Π΅Π½Ρ‹ ΠΏΠ°Ρ€Π°Π»Π»Π΅Π»ΡŒΠ½ΠΎ, ΠΎΠ½ΠΈ Π±ΡƒΠ΄ΡƒΡ‚ ΠΈΠΌΠ΅Ρ‚ΡŒ ΠΎΠ΄ΠΈΠ½Π°ΠΊΠΎΠ²ΠΎΠ΅ напряТСниС, поэтому напряТСниС Π½Π° R ΠΈ Π±ΡƒΠ΄Π΅Ρ‚:
V y = 12 Π’
I y = V y / R y
I y = 12 / 8.077
I y = 1.486A
ΠŸΠΎΡΠΊΠΎΠ»ΡŒΠΊΡƒ R y прСдставляСт собой ΠΏΠΎΡΠ»Π΅Π΄ΠΎΠ²Π°Ρ‚Π΅Π»ΡŒΠ½ΡƒΡŽ ΠΊΠΎΠΌΠ±ΠΈΠ½Π°Ρ†ΠΈΡŽ рСзисторов R x ΠΈ 5 Ом, Ρ‚ΠΎΠΊ Π½Π° R x Π±ΡƒΠ΄Π΅Ρ‚ Ρ‚Π°ΠΊΠΈΠΌ ΠΆΠ΅, ΠΊΠ°ΠΊ Ρƒ R y . Π’Π°ΠΊΠΈΠΌ ΠΎΠ±Ρ€Π°Π·ΠΎΠΌ, Ρ‚Π΅ΠΊΡƒΡ‰ΠΈΠΉ Π² R x Π±ΡƒΠ΄Π΅Ρ‚:
Π― Ρ… = 1.486A
V x = I x R x
V x = (1,486) (3,077)
V x = 4,57V
НаконСц, ΠΏΠ΅Ρ€Π΅ΠΉΠ΄Π΅ΠΌ ΠΊ исходной схСмС, ΠΏΠΎΡΠΊΠΎΠ»ΡŒΠΊΡƒ R x прСдставлял собой ΠΏΠ°Ρ€Π°Π»Π»Π΅Π»ΡŒΠ½ΡƒΡŽ ΠΊΠΎΠΌΠ±ΠΈΠ½Π°Ρ†ΠΈΡŽ рСзисторов 5 Ом ΠΈ 8 Ом, поэтому напряТСниС Π½Π° ΠΎΠ±ΠΎΠΈΡ… этих рСзисторах Π±ΡƒΠ΄Π΅Ρ‚ Ρ‚Π°ΠΊΠΈΠΌ ΠΆΠ΅, ΠΊΠ°ΠΊ Ρƒ R x . Но нас интСрСсуСт Ρ‚ΠΎΠ»ΡŒΠΊΠΎ рСзистор 8 Ом, поэтому напряТСниС Π½Π° рСзисторС 8 Ом Π±ΡƒΠ΄Π΅Ρ‚:
Π’ 8 = 4,57 Π’
А Ρ‚ΠΎΠΊ Π² рСзисторС 8 Ом Π±ΡƒΠ΄Π΅Ρ‚:
Π― 8 = V 8 / R 8
Π― 8 = 4.57/8
I 8 = 0,57 A

Π­ΠΊΠ²ΠΈΠ²Π°Π»Π΅Π½Ρ‚Π½ΠΎΠ΅ сопротивлСниС сСти (R) = 2,0188 Ом
ΠΈ Ρ‚ΠΎΠΊ Ρ‡Π΅Ρ€Π΅Π· рСзистор 8 Ом (I 8 ) = 0,57 A (Ans)

.
Π Π°Π·Π½ΠΎΠ΅

Π”ΠΎΠ±Π°Π²ΠΈΡ‚ΡŒ ΠΊΠΎΠΌΠΌΠ΅Π½Ρ‚Π°Ρ€ΠΈΠΉ

Π’Π°Ρˆ адрСс email Π½Π΅ Π±ΡƒΠ΄Π΅Ρ‚ ΠΎΠΏΡƒΠ±Π»ΠΈΠΊΠΎΠ²Π°Π½. ΠžΠ±ΡΠ·Π°Ρ‚Π΅Π»ΡŒΠ½Ρ‹Π΅ поля ΠΏΠΎΠΌΠ΅Ρ‡Π΅Π½Ρ‹ *